The Neurologic Diagnosis wwwwwwwwww Jack N. Alpert

The Neurologic Diagnosis

A Practical Bedside Approach Jack N. Alpert, MD St. Luke’s Episcopal Hospital Department of Neurology University of Texas Medical School at Houston Houston, TX, USA [email protected]

ISBN 978-1-4419-6723-7 e-ISBN 978-1-4419-6724-4 DOI 10.1007/978-1-4419-6724-4 Springer New York Dordrecht Heidelberg London

Library of Congress Control Number: 2011941214

© Springer Science+Business Media, LLC 2012 All rights reserved. This work may not be translated or copied in whole or in part without the written permission of the publisher (Springer Science+Business Media, LLC, 233 Spring Street, New York, NY 10013, USA), except for brief excerpts in connection with reviews or scholarly analysis. Use in connection with any form of information storage and retrieval, electronic adaptation, computer software, or by similar or dissimilar methodology now known or hereafter developed is forbidden. The use in this publication of trade names, trademarks, service marks, and similar terms, even if they are not identifi ed as such, is not to be taken as an expression of opinion as to whether or not they are subject to proprietary rights. While the advice and information in this book are believed to be true and accurate at the date of going to press, neither the authors nor the editors nor the publisher can accept any legal responsibility for any errors or omissions that may be made. The publisher makes no warranty, express or implied, with respect to the material contained herein.

Printed on acid-free paper

Springer is part of Springer Science+Business Media (www.springer.com) In Memory of Morris B. Bender, a superlative clinical neurologist of the twentieth century, who stimulated my lifelong interest in the manifestations, evaluation, and diagnosis of patients with neurologic diseases. His infl uence permeates this text. wwwwwwwwww Foreword

Assessment of neurological complaints, a high percent of a family practitioner and internist’s practice, requires an accurate history and a careful neurological examination. As Dr. Alpert expertly discusses in detail, the history provides clues to the clinical diagnosis while the complementary neurological exami- nation localizes the lesion(s). Dr. Alpert is able to distil out from his over four decades of a busy practice and from his excellent teaching of medical stu- dents and neurological trainees a logical, readable, and provocative approach to each neurological complaint. In a chapter of 36 cases of “Diagnostic Dilemmas ,” one’s clinical acumen is challenged with practical questions and astute observations. By discussing neurological disorders in terms of ten “ Neuroanatomic Diagnoses,” Dr. Alpert compartmentalizes neurological diseases into conve- nient and manageable discrete entities. By building upon the unique anatomy and physiology of each unit, greater logic is made in one’s deductive reason- ing for a diagnostic conclusion. Special emphasis is given to the “Six Major Decussations” with clinical correlations, as with strokes, autonomic disor- ders, neuromuscular diseases and the poorly responsive patient. A separate chapter on “ Common Symptoms in the Neurology Clinic” is a potpourri of frequently seen and rare cases which will challenge the beginner and the experienced practitioner of neurology. This textbook is highly recommended to the serious student of clinical neurosciences, be they medical students, trainees, or practitioners.

Houston, TX, USA Frank M. Yatsu Chairman, Emeritus Department of Neurology University of Texas Medical School at Houston

vii wwwwwwwwww Preface

The purpose of this introductory text is to simplify the method of making a neurologic diagnosis. Medical students are often intimidated by a deluge of data and extensive differential diagnoses. They have no organizational struc- ture to follow. Diagnostic techniques of general medicine are not applicable. Neurology is a unique specialty since it requires the intermediary step of an anatomic diagnosis prior to proffering a differential diagnosis. Yet, the required knowledge of neuroanatomy need not be profound for the student who will specialize in any fi eld other than neurology and neurosurgery. This text is directed to medical students and residents who will all be regu- larly faced with numerous patients who have neurologic symptomatology. Typical one month rotations out of 4 years of medical school are clearly inad- equate training to make a cogent neurologic diagnosis, especially since subse- quent instruction is commonly provided by upper level residents who have the same background and numerous misconceptions. This is not a comprehensive text. The emphasis will be on establishing a neuroanatomic diagnosis before formulating a differential diagnosis. A thorough discussion of diseases will not be included. Treatment will seldom be discussed since it is likely to change radically over time. Redundancy of basic principles is purposeful. Repetition is the mother of learning or as William Gowers, an eminent neurologist of the early twentieth century, remarked, “A teacher who hesitates to repeat, shrinks from his most important duty, and a learner who dislikes hearing the same thing twice over lacks his most essential acquisition.”

Houston, TX, USA Jack N. Alpert

ix wwwwwwwwww Acknowledgments

Without the encouragement, assistance, and patience of my wife, Ruth, this book would never have been realized. My children, Richard, Daniel, Ariela, Jessica and my sister Barbara supplied never-ending interest and support. After a desultory beginning, the urging of Jessica, in particular, prompted me to actively pursue publication. Dr. Srinivasa Kandula contributed crucial assistance in selecting, creating, and sketching many of the illustrations. His eager, enthusiastic collaboration was vital to the completion of this text. Mike de la Flor added his expertise to several illustrations. Drs. Ernesto Infante and Frank Yatsu provided me with sound and indis- pensable advice with their reviews of the entire manuscript. I am grateful for their hard work, assistance, and positive feedback. Donna J. Williams was my invaluable, conscientious transcriptionist. Her technical expertise was outstanding. Her willingness to work late at night, answer questions, and complete just another chapter revision was truly remarkable. Additional neurologists who added useful critiques, interest, and sugges- tions include Drs. Stanley Appel, Randolph Evans, James Grotta, Victor Rivera, and Loren Rolak. Dr. Irving Fishman assisted in reviewing the physi- ology of the genitourinary system. I am most appreciative of their help and encouragement.

xi wwwwwwwwww Contents

1 Introduction ...... 1 2 The Ten Neuroanatomic Diagnoses ...... 3 Common Neurologic Signs: Localizing Value ...... 4 Central Nervous System Disease (CNS), Peripheral Nervous System (PNS), and Muscle Disease (M) ...... 4 Cerebral ...... 4 Extrapyramidal ...... 7 Brainstem/Cerebellum ...... 8 Spinal Cord Lesions (Myelopathies) ...... 11 Radiculopathy ...... 14 Plexopathy ...... 16 Neuropathy ...... 17 Neuromuscular Junction ...... 19 Myopathy ...... 20 Meningeal Disease ...... 21 Synopsis ...... 23 Multiple Choice Questions ...... 24 Answers ...... 25 Bibliography ...... 26 3 The Neurologic History Holds the Diagnostic Keys ...... 27 Neurologic Symptoms in Psychiatric Disease ...... 33 Bibliography ...... 35 4 Neurologic Examination ...... 37 Vital Signs ...... 37 Mental Status Examination ...... 38 Visual Observations ...... 38 Auditory Observations ...... 39 Aphasia ...... 39 Agnosia ...... 41 Apraxia ...... 41 Cranial Nerve Examination ...... 45 Olfactory Nerve (I) ...... 45 (II) ...... 45

xiii xiv Contents

Optic Nerve: Evaluation of Central Vision (See Fig. 4.2) ...... 46 Visual Field Examination (See Fig. 4.3) ...... 47 3rd, 4th, and 6th Cranial Nerves ...... 51 Trigeminal Nerve ...... 66 7th Cranial Nerve (Facial) ...... 68 8th Cranial Nerve (Vestibulocochlear) ...... 70 Glossopharyngeal Nerve (9th Cranial Nerve) ...... 75 Vagus Nerve (10th Cranial Nerve) ...... 76 Spinal Accessory Nerve (11th Cranial Nerve) ...... 76 Hypoglossal Nerve (12th Cranial Nerve) ...... 77 Motor Examination ...... 77 General Observations ...... 78 Close Inspection ...... 81 Coordination ...... 81 Strength ...... 82 Tone ...... 83 Gait and Station Examination ...... 85 Common Gait Disorders ...... 85 Uncommon Gait Disorders ...... 86 Refl ex Examination ...... 88 Method of Examination ...... 88 Enhancement Technique ...... 90 Refl ex Aberrations ...... 90 Superfi cial Refl exes ...... 90 Abnormal Refl exes ...... 90 Refl exes of Questionable Signifi cance ...... 92 Synopsis ...... 92 Sensory Examination ...... 93 Sensory Abnormalities According to Lesion Site ...... 94 Nonorganic Sensory Loss ...... 96 Sensory Terminology ...... 97 Dermatome Patterns (Fig. 4.18) ...... 97 Mechanical Signs ...... 98 Meningeal Disease ...... 98 Cervical and Lumbar Radiculopathy ...... 98 Questions (True or False) ...... 99 Answers ...... 100 Bibliography ...... 100 5 Evaluation of the Poorly Responsive Patient ...... 101 Clouding of Consciousness (Lethargy) ...... 101 Delirium ...... 101 Obtundation ...... 102 Stupor ...... 102 Coma ...... 102 Neurologic Examination ...... 102 Inspection ...... 102 Contents xv

Respiration (see Fig. 5.1) ...... 102 Mental Status Examination ...... 103 Cranial Nerves ...... 104 Utility of Caloric Testing ...... 106 Spontaneous Eye Movements in a Poorly Responsive Patient ...... 106 Eye Deviations ...... 107 ...... 107 Corneal Refl exes ...... 107 ...... 108 Palate ...... 108 Motor Function ...... 108 Refl exes ...... 108 Sensory ...... 109 Meningeal Signs ...... 109 Metabolic and Hypoxic-Ischemic Encephalopathies ...... 109 Metabolic Encephalopathy ...... 109 Hypoxic-Ischemic Encephalopathy ...... 114 Prognostic Factors in Comatose Survivors After CPR ...... 114 Herniation Syndromes ...... 116 Chronic Disorders of Consciousness ...... 119 Brain Death ...... 121 Questions (True or False) ...... 124 Answers ...... 125 References ...... 125 6 The Six Major Anatomic Decussations with Clinical Correlation ...... 127 The Corticospinal Tract (Fig. 6.1) ...... 127 The Corticobulbar Pathways (Fig. 6.2) ...... 129 The Oculomotor Decussation and Associated Pathways (Fig. 6.3) ...... 131 The Visual Pathways (Fig. 6.4) ...... 134 The Sensory Systems ...... 136 Questions (True or False) ...... 139 Answers ...... 139 Bibliography ...... 139 7 Cerebrovascular Anatomy with Clinical Correlation...... 141 Ischemic Stroke (Cerebral Infarction) and Transient Ischemic Attack ...... 141 Summary ...... 148 Venous Sinus Disease ...... 155 Intracerebral Hemorrhage ...... 156 Subarachnoid Hemorrhage ...... 159 Questions (True or False) ...... 163 Answers ...... 163 References ...... 164 xvi Contents

8 Autonomic Nervous System Anatomy with Clinical Correlation ...... 165 ...... 165 Case Reports ...... 169 Blood Pressure ...... 172 Genitourinary System ...... 174 Anatomic and Physiologic Overview...... 174 Sexual Function ...... 175 Bowel Function ...... 176 Questions (True or False) ...... 176 Answers ...... 177 References ...... 177 9 Neuromuscular Diseases: Neuroanatomic and Differential Diagnoses ...... 179 Cases ...... 180 Appendix ...... 204 Bibliography ...... 204 10 Common Symptoms in the Neurology Clinic ...... 207 Headache ...... 207 Migraine ...... 216 Trigeminal Neuralgia ...... 217 Temporal Arteritis ...... 217 Cluster Headache ...... 217 Chronic Paroxysmal Hemicrania ...... 217 Hypnic Headache ...... 217 Short-Lasting, Unilateral Neuralgiform Headaches with Conjunctival Injection and Tearing (SUNCT) ...... 217 Glossopharyngeal Neuralgia ...... 218 Vertigo ...... 219 Near-Syncope, Syncope, and Seizure ...... 227 ILAE Classifi cation of Epileptic Seizures (Modifi ed) ...... 230 Classifi cation According to Etiology ...... 231 Sleep Disorders ...... 235 Sleep Apnea ...... 236 Parasomnias ...... 238 Chronic Primary Insomnia ...... 241 Narcolepsy and Cataplexy ...... 242 Diagnostic Criteria for Narcolepsy ...... 242 Idiopathic Hypersomnia ...... 243 Other Transient Neurologic Symptoms ...... 243 “Drop Attack” (The Sudden Inexplicable Fall) ...... 243 Confusion/Amnesia ...... 245 Transient ...... 246 Transient Visual Illusions and Hallucinations ...... 247 Transient Motor and Sensory Disturbances ...... 248 Paroxysmal Motor Phenomena ...... 250 References ...... 251 Contents xvii

Headache ...... 251 Vertigo ...... 251 Near-Syncope, Syncope and Seizure ...... 251 Sleep Disorders ...... 251 Transient Neurologic Symptoms ...... 251 11 Diagnostic Dilemmas ...... 253 References ...... 280 12 Neurologic Terminology ...... 283 Acronyms ...... 283 Signs ...... 284 Syndromes ...... 289

Index ...... 295

Introduction 1

Keywords Neuroanatomy ¥ Neuroimaging ¥ Diagnostic methods

Neurology has made major advances over the last forms of cancer can be cured, usually through few decades in understanding some of the com- excision of the abnormal tissue. plexities of neuroanatomic structures and their Only when physicians have been in practice physiologic ramifi cations. This has resulted in for a few years do they recognize the importance the development of effi cacious treatment options of neurologic symptoms and their inability to for common neurologic disorders such as epi- analyze them. In fact, the majority of symptoms lepsy and migraine and for previously debilitat- that affect the body, other than the chest and abdo- ing diseases, for example, myasthenia gravis and men, have neurologic sources or neurologic rami- multiple sclerosis. Nevertheless Neurology has fi cations. Furthermore, patients with unexplained been bedeviled with an unjustifi ed accusation chest and abdominal pain may benefi t from a that diseases of the nervous system are observ- neurologic consultation. Thus, common complaints able but untreatable. This misconception has been such as loss of consciousness, impairment of handed down through generations of medical stu- memory and speech, headache, dizziness, numb- dents because of the dearth of suffi cient instruc- ness, paresthesias, weakness, neck pain, pain of tion in the early years of medical school. the extremities, and low back pain require a clini- Furthermore, it is commonly asserted that cal assessment, not necessarily neurophysiologi- neurologic diseases are uniquely incurable. This cal and neuroimaging procedures. fallacious argument, often promulgated by Modern technology with CAT and MRI scans, poorly educated physicians, conveniently for- angiography, electromyography, and electroen- gets that most illnesses are incurable. For cephalography are wonderful adjuncts to making instance, are there cures for people with asthma, a diagnosis. They cannot be substituted for the diabetes, hypertension, coronary artery disease, initial assessment. Here is an example: and rheumatoid arthritis, just to name a few? Some examples of curable diseases are infec- Case report A 55-year-old man arrives in the tious diseases which can be eradicated by anti- emergency room with left-sided weakness after a biotics or run a natural course of spontaneous motor vehicle accident. His car was struck from recovery. Gallstones, appendicitis, and some the passenger side where he was sitting. He has

J.N. Alpert, The Neurologic Diagnosis: A Practical Bedside Approach, 1 DOI 10.1007/978-1-4419-6724-4_1, © Springer Science+Business Media, LLC 2012 2 1 Introduction a medical history of diabetes and hypertension. locations in brain and spinal cord. How can one A neurologic examination disclosed a blood distinguish lesions of the medial lemniscus from pressure of 150/110. STAT blood sugar was the lateral spinothalamic tract? The presence of a 280 mg/dl. Neurologic abnormalities included a “long tract sign,” such as a Babinski sign, indi- moderate left , arm greater than leg, cates a lesion of the corticospinal tract which is a left hyperrefl exia with left Hoffmann and Babinski few feet in length. As the tract extends from the signs, and impaired rapid alternating movements frontal cortex to the caudal portion of the spinal of the left arm. There was no sensory loss. cord, additional abnormal signs are required for A prompt CT scan of the brain revealed prominent the precise localization of pathology. subcortical ischemic changes, right greater than The predominant principle of this text will be left. A subsequent MRI scan of the brain confi rms an emphasis on the existence of only ten neuro- vascular pathology because of multiple scattered anatomic diagnoses. The second chapter will T2 hyperintensities. The patient was admitted to a introduce this concept. A simplifi cation, how- stroke unit and a neurological consultant was ever, does not obviate the necessity of acquiring requested. The neurologist promptly ordered an at least a modest knowledge of neuroanatomy. MRI scan of the cervical spine which revealed a After a discussion of methods to obtain a neuro- herniated disk, C3ÐC4 on the left side, compress- logic history and perform a neurologic examina- ing the spinal cord. Surgery was performed and tion, four subsequent chapters will focus on the the patient recovered completely. major elements of neuroanatomy and their clini- cal relevance. A separate chapter will review the Lessons: common neurologic symptoms of an outpatient 1. Hemiparesis is not a localizing abnormality clinic, applicable both to general medicine and other than supporting the presence of central neurology. Each subtopic will be dealt with nervous system disease. cursorily since the purpose of this text is only 2. The past medical history, in this case diabetes to provide an overview of the spectrum of neuro- and hypertension, leads the attending physi- logic disorders. In Chap. 11 , Diagnostic cian astray. Dilemmas, a variety of case reports will be pre- 3. CT or MRI abnormalities in the wrong loca- sented to expose the reader to the wide spectrum tion promote inaccurate diagnoses. of neurologic diseases. As neurologic syndromes Neuroanatomy frightens the fl edgling medical and terminology are so ingrained in medical lore, student. For the beginner, there are numerous, a separate, fi nal chapter to review them is unfathomable tracts criss-crossing at different obligatory. The Ten Neuroanatomic Diagnoses 2

Keywords Neuroanatomy ¥ Neuropathy ¥ Localization ¥ Neuromuscular junction ¥ Encephalopathy ¥ Myopathy ¥ Myelopathy ¥ Radiculopathy

This chapter aims merely to introduce the student of a neurologic disease. There is a plethora of to the unique diagnostic method used by neurolo- tracts and “centers” of function which suppos- gists. Detailed discussion of anatomic pathways, edly require the acquisition and recall of abnormal fi ndings, and differential diagnosis is detailed anatomic minutia. In fact, only a rough provided in subsequent chapters. Sections on localization to one of ten anatomic sites is case reports will allow the student or physician to required for practical purposes. An inaccurate utilize the principles discussed in the foregoing selection of one of these diagnoses will make a chapters. differential diagnosis nearly impossible. This The immediate linkage of symptoms and signs choice does require a suffi cient knowledge of to the differential diagnosis is the common neuroanatomy and neurophysiology which will approach in the practice of internal medicine. For be reviewed in subsequent chapters. The outline neurology, this practice is fraught with hazards and below is a summary of essential diagnostic contains the seeds of misdiagnosis. A neuroana- principles. tomic diagnosis is a required preliminary step. In Ten neuroanatomic sites to choose from: addition, there are numerous misconceptions about 1. Cerebral neurologic symptoms and signs which trigger 2. Extrapyramidal errors when making a neuroanatomic localization. 3. Brainstem/Cerebellum The consequences are often unnecessary, exorbi- 4. Spinal cord tant expenditures for inapplicable neuroimaging or 5. Root neurophysiologic testing, and inappropriate refer- 6. Plexus rals. This results in delayed care, sometimes with 7. Nerve serious adverse consequences. 8. Neuromuscular junction Most medical students are worried, if not ter- 9. Muscle rifi ed, about making an inaccurate localization 10. Meninges

J.N. Alpert, The Neurologic Diagnosis: A Practical Bedside Approach, 3 DOI 10.1007/978-1-4419-6724-4_2, © Springer Science+Business Media, LLC 2012 4 2 The Ten Neuroanatomic Diagnoses

(c) Degenerative Common Neurologic Signs: (d) Degenerative, structural/compressive Localizing Value (e) Developmental (f) Genetic Central Nervous System Disease (CNS), (g) Infectious Peripheral Nervous System (PNS), (h) Metabolic and Muscle Disease (M) (i) Neoplasm (j) Nutritional 1. Pathologic signs indicating CNS disease with (k) Toxic localizing value (l) Trauma (a) Dementia: Cerebral (m) Vascular (b) Aphasia: Cerebral 5. The paroxysmal disorders (c) Seizure: Cerebral (a) Headache and facial pain (d) Abnormalities of mood, personality, and (b) Seizure and syncope behavior associated with abnormal neuro- (c) Sleep disorders logic signs: Cerebral (d) Transient ischemic attack (requires ana- (e) Homonymous : Cerebral tomic defi nition) (f) Abnormal involuntary movements/pos- (e) Transient global amnesia tures: Extrapyramidal (f) Vestibular disorders (requires anatomic (g) Abnormal eye movements and/or pupils defi nition) localizing to: Brainstem or cerebellum (h) Crossed fi ndings, ipsilateral cranial nerve, and contralateral long tract: Brainstem Cerebral (i) Sensory levels: Spinal cord, rarely brainstem 2. Pathologic signs indicating CNS disease but The fi ve pathognomonic signs of cerebral hemi- nonlocalizing within the CNS sphere disease are dementia or a confusional state, (a) Hemiparesis abnormalities of mood, personality and behavior (b) Hemisensory loss associated with abnormal neurologic signs, aphasia, (c) Hemiataxia seizure, and . The absence (d) Unilateral hyperrefl exia including unilat- of these abnormalities does not exclude cerebral eral Hoffmann signs disease but must allow for other possibilities. (e) Babinski sign (f) Eye movement abnormalities of cerebral, Pathognomonic signs of cerebral disease are brainstem, or cerebellar origin dementia/confusion, aphasia, seizure, hom- (g) Position sense loss exceeding loss of onymous hemianopsia, and abnormalities of vibration perception mood, personality and behavior accompanied 3. Pathologic signs of CNS or PNS/M origin by abnormal neurologic signs. (a) Dysarthria (b) Dysphagia Case 1 A 67-year-old retired male accountant is (c) Monoparesis, paraparesis, , brought to the Emergency Department at quadriparesis, and crossed 10:00 p.m. because of slurred speech and confu- (d) Dysdiadochokinesis (impaired rapid alter- sion. He is unable to give a coherent history. His nating movements) wife says that she noticed slurred speech shortly (e) Positive Romberg after dinner followed promptly by confusion and (f) Gait disorders asking the same question repeatedly. He has dia- 4. Common differential diagnoses betes controlled by diet only and has just recov- (a) Autoimmune/infl ammatory ered from a severe gastroenteritis causing (b) Demyelinating prolonged watery diarrhea. Cerebral 5

Neurologic examination : 7. Postural and may occur with The patient is oriented to person, year, not month any metabolic encephalopathy. or place. Speech is severely dysarthric but con- CVA is no longer an acceptable diagnosis. tent and syntax are normal. He is able to add A stroke is either an infarction or hemorrhage, 14 + 6 but not 14 + 17. He spells “hand” forward not an accident. but not backward. Short-term recall is one of Summary : three words given after 3 min have elapsed. He This patient has an acute encephalopathy which has impaired rapid alternating movements of both indicates bilateral cerebral dysfunction. Bilateral hands and is clumsy when removing his shirt. He cerebral dysfunction can be due to a slowly pro- has a mild postural tremor and asterixis. gressive disorder such as Alzheimer’s disease The third-year medical student’s diagnosis : which would not be characterized as an encephal- Cerebrovascular accident (CVA) in the left mid- opathy. The term encephalopathy implies a rap- dle cerebral artery distribution. idly developing confusional state. An acute cerebral ischemic event rarely produces an Neurologist’s diagnosis : encephalopathy. Furthermore, the term CVA is Encephalopathy, probably metabolic. no longer an acceptable diagnosis. A stroke is an Laboratory data : infarction or hemorrhage, not an “accident.” Serum sodium is 162 mEq/L and BUN 70 mg/dl. Categories of diseases which are likely to cause an encephalopathy include metabolic alter- Final diagnosis : ations, infections such as an encephalitis, nutri- Encephalopathy secondary to hypernatremia tional disorders, toxic substances or drugs, and caused by volume contraction due to severe trauma. diarrhea. Case 2 A 73-year-old man reports the sudden Lessons : onset of left-sided weakness beginning 2 weeks

ago with a mild increase in weakness since then. The differential diagnosis of cerebral hemi- He has a history of diabetes and hypertension. sphere disease depends on whether it is unilat- eral or bilateral. Examination : Blood pressure 150/100, left hemiparesis with 1. The fi rst task is to determine whether there is greatest involvement of hand and foot, left hyper- focal or diffuse disease, in this case unilateral refl exia including a left Hoffmann and Babinski or bilateral cerebral disease. signs. 2. Confusion indicates bilateral cerebral MRI (brain) : dysfunction. Multiple subcortical T2 hyperintensities typical 3. Dysarthria is nonlocalizing. Aphasia indicates of vascular disease but no discrete infarction. a dominant hemisphere lesion. 4. Acalculia and errors reversing words are usu- MRA (head and neck) : ally not focal signs. Occasionally, they can be 60% stenosis of right internal carotid artery and part of an aphasia. 85% stenosis of left internal carotid artery. 5. Short-term recall defi cits indicate bilateral Neurologic consultation is requested with cerebral dysfunction. regard to treatment options. The neurologist takes 6. Impaired rapid alternating movements are of additional history as a neurologic review of sys- localizing value when unilateral. This tems is mandatory for every patient even if seem- impairment occurs with any motor system ingly irrelevant. The patient reports a 2-year involvement, corticospinal, cerebellar, or extra history of periodic severe neck pain with pyramidal. Proprioception loss and weakness occasional radiation to the left shoulder. The neu- from any etiology may be the source of this rologist’s examination shows no other fi ndings. fi nding. Recognizing that a hemiparesis is a nonlocalizing 6 2 The Ten Neuroanatomic Diagnoses sign of central nervous system disease, an MRI of the year he repeats the month. On the fourth the cervical spine is obtained. This reveals an request to state the year he responds correctly. extramedullary mass at C3ÐC4 on the left side When asked to repeat “no ifs, ands or buts,” he compressing the spinal cord. Extramedullary replies “no ifs, ands, and buts.” The remainder of tumors, outside the spinal cord and within the a complete neurologic examination is normal. dura, are nearly always benign such as a schwan- Laboratory fi ndings : noma or meningioma. The EKG monitor shows intermittent atrial fi bril- Diagnosis : lation. CT (head) reveals a small, ill-defi ned Cervical myelopathy secondary to a schwannoma hypodensity in the left temporal lobe. with spinal cord compression at C3ÐC4. Diagnosis : Treatment : Left cerebral infarction, temporal lobe, due to Cervical laminectomy and resection of the neo- cardioembolism to a branch of the left middle plasm. The patient makes a complete recovery. cerebral artery. Atrial fi brillation, a common complication after heart surgery, is the etiology. Lessons : Lessons : A hemiparesis may occur with cerebral, brain- stem, or spinal cord disease. Aphasia is often misinterpreted as confusion. Aphasia suggests stroke whereas confusion 1. Take a complete history. A seemingly irrele- implies metabolic, toxic or hypoxic-ischemic vant symptom may be pertinent. Pain radiating factors. to the shoulder may be radicular, particularly common with a schwannoma. 1. Accurate distinction between confusion and 2. Recognize that a spastic hemiparesis localizes aphasia is critical. Confusion means bilateral the disease only to the central nervous system, cerebral dysfunction and therefore metabolic, specifi cally cerebral, brainstem and spinal cord, toxic or hypoxic-ischemic etiologies. Aphasia the location of the corticospinal tract. indicates a stroke in this clinical setting. 3. The differential diagnosis of spinal cord disease 2. The fi rst clue to a neurologic disorder is acute associated with a 2-year history of neck pain anxiety occurring many hours after surgery, a begins fi rst with structural degenerative disease. not unusual occurrence in a patient with the Neoplasm would be a secondary choice, but sudden onset of aphasia. This prompts a neu- nevertheless always a diagnostic consideration. rologic evaluation. 4. MRI scans show much pathology which may be clinically irrelevant. Anxiety and agitation often occur with the acute onset of aphasia. Case 3 A 66-year-old man with coronary artery disease undergoes coronary artery bypass surgery 3. The patient exhibits a paraphasia which, in (CABG). Thirty-six hours later he becomes this case, is the substitution of the word “dis- acutely anxious and appears confused to the nurse placed” for “discharged.” This is a semantic who requests a sedative for her patient. A well- (verbal) paraphasia. A phonemic paraphasia trained, careful medical resident decides to evalu- would be, for example, to say “plor” instead of ate the patient before prescribing a sedative. “fl oor” or “fl eet” instead of “street.” He dem- Neurologic examination : onstrates perseveration which is giving the The patient asks the resident, “When will I be same response despite a new question being displaced?” then says, “I mean discharged.” The asked. Additionally, the patient is unable to patient is alert, has fl uent speech, and is oriented repeat a simple phrase. These three fi ndings, to person, place, and month. When asked to state paraphasia, perseveration, and poor repetition Extrapyramidal 7

are characteristic of aphasia although perse- The extrapyramidal system is responsible for veration may occur in demented patients. See the maintenance of posture. Lesions of the detailed discussion of aphasia in Chap. 4 . extrapyramidal system cause abnormal involun- 4. The presence of aphasia ordinarily indicates a tary movements and abnormal postures. This sys- left cerebral lesion. Only about 50% of left- tem is represented mainly by specifi c nuclei, the handed individuals are right hemisphere domi- basal ganglia, which are located in deep subcorti- nant. Since about 10% of the population is cal regions. The primary nuclei are the caudate, left-handed there is about a 5% chance of a globus pallidus, subthalamus, and putamen. An right cerebral lesion. additional structure, the substantia nigra, is 5. The acute onset of a focal cerebral defi cit in a located in the midbrain. Lesions in these regions patient who just underwent coronary artery result in abnormal postures or abnormal bypass surgery is nearly always an infarction. involuntary movements. Examples include The most common etiology in the postopera- Parkinson’s disease, chorea, hemiballism, atheto- tive period is cardioembolism associated with sis, and dystonia. These disorders will be dis- atrial fi brillation. Atrial fi brillation occurs in cussed in the chapter on neurologic examination. one third of post-CABG patients. 6. Thus, a single word, “displaced,” prompts a Lesions of the extrapyramidal system cause diagnosis of cardioembolism to the left abnormal involuntary movements such as bal- middle cerebral artery due to postoperative lism, , chorea, dystonia, and tremor. atrial fi brillation! Case 4 An 85-year-old woman complains of Signs of aphasia are paraphasias, dysno- sudden involuntary movements of the right arm mia, abnormal speech fl uency, and poor which began 5 days ago. They occur every few repetition. minutes. At about the same time she noted blurred vision affecting the right eye. She has a history of 7. The common diagnostic hallmarks for aphasia hypertension controlled with a diuretic. include one or more of these fi ndings: para- Examination : phasias, dysnomia, abnormal speech fl uency, Neurologic examination discloses a right hom- and poor repetition. Any one of these fi ndings onymous hemianopsia. There are abrupt fl inging indicates the likelihood of focal disease and movements of the right arm over her head. thus rules out a solely metabolic or toxic MRI (head) reveals infarctions in the left etiology. occipital lobe and in the region of the left subtha- lamic nucleus. The MRA (head and neck) dis- closes a severe left posterior cerebral artery stenosis. Extrapyramidal Diagnosis : Left subthalamic nucleus and occipital lobe infarctions due to atheromatous disease causing There are three motor systems: corticospinal, severe stenosis with subsequent thrombosis in the extrapyramidal, and cerebellar mediating ini- proximal portion of the left posterior cerebral tiation of movement, maintenance of posture, artery. and coordination, respectively. Lessons : There are three motor systems, corticospinal, 1. Branches of the left posterior cerebral artery extrapyramidal, and cerebellar. The corticospinal supply the left subthalamic nucleus, thalamus, tract mediates initiation of movement, and the portions of the midbrain, inferior temporal, cerebellar system, coordination of movement. and occipital lobes. 8 2 The Ten Neuroanatomic Diagnoses

Lessons : Ballism produces sudden movements at 1. The patient describes a festinating gait, an proximal joints, chorea affects distal mus- uncontrollable increase in speed in an attempt culature, athetotic movements are writh- to catch up with his center of gravity. ing, and dystonia is marked by abnormal 2. He has a fl at affect, hypomimia, which is also postures. known as masked facies. This is a decreased mobility of facial musculature. Yet the patient 2. Hemiballism is a fl inging irregular movement has a positive glabellar refl ex, known as at proximal joints, especially shoulder and Myerson’s sign, which is a rapid, sustained hip, most often due to a lesion in the subtha- blinking when the bridge of the nose is tapped. lamic nucleus. Thus refl ex responses can be increased in the 3. Differential diagnoses of the patient’s involun- presence of lack of mobility. tary movements include chorea, athetosis, and 3. Bradykinesia is exhibited by the patient slowly dystonia. crossing his legs. 4. Choreiform movements are rapid, jerky move- 4. Thus pure observation on entering the exami- ments of the extremities primarily involving nation room makes the diagnosis. Paying atten- distal musculature. tion to the patient’s facial mobility and leg 5. Athetosis is characterized by slow, writhing movements furnishes the two primary diagnos- movements of the extremities. tic clues. 6. Dystonic movements are persistent or inter- mittent maintenance of an abnormal posture. 7. The patient describes visual loss in only one eye, a relatively common complaint of patients with Brainstem/Cerebellum homonymous hemianopsias. The temporal fi eld loss is larger and more apparent to the patient. Crossed fi ndings and abnormal eye move- 8. Treatment of the involuntary movements with ments typify brainstem lesions. a dopamine antagonist is useful, and the syn- drome usually subsides over several weeks to The cardinal features of diseases affecting the a few months. brainstem are: (1) crossed fi ndings and (2) spe- Case 5 A 70-year-old man complains of running cifi c types of abnormal eye movements. Examples into walls. For the last 3 months, while walking of the former include right facial weakness and in the corridors at work, he involuntarily picks up left hemiparesis, right third nerve palsy and left speed and has to catch himself by aiming at the hemiparesis (Weber’s syndrome), ipsilateral gaze nearest door or wall. He has no other complaints. palsy, and contralateral hemiparesis, i.e., gaze to When specifi cally queried about turning in bed or the right with a right hemiparesis (Foville’s syn- getting out of a chair he reports that he has slowed drome) and a third nerve lesion with contralateral down. tremor or choreiform movements (Benedikt’s syndrome). Examination : Most forms of pathologic occur The patient has a fl at affect and his shoulders with brainstem lesions and less often pure cere- droop. He walks with a stooped posture and leans bellar lesions. The direction of nystagmus refers forward. He crosses his legs slowly. Tapping the to its quick phase. Quick phases of nystagmus bridge of his nose elicits repeated blinking. A that are pure rotatory, multidirectional, oblique, normal individual may blink once or twice but disconjugate, or vertical indicate brainstem/cere- not persistently and uninterruptedly. There is bellar pathology. An important exception is drug mild cogwheel rigidity and no tremor. toxicity which most often causes horizontal, Diagnosis : gaze-evoked nystagmus. This means left-beating Parkinson’s disease. nystagmus on left lateral gaze and right-beating Brainstem/Cerebellum 9 nystagmus on right lateral gaze. Nystagmus in the 3 mm with both reacting briskly to light. The vertical plane may also occur with drug toxicity. right palpebral fi ssure is larger than the left. The left face is warmer than the right. The left corneal Peripheral vestibular disease typically produces response is absent. The left soft palate elevates horizontal, direction-fi xed nystagmus with a poorly. There is mild left heel-to-shin . slight rotatory element. There is slight impairment of pinprick sensibility on the right side of the body. If the quick phases of nystagmus are horizontal and unidirectional with any ocular deviation, an Dysphagia, vertigo, , and facial numb- eighth nerve or semicircular canal lesion may be ness are common symptoms of brainstem present. The proviso is the absence of other neu- ischemia. rologic symptoms or signs which would indicate central nervous system disease. A left eighth nerve Diagnosis : lesion, for instance, produces direction-fi xed, Brainstem infarction, left dorsolateral medulla, contralateral (right-beating), horizontal nystag- secondary to a left vertebral artery occlusion with mus with a slight rotatory element which is ischemia in the left posterior inferior cerebellar counterclockwise and most prominent on right artery distribution. The diagnosis is confi rmed by lateral gaze, usually present on up or down gaze MRI scan. and least often on left lateral gaze. The reverse occurs with a right eighth nerve lesion. Lessons : 1. This patient has a partial Wallenberg’s Pendular nystagmus, equal velocity in both syndrome. directions, is usually congenital. Nystagmus due to CNS lesions is most Pendular nystagmus is usually of congenital often secondary to brainstem lesions, less origin. Manifestations are nystagmus which has often pure cerebellar pathology. equal velocities in both directions which means no separation into quick and slow phases. There 2. He has a left Horner’s syndrome manifested may be also jerk nystagmus with quick and slow by , , and anhidrosis (warmth) on phases which vary in intensity with changes of eye the left side. The sympathetic system begins in position. This nystagmus is accentuated by visual the hypothalamus, descends through the brain- fi xation, and there is commonly a null point, a stem, coalesces in the dorsolateral medulla location where it disappears. The patient seldom before it passes down the spinal cord. See has symptoms. The etiology is unknown. Chap. 8 on autonomic nervous system. 3. Horizontal gaze-evoked nystagmus indicates Case 6 A 62-year-old man complains of diffi - involvement of vestibular nuclei. culty swallowing of 3 days’ duration. At the onset 4. The absent left corneal response indicates of this problem he was briefl y dizzy and had involvement of the left spinal tract and nucleus numbness in his mouth. He describes the dizziness of the fi fth cranial nerve. Numbness in the as a tilting of the horizon, a visual illusion. He mouth at the onset of the stroke is caused by walked unsteadily. There was rapid improvement the same lesion. except for residual diffi culty swallowing liquids.

His past medical history includes hypertension and hypercholesterolemia. Medications are often occurs with lesions lisinopril and atorvastatin. outside the cerebellum proper.

Examination : 5. Left heel-to-shin ataxia is consistent with The patient has mild horizontal, gaze-evoked involvement of the left inferior cerebellar nystagmus. The left pupil is 2 mm and the right is peduncle (restiform body). Of note is the com- 10 2 The Ten Neuroanatomic Diagnoses

mon presence of ataxia without intrinsic cere- abducens nucleus which receive input from the bellar disease. ipsilateral PPRF and send fi bers across the mid- 6. Weakness of the left soft palate indicates line to form the contralateral MLF. Thus, the involvement of the nucleus ambiguus on the MLF is responsible for ipsilateral adduction. left side which contains the motor portion of Case 7 A 52-year-old man is brought to the the tenth cranial nerve. emergency room after the sudden onset of diplo- 7. Hypesthesia on the right side of the body pia, vertigo, and vomiting followed by right-sided points to involvement of the lateral spinotha- weakness. He has a 10-year history of hyperten- lamic tract on the left side. sion and smokes one pack of cigarettes per day.

He has had his hypertension under control with There are quick and slow eye movement sys- amlodipine. tems. Saccades (quick) are used to scan the environment. Visual pursuit (slow) follows a Examination : moving target. The vestibular system gener- Examination reveals a blood pressure of 160/100 ates slow eye movements when the head is and he has bilateral carotid bruits. The eyes are turned quickly. deviated to the right and he is unable to look to the left. He has a right spastic hemiparesis. A brief comment about the two main eye Laboratory studies : movement systems is now required. The quick or Carotid Dopplers show bilateral 60Ð70% stenoses saccadic system is used to place the fovea on the of the internal carotid arteries. A CAT scan of the object of interest when scanning the environment. brain is normal. An example of the use of saccades is inspecting parts of a stationary object such as a painting. The Diagnosis : slow system includes visual tracking such as fol- Left pons ischemia secondary to basilar artery lowing the course of a plane through the sky. The disease. vestibular system also generates a slow phase with Lessons : quick head movement, part of the vestibulo-ocu- 1. Diplopia, vertigo, and vomiting are character- lar refl ex arc. The cerebellar fl occulus and vermis istic symptoms of brainstem ischemia. play a role in generating slow eye movements. 2. Eye deviation to the right is compatible with either a right cerebral or a left pontine (PPRF) The crossed fi ndings of brainstem lesions are lesion. usually cranial nerve involvement on one side 3. A right hemiparesis can be due to a left cere- and hemiparesis or hemihypesthesia on the bral, internal capsule, midbrain, pontine or other. medullary lesion. Below the decussation of the pyramids at the cervical medullary junc- A voluntary, contralateral saccade begins with tion, a right hemiparesis is caused by a right- a signal produced, for example, in the right fron- sided spinal cord lesion. toparietal eye fi elds, descends to the contralateral left PPRF (paramedian pontine reticular forma- The oculomotor pathway between fron- tion) after crossing the midline between the mid- toparietal eye fi elds and the PPRF decus- brain and pons. The left PPRF generates a saccade sates at the pontomesencephalic junction. to the left by sending fi bers to the left sixth nucleus and the right third nucleus. The left 4. Right cerebral ischemia would produce eye sixth nerve innervates the left lateral rectus and deviation to the right and a left hemiparesis, the right third nerve innervates the right medial but left pontine ischemia would explain the rectus muscle. The medial longitudinal fascicu- fi ndings because of involvement of the left lus (MLF) connects the left PPRF to the right PPRF and left basis pontis (corticospinal third nucleus. There are interneurons in the tract). Thus, when a patient looks at his weak Spinal Cord Lesions (Myelopathies) 11

side he most likely has a contralateral pontine Loss of vibration sense precedes loss of posi- lesion. The oculomotor pathway will be tion sense except with cerebral lesions. reviewed in detail in Chap. 6 .

Neurologic examination : Carotid artery stenoses are unrelated to The patient has impaired color vision OD with a brainstem ischemia. visual acuity of 20/20. She has impaired rapid alternating movements of the right arm (dysdiado- 5. Carotid stenoses are unrelated since the verte- chokinesis), 4+/5 strength of right interossei and brobasilar system supplies the pons. wrist extensors, absent vibratory perception at right 6. A powerful risk factor for carotid stenosis is toes, ankle, and knee and she makes a few position smoking. sense errors at the right toes. She has right Hoffmann and right Babinski signs. There is a sensory level Spinal Cord Lesions (Myelopathies) of pain and temperature at T4 on the left side. MRI of the cervical spine shows an intramed- ullary lesion (within cord) as opposed to Myelopathies can be manifested by pure involve- extramedullary (outside cord, inside dura) and ment of motor or sensory pathways but com- extradural (outside dura). monly a combination of both (see Fig. 2.1 ). Diagnosis (by radiologist) : Case 8 A 33-year-old woman complains of Intramedullary neoplasm versus multiple scalding her left foot without realizing it when sclerosis. testing her bath water 1 week ago. The burn is healing well, but she has an uncomfortable Diagnosis : tingling sensation on the left leg. A thorough Cervical myelopathy with partial Brown- review of neurologic symptoms elicits what the Séquard’s syndrome and OD patient describes as a “minor problem,” blurred secondary to multiple sclerosis. vision OD when running her usual 5 miles before An MRI of the brain shows four periventricular going to work. This began 1 year ago. The patient T2 and FLAIR hyperintensities, perpendicular to has insulin-dependent diabetes mellitus. She the ventricles. These fi ndings on brain MRI sup- takes no medication other than insulin. port the diagnosis of multiple sclerosis.

Anatomy of Spinal Cord Fasciculus gracilis Fasciculus cuneatus S L Th C Central canal Dorsal spinocerebellar tract S Lateral corticospinal tract L Intermediolateral Th gray column C Ventral spinocerebellar tract L S Lateral spinothalamic tract C Th

Ventral corticospinal tract Anterior white commissure

Fig. 2.1 Cross-section of the spinal cord at T2. The three lateral spinothalamic tracts have the same pattern. major tracts are shown with their lamination. C cervical; T The intermediolateral gray column contains sympathetic thoracic; L lumbar; S sacral. The lateral corticospinal and system neurons 12 2 The Ten Neuroanatomic Diagnoses

Lessons : 6. Dysdiadochokinesis (impaired rapid alter- 1 . The inadvertent discovery. A thorough review nating movements) is a common sign of of systems elicits a history of visual impair- impairment of any of the three motor sys- ment with exercise. Awaiting the patient’s tems, corticospinal tract, extrapyramidal sys- spontaneous complaint is unsatisfactory and tem, and cerebellar system. In this case it is never an excuse for being unaware of a due to involvement of the corticospinal tract diagnostic symptom. The symptoms and in the spinal cord. neurologic fi ndings are suffi cient to make a 7. Vibration sense loss out of proportion to pro- diagnosis without an MRI scan as the patient prioception (position sense) is a typical pat- has two lesions separated in space and time, tern with any lesion located at or below the confi rmed by bedside examination. This is thalamus. Conversely, position sense loss the hallmark of multiple sclerosis. with preserved vibration sense is common with cerebral hemisphere lesions. Temporary impairment of neurologic 8. Asymmetric Hoffmann signs are abnormal function with fever or exercise is Uhthoff’s whereas bilateral symmetrical Hoffmann sign, common with patients who have signs can be normal. A clearly unilateral multiple sclerosis. Hoffmann sign indicates pathology affecting the corticospinal tract whether cerebral, 2. The patient has Uhthoff’s sign, a temporary brainstem, or spinal cord. impairment of neurologic function associ- 9. Babinski signs, unilateral or bilateral, are ated with exercise or fever and common with always abnormal and only indicate central multiple sclerosis. nervous system disease involving the corti- cospinal tract. Optic nerve disease often causes defi cits in color perception (color desaturation); When spinal cord disease is suspected yet visual acuity may remain 20/20. there must be a search for a sensory level. 3. Optic neuropathy is present because of 10. A discrete sensory level is usually diagnostic impaired color vision even though visual of myelopathy and may be one to several lev- acuity is normal. Depending on visual els below the level of the lesion. When spinal acuity to diagnose optic nerve disease is not cord disease is suspected there must be a acceptable. careful search for this level. 4. Loss of temperature sensation on one side of

the body may be the initial symptom of a Brown-Séquard’s syndrome occurs with myelopathy. Patients are often aware of this spinal cord lesions. There is ipsilateral problem prior to loss or impairment of pin- impairment of motor function, vibration/ prick sensibility. Both pain and temperature position sense, increased refl exes with are mediated through the lateral spinotha- Babinski sign, and contralateral loss of lamic tract. pain and temperature. 5. Unilateral distal weakness usually occurs with corticospinal tract involvement in any 11. Brown-Séquard’s syndrome is manifested of its three locations, cerebral, brainstem, by ipsilateral abnormalities of motor func- and spinal cord. tion, refl exes, vibration/position sense, and

contralateral impairment of pain/temperature. Dysdiadochokinesis (impaired rapid There is contralateral impairment of pain and alternating movements) occurs with temperature sensation because of the already lesions affecting any of the three motor decussated lateral spinothalamic fi bers. systems, corticospinal, extrapyramidal, 12. Lastly, multiple sclerosis involves only the and cerebellar. central nervous system. Initial involvement Spinal Cord Lesions (Myelopathies) 13

with most patients affects the optic nerve, Past medical history is remarkable for hyperten- brainstem, or spinal cord. sion. A recent medical examination shows a Motor dysfunction exhibits a predilection for normal size prostate. The Emergency Room phy- involvement of the legs with monoparesis or sician examines the patient in the supine position. paraparesis of spinal cord origin. This may occur Examination reveals normal eye movements, with either cervical or thoracic lesions. Triparesis strength, and sensation. Refl exes are 3+ and sym- or quadriparesis obviously occurs only with metrical and no pathologic refl exes are found. cervical lesions. Bilateral arm weakness alone, A neurology consultant queries the patient in “man-in-the-barrel” syndrome, is a rare occur- some depth about his dizziness. The patient rence which may be a manifestation of high denies vertigo and lightheadedness, but he feels cervical cord lesions. It should be emphasized unsteady and his knees are “stiff.” that weakness is usually most prominent in distal musculature and increased tone of spastic type is The neurologic examination is performed best a common associated feature. Acute spinal cord with the patient sitting. lesions may produce greater proximal weakness.

Neurologic examination : High cervical spinal cord lesions, C1ÐC4, Neurologic examination with the patient sitting often cause symptoms in both arms. reveals normal strength but prominent spasticity. Refl exes are 3+, symmetrical and bilateral sus- The typical sensory symptom is a sensory tained ankle clonus is obtained. The patient is level to pain and temperature due to involvement encouraged to walk and, after briefl y refusing, of the lateral spinothalamic tract in either cervi- walks stiffl y with circumduction of his legs. The cal or thoracic cord. Although the prototypical neurologist suspects a myelopathy. An MRI scan of sign is a sensory level one or two segments below cervical and thoracic spine reveals severe cervical the level of the lesion, a thoracic sensory level spinal stenosis with cord compression at C4ÐC5. may occur with cervical lesions. Involvement of the dorsal column, fasciculus gracilis, and cune- Diagnosis : atus causes ipsilateral loss of vibration and occa- Cervical myelopathy secondary to cervical spinal sionally position sense. Nearly always, vibration stenosis. sense to a 128 cps tuning fork is lost fi rst, similar The patient undergoes a cervical laminectomy to neuropathies. Consequently, depending on and both “dizziness and arthritis” resolve over proprioception, to evaluate this system is not 3 months. acceptable. Sensory loss may begin in the legs Lessons : and gradually rises as the lesion progresses. High cervical cord lesions, C1ÐC4, may result in Dizziness has numerous meanings including motor and sensory symptoms involving both poor balance, vertigo, blurred vision, light- arms. The sensory loss may uniquely include so- headedness, anxiety, and therefore multiple called “cortical” sensory signs such as astereog- etiologies. nosis, poor two-point discrimination, and agraphesthesia. Position sense loss may exceed 1. Dizziness is a term requiring detailed interro- vibratory impairment. The essential principal is gation of the patient. This patient describes to think cervical myelopathy or radiculopathy dizziness as a gait disorder. fi rst and neuropathy second with bilateral arm 2. The neurologic examination is best performed symptomatology. in the sitting position unless the patient is Case 9 A 76-year-old man is admitted to the unable to support himself. hospital because of diffi culty walking. He com- 3. Arthritis is a commonly used term which can- plains of dizziness and arthritis of the knees. He not be blindly accepted as accurate without adds a history of urinary urgency and frequency. thorough investigation. 14 2 The Ten Neuroanatomic Diagnoses

4. Sustained clonus is pathologic and indicates roots are secondarily affected. When nerve root involvement of the corticospinal tracts. pain extends from the neck to below the elbow or Symmetrical unsustained clonus is normal. from the back to below the knee, nerve root origin 5. Spasticity is detected by a free interval when is likely. When pain radiates from the neck down passively moving a limb followed by increased to the shoulder into the upper arm, or low back resistance to further attempted movement, then pain extends to hip or upper thigh, then pain of a quick giving way (clasp-knife reaction). muscular origin must also be considered. Pain 6. Pure bilateral corticospinal tract involvement aggravated solely with arm and leg movement at a of the legs points to a myelopathy which may specifi c joint suggests muscle or joint pathology. be cervical or thoracic. Radicular pain may be provoked by prolonged 7. An MRI of the cervical and thoracic spine (1Ð3 min) head extension which narrows the inter- covers the entire spinal cord. The lumbar vertebral foramina. Head turning is less likely to region need not be imaged unless there are provoke radicular pain. Head fl exion maintained signs of lumbar root disease (radiculopathy). for a few minutes gradually relieves the pain. This This will save the patient’s insurer a few thou- is a practical and simple diagnostic bedside sand dollars. technique. 8. Cervical myelopathy secondary to spinal stenosis frequently presents with only involve- The primary symptom of radiculopathy is ment of the motor system. pain which radiates below the elbow or knee. Pain which radiates only to the upper arm or Autonomic dysfunction is common with spi- thigh is less certainly radicular. nal cord disease. The abnormalities may involve the pupil, blood pressure, bowel, blad- der, and sexual function. Focal weakness, , and atrophy may occur. In fact, fasciculations are most often Autonomic nervous system involvement is observed with cervical and lumbar root pathol- common, particularly urinary tract symptoms. ogy since diseases which affect the anterior horn Urinary urgency due to a small spastic bladder is cell, such as amyotrophic lateral sclerosis (ALS), the fi rst development. Urinary retention and are rare. Refl exes affected by the involved nerve overfl ow incontinence is a late development. root may be decreased. Quite often deep tendon Constipation is the almost invariable manifesta- refl exes are not affected. tion of bowel dysfunction and is frequently overlooked in view of its commonplace occur- Fasciculations occur with root and anterior rence. The sympathetic nervous system is repre- horn cell disease primarily. sented in the intermediolateral cell column of the spinal cord. Hence orthostatic hypotension and Well-defi ned sensory loss in the involved der- Horner’s syndrome may be important elements matome is infrequent. Paresthesias are common. of spinal cord disease. Cervical lesions between Vibration and proprioception loss is extremely C1 and T2 can cause Horner’s syndrome, an rare since many nerve roots would have to be excellent localizing sign. The sympathetic fi bers affected to cause this type of sensory defi cit. exit at C8, T1ÐT2 levels. Sexual dysfunction, mainly impotence, may occur. Case 10 A 57-year-old man complains of the acute onset of severe right hip and groin pain followed by right leg weakness. He is able to Radiculopathy walk only with assistance. He has had chronic low back pain for 2 years, obtaining relief with Radiating pain is the cardinal manifestation of periodic use of ibuprofen. The patient’s past radiculopathy whereas spinal cord pathology is medical history is negative and he takes no much less likely to cause pain unless sensory nerve medication. Radiculopathy 15

Fig. 2.2 Brachial plexus

Neurologic examination : Lessons : Strength of iliopsoas and quadriceps is 4/5, ante- 1. Groin pain is unusual in radiculopathies and rior tibialis 4+/5, and there is no knee refl ex on immediately raises a warning fl ag about the the right side. The patient has sensory loss to pin- signifi cance of the MRI fi ndings. prick along the right medial calf. 2. The herniated disk is at a level below the loca- tion of the abnormal signs. MRI (Lumbar) : 3. Iliopsoas weakness implies involvement of There is a large herniated disk at L5ÐS1 which is L1ÐL3 nerve roots. The absent knee refl ex is centrally located. A 2-h glucose tolerance test consistent with L2ÐL4 pathology. reveals a fasting glucose of 105 mg/dl and a 2-h 4. Perhaps the most helpful fi nding is numbness glucose of 220 mg/dl. on the medial calf indicating involvement of Diagnosis : the saphenous nerve. This is a sensory branch Lumbosacral plexopathy due to diabetes. This of the femoral nerve and is seldom involved in syndrome has been called diabetic amyotrophy. lumbar radiculopathies. Iliopsoas musculature is innervated by L1ÐL3 5. Anterior tibialis weakness indicates involve- roots. The knee jerk is L2ÐL4. The medial calf ment of the peroneal nerve, a branch of the sensory loss is due to involvement of the saphen- sciatic nerve, and thus rules out a purely femo- ous branch of the femoral nerve. All of these ral neuropathy. fi ndings indicate involvement of the femoral 6. Diabetic lumbosacral plexopathies are not nerve and the peroneal nerve (anterior tibialis uncommon, may occur as the fi rst sign of dia- weakness), a branch of the sciatic nerve. Plexus betes and frequently cause excruciating pain lesions affect multiple nerves. The patient which is often diffi cult to control. It carries a achieves good control of his diabetes and is good prognosis for recovery over 6 months. treated with analgesics and physical therapy. He See Figs. 2.2 and 2.3 for a review of the cer- recovers in 6 months. vical and lumbosacral plexus anatomy. 16 2 The Ten Neuroanatomic Diagnoses

Lumbosacral plexus

L1

Iliohypogastric nerve Ilioinguinal nerve L2 Genitofemoral nerve

Lateral cutaneous L3 nerve of thigh

L4

Nerves to iliacus and psoas muscles L5

Femoral nerve Obturator nerve Lumbosacral trunk S1 Superior gluteal nerve Inferior gluteal nerve S2

S3 Sciatic nerve Posterior cutaneous S4 nerve of thigh Pudendal nerve

Fig. 2.3 Lumbosacral plexus

The lumbosacral plexus includes T12ÐS4 Plexopathy roots and is located within the psoas major mus- cle. As with brachial plexopathy more than one The brachial plexus incorporates C4ÐT1 roots nerve is affected. Pain, weakness in more than and extends from the spinal column to the axilla. one nerve distribution, and decreased refl exes Lesions which affect the brachial plexus must, by are the most common fi ndings. Sensory loss may defi nition, affect more than one peripheral nerve. be present in more than one nerve originating Shoulder and arm pain may occur and this can be from the lumbar plexus, but this is usually a aggravated by arm movement. Refl exes are minor presenting sign. Diabetes is a common decreased. There may be patchy sensory loss etiology. especially to pin and touch. Trauma of various origins is the most common etiology and malig- Diabetes is a common cause of lumbosacral nant neoplasm such as carcinoma of the lung is plexopathy which may precede its diagnosis. next. Viral etiology is the least common.

Case 11 A 60-year-old man complains of pro- The brachial plexus incorporates C4ÐT1 roots gressive diffi culty walking over a period of and the lumbosacral plexus T12ÐS4 roots. 2 years. He states that this is a problem of Neuropathy 17 arthritis manifested by aching in the thighs and conduction velocities. A lumbar puncture is calves. In addition, he reports that he has poor remarkable for a protein of 85 mg/dl but is other- circulation in his legs and this was discovered wise normal. by his previous doctor who has just retired. The Treatment : patient has type II diabetes and coronary artery Intravenous immunoglobulin restores the patient to disease. near normal over 8 months. His new family physician does a “focused” examination and fi nds no dorsalis pedis pulses. Lessons : He notes pain with hip rotation. Hip X-rays disclose in both hips. He then refers Numbness and paresthesias are neurologic the patient to a vascular surgeon and an orthope- not vascular symptoms. dic surgeon. Studies disclose evidence for small vessel disease and minor osteoarthritis in both 1. Never accept the patient’s diagnosis. hips. Physical therapy is prescribed. 2. Poor circulation does not cause numbness and The patient returns to his family physician tingling. These symptoms are typical of involve- 1 month later. He walks in with some diffi culty ment of the sensory portion of either the central because of bilateral footdrop. A neurology con- or peripheral nervous systems. The localization sultation is requested. depends on the pattern of loss. In this case the One additional month later the patient is distal portions of all extremities are involved, brought in to the neurologist’s offi ce in a wheel- most compatible with neuropathy. chair. The patient reports rapid worsening of both 3. Weakness does not occur with arthritis unless circulation and arthritis. Moreover, he is now there is an associated myositis. This patient’s bothered by poor circulation in his hands. weakness is prominent in distal musculature, The neurologist takes a full history. He ques- common with involvement of any portion of tions the patient about his diagnosis of poor cir- the central and peripheral nervous systems culation. Initially, the patient states that his except focal radiculopathy and plexopathy. previous doctor diagnosed it but eventually The pattern which, in this case, involves all divulges his symptoms which are numbness and extremities suggests neuropathy. Absent tingling. He adds that his arthritis is causing mus- refl exes clinch the diagnosis. cle aches and weakness. The patient’s examina- 4. An incomplete initial history by the family tion is remarkable for absent refl exes in arms physician triggers referral to the wrong spe- and legs. There is 3/5 strength of anterior tibialis, cialists which results in enormous extra costs gastrocnemius, interossei, and wrist extensors. and increased patient disability. Strength of the iliopsoas and quadriceps is 4/5. The patient has no vibratory perception at toes, ankles, and knees. He makes position sense errors at the toes. There is distal sensory loss to pain and tem- Neuropathy perature in all extremities. The most common neuropathy is bilateral, begins

in the feet, and is usually symmetrical or nearly CIDP causes diffuse weakness, distal greater so. Tingling, burning, wet or cold sensations or than proximal. a feeling of walking on gravel are just some of the descriptive terms used by patients. Involve- Diagnosis : ment of the hands may occur soon afterward or Chronic infl ammatory demyelinating polyneu- be delayed by years depending on the etiology of ropathy (CIDP). the neuropathy. The sensory signs depend on Laboratory studies : whether it is mainly a small fi ber neuropathy An electromyogram (EMG) and nerve conduc- (pain and temperature) or large fi ber (vibration, tion study reveals marked slowing of nerve position, and touch). 18 2 The Ten Neuroanatomic Diagnoses

Focal neuropathies occur due to trauma and Distal weakness, sensory loss, and hypore- compressive lesions. The median nerve is easily fl exia support the diagnosis of neuropathy. compressed at the carpal tunnel. The ulnar nerve is frequently subjected to trauma in the Strength is usually lost fi rst in distal muscula- cubital tunnel at the elbow. The femoral nerve ture. Combined sensorimotor neuropathies are can be affected due to a stretch injury during common but the sensory symptoms and signs childbirth. The lateral femoral cutaneous nerve are usually more apparent. Refl exes are decreased of the thigh can be compressed under the or absent. Rarely, there are pure motor neuropa- inguinal ligament. This diagnosis is meralgia thies such as multifocal motor neuropathy and paresthetica, a very common disorder often some acute infl ammatory demyelinating poly- mistaken for radiculopathy and has subjected neuropathies (AIDP) known as Guillain-Barré patients to needless laminectomies. The per- syndrome, an autoimmune disorder. Although oneal nerve can be compressed in the popliteal Guillain-Barré syndrome can be mostly a motor fossa which causes a dropped foot and “step- neuropathy which produces an ascending paraly- page” gait. sis, sensory symptoms are common. Autonomic dysfunction, such as tachycardia, hypertension, Case 12 A 68-year-old man complains of pro- and urinary retention, is not infrequent. CIDP is a gressive leg weakness, painful and stiff muscles sensorimotor neuropathy which may begin insid- over the previous 3 months. He has diffi culty iously and progress steadily over months to years. climbing stairs. For the last year he has been It manifests itself with neurologic signs often impotent, complains of a dry mouth, and has con- quite similar to AIDP. stipation. Past medical history is unremarkable. Neurologic examination : The Romberg test is positive most often with The patient has mild bilateral ptosis. Strength of neuropathy, less often with vestibular dys- iliopsoas is 4/5, deltoid 4+/5, and infraspinatus function, infrequent with most spinal cord 4+/5, bilaterally. Refl exes in the legs are absent. lesions, and rare with cerebellar pathology. Diagnosis : LambertÐEaton syndrome, a disorder of the neu- When neuropathies signifi cantly involve large romuscular junction. fi bers, vibration and position sense loss with an ataxic gait, aggravated in the dark, is often the Lessons : result. Eye closure in the shower, typically when 1. Unfortunately, not every patient presents a shampooing, may precipitate falls. A positive history diagnostic of a specifi c anatomic site Romberg and poor tandem gait are frequent of pathology. abnormalities. Probably the most common cause 2. Proximal weakness usually indicates a myo- of a positive Romberg is a neuropathy followed pathy or a lesion of the neuromuscular by acute vestibular disease and, third, a junction. myelopathy. Cerebellar pathology may cause a 3. Ptosis suggests myasthenia gravis. positive Romberg probably due to lesions involving 4. Absent refl exes are most compatible with a vestibular-cerebellar pathways. That cerebellar neuropathy, not myasthenia gravis. disease can cause a positive Romberg is 5. Autonomic nervous system dysfunction is controversial. present since the patient has impotence, constipation, and a dry mouth. This is not expected in either myopathy or myasthenia Meralgia paresthetica is a compressive neu- gravis. ropathy which causes variable degrees of pain, 6. This mixed picture occurs with LambertÐ paresthesias, and numbness on the lateral Eaton syndrome, a neuromuscular junction thigh. disorder. It is caused by an insuffi cient release Neuromuscular Junction 19

of acetylcholine at the presynaptic terminal, Ptosis, extraocular muscle weakness, dys- and 50% of patients have an underlying phagia, dysarthria, fatigue, and facial and malignancy. proximal extremity weakness are common in patients with myasthenia gravis. Neuromuscular Junction LambertÐEaton syndrome is a rare disorder Discussion of the neuromuscular junction must due to inadequate release of acetylcholine from deal with a differential diagnosis since there are the presynaptic terminal. It is more common in just three main disorders to consider. men. Typical symptoms as described in the case report include proximal weakness, paresthesias, and autonomic dysfunction such as impotence Neuromuscular junction disorders such as and hypohidrosis. About 50% of patients have myasthenia gravis and LambertÐEaton syn- small cell carcinoma of the lung. There is pro- drome usually cause proximal more than duction of antibodies directed against voltage- distal weakness. gated calcium channels in the peripheral nerve terminals. This results in inadequate release of First and foremost is myasthenia gravis. There acetylcholine into the synaptic cleft from the are numerous presentations, some of which presynaptic terminal. Hence, repetitive nerve include droop due to weak levator palpe- stimulation results in an incremental response brae superioris muscles and extraocular muscle with the amplitude usually increasing by 2Ð20 palsies causing fl uctuating diplopia even, at times. times, during the same examination. Pseudointernuclear ophthalmoplegia and gaze paretic nystagmus or nystagmoid jerks are not LambertÐEaton syndrome is associated with uncommon. Pseudointernuclear ophthalmoplegia proximal weakness, autonomic dysfunction, refers to isolated medial rectus weakness caused hyporefl exia, and rarely ptosis. by pure muscular weakness due to myasthenia gravis. An internuclear ophthalmoplegia is caused The third important disorder of the neuromus- by a lesion of the MLF in the pons or midbrain cular junction is the toxic effects of botulism and ipsilateral to the medial rectus weakness. aminoglycoside antibiotics. Botulism prevents Weakness of neck musculature may cause a release of acetylcholine from the presynaptic persistent head fl exion posture associated with terminal. Antibiotic-induced neuromuscular weakness of neck extensor muscles. More often disease is related to excessively high drug levels there is weakness only of head fl exion. Acute which most often occur with renal failure. This respiratory failure with a prior history of neglected also results in a decreased release of acetylcho- neurologic symptoms is a rare presentation. line from the presynaptic terminal. Postjunctional Dysarthria, dysphagia, and facial and extremity membrane sensitivity to acetylcholine is also weakness are additional common symptoms. All reduced. of the symptomatology is likely to worsen later in the day or when the patient is fatigued. Perhaps Myopathies usually cause proximal weakness a key to the diagnosis is the invariable presence but there are important exceptions such as of normal mental status, refl ex, and sensory inclusion body myositis and myotonic examinations. The etiology is an autoimmune dystrophy. disorder due to the production of antiacetylcho- line receptor antibodies which destroy the acetyl- Case 13 A 51-year-old woman complains of choline receptors located on the postsynaptic neck, hip, and knee pain. Over 3 months she terminal. The antibodies arise from a hyperplas- reports that the pain prevents her from getting out tic thymus or thymoma. of a chair and even off the toilet. Additionally, 20 2 The Ten Neuroanatomic Diagnoses she has numbness and tingling of both feet. She has had rheumatoid arthritis for 12 years and has Myopathy numerous joint deformities affecting the wrists, The key to the diagnosis of myopathy is usually proximal interphalangeal joints, and knees. She the absence of sensory complaints. Myopathies has been on long-term treatment with low-dose are ordinarily slowly progressive disorders. The prednisone and methotrexate. history should focus on how the weakness affects Examination : the patient. Is there diffi culty raising the arms to Neurologic examination reveals several abnor- comb their hair? Is climbing steps the main prob- malities which include absent refl exes in the legs lem or is it opening jars? Are eye movements, and loss of vibration perception at the toes. chewing, and swallowing affected? Strength of head fl exion is 4+/5, iliopsoas 4/5, The examination most often reveals proximal quadriceps 4+/5, and anterior tibialis 4/5. muscle weakness. However, distal weakness is What is causing her disability? not rare and occurs in some myopathies such as Neurologic diagnoses : myotonic dystrophy and inclusion body myositis. 1. Steroid myopathy. The major differential diagnosis to consider on 2. Neuropathy associated with rheumatoid the basis of anatomy is involvement of the ante- arthritis. rior horn cell such as in ALS or one of its variants, spinal muscular atrophy. Fasciculations, often Laboratory data : associated with atrophy, are hallmarks of anterior EMG confi rms both diagnoses as there are poly- horn cell disease and radiculopathy but not pres- phasic low amplitude muscle potentials in proxi- ent in myopathy. A disorder of the neuromuscular mal leg musculature which are typical for junction, particularly myasthenia gravis, must be myopathy. There are also mildly slow nerve con- excluded although progressive weakness which is duction velocities with occasional fi brillations in not variable during the day would be quite uncom- the anterior tibialis muscle which are characteris- mon. Sensory symptoms, such as numbness and tic for neuropathy. tingling, do not occur with myasthenia gravis. Lessons : Wasting and decreased to absent refl exes can 1. Proximal weakness is common in myopathy be present but are usually late in the course of a and infrequent in neuropathy. myopathy. Painful are not unusual. Autonomic dysfunction such as a functional blad- der disorder does not occur. Weakness of head fl exion and less often head extension are often affected in myopathy. Low back pain without radiation is more often 2. Testing head fl exion and extension is essential of muscular origin rather than a herniated disk when myopathy is suspected as these muscles or spinal stenosis. Unusual causes include sub- are often involved. Myasthenia gravis, acute arachnoid hemorrhage, rigidity of Parkinson’s infl ammatory demyelinating polyneuropathy, disease, and metastatic bone disease. and chronic infl ammatory demyelinating polyneuropathy also commonly affect these Case 14 An 89-year-old man complains of neck muscles. and severe low back pain for the last 2 weeks. After 3. The patient’s disability relates to strength but a 10-min evaluation, a harried primary care physi- not sensory symptoms. cian refers him to an orthopedic surgeon. An MRI 4. The patient has a neuropathy because of absent scan reveals marked spinal stenosis as well as refl exes, distal sensory loss to vibration per- severe stenosis of neural foramina at levels L3ÐL4 ception, and weak distal musculature. and L4ÐL5. He is referred for physical therapy. 5. In summary, this patient has two diseases, cer- The pain resolves gradually over 3Ð4 days but 1 tainly not unusual in this clinical setting. week later suddenly increases and is unbearable. Meningeal Disease 21

A neurology consultation is requested before patients, however, who have subtle presenting considering a lumbar laminectomy. The neurolo- manifestations. Of necessity, this section will gist takes a full history. The low back pain is non- focus on differential diagnoses, cerebrospinal radiating. The neck pain is nearly as severe. Just fl uid studies, and practical management issues prior to the onset of his symptoms the patient but not on treatment since, as previously noted, now recollects that he had brief double vision radical changes will likely occur in the future. lasting about 2 h. He had forgotten about it. Examination : Acute meningeal disease is due to either infec- Neurologic examination reveals asymmetric tion or hemorrhage. Leptomeningeal carcino- pupils, OS 4 mm with a 1+/4 reaction to light, matosis usually develops gradually. and OD 2.5 mm with a 4+/4 reaction to light. The patient’s neck is mildly stiff on fl exion but not Five percent of patients with subarachnoid with head turning. hemorrhage have normal CAT scans. A CT scan (brain) shows blood in the suba- Therefore, a lumbar puncture is required to rachnoid spaces. Angiography demonstrates a exclude hemorrhage. left posterior communicating aneurysm. Lessons : Case 15 A 38-year-old woman complains of peri- 1. It is essential to take a complete neurologic odic impairment of speech for 1 week. An outpa- history even when the patient has low back tient evaluation included a carotid Doppler study pain. Questions regarding memory, speech, which revealed a high-grade left internal carotid vision, etc., may yield diagnostic information artery stenosis. CAT scan (head) was normal. as it did in this case. A neurology consultant elicits additional his- 2. Low back pain which does not radiate is not a tory after the review of all neurologic symptoms. common presenting symptom for spinal steno- The patient reports a mild (2/10) intermittent sis or a herniated disk. Radicular pain is ordi- occipital headache for 4 weeks, easily relieved with narily present with either condition. acetaminophen. She did not mention it to her pri- 3. Never accept as fact another doctor’s diagno- mary care physician because she felt it was insig- sis without personal investigation. A physi- nifi cant. Her physician did not question her about cian may have a stellar reputation but who has headache. The headache had increased in severity not made mistakes? over a few days just prior to this evaluation. 4. Neck and low back pain are symptoms which Neurologic examination : are not unusual in patients who have a suba- Neurologic examination is signifi cant for bilat- rachnoid hemorrhage. eral high cervical carotid bruits. She makes an 5. The large pupil with a slow reaction to light occasional verbal paraphasia such as calling her supports compression of the left third nerve by elbow a bone and her wrist a knee. There is mild an aneurysm, typically internal carotid or pos- end nuchal rigidity, i.e., her neck is initially sup- terior communicating. The pupillary fi bers are ple but toward the end of passive fl exion there is superfi cial and most often the fi rst sign with increased resistance. As noted above a CAT scan aneurysms in this location since they are easily of the brain was normal. compressed.

Nuchal rigidity can be a subtle fi nding, present Meningeal Disease only with full head fl exion.

The diagnosis of meningeal irritation may The initial impression of the neurologist is appear simple since most patients have fl agrant suspected subarachnoid hemorrhage possibly due neurologic symptoms and signs. There are many to a left middle cerebral artery aneurysm. 22 2 The Ten Neuroanatomic Diagnoses

The studies performed included a lumbar rhage has occurred. Xanthochromia persists puncture which revealed bloody fl uid. When cen- for 2Ð4 weeks. Increased bilirubin due to liver trifuged, the supernatant was xanthochromic disease produces a yellowish color but a serum (yellowish). This secured the diagnosis of sub- bilirubin of 10 mg/dl or more is required before arachnoid hemorrhage. The protein was 102 mg/dl, the spinal fl uid changes color. Markedly glucose 70 mg/dl, red blood cells 17,000/cu mm, elevated spinal fl uid protein greater than and wbc 1,320/cu mm with 90% neutrophils. 150 mg/dl may also produce a faint degree of Angiography demonstrated band-like constric- xanthochromia. In the fi rst few hours after tions of both internal carotid arteries at the C2, C3 subarachnoid hemorrhage the spinal fl uid often level associated with bilateral moderate stenoses. appears pink which is due to oxyhemoglobin. In addition, there was a left middle cerebral artery A second method of ruling out a traumatic tap aneurysm. An MRI of the brain revealed a small is comparing the red blood cells to the white left temporal lobe infarction. blood cells. The ratio in blood is 700:1, respec- tively. If this ratio is decreased, meaning an Lessons : increased wbc count due to infl ammation, a diag- 1. Subarachnoid hemorrhage is not necessarily nosis of subarachnoid hemorrhage is supported. an acute catastrophic event. 2. Angiography disclosed fi bromuscular dysplasia Case 16 A 72-year-old man enters the emergency which is associated with cerebral aneurysms. room because of a 12-h history of headache and 3. This patient’s aphasia and temporal lobe horizontal double vision. This began the previous infarction may have been due to vasospasm night. At that time he had a severe occipital head- associated with subarachnoid hemorrhage, ache and double vision when looking to the left. embolism from a thrombus in the aneurysm to In addition, he reports a chronic cough with pro- a distal middle cerebral artery branch, or duction of the yellowish sputum over the last few embolism from fi bromuscular dysplasia which weeks. His past medical history is remarkable for has caused a left internal carotid artery steno- hypertension and heavy smoking. sis. Vasospasm is the most likely etiology Examination : since embolism from a thrombus in an aneu- Examination reveals a blood pressure of 230/120 rysm is not as common and stroke caused by and temperature of 102¡F. He has rales at the left fi bromuscular dysplasia is rare. base. There is mild nuchal rigidity. Funduscopic 4. Normal CAT scans of the head occur in 5% of examination shows blurred optic disks bilater- patients with subarachnoid hemorrhage and ally, absent venous pulsations, and one fl ame- one cannot depend on it to make a diagnosis. shaped hemorrhage at the right optic disk margin. A lumbar puncture is essential. Vision is normal. The patient has a left lateral rectus paresis due to left sixth nerve involvement. Cerebrospinal fl uid xanthochromia (yel- A CAT scan of the brain is ordered stat, but the low) is detected about 10 h after hemor- unit has just broken down. The medical resident’s rhage. This is found in the supernatant after thoughts are as follows: The funduscopic picture centrifuging bloody CSF. It persists for suggests . Furthermore, the sixth 2Ð4 weeks. nerve palsy is an early sign of increased intracra- nial pressure. If that is the case, is there a risk of 5. A traumatic tap is virtually excluded because uncal herniation by doing a lumbar puncture? of the presence of xanthochromia present in What do these neurologic signs mean and the supernatant after centrifuging of spinal what is the best management: fl uid. Xanthochromia is due to the presence of 1. Nuchal rigidity and fever indicate meningeal bilirubin which causes a yellowish color and is disease, in this instance, hemorrhage or present approximately 10 h after the hemor- infection. Synopsis 23

2. A high fever, over 101¡F, usually indicates (a) Cerebral dysfunction, right, left, bilateral, infection. or encephalopathy 3. A is an early sign of increased (b) Extrapyramidal intracranial pressure. (c) Brainstem/cerebellum (d) Spinal cord (myelopathy) Ophthalmoscopic fi ndings in papilledema (e) Root (radiculopathy) include: blurred disk margins, especially (f) Plexus (plexopathy) temporal side, absent venous pulsations, (g) Nerve (neuropathy) and hemorrhages at the disk margins. (h) Neuromuscular junction (i) Muscle (myopathy) 4. Papilledema is present. The three major fi nd- (j) Meninges (meningitis) ings of papilledema include: 3. Characteristic and diagnostic features of: (a) Blurred optic disk margins. Initially the (a) Cerebral disease nasal margin is blurred followed later by i. Aphasia the temporal margin. Nasal blurring is ii. Dementia also a common normal variation and, iii. Seizures therefore, the temporal margin is the focus iv. Homonymous hemianopsia of attention. v. Abnormalities of mood, personality (b) Venous pulsations are absent in papille- and behavior associated with abnor- dema, but approximately 5-10% of nor- mal neurologic signs mal people have no visible venous (b) Extrapyramidal disease pulsations. i. Abnormal involuntary movements/ (c) Hemorrhages at the disk margins are vir- postures tually diagnostic of papilledema. The ii. Hypokinetic or hyperkinetic hemorrhages are usually splinter or fl ame- iii. Gait disorder can be the sole shaped. manifestation 5. The risk of uncal or tonsillar herniation, espe- (c) Brainstem/cerebellar cially in the absence of a cerebral, brainstem, i. Crossed fi ndings, e.g., right facial or cerebellar lesion on a clinical basis, is weakness and left hemiparesis negligible. ii. Abnormal eye movements and/or A lumbar puncture is performed. There are pupils indicating this localization 800 wbc/cu mm, 95% of which are neutrophils. iii. Pure cerebellar system signs which The glucose is 30 mg/dl with a simultaneous occur with brainstem and/or cerebel- blood sugar of 110 mg/dl, signifi cantly less than lar pathology the two-thirds of the blood sugar which would be (d) Myelopathy expected in normals. The protein is 120 mg/dl, i. Sensory level markedly elevated. The Gram stain shows Gram- ii. Monoparesis, paraparesis, triparesis, positive organisms. Appropriate treatment is crossed paresis (e.g., right arm and left initiated for bacterial meningitis. leg), quadriparesis, and hemiparesis iii. Associated signs include spasticity, asymmetric hyperrefl exia, Babinski signs, and asymmetric Hoffmann signs Synopsis (e) Radiculopathy i. Root pain and/or paresthesias typi- 1. Technique of making a neurologic diagnosis cally affecting one limb (a) Anatomic localization ii. Decreased refl ex and focal weakness (b) Differential diagnosis determined by ana- in root distribution tomic localization iii. Sensory symptoms/abnormalities in 2. Anatomic diagnosis dermatome pattern 24 2 The Ten Neuroanatomic Diagnoses

(f) Plexopathy 2. A 77-year-old man has the sudden onset of i. Disorder affecting multiple roots ordi- slurred speech 24 h after a transurethral resec- narily in only one limb tion of the prostate. He has diabetes and ii. Motor signs predominate hypertension. iii. Pain often severe Neurologic examination reveals a blood (g) Neuropathy pressure of 170/110 and a left carotid bruit. i. Ordinarily begins in both feet unless The patient is oriented to person only. His there is localized entrapment speech is dysarthric. He remembers 1 of 3 ii. Decreased to absent refl exes words after 1 min has elapsed. iii. Early signs may be distal sensory loss (a) Where is the lesion? to temperature and pin (small fi ber) i. Left cerebral hemisphere and/or diminished vibratory perception ii. Right cerebral hemisphere distally (large fi ber). Position sense iii. Brainstem loss is a much later development. iv. Bilateral cerebral hemispheres iv. Light touch loss distally is virtually (b) What test should be ordered urgently? diagnostic i. CAT scan (brain) (h) Neuromuscular junction (myasthenia gravis) ii. Carotid Doppler i. Fluctuating weakness worse later in iii. CBC, electrolytes, blood urea nitro- the day gen, glucose ii. Extraocular muscle weakness, dysar- iv. MRI (brain) and MRA (head and neck) thria, dysphagia, and especially head 3. A 41-year-old man is admitted to the hospital fl exion/extension weakness. Proximal because of acute onset of vertigo and vomit- muscles are preferentially affected. ing. He has diabetes and smokes one pack of (i) Myopathy cigarettes per day. i. Weakness commonly proximal and Neurologic examination reveals a blood often involving head fl exion pressure of 150/100. He has horizontal gaze- ii. Distal weakness may occur, mostly evoked nystagmus. notably in myotonic dystrophy and Where is the lesion? inclusion body myositis (a) Semicircular canals iii. No change in refl exes and no sensory (b) Brainstem loss (c) Eighth nerve (d) Temporal lobe (e) Brainstem or eighth nerve Multiple Choice Questions 4. If the lesion is in the brainstem, what are the two most likely etiologies? 1. A 47-year-old woman complains of a 1-week (a) Vascular history of left leg weakness. The neurologic (b) Infectious examination discloses mild weakness of all (c) Neoplasm muscles of the left leg. The right ankle jerk is (d) Demyelinating 1+ and the left is 2+. (e) Metabolic Which one or more of these locations could 5. An 80-year-old man complains of progressive be the site of the lesion? weakness and clumsiness of the left arm of 2 (a) Cerebral months duration. (b) Cerebellum Examination discloses mild weakness of (c) Spinal cord interossei and wrist extension, impaired rapid (d) Root alternating movements of the left arm, 3+ (e) Nerve refl exes in the left arm and 2+ in the right arm. (f) Neuromuscular junction Where is the most likely site of the lesion? Answers 25

(a) Cerebral 10. What is the most important factor for an (b) Brainstem accurate neurologic diagnosis? (c) Cerebellum (a) Past medical history (d) Spinal cord (b) Family history (e) Nerve (c) Present illness and neurologic examination 6. A 55-year-old woman complains of severe (d) Neuroimaging right anterior thigh pain which is aggravated (e) Blood test results by prolonged sitting and relieved by walking. This began insidiously about 8 months ago and is progressing in severity. She has devel- Answers oped weakness over the last 2 weeks and fi nds it diffi cult to climb stairs. 1. (a) and (c). Neurologic examination: Strength of iliop- Leg weakness will occur with all of the diag- soas and quadriceps is 4+/5. There are promi- noses except cerebellum. Nerve, root, or nent fasciculations over the anterior thigh. plexus lesions produce a decreased not an The right knee refl ex is 1+ and the left is 3+. increased refl ex. Neuromuscular junction Where is the lesion? disorders do not affect refl exes and usually (a) Cerebral cause proximal weakness. The increased (b) Spinal cord ankle refl ex indicates involvement of the cor- (c) Root ticospinal tract which is cerebral, brainstem, (d) Plexus or spinal cord. (e) Nerve 2. (a) iv. 7. In the case described in no. 6, what etiologies Disorientation and poor short-term recall indi- should be considered and why? cate bilateral cerebral dysfunction (encephal- (a) Neoplasm opathy). Dysarthria is nonlocalizing. Carotid (b) Structural/degenerative bruits are not relevant in this case. (c) Degenerative When there is bilateral cerebral dysfunc- (d) Infl ammation tion, a search for metabolic derangement (e) Infectious comes fi rst. Abnormalities of electrolytes are 8. A 40-year-old man has a 3-day history of not rare after transurethral prostate resection. rapidly progressive slurred speech and diffi - This patient has hyponatremia. culty swallowing liquids. He has insulin- (b) iii. dependent diabetes mellitus. 3. (b). Neurologic examination: The patient has The nystagmus has two directions which dysarthria, bilaterally sluggish palate move- indicate brainstem or cerebellar disease. Eighth ments but a normal gag response, and mild nerve or semicircular canal lesions are unidi- weakness of head fl exion. rectional. Cerebral lesions do not cause nys- Where is the lesion? tagmus except as a manifestation of a seizure. (a) Cerebral 4. (a) and (d). (b) Brainstem Vascular disease is suspected because of dia- (c) Cranial nerve betes and a smoking history. Multiple sclero- (d) Neuromuscular junction sis is possible since he is in the right age 9. What is the etiology of the case described range (20Ð50 years) and the brainstem is in No. 8? often affected. (a) Vascular 5. (a). (b) Autoimmune A brainstem lesion is possible but usually (c) Neoplasm affects other functions, especially eye move- (d) Demyelinating ments and causes crossed fi ndings, especially 26 2 The Ten Neuroanatomic Diagnoses

when progressing in severity. Cerebellar involvement might explain the fi ndings but lesions cause neither weakness nor asymmet- the etiology, Guillain-Barré syndrome, rarely ric refl exes. Spinal cord disease is possible presents in this fashion. This patient has a dis- but usually involves a leg fi rst. Nerve pathol- ease affecting the neuromuscular junction. ogy would decrease the refl exes. Distal weak- 9. (b). ness associated with poor coordination and Myasthenia gravis is the prototypical neuro- hyperrefl exia of an arm is most consistent muscular junction disorder. It is caused by with a cerebral lesion. Impaired rapid alter- antiacetylcholine receptor antibodies which nating movements occur with involvement of attack the acetylcholine receptor site on the any of the three motor systems, corticospinal, postsynaptic membrane. extrapyramidal, and cerebellar. The steadily 10. (c). progressive worsening suggests neoplasm. The past medical history may aid in the dif- 6. (c). ferential diagnosis but many patients with Cerebral disease does not cause fascicula- diabetes, for example, may develop cancer, tions or pain and usually affects distal mus- demyelinating disease, or degenerative dis- culature. Spinal cord pathology does not ease. The family history is much less likely to cause pain sitting and relief standing. Also it be of value for similar reasons. Neuroimaging seldom produces severe pain. Plexus lesions is precise but if the wrong structure is imaged, are painful but the pain persists in any posi- it is of no value. Blood test results are valu- tion. It also affects more than one nerve. able if the ones selected pertain to the differ- Fasciculations and pain are characteristic ential diagnosis. The present illness and features of radiculopathy. neurologic examinations are likely to lead to 7. (a) and (b). an accurate neuroanatomic diagnosis and The focal fi ndings suggest L3ÐL4 radiculop- thus to the appropriate diagnostic tests. athy. This slow progressive course raises the possibility of benign neoplasm such as men- ingioma or schwannoma but not the elimina- Bibliography tion of pain with walking. A herniated disk explains the alteration in pain with changes Blumenfeld H. Neuroanatomy through clinical cases. of position although the usual course is not Sunderland, MA: Sinauer Associates; 2002. one of steady progression. Neuroimaging Brazis PW, Masdeu JC, Biller J. Localization in clinical neurology. 5th ed. Philadelphia, PA: Lippincott will determine the diagnosis. Williams & Wilkins; 2007. 8. (d). Goetz CG. Textbook of clinical neurology. 2nd ed. Cerebral disease seldom affects swallowing. Philadelphia, PA: Saunders; 2003. Brainstem lesions usually cause crossed fi nd- Jankovic J, Tolosa E, editors. Parkinson’s disease and movement disorders. 5th ed. Baltimore, MD: Williams ings, abnormal eye movements and affect & Wilkins; 2007. long tracts such as the corticospinal and lat- Karpati G, Hilton-Jones D, Bushby K, Griggs RC. eral spinothalamic tracts. Bilateral palate Disorders of voluntary muscle. 8th ed. New York, NY: weakness due to a medullary lesion is not in a Cambridge University Press; 2010. Mohr JP, Choi D, Grotta JC, Weir B, Wolf PA, editors. single arterial distribution. Thus a brainstem Stroke. In: Pathophysiology, diagnosis and management. infarction is extremely unlikely. Cranial nerve 4th ed. New York, NY: Churchill Livingstone; 2004. The Neurologic History Holds the Diagnostic Keys 3

Keywords Chief complaint • Hypochondriasis • Present illness • Munchausen • History • Psychogenic symptoms • Interview • Conversion reaction

Obtaining a complete history by employing a by a detailed history of the present illness remains methodical, unhurried approach is the key to an the bedrock necessity of a precise diagnosis. In an accurate diagnosis. The initial focus must be on outpatient setting perhaps 75–90% of the time the anatomic localization which invariably deter- spent with the patient should focus on this history. mines the differential diagnosis. Paresthesias of Physicians are often confronted by patients who the left leg, for example, may indicate a compres- rapidly spew forth symptoms that he or she feels sive peroneal neuropathy, lumbar radiculopathy are valuable. They may censor out critical infor- from a herniated disk, spinal cord disease, or a mation due to fear of time constraint. Scarcity of right cerebral lesion. This type of diagnostic time will therefore undermine the physician’s dilemma is a frequent reason to request a neurol- ability to care for his patient as well as interfere ogist’s opinion. An MRI scan of the lumbar spine with the doctor–patient relationship. Adminis- may be completely normal, dumbfounding the trative mandates of time allotment for an offi ce patient’s physician. Conversely, a herniated disk visit have already reduced the quality of care. Is with nerve root compression may be treated sur- there time for demonstrating empathy which will gically without alleviation of the patient’s symp- allow the patient to more easily divulge the symp- toms which are due to a brain tumor. Furthermore, tomatology that may be the key to the diagnosis it is quite common to fi nd a herniated disk which and effi cacious treatment? is asymptomatic. Needless to say, technology The following comments will digress from the with all of its benefi ts has major limitations. It specifi c fi eld of neurology, but instead review my resolves numerous diffi culties of a differential personal opinions of interviewing a patient. The diagnosis but can easily lead to inappropriate physician should be groomed and dressed in a treatment by an unwary physician. style which is compatible with the geographic Amongst experienced neurologists there is a location of the practice and the type of patient fear, with some justifi cation, that current neurol- population. Signifi cant deviation may engender ogy residents are better trained in interpretation of distrust or focus on the physician to the detriment MRI scans and pharmacological applications than of free communication. obtaining a history suffi cient to make an accurate Entry into the room should not be rushed but anatomic diagnosis. The chief complaint followed deliberate and direct. Use of the patient’s fi rst

J.N. Alpert, The Neurologic Diagnosis: A Practical Bedside Approach, 27 DOI 10.1007/978-1-4419-6724-4_3, © Springer Science+Business Media, LLC 2012 28 3 The Neurologic History Holds the Diagnostic Keys name on fi rst meeting may be appreciated by The history of the present illness remains the some patients but sound discourteous to others. foundation of an accurate neurologic diagnosis. The decision about how to greet the patient in the Emphasis must be placed on the chief complaint. future should be made after the fi rst visit. Here are a few interesting examples followed by Although a woman may not expect a handshake brief explanations (see Table 3.1 ). at fi rst meeting, that tradition is fast becoming obsolete and rightly so. The initial physically Explanations : fi rm grasp with eye contact sets the stage for a 1. Any symptom which is repetitive, stereo- frank discussion of all symptoms. The door must typed and with brief duration of a few sec- then be closed to ensure privacy. onds to a few minutes may potentially be of The physician and the patient should be seated epileptiform origin. This can be a movement, comfortably in a chair which will give the patient sensation, or psychic experience. a sense that the doctor will spend some time 2. Movement or gait disorders require a thor- listening to him before moving on to an examina- ough inquiry about other normal movements tion or, for that matter, out of the room. Today, such as turning in bed, turning around while many patients simply jump on the examining walking, getting out of a chair, writing, and table expecting only a cursory review of symp- using eating utensils. toms and are often astonished at a relaxed inter- 3. A history obtained only from family mem- view. Since the neurologic history is likely to be bers is likely to include their interpretation of lengthy, probably 15–30 min for an initial visit, it the patient’s symptoms which may not be is best for the physician to be seated facing the accurate. The basis for their conclusion patient to enable him to watch for expressions should be clarifi ed. that may tip off the relative importance of some 4. Jaw pain may be due to local pathology in symptoms. Keeping one’s head down or turned to the teeth, gums or temporomandibular joint, a computer screen may be a turnoff for many neuropathic pain such as the knife-like elec- patients. How does one express interest without trical pain of trigeminal neuralgia or isch- eye contact? Despite improved documentation emic pain of muscular origin which occurs and effi ciency the consequence is rising patient with prolonged, repetitive muscular activity. dissatisfaction and often a search for alternative In the absence of trauma ischemic pain must therapies which provide a soothing physical be considered in this case. Infl ammatory dis- touch and ample time for communication. ease of the internal maxillary artery is a rela- Chewing gum is obviously unacceptable as it tively common feature of temporal arteritis. implies boredom with “just another patient.” 5. The key word here is “bilateral,” hence sus- Irritating habits such as tapping one’s fi nger or pected spinal cord disease. The second ele- pen indicates impatience, which is likely to either ment is inability to feel texture yet perceive irritate or intimidate some people. Frequent inter- pain, temperature, and light touch. This is ruptions disrupt the patient’s thinking and pre- termed “astereognosis,” an uncommon but pared organization for the visit. This may provoke well-described consequence of damage to omission of major portions of the history. On the the posterior columns. other hand, gentle prodding back to essentials 6. As an isolated symptom, impairment of tem- may be necessary in many cases. Comments that perature perception is nearly always due to a I have found useful for verbose patients who are myelopathy with localized pathology affect- circumstantial or tangential in their replies are: ing the lateral spinothalamic tract. “Please slow down as you are going a little too 7. The sudden collapse without prodromal fast for me.” and “Could you describe your main symptoms has also been called a “drop problem again so I can understand it more attack” which is associated with several dis- completely?” A few smiles often relaxes and orders. Other than cataplexy, vertebrobasilar encourages the patient to be candid. disease, complete heart block, Ménière’s 3 The Neurologic History Holds the Diagnostic Keys 29

Table 3.1 Analysis of the chief complaint Follow-up questions Interesting examples of chief by an experienced complaints neurologist Answers Diagnosis My teeth periodically chatter For how long do they 1 or 2 min Simple partial seizures of the even if I am not cold chatter? jaw caused by a convexity meningioma I sometimes have to run to Do you have diffi culty Yes Parkinson’s disease with catch up with myself getting out of a chair? festinating gait or propulsion Mother becomes very thirsty How do you know she She licks her lips Complex partial seizures with and does not know it (history is thirsty? frequently automatism, idiopathic from daughter) I cannot chew chicken Why not? My jaws ache Temporal arteritis with jaw claudication My hands feel as though they Do you have any Yes Cervical myelopathy due to a are dipped in wax problem identifying meningioma compressing the coins in your pocket? dorsal portion of the spinal cord at C3–C4 I have to test the bath water Why not your left leg? I have scalded it Thoracic myelopathy due to temperature with my right leg multiple sclerosis When my friend told me she Have you suddenly Yes, when I am told a Cataplexy, a near constant had breast cancer, I collapsed collapsed at other joke fi nding in patients with and fell on the fl oor times? narcolepsy My brain is turning at the When does this occur It always occurs in bed, Benign paroxysmal positional base of my skull and how long does it sometimes when I turn vertigo last? over and it lasts less than 1 min When I laugh, the back of my Do you have any Yes, I have arthritis and Chiari Malformation Type 1 head hurts diffi culty walking? my legs are getting stiff causing spasticity due to spinal cord compression I hear the same phrase spoken Do you have any Yes, that seems to have Glioma, left posterior even after changing the TV trouble understanding been a problem temporal region, associated channel conversation? developing over the last with auditory perseveration few months I found a hand lying on my Was anyone with you No, my husband was out Right cerebral infarction with stomach one night when I at the time? of town at a meeting the syndrome of woke up early autotopagnosia I see Mickey Mouse fl ying How often do you see It has been occurring a Simple partial seizures with into my left eye him and for how long few times a week for the formed visual hallucinations on each occasion? last 2 weeks and I see due to a right cerebral him for about 15 s infarction, parietotemporal

disease with otolithic crisis and basilar intracranial pressure which is transmitted migraine are additional etiologies and require throughout the brain and spinal cord. In this specifi c questions to ascertain the diagnosis. instance the cerebellar tonsils are displaced 8. “Turning” is the key word. Although not downward and are impacted on the foramen invariably the case, it implies vertigo. A brief magnum causing pain. Arthritis is so com- duration of vertigo, less than 1 min, is typical mon that many patients use that term to of benign paroxysmal positional vertigo. describe a variety of symptoms which may Provocation with movement is another fea- include spasticity. ture and will be discussed in Chap. 10 . 10. Complex abnormalities of audition suggest 9. Laughing, coughing, sneezing, or any central nervous system disease. Impaired strain will trigger a momentary increase in comprehension, an element of aphasia, with 30 3 The Neurologic History Holds the Diagnostic Keys

intact hearing secures the diagnosis of tem- Past medical history includes diabetes, morbid poral lobe disease, posterior location. obesity with a body mass index of 40, hypothy- 11. Inability to recognize a part of one’s own roidism. Medications are metformin, losartan, body is called “autotopagnosia” which is and levothyroxine. He smokes one pack of ciga- generally associated with a nondominant rettes per day. hemisphere lesion. Neurologic examination: Blood pressure 110/70. 12. Formed visual hallucinations such as Mickey Pulse 88, regular. He has bilateral high-pitched Mouse in this case are known to be associ- mid-cervical carotid bruits. Ankle refl exes are ated with temporal lobe lesions. Unformed absent. Vibration sense is absent at toes and visual hallucinations such as geometric ankles. shapes occur with occipital lobe pathology. Carotid Dopplers show 50–70% stenoses, The signal feature here is the localization to bilateral. the left eye which implies the left visual fi eld, hence a probable focal lesion rather than an Diagnosis : Transient ischemic attacks. organic psychosis. Thus a right temporal lobe Neurology consultation is requested regarding: lesion is likely. Stereotypical brief events 1. Should carotid angiography be performed? with preservation of consciousness indicates 2. If stenosis is confi rmed, should a carotid stent a simple partial seizure. or surgery be performed? For a neurologist the chief complaint is best 3. Which side should be treated fi rst? given without the past medical history or irrele- The neurologist’s assessment focuses on the vant comments on race or ethnicity. In any of the history, repeating many of the questions already above-cited examples would it be immediately asked. essential to know that the patient was diabetic, 1. Are you certain that you did not lose con- hypertensive, or that he or she was Asian- sciousness? “Yes.” American or African-American? Why not use 2. Did you lose vision? “Yes.” For how long? the most revealing exact words of the patient? “Less than 1 min.” The past medical history can be expanded upon 3. Did everything turn pitch black or just blank, in the section of past medical history. Ethnicity like nothing was there? “Pitch black.” can be added to the social history if deemed rel- 4. You fell twice. What happened the other time? evant although it is seldom pertinent for diag- “I just sat down.” nostic purposes. Can a 45-year-old Native 5. Why did you sit down? “I was short-winded American man with hypertension have a brain and felt dizzy.” tumor? Certainly. Can a diabetic African- 6. Did you have a spinning sensation, lighthead- American woman with painful paresthesias of edness or loss of balance? “Lightheadedness.” one arm have a herniated cervical disk rather 7. What were you doing when you fell? “I was than neuropathy? Absolutely and most probable. walking out of the bathroom once and the Diabetic neuropathies ordinarily are symptom- other time I had just gotten out of bed and atic in the feet fi rst. walked outside to get the newspaper.” A typical example of misdiagnosis with superb 8. Had you urinated or moved your bowels although uninformed medical care follows: before walking out of the bathroom? “I had just urinated.” Case 1 : A 79-year-old man complains of three 9. How long had you been out of bed when you “blackouts” in the last 2 weeks. He fell twice as picked up the newspaper? “2–3 min.” his legs “gave way.” He did not lose conscious- ness, denies memory loss, impaired speech, Analysis : numbness, tingling or subsequent weakness. 1. Visual loss can be due to retinal, optic nerve, There was no tongue biting or urinary inconti- optic tract, optic radiation, or occipital lobe nence. He momentarily lost his vision. disease. 3 The Neurologic History Holds the Diagnostic Keys 31

2. This is not which affects (d) The source : Hypotension or cardiac one eye due to retinal ischemia. arrhythmia with decreased cardiac output. 3. Transient blackness usually indicates retinal (e) Risk factor : Diabetes. ischemia. Absent vision (blank) occurs with 11. Diagnosis : Diabetic autonomic neuropathy. occipital lobe disease. 4. Bilateral ischemia implies poor retinal perfu- Lessons : sion which occurs with hypotension. Usually 1. A meticulous history discloses a probable brief blindness, described as nothing there, is diagnosis. most common with transient ischemic attacks 2. The blood pressure should be checked over in the basilar artery-bilateral posterior cere- time, at least 3 min in this case since symp- bral artery distributions. toms occurred after 2–3 min of walking. 5. Prodromal symptoms of dizziness occur with 3. The fi nal diagnosis is established after both hypotension. Shortness of breath is probably history and examination are completed. due to insuffi cient perfusion of the lung Case 2 : A 55-year-old woman complains of three apices. episodes of loss of vision affecting the right eye. 6. After urination there is commonly a drop in She says, “They don’t last very long,” but her vision blood pressure associated with a vagal is blurry and she cannot read. Her past medical his- response since bladder contraction is para- tory includes diabetes, and she smokes two packs of sympathetic and the latter causes a decreased cigarettes per day. Examination discloses bilateral heart rate. A contributing factor is diminished carotid bruits but is otherwise normal. Laboratory sympathetic tone associated with inadequate evaluations include an MRI of the brain which peripheral vasoconstriction. reveals moderate subcortical ischemic change. 7. Three episodes in 2 weeks implies a predis- Carotid Dopplers reveal mild carotid stenoses bilat- position, an underlying dysfunction of the erally. Angiography demonstrates moderate-to- autonomic system. severe bilateral external carotid stenoses and minor 8. The neurologic examination is identical but atherosclerotic changes at the bifurcation. further assessment of blood pressure is A neurologic consultation is requested. The indicated. neurologist queries the patient who expresses Blood pressure is 115/70 sitting with a some annoyance and is oppositional at his insis- pulse of 84. tence of clarifying her comment, “They don’t last Blood pressure standing at 1 min is 110/65 very long.” Eventually she responds to the ques- with a pulse of 88. tion, “Do they last seconds, minutes or closer to an Blood pressure after standing for 3 min is hour?” She responds, “Closer to an hour.” “Do 78/50 with a pulse of 100. (The patient feels you have a headache?” “No.” “Is your eye or head dizzy.) sore afterwards?” “Of course my eye and forehead 9 . Diagnosis : and auto- are a little sore.” “Have you ever had severe head- nomic neuropathy. aches?” “I had severe sinus headaches in my 20s 10. Summation . during the allergy season.” “Were you ever nause- (a) Duration is less than 1 min. The differen- ated with them?” “Sometimes, but usually in the tial diagnosis is near-syncope, TIA and sun when my headache was worse.” The diagnosis seizure. is migraine. Had there been no pain whatsoever, (b) The location is both eyes. The differen- the diagnosis would be ophthalmic migraine. tial diagnosis is near-syncope with reti- nal ischemia, TIA involving occipital Lessons : lobe. 1. Long duration of visual symptoms, more than (c) The character is black. The diagnosis is 5 min, suggest migraine. A TIA is typically near-syncope as blackness implies reti- less than 5 min although by defi nition it may nal hypoperfusion. last up to 24 h. 32 3 The Neurologic History Holds the Diagnostic Keys

2. Despite an initial refusal to answer a question, occurrence. Finally, a review of current medi- giving the patient a few choices may elicit a cines, dosages included, is mandatory. response. The family history is of interest but seldom 3. A diagnosis of “sinus headaches” given by a yields diagnostic information. A positive family patient should always be suspect. Details are history of tremor is useful if that is the patient’s required and, in this case, the presence of nau- primary complaint. Conversely, if a female sea and indicates migraine which patient has a chief complaint of headache, a is far more common. family history of migraine is of questionable A neurologic history is incomplete without pertinence since close to 20% of the female pop- questioning the patient about all neurologic ulation has migraine. Certainly a strongly posi- symptomatology even if deemed irrelevant. If a tive family history of malignancy or vascular 62-year-old patient seeks attention because of a disease conveys relative risk but does not obvi- weak right leg, should he be questioned about ate the need to address each symptom on its own memory or speech disturbances? A left cerebral merits. lesion may cause elements of aphasia as well as The social history provides important infor- right leg weakness. If a 40-year-old woman com- mation. Aside from smoking and alcohol con- plains of leg weakness associated with stiffness, sumption, the patient’s sleep history and changes questions about vision may uncover the diagno- in appetite convey meaningful data. For instance, sis. For example, she may have blurred vision restless sleep and daytime somnolence may affecting one eye with poor color perception indi- indicate obstructive sleep apnea or periodic leg cating an optic nerve lesion. Stiff, weak legs point movements of sleep. Chronic fatigue associated to a myelopathy with spasticity. Widely separated with muscle pain and insomnia with early lesions suggest a demyelinating disorder such as morning awakening strongly suggests depres- multiple sclerosis or, in this case, possibly neuro- sion with somatic manifestations. Occupational myelitis optica (Devic’s disease). exposures to toxic agents should always be Consequently, a neurologic history requires documented. A list of allergies to medications questioning all patients about episodes of loss of is routine. consciousness, memory loss, sleep patterns, may contain useful infor- speech or language function, headache, visual mation. Palpitations of the heart may tip the phy- disturbances, hearing disorders, diffi culty swal- sician off to the possibility of cardioembolism as lowing, dizziness, impairment of balance and a cause of stroke. Genitourinary symptomatology gait, strength, numbness and tingling, and fatigue. is often of clinical importance. Urinary inconti- The presence of neck or back pain may be impor- nence associated with a vague history of brief tant. Additional questions regarding bladder and altered mentation may indicate a complex partial bowel function may be pertinent. seizure. Diffi culty initiating urination may occur The past medical history is obviously essential with lumbosacral root lesions. Unawareness of to obtain in depth. Nevertheless, there are numer- urinary incontinence during an alert state sug- ous pitfalls. Open-ended questions such as, “Do gests cerebral pathology. For example, diffi culty you have any illnesses?” may be answered nega- getting out of a chair and a shuffl ing gait with tively. For instance, many people do not consider start hesitation accompanied by unawareness of hypertension an illness. Others may deny it when urine-stained pants suggests normal pressure specifi cally asked because treatment has been hydrocephalus. Gastrointestinal disorders such as successful. If recent blood pressures are normal Crohn’s disease can be associated with a stroke. on medication, then, by their analysis, they no Whipple’s disease may be the cause of a brain- longer have hypertension. Prior surgeries are stem lesion. Diabetic enteropathies with diarrhea often forgotten such as a mastectomy for breast may be a tip off for the etiology of a neuropathy. cancer. Metastases from carcinoma of the breast There are special situations which require a 10–15 years after mastectomy is a well-known solicitous, caring approach. This is especially Neurologic Symptoms in Psychiatric Disease 33 applicable to minority patients. Many Hispanic Case 4 : A 26-year-old man complains of numerous patients who have lived in the United States for “blackouts.” While watching TV he reports sud- years and speak English fairly well may still denly losing consciousness and being unarous- insist on a translator. When the physician can able by family members for 10–15 min. No overt speak a little Spanish, even if not fl uent, patients seizure phenomena have been described by wit- will be exceptionally pleased and may be willing nesses. He has taken short-term disability since to communicate in faulty English without assis- these “blackouts” are occurring at least a few tance, very useful if no translator is available. times a day. He is desperate to return to work. African-American patients will often distrust Neurologic examination reveals an alert, athletic- Caucasian physicians because of preconceived appearing man with normal speech. There are no notions or prior disagreeable experiences. This abnormal neurologic signs. An EEG shows a few may affect how they present their symptoms as suspicious sharp waves in the right temporal described in the following case. region. A tentative diagnosis of complex partial seizures is made and treatment with phenytoin is Case 3 : A 56-year-old African-American woman initiated. Despite good therapeutic levels, the was leaving a supermarket on a rainy day. She patient continues with these episodes. On the slipped on the wet pavement and landed on her patient’s fourth visit his sister accompanies him back. She went to a busy local emergency room and adds critical information. When queried where an X-ray of her lumbar spine was obtained about the patient’s sleep habits she adds that he because of her complaint of severe low back pain. snores loudly. Polysomnography discloses This was normal. The Emergency Room physi- obstructive sleep apnea. cian suspected she was malingering and planning a lawsuit. The following day she decided to seek Lessons : a neurologic opinion and, in the waiting room, 1. Always obtain the report of witnesses when was screaming in agony. When she was fi rst seen the history is unclear. her behavior was hysterical, appearing dispro- 2. Queries about sleep habits should be a routine portionate to any conceivable injury. The neuro- part of any neurologic history. logic examination was normal, but she was explicitly tender in the mid-to-lower thoracic region. Plain fi lms revealed a T9 compression Neurologic Symptoms in Psychiatric fracture. This patient displayed the cry-for-help Disease syndrome, i.e., “take me seriously.” The take- home lesson is never dismiss a complaint despite Many if not most cultures, including western circumstances implying dishonest behavior. civilization, consider psychiatric illness a failure The nearly deaf patient can be extraordinarily of the individual to cope with the normal stresses diffi cult to evaluate. Many doctors shout loudly at of everyday life. The eradication of this stigma is the patient forgetting that speaking directly into in progress throughout the western world but has the patient’s ear from a distance of 1 or 2 in. is been only partially successful. This mind-set much more effective than shouting at a distance. It facilitates somatization which legitimizes the ill- is also considerably less disturbing to other patients ness for the patient. Thus every neurologic symp- or personnel in the vicinity. If this is also ineffec- tom has the potential of being the physical tive, short written questions will be necessary. manifestation of a psychiatric illness. Sometimes The nonspecifi c complaint may be particularly considered the bane of a neurologist’s practice, challenging and frustrating. A good example is somatization must be quickly recognized before the patient who says, “I blacked out.” This could extensive, expensive and, especially, invasive mean loss of vision, loss of consciousness or a tests with an element of risk to the patient are sleep disorder. An intriguing case report is ordered. This requires a confi dent physician will- described below. ing to oppose a patient’s demand for an unneces- 34 3 The Neurologic History Holds the Diagnostic Keys sary examination. Conversely, a hasty, reckless cially depression, as well as underlying neuro- diagnosis of a psychiatric disease when con- logic conditions which may even be the fronted with peculiar but nonetheless realistic precipitating factor. Consequently, the examining symptoms is not unusual. Hence the neuropsy- physician is easily led astray and quick diagnoses chiatric illness can be the most challenging and can be hazardous. La belle indifference, demeanor intriguing part of a neurology practice. inconsistent with the illness, once considered a Depression can manifest itself in innumerable classical sign is not a reliable indicator since ways without an acknowledgment of a mood dis- many patients with severe discomfort will be order by the patient. There are several symptoms, stoic and minimize symptoms. More useful signs however, which patients are not reluctant to dis- include collapsing leg weakness, a sudden drop cuss and can provide the necessary clues for a of an arm when evaluating for arm drift, overac- diagnosis. They often feel free to acknowledge tive facial movements such as grimacing or blink- diffi culty with concentration and may use the ing and Hoover’s sign. When one symptom is leg term “brain fog” to explain slowing of thought weakness, the patient is examined supine and processes. Not unusual spontaneous complaints requested to lift one leg. The opposite leg should are derealization, a feeling of being disconnected exhibit downward pressure; thus the examiner from the environment, and depersonalization, a places his hand under the heel to detect this com- sense of detachment from the body. If only brief, pensatory movement. The absence of detectable seconds to a few minutes, simple partial seizures downward pressure, Hoover’s sign, supports a must also be considered. A change in sleeping diagnosis of a conversion reaction or malingering. pattern, particularly early morning awakening Panic disorders are recurrent attacks of fear, if and an alteration in appetite, most often anorexia, not terror, which are associated with physical are common. Loss of motivation, fatigue, restric- symptomatology. Occasionally the patient latches tion of interests, loss of libido, and poor perfor- onto these complaints as a major issue. Palpitations, mance at work are additional features. tremor, faintness, diffi culty swallowing due to a Memory loss is a frequent complaint of the “lump” or fullness in the throat, shortness of depressed patient and this inevitably raises the breath, and paresthesias of the extremities due to question of the pseudodementia of depression hyperventilation are some of the manifestations. versus the mild cognitive impairment of incipient Differentiation from simple partial seizures is Alzheimer’s disease. Slow responses to questions ordinarily not diffi cult since the duration of symp- may refl ect the bradyphrenia (slowed thinking) toms is usually more than 10 min. of dementia or the psychomotor retardation of Hypochondriasis is a well-known and usually depression. In this instance neuropsychological easily recognized disorder because of multiple, testing is usually required to differentiate between frequent visits to a physician with a multiplicity structural disease and a “functional” disorder of complaints. Diagnostic studies which refute although the latter is physiological, possibly these worries are rejected as insuffi cient or unre- structural, and most often precipitated by exter- liable. Other uncommon conditions such as facti- nal events. tious disorders and malingering, which must Conversion disorders are manifested by motor always be considered, requires practical judg- or sensory symptoms that suggest neurologic or ment by an alert physician. A detailed analysis is medical disease and are provoked by psychoso- not possible in this introductory text. cial stresses. There is often a secondary gain Factitious disorders involve a psychological through this behavior. Some physicians believe urge to be ill. Self-medication, falsifying test they are less common than a century ago since results, and use of artifi cial methods are utilized the lay public is exposed if not deluged with to feign illness. Munchausen syndrome, one form medical information. Conversely, the presenta- of this disorder, is associated with traveling from tions may be more sophisticated. There is high one physician or hospital to another to seek treat- comorbidity with other psychiatric disease, espe- ment, even surgical, through lying (pseudologia Bibliography 35 fantastica), and self-medication. Munchausen by materialistic gain for being declared disabled. proxy, a horrifi c disorder, involves a mother or These patients commonly have an antisocial caregiver who produces a disease in a child by personality disorder. medication or other manipulations. Malingering is seldom seen in most neurol- ogy clinics but not unusual in practices which Bibliography are visited by patients seeking workmen’s com- pensation. The malingerer often arrives with a Critchley M. The parietal lobes. New York, NY: Hafner; cane, crutches or even in a wheelchair. He or she 1966. Weintraub MI, editor. Malingering and conversion reac- tends to be hostile, sullen and exhibits a low tions. Neurologic clinics. Philadelphia, PA: WB pain threshold. There is ordinarily an obvious Saunders; 1995. Neurologic Examination 4

Keywords Neurologic examination • Reflexes • Mental status • Sensation • Cranial nerves • Motor function • Gait

The purpose of this chapter is to review the tech- 9. Pin. niques of observation and examination. As Max 10. Cotton or Q-tip. Wintrobe, an eminent internist, remarked, “The Useful specialized equipment: physical examination must be carried out with a 1. Tape measure. watchful eye, a sensitive touch, discerning ears, 2. Two-point discriminator. and an alert sense of smell. Above all, what is 3. Coins, rubber band, key, or other small needed is an alert mind free of dogma and rou- objects. tine.” Nowhere is this more essential than in the 4. Red glass for testing diplopia. specialty of neurology. 5. 512 frequency tuning fork. Despite the detailed description of required 6. Tube of cinnamon or other spice. observations, a thorough neurologic examination Categories of examination: is easily completed in an uncomplicated patient 1. Vital signs, cervical bruits, heart examination if within 15 min. Only consistent practice on all vascular disease is a diagnostic consideration. history and physicals can this allotted time allow- 2. Mental status. ance be achieved. If complicated abnormal signs 3. Cranial nerves. are elicited, a return to a portion of the examina- 4. Motor function. tion may be necessary to ensure accuracy and 5. Gait and station. thus entail a more lengthy assessment. 6. Reflexes. The neurologic examination requires specific 7. Sensory testing. instruments. These are: 8. Mechanical signs. 1. Watch. A watch is preferred over fumbling with a cell phone or other device when taking a pulse. Vital Signs 2. Stethoscope. 3. Ophthalmoscope. Vital signs can be taken either at the beginning or 4. Penlight. the end of the examination. If there is any history 5. 128 frequency tuning fork. of impaired cognition, taking of vital signs is 6. Reflex hammer. often best deferred to the end of the neurologic 7. Visual acuity card and pinhole. examination. There is a natural merging from the 8. Two red matches or other small red targets. history to the evaluation of the mental status.

J.N. Alpert, The Neurologic Diagnosis: A Practical Bedside Approach, 37 DOI 10.1007/978-1-4419-6724-4_4, © Springer Science+Business Media, LLC 2012 38 4 Neurologic Examination

Relegation of vital signs to a nurse or other assis- of the heart is necessary in any patient who gives tant can detract from the patient’s perception of a history suggestive of cerebrovascular disease. the physician’s interest. Saving 1 min or less for blood pressure and pulse may not be worthwhile as many patients benefit from the reassurance of Mental Status Examination the physician’s touch. A detailed mental status examination is not Checking the blood pressure and pulse is always required. mandatory for a neurologic examination. This examination done in its entirety is Any history of neurologic symptoms such as optional if the patient’s major complaint is com- faintness, dizziness, nausea, weakness, loss of pletely unrelated to cerebral function such as low balance, or impairment of vision with a change of back pain. The decision assumes that the past position requires blood pressure evaluation in medical history includes no previous illness supine, sitting, and standing positions. The blood affecting cognition. The patient is certain to be pressure standing should be taken every minute annoyed if a full mental status examination is for 3 min and, if the blood pressure has not stabi- performed unnecessarily. lized, up to 5 min. The blood pressure should be Preliminary observations begin when the taken in both arms to assess for the subclavian patient is first seen or heard. steal syndrome. This occurs when there is a rever- sal of blood flow in the vertebral artery to com- pensate for a proximal occlusion or high-grade Visual Observations stenosis of one subclavian artery impairing blood flow to the arm. The systolic pressure difference Facial symmetry and expression are key ini- is usually greater than 30 mm of Hg. A pulse tial observations. delay when both radial pulses are palpated simul- taneously is often detectable. 1. Facial asymmetry, spontaneous or with emo- tion, especially a laugh. A history of loss of balance requires checking 2. Facial expression. For example, is the effect blood pressure standing if no other obvious sad or flat? Is the face immobile? etiology is found. 3. Facial features. For example, is there prog- nathism or proptosis? Examination of the neck for cervical bruits is 4. Eye position. Look for skew deviation (verti- essential even though a direct correlation to steno- cal separation of the globes). Is there dyscon- sis cannot be made. The first decision is to jugate gaze or forced eye deviation, up or distinguish between an arterial bruit and venous down, right or left? hum. The latter is low pitched and heard through 5. Head position. Is the head tilted or turned to both systole and diastole. The former is usually left or right, up or down? high-pitched and audible only during systole. 6. Body position. Is the patient stooped, hyper- Each carotid artery should be auscultated in low, extended, turned to one side, or is there a dys- middle­, and high cervical regions. If the bruit is tonic (twisted) posture? very localized, it is more likely to be associated 7. Arm and leg positions. with stenosis. Bruits heard high in the neck 8. Note the presence of abnormal involuntary are more often correlated with stenosis. movements of eyes, trunk, or limbs. For exam- Supraclavicular bruits are usually transmitted ple, is there nystagmus, tremor, tic, myoclonus, from the chest, but occasionally indicate a signifi- athetosis, chorea, or ? Description cant subclavian stenosis. The bell is more useful of these entities will be located in the sections than the diaphragm for cervical bruits. Auscultation of cranial nerves and motor function. Mental Status Examination 39

labial muscles, especially as occurs with peripheral Observing the patient for involuntary eye, 7th nerve lesions, will cause difficulty repeating trunk, and limb movements is essential. words such as “mama” and “papa.” Pseudobulbar speech is a spastic dysarthria which is slow, harsh, strained, and implies bilateral upper Auditory Observations motor neuron lesion involvement. Associated findings are forced, uncontrollable crying and laughing. Patients with Parkinson’s disease fre- Dysarthria (nonlocalizing) must be distin- quently have rapid, mumbled and low volume guished from aphasia (localizing). speech. Spas­modic dysphonia is characterized by involuntary causing intermittent vocal The most obvious and first element of the cord abduction or adduction and sometimes trem- mental status examination is the assessment of ulous speech. Absence of inflection or monotone speech and the first hurdle is to distinguish dysar- speech is called a dysprosody, a common mani- thria (nonlocalizing) from aphasia (localizing). festation of nondominant hemisphere lesions. Ninety to ninety-five percent of the population is left-hemisphere dominant and this includes about Pseudobulbar speech is a spastic dysarthria. one-half of left-handed people. Aphasia affects all language functions, verbal, reading, and writ- ing. Dysarthria is an impairment of articulation Aphasia and pronunciation. Evaluation of the aphasic patient has been initi- ated prior to formal testing since spontaneous Dysarthria speech yields much of the needed information. Typical types of dysarthria include hoarse, nasal, breathy, wavering, and tremulous speech. Detection of paraphasias is the first task when Disorders of strength, speed, inflection, pitch, evaluating for aphasia. and rhythm are additional elements to observe. Commonly used words to test for dysarthria are Spontaneous speech includes propositional, “Methodist Episcopal” and “baby hippopota- emotional, and automatic forms. A patient with a mus.” Spontaneous speech provides most, if not nonfluent aphasia may still be able to curse flu- all, of the evaluation. ently. Patients with global aphasia may repeat the same simple word or a neologism to every ques- Nasal speech is characteristic of soft palate tion. This has been termed monophasia or a ver- weakness. bal automatism. The most important observation is the detection of paraphasias. These may be Hoarseness, breathiness, and low pitch occur ­frequent and obvious, but occasionally barely with weakness of laryngeal muscles and vocal recognizable. Three types of paraphasias are cord paralysis. Nasal speech is characteristic of ­phonemic, semantic (verbal), and neologisms. soft palate weakness. All of these findings may A phonemic paraphasia might be to say, for occur with lesions of the brainstem, 10th cranial example, “sleet” instead of “street.” A semantic nerve (vagus), , and vocal (verbal) paraphasia would be substituting an cord paralysis; the last is commonly due to lesions incorrect word such as “door” for “window.” of the recurrent laryngeal nerve. Wavering, irreg- A neologism is a manufactured word with no ular speech with variable loudness suggests meaning such as “flubsum” for “shoe.” Other ­scanning speech, characteristic of cerebellar sys- errors which may occur include perseveration, tem dysfunction. Fatigue of speech is demon- circumlocutions, excessive pauses, hesitation, strated by counting up to 50 or 100 and, if present, and agrammatism. Perseveration is repeating the suggests myasthenia gravis. Any involvement of response to a first question when a new request is 40 4 Neurologic Examination given. Circumlocution would be, for example, to localize a lesion to the perisylvian region. Inability say “what you tell time with,” instead of “watch.” to read, alexia, can be congenital or acquired. It Agrammatism is defective syntax. Completely may occur with or without agraphia. Alexia with- unintelligible speech is termed jargon aphasia. out agraphia is a classical disconnection syn- Comprehension is tested by giving verbal drome due to a lesion most often in the dominant instructions such as “open your mouth” and “put medial occipital cortex involving the splenium of out your tongue.” These are simple commands. the corpus callosum. A right homonymous hemi- A two-step command is “raise your arm and open is usually an associated finding. In this your mouth.” A three-step command would be instance, the patient may write a sentence and, adding a third instruction such as “cross your after a delay of a few minutes plus two or three legs.” A four-step command would be to add a intervening questions, may be unable to read it. fourth instruction. In the absence of right–left Alexia with agraphia is more common and is usu- confusion, a four-step command could be “touch ally associated with a lesion of the angular gyrus your right (1) ear (2) with your left (3) hand (4)”. or with Wernicke’s aphasia. The latter occurs Finally, a statement and question could be, “The with a lesion of the superior temporal gyrus. dog bit the cat. Which animal is injured?” Occasionally, gestures are understood but not Alexia without agraphia is a classical discon- verbal instructions; thus, the patient is able to nection syndrome due to a lesion in the domi- imitate the examiner. nant medial occipital cortex involving the Naming is impaired in nearly all types of splenium of the corpus callosum. aphasia. This may be for objects or body parts. Testing may elicit paraphasias of any type and Aphasia classifications require specific obser- circumlocutions. Responsive naming, for exam- vations. The precise identification of the aphasia ple, would be, “What do you do with a glass?,” has limited practical value since there are many “What do birds lay in their nest?,” “How does a instances of lesions in approximately the same lemon taste?” These questions test verb, noun, anatomic location with different manifestations. and adjective retrieval, respectively. Nevertheless, important distinctions include flu- ent vs. nonfluent, good vs. poor repetition, and Naming is impaired in nearly all types of good vs. poor comprehension. Naming is usu- aphasia. ally impaired with all types of aphasia. Paraphasias are common in fluent and global The ability to repeat simple phrases such as aphasias. The general anatomic correlations are the standard, “no ifs, ands or buts,” may help to (see Table 4.1):

Table 4.1 Aphasia classifications Aphasia type Fluency Repetition Comprehension Naming Global A A A A Broca A A NL A Transcortical motor A NL NL A Transcortical sensory NL NL A A Transcortical mixed A NL A A Conduction NL A NL Variable Wernicke’s NL A A A Anomic NL NL NL A Subcortical Variable NL Variable A A abnormal; NL normal Mental Status Examination 41

due to lesions which interfere with information Useful distinctions in aphasia analysis are fluent transfer from primary sensory cortical areas vs. nonfluent, good vs. poor repetition, and to language centers. Consequently, there are good vs. poor comprehension. descriptions of visual, auditory and tactile agnosia, phonagnosia, autotopagnosia, finger agnosia 1. Nonfluent. Posterior inferior frontal lobe lesions. which is a focal type of autotopagnosia, prosopag- 2. Fluent. Posterior temporal and parietal lobe nosia, astereognosis, and visual spatial agnosia. lesions. The concept of visual agnosia has been questioned 3. Poor repetition. Perisylvian. since many of these patients have some disorder, 4. Good repetition. Outside the perisylvian region. often subtle, of visual perception. Tactile agnosia 5. Poor comprehension. Posterior temporal and can be considered astereognosis, the inability to parietal. recognize a stimulus by touch even though the 6. Good comprehension. Posterior frontal. perception of touch is intact. Additionally, there Global aphasias are typically due to large peri- are patients diagnosed with visual or tactile agno- sylvian lesions. Broca’s aphasia is associated sia who have dysnomia and thus are inaccurately with lesions of the posterior inferior frontal gyrus labeled. Visual agnosia implies normal naming by and anterior perisylvian region. Wernicke’s apha- touch of an unrecognizable visual stimulus. sia is due to a lesion in the superior temporal Clearly, agnosia must be accompanied by normal gyrus. Transcortical aphasias have normal repeti- naming using a different sensory stimulus. tion and are due to lesions outside the perisylvian region, transcortical motor is located anteriorly, Agnosia must be accompanied by normal transcortical sensory posteriorly, and transcorti- naming using a different sensory stimulus. cal mixed is a combination. Echolalia, the repeti- tion of the examiner’s words or phrase, is not unusual in the transcortical aphasias because of Autotopagnosia (body image) is a patient’s the maintained ability to repeat. impaired ability to name or recognize his or her own body parts in the absence of dysnomia for Lesions of the posterior inferior frontal gyrus objects, an exceedingly rare phenomenon. A spe- and the superior temporal gyrus of the domi- cific type of autotopagnosia, finger agnosia, is nant hemisphere may cause Broca’s aphasia one of the four parts of Gerstmann’s syndrome and Wernicke’s aphasia, respectively. which also includes right–left confusion, agraphia, and acalculia. This syndrome is due to a lesion of Lesions of the arcuate fasciculus, a tract the dominant angular gyrus (parietal lobe). ­running between the posterior and anterior lan- Phonagnosia is the inability to recognize voices guage areas, may result in conduction aphasia, a and prosopagnosia, the inability to recognize fluent aphasia with good comprehension but poor faces. Prosopagnosia has been popularized by the repetition. Subcortical aphasias are those due to book, The Man Who Mistook His Wife For A Hat, lesions of the thalamus, caudate, putamen, and by Oliver Sachs. Lesions, commonly bilateral, of internal capsule. These are usually of vascular the occipitotemporal regions and nondominant origin and too variable to categorize. posterior hemisphere lesions are associated with visual agnosia and prosopagnosia. Agnosias Apraxias Agnosia indicates normal reception of a sen- Apraxia is the inability to perform an act despite sory stimulus but inability to identify it. intact comprehension, attention, motor, and ­sensory function. Apraxias are common with any Agnosia indicates normal reception of a sensory type of dementia which indicates bilateral cerebral stimulus, but inability to identify it. These can be disease. Specific types may be caused by unilat- considered disconnection syndromes. They are eral hemisphere disease. The most recognizable 42 4 Neurologic Examination are the motor apraxias; they are limb-kinetic, lesions. It becomes obvious when requesting the ­ideomotor and ideational. These apraxias are patient to close his eyes in order to check position associated with dementia or dominant hemi- sense. The patient does so for 1 or 2 s only, no sphere lesions. matter how many requests are given. Checking visual fields will be nearly impossible when the Apraxia is the inability to perform an act patient maintains eye closure. This latter problem despite intact comprehension, attention, also occurs with blepharospasm from which it motor, and sensory function. must be distinguished. Motor impersistence is a term that has been used for inability to maintain Limb-kinetic apraxia is actually clumsiness eye opening or eye closure. and thus not a true apraxia, but a common initial abnormality of one of the motor systems, corti- Disorders of Attention and Recognition cospinal, extrapyramidal, and cerebellar. Under the rubric of disorders of attention and Ideomotor apraxia is inability to perform an act recognition are anosodiaphoria, anosognosia, such as wave good-bye, brush your teeth, or asomatognosia, autotopagnosia, extinction, inat- comb your hair. Occasionally, the patient may be tention, and neglect. Anosodiaphoria refers to unable to imitate an act demonstrated by the recognition of the deficit but an absence of con- examiner. Ideational apraxia is the inability to cern, a dismissal of its importance. Patients with plan an act requiring several steps such as “fold a neglect pay no attention to the involved limb letter, put the letter in an envelope, seal the enve- which is weak, but when given a specific request lope, stamp it and mail it.” Sympathetic apraxia to move it will do so. Anosognosia is utter denial has been identified with left frontal lesions; there of any deficit. is a right hemiparesis and left limb apraxia. Anosognosia refers to denial of illness which Ideomotor apraxia is inability to perform an act is a common finding associated with degen- such as wave good-bye or imitate a gesture. erative brain disease and nondominant hemi- sphere lesions. Constructional apraxia is the inability to copy diagrams such as intersecting pentagons or drawing Asomatognosia is loss of awareness of one-half a clock face including the numbers appropriately of the body. Autotopagnosia refers to a failure of placed and requesting the patient to add a selected the patient to recognize his own limb or body time with the hands drawn in. Constructional part. Finally, a specific and dramatic symptom is apraxia (apractagnosia) is commonly associated denial of blindness, Anton’s syndrome. Extinction with nondominant hemisphere lesions. is a sensory phenomenon whereby the patient Dressing apraxia can be evaluated by turning a perceives a single sensory stimulus whether shirt sleeve inside-out and asking the patient to visual, auditory, or tactile, but when given a com- put the shirt on. Hemineglect is observed when peting stimulus on a contralateral side, perceives the patient dresses just one-half of his body. This only the contralateral stimulus. Hemi-inattention phenomenon occurs most often with right pari- is another word employed for this finding. This etal lesions. More often, the patient is unable to may be intramodal, e.g., two touch stimuli, or put his shirt on because of a confusional state. intermodal, e.g., auditory on the right and touch on the left. There is increased sensitivity of the Apraxia of eye opening or eye closure is not intramodal stimulus when the touch stimuli are an infrequent symptom of nondominant hemi- on different parts of the body such as right hand sphere lesions. and left leg. All of these perceptual disorders are nearly always due to nondominant hemi- Apraxia of eye opening or eye closure is not an sphere lesions, particularly but not exclusively, infrequent symptom of nondominant hemisphere parietal lobe. Mental Status Examination 43

To summarize, some of the above-noted terms might be expected to perform serial 7s. The are used almost interchangeably such as neglect, educational status of the local community anosognosia, and anosodiaphoria. Hemi- should be assessed before deciding on normal inattention is another commonly used term for parameters. both motor and sensory phenomena. These issues 6. Spelling and reversing five-letter words such have been discussed and debated by generations as “table” or “world” and, if the patient fails of neurologists and differences of opinion persist. the test, using four, then three, and finally, Most important is the ability to recognize or elicit two-letter words. this phenomenon, particularly in near-isolation, 7. Short-term recall. The patient is given three in order to confirm the presence of neurologic words, nouns, and is asked to repeat them in pathology. 3–5 min. To avoid interrupting the examina- The Mini-Mental State Examination should tion as well as losing time, it is useful to ask be completed if memory loss is a primary or the patient to repeat the words during the significant element of the present illness (see reflex examination since this section does not Table 4.2). If memory loss is a minor or question- require any active response from the patient. able concern, a screening exam is ordinarily suf- This test is critical since it may be the only ficient. This assumes that the patient’s history as abnormal sign in early dementia. well as any observation made by a family mem- ber elicits no information that suggests signifi- Short-term recall deficits are often the first cant abnormal cognitive function. and only abnormal finding in patients with early dementia. The Mini-Mental State Examination should be completed if memory loss is a primary or 8. Optional. Proverb interpretation. This is pri- significant element of the present illness. marily useful for concrete responses. For example, when asked, “What does it mean if The screening exam may include: someone tells you ‘you hit the nail on the 1. Orientation to person, place, and date. head’?,” the patient may answer, “You took 2. Following commands such as: the hammer and hit the nail into a log.” Another (a) Open your mouth (simple or one-step). commonly used test phrase is “don’t cry over (b) Touch your left ear (two-step). spilt milk.” Abnormalities suggest the pres- (c) With your left hand touch your nose ence of frontal lobe disease. (three-step). 9. Optional. Face-hand test. The patient is given (d) With your right hand touch your left ear two touch stimuli, one on each side of the (four-step). body, on different parts. Typically, these are Should right–left disorientation be prominent, face, chest, hand, and leg. When given two different commands are necessary such as “raise stimuli, the patient may extinguish all the your hand,” “open your mouth,” “put out your stimuli on one side which indicates the tongue,” and “cross your legs.” (four-step presence of a contralateral cerebral lesion. command without need for right–left aware- Repeated bilateral errors of extinction may ness). Additionally, perseveration may be uncov- occur if the patient’s ability to perceive ered. This is the repetition of the same response more than one stimulus is defective. (verbal or motor) despite a new request. Persistent bilateral extinction phenomena 3. Naming objects and body parts. suggest generalized cerebral dysfunction. The 4. Repetition of simple phrases such as: “No ifs, least sensitive sign for unilateral extinction is ands or buts.” touching both hands, both legs, or both sides 5. Calculations. Addition of two-digit numbers, of the face. However, the use of sensory hier- such as 14 + 17, is a reasonable expectation for archy in descending order of dominance, face, high school graduates. College graduates chest, leg and hand, may elicit focal signs not 44 4 Neurologic Examination

Table 4.2 Mini-mental state examination (MMSE)a,1,2

References: (1) F olstein MF, Folstein SE, McHugh PR. “Mini-Mental State”: a practical method for grading the cognitive state of patients for the clinician. J Psychiatr Res. 1975;12:189–98. (2) Cockrell JR, Folstein MF. Mini-Mental State Examination (MMSE). Psychopharm Bull. 1988;24:689–92. (3) Becker JT, Huff FJ, Nebes RD, Holland A, Boller F. Neuropsychological function in Alzheimer’s disease: pattern of impairment and rates of progression. Arch Neurol. 1968;45:263–68. (4) Mortimer JA, Ebbit B, Jun S-P, Finch MD. Predictors of cognitive and functional progression in patients with probable Alzheimer’s disease. Neurology. 1992;42:1689–96 aAdapted from Folstein et al1 and Cockrell and Folstein.2 © 1975, 1998 Mini Mental LLC. Used with permission Cranial Nerve Examination 45

discernable by other means. An example would be touching the patient’s right face and left hand and vice versa. If only the left hand is not perceived on repeated testing, a right cerebral lesion would be suspected.

Cranial Nerve Examination

Olfactory Nerve (I)

Anosmia is an early sign in some patients with Parkinson’s and Alzheimer’s disease. Fig. 4.1 Ophthalmoscopic examination. The examiner should be close enough to touch the patient’s cheek with a The examination of this nerve is not routine, finger holding the ophthalmoscope but its function may be impaired with several neurologic disorders. Usually the patient’s main normal optic disk has distinct edges, but the complaint is loss of taste as olfaction provides nasal margin may occasionally be blurred. much of the nuance of taste. It is commonly There is a physiologic cup in the center which impaired after a serious head injury and may be shows a latticework due to the underlying lost on one or both sides with subfrontal or para- lamina cribrosa. The temporal margin of the sellar neoplasms. Anosmia is an early sign in disk is often pale; thus, observation of tempo- some patients with Parkinson’s and Alzheimer’s ral pallor is very subjective and often inaccu- disease. Diseases of the sinuses and nasal mucosa rate. The temporal portion of the optic disk is must be excluded. This sense is easily tested with located where the papillomacular bundle a small tube of spice such as cinnamon. enters the nerve. If there are any visual symp- toms, the macula should be inspected by requesting the patient to look directly into the Optic Nerve (II) light. The macula is located about 2 disk diam- eters temporal to the optic disk. 1. Ophthalmoscopic technique (see Fig. 4.1). 3. Attenuated arteries and arteriovenous nicking A dimly lit room is advantageous. The patient imply arteriolar disease in similar-sized intrac- must keep both eyes open and fixed on a distant erebral vessels. Hollenhorst plaques are bright object, preferably at least 10 ft. away, to prevent yellow, seen within arterioles especially at a near response with pupillary constriction. branching points, and are composed of choles- The best visualization of the fundus occurs terol. This implies embolism originating from when the examiner is close enough to touch the atherosclerotic plaques in the carotid artery or patient with a finger holding the ophthalmo- aortic arch. scope. The examiner should approach the patient from the side, adjacent to the shoulder, Absence of venous pulsations is expected to prevent obstruction of the patient’s central when intracranial pressure exceeds 200 mm

vision. If the pupil is small, less than 3 mm, a H2O. Ten percent of the normal population small aperture should be selected. display no venous pulsations.

The funduscopic examination should empha- 4. Papilledema. The main features are bilateral size the optic disk and immediate vicinity. blurring of the optic disk margins, especially the temporal side which is rarely blurred in the 2. The funduscopic examination should empha- normal population. Nasal margins of the disk, size the optic disk and immediate vicinity. The often blurred in the normal population, are 46 4 Neurologic Examination

also affected first in the patient with papille- dema. Absence of venous pulsations is expected when intracranial pressure exceeds

200 mm H2O, but they are not seen in approxi- mately 10% of the normal population. Venous pulsations are best visualized in or near the center of the optic disk. Flame and splinter hemorrhages at the disk margins ordinarily secure the diagnosis of papilledema. Hemorrhages only in the peripheral portion of the fundus are not characteristic of papille- dema. Without hemorrhages, the diagnosis of papilledema is suspect. There are other fea- tures which support the diagnosis, however, such as tortuous veins, angulation of vessels at the disk margins, and hyperemia of the disk. Vision is normal in early papilledema with the Fig. 4.2 Testing for a central with a red-tipped exception of enlarged blind spots, very diffi- match. Each eye is tested alone with the patient focused cult to determine at the bedside. Chronic on the match placed on the examiner’s nose. The match which should be perceived as brightest is on the examin- papilledema will cause visual field constric- er’s nose. If the match on the nose is dull or darker in tion. Fluorescein angiography with fundus color, the patient has a relative central scotoma photography can be utilized in questionable cases, specifically to rule out pseudopapille- in macular disease color perception is dema. Dye leakage beyond the disk margins is impaired, but light perception is normal since noted in true papilledema. False negative tests the entire perceives light. rarely occur in early papilledema. 2. Color comparison of two red matches. One is held on the examiner’s nose and one on his Vision is normal in early papilledema with the cheek. The patient is instructed to fixate on exception of enlarged blind spots. the match placed on the examiner’s nose and compare the brightness of the red color. If the Optic Nerve: Evaluation of Central match on the cheek appears brighter, then Vision (See Fig. 4.2) there is a defect in central vision or a relative central scotoma assuming the match is per- ceived. Obviously, inability to maintain focus Impairment of color and light perception is invalidates the examination. It is quite easy, often the earliest sign of optic nerve disease. however, to see the patient shift focus and two or three attempts may be required to Light perception is normal in macular disease. complete the test. 3. The patient is simply requested to look at the 1. Each eye is evaluated independently by cov- examiner’s nose and simultaneously ering the unexamined eye. A comparison is instructed to point out features on the exam- made of light and color perception. A small, iner’s face that appear indistinct. This is often bright red match is most often used to evalu- difficult for patients, but when there is a con- ate this quality. An abnormal eye sees less sistent abnormality, a scotoma is easily iden- light and colors are less bright. The patient tified and precisely localized. may interpret the defective eye perceiving 4. The patient is requested to focus on the more color as there is a “deeper” red. Thus, examiner’s nose and a red match is moved in clear definitions are essential. Furthermore, a circular fashion around the nose. When Cranial Nerve Examination 47

the color is lost or the matchstick not even i.e., the direct response is better than the perceived, a scotoma can be outlined. ­consensual response. 5. may be central, paracentral, or ceco- central, the latter extending to the blind spot. An afferent pupillary defect is present 6. Visual acuity. An accurate assessment with optic nerve disease but rarely macu- requires an evaluation with a pinhole to cor- lar disease. rect for a refraction error. The pinhole should be 2–2.5 mm in diameter. Despite normal 10. Photostress test. This test is performed to dis- visual acuity, a central scotoma may still be tinguish between macular and optic nerve dis- present, especially for color. ease. A light is shined directly into one eye for 10 s with the other eye covered. Visual acuity The central scotoma is the hallmark is then tested and timed. When the visual acu- ­finding of optic nerve disease and a more ity returns to the next larger line, the time is reliable sign than visual acuity. recorded. For instance, if visual acuity is 20/20, the time recorded to achieve a visual 7. Ischemic optic nerve disease most often pro- acuity of 20/25 is noted. The same is done duces altitudinal or quadrantic defects, with the other eye. With optic nerve disease, whereas demyelinating or metabolic optic the time to return to pretest visual acuity is the neuropathies more often produce central same as a normal eye. With macular disease, scotomas. light causes bleaching of the visual pigments 8. Central vision and macular disease. A central and significantly lengthens the recovery time. scotoma is often perceived by the patient as a 11. Flight of colors. A light is shined directly dark spot (a positive scotoma) when there is into one eye for 10 s. The patient closes his macular disease. Optic nerve pathology pro- eyes and reports the observed colors. Patients duces blurry vision (a negative scotoma). with optic nerve disease perceive few colors. Metamorphopsia occurs frequently with A definite asymmetry between eyes supports macular disease. An example is a distortion a diagnosis of optic nerve disease. This test of straight lines which appear wavy, crooked, has been found to be more sensitive than or angulated due to . visual-evoked potentials. Photopsias and micropsia may occur. 9. Afferent pupillary defect. This abnormality defines the presence of optic nerve disease. Visual Field Examination (See Fig. 4.3) It is almost never found in macular disease unless there is extensive damage to the entire Detecting motion is the least sensitive method retina. The patient is best examined in a of evaluating visual fields. dimly lit room and is focusing at a distant object. The light should be bright and shined 1. In the asymptomatic patient, the visual field from below the level of the eyes to prevent it examination remains an essential part of a routine from being a near stimulus. The light should neurologic examination. It would be both tragic be moved quickly from eye to eye several and embarrassing if the patient has an asymptom- times after holding the light on the pupil for atic brain tumor, usually benign, or a stroke that about 2 s. An obviously positive test is pres- affected the visual pathway despite a primary ent when one pupil dilates. A test is consid- complaint, for instance, of arm pain. A quick ered positive if there is a reduced amplitude screen with double simultaneous motion is the of constriction and a more rapid dilation least sensitive first step. Adding rapid finger- (escape). It may also be considered positive counting in four quadrants with both eyes open is when the normal eye constricts briskly when sufficient in an­asymptomatic patient when there the light is moved from the abnormal eye, is no history or evidence of cerebral disease. 48 4 Neurologic Examination

Fig. 4.3 (a–f) Retinal or optic nerve lesions. (a) Cecocentral (m) Homonymous inferior quadrantanopsia, right side. scotoma. (b) Central scotoma. (c) Arcuate scotoma. Left parietal or occipital lobe lesion. Occipital pathology is (d) Paracentral scotoma. (e) Inferonasal quadrant defect. much more common. (n) Homonymous macular sparing (f) Altitudinal defect. (g) Optic chiasm lesion (a junctional hemianopsia, left side. Optic radiation or occipital lobe defect). An optic nerve lesion on the left side just anterior to lesion. (o) Bilateral inferior altitudinal hemianopsia. the chiasm affecting crossing nasal fibers which receive Bilateral occipital (superior), bilateral optic nerve and, least visual information from the right temporal field. common, optic chiasm mass lesions. (p) Bilateral superior (h) and central scotomas. Optic altitudinal hemianopsia. Bilateral occipital (inferior), bilat- chiasm lesion. (i) Bitemporal hemianopsia and bilateral eral optic nerve and, least common, optic chiasm mass paracentral scotomas. Optic chiasm lesion. (j) Incongruous lesions. (q) Homonymous hemianopsia, splitting of fixa- homonymous hemianopsia. Left optic tract lesion. tion, right side. Optic radiation. (r) Temporal crescent (k) Homonymous scotomata, right side, due to a left defect, right side. Occipital lobe lesion, anterior calcarine occipital or optic tract lesion. (l) Homonymous superior cortex lesion. (s) Homonymous hemianopsia, right side, quadrantanopsia, right side. Left temporal or occipital lobe with sparing of the temporal crescent. Left occipital lobe lesion. Occipital pathology is much more common. lesion with sparing of the anterior calcarine cortex Cranial Nerve Examination 49

Fig. 4.3 (continued)

The patient may respond to two fingers in one Visual fields at the bedside can be examined quadrant and one finger in a contralateral quad- by motion, rapid finger-counting, object rant as four fingers when one side is not clearly identification, and color, e.g., red matches. seen. It should be understood that even a func- tionally blind person can pick up motion but not 2. The symptomatic patient. Each eye is examined form. This is called Riddoch’s phenomenon and individually with both double simultaneous thus the additional test of finger-counting is stimulation using motion and rapid finger- valuable. counting in two quadrants such as inferior Optional tests include using different objects, nasal and superior temporal. e.g., a coin in the superior temporal quadrant OS The completion phenomenon must be kept in and a key in the superior nasal quadrant OS. This mind. Many patients will see only one side mov- test has the advantage of eliminating the comple- ing, but assume that the other side also moves. tion phenomenon as an interfering factor. Rapid 50 4 Neurologic Examination double simultaneous exposure to two bright red Homonymous, which means both eyes matches flashed in one nasal and one temporal involved, hemianopsias are caused by lesions quadrant of one eye may elicit an extinction affecting the visual pathways behind the chiasm. phenomenon, perception of only one red-tipped For instance, a left optic tract, lateral geniculate, match. A third and perhaps most useful subtle test optic radiation, or occipital lobe lesion will pro- is color comparison. For example, the examiner duce a partial or complete right homonymous can hold one hand in each field or quadrant and hemianopsia. request the patient to state which hand is brighter (i.e., the normal side). Homonymous hemianopsias are due to retro- Finally, the time of the stimulus exposure chiasmal lesions. can be critical in eliciting a visual field defect. Extinction, the obliteration of a percept, may There are two types of homonymous hemian- occur when the stimulus is held up in a flash fash- opsias, congruous and incongruous. Congruous ion. The longer the stimulus is kept in view, the hemianopsias have the same defect in both eyes, less likely the possibility of discovering a visual whereas incongruous hemianopsias are different field defect. The exception, of course, is color in each eye. Examples are noted in the figures. comparison. Lesions of the optic tract are often incongruous. Lesions of the optic chiasm are most often Thus, the presence of an incongruous homony- associated with neoplasm, especially of pituitary mous hemianopsia raises the serious question of origin. The earliest sign is ordinarily a central a mass lesion, neoplasm or aneurysm, which are scotoma. Macular fibers are most dense in the often located adjacent to the optic tract. Ischemic central and posterior region of the chiasm and stroke and multiple sclerosis are uncommon eti- constitute a large portion of the structure, thus ologies. Lesions of the lateral geniculate nucleus explaining this early sign. A typical pattern of are rare, but may also cause incongruous hom- visual loss is the junctional scotoma, a central, onymous hemianopsias. paracentral, or cecocentral scotoma in one eye, and a superior temporal field defect in the other Congruous hemianopsias have the same defect eye. This is due to a lesion, ordinarily a neoplasm, in both eyes, whereas incongruous hemianop- which involves one optic nerve close to the chi- sias are different in each eye. asm and the inferonasal fibers from the contralat- eral eye as they loop a short distance anteriorly (Wilbrand’s knee) before they enter the chiasm. The principle of unilateral visual loss being The classic bitemporal hemianopsia is typically a caused only by retinal, optic nerve and, infre- later development. Inferior fibers of the chiasm quently, optic chiasm disease is violated by serve the superior fields. Thus, bitemporal supe- the temporal crescent defect. rior homonymous quadrantanopsias usually pre- cede a complete bitemporal hemianopsia when Patients with postchiasmal lesions usually there is a pituitary neoplasm. The reverse is the complain of loss of vision rather than darkness. case, for example, with a craniopharyngioma The defect is often perceived in only one eye, which compresses the chiasm from above. ordinarily the eye with the temporal defect which is nearly always larger than the nasal defect. The earliest sign of an optic chiasm lesion is Careful examination can usually uncover the commonly a central scotoma. nasal defect. The principle of unilateral visual loss being caused only by retinal, optic nerve and, infrequently, optic chiasm disease is violated The classic bitemporal hemianopsia is typically by the temporal crescent defect. This is due to an a later development with optic chiasm lesions. anterior, medial occipital lobe lesion. This loca- tion explains why the temporal field is larger than the nasal field. Homonymous hemianopsias with Cranial Nerve Examination 51 sparing of the temporal crescent occur when the diopulmonary resuscitation by a physician who anterior medial occipital lobe is spared. happened to be shopping in the next aisle. In the Homonymous hemianopsias or quadrantanop- Emergency Department, an EKG disclosed ST sias with or without macular sparing occur with wave abnormalities and he was admitted to the lesions behind the optic chiasm. Lesions of the Coronary Care Unit with a diagnosis of an acute optic radiations in the temporal lobe produce myocardial infarction and hypoxic-ischemic “pie-in-the-sky,” superior contralateral quadran- encephalopathy. A stat CT (head) scan was nor- tanopsias. Lesions of the optic radiations in the mal. The neurologic consultant found an inter- parietal lobe produce “pie-on-the-floor,” inferior mittently agitated patient who responded to contralateral quadrantanopsias. painful stimuli by pushing the examiner’s hand away. He had a positive Brudzinski sign and Occipital lobe lesions produce a large variety bilateral Babinski signs. A funduscopic examina- of visual field defects. tion revealed bilateral blurring of the optic disk margins, splinter hemorrhages at the margins, Occipital lobe lesions produce a large variety and a subhyaloid hemorrhage O.D. (see Fig. 4.8). of visual field defects. These are listed in the adja- A lumbar puncture yielded bloody spinal fluid. cent table. When there is an abrupt onset of bilat- The patient steadily improved over 4 days return- eral superior or inferior altitudinal hemianopsia, ing to a normal mental status. Angiography dem- vascular disease in the basilar artery distribution onstrated a posterior communicating aneurysm producing infarctions of the inferior or superior which was coiled successfully. banks of the calcarine fissure, respectively, is the usual etiology. Naturally, if the visual loss is stag- Lessons: gered, one eye alone at first followed by the sec- 1. Electrocardiograms may show abnormal ST ond eye, bilateral optic neuropathies often of segments after a subarachnoid hemorrhage. ischemic origin may be present. An infrequent This is probably due to an outpouring of nor- etiology would be a large prechiasmal mass lesion. epinephrine which causes contraction band Cortical blindness and Anton’s syndrome (denial necrosis. of blindness) may occur when there are bilateral 2. This patient had papilledema and a subhyaloid occipital lobe lesions. Macular sparing is most hemorrhage, the latter finding virtually diag- often present with occipital lobe lesions. nostic of a subarachnoid hemorrhage. Explanations include dual vascular supply to the 3. In this case, only a thorough neurologic exam- occipital pole by branches of either middle or pos- ination unmasked the diagnosis. terior cerebral arteries as well as the intermixture See Figs. 4.4 through 4.8 for a look at a few of fibers from retinal ganglion cells projecting to important funduscopic abnormalities. both ipsilateral and contralateral occipital poles. See Tables 4.3 through 4.6 for reviews of reti- nal, macular, and optic nerve diseases. Ophthalmoscopic Findings and Visual Field Defects with Lesions of the Visual 3rd, 4th, and 6th Cranial Nerves System Effective use of an ophthalmoscope is a declining Eyelids: A Critical Initial Observation art. But it should be a routine part of any neuro- logic consultation even if there is a low yield of When ophthalmologic disease is absent, obvi- diagnostic information. The case described below ous asymmetric palpebral fissures have just is an example of its potential diagnostic role in an four etiologies. unexpected clinical scenario. Case 1: A 52-year-old man is brought to the Are there asymmetric palpebral fissures, i.e., the Emergency Department after a cardiorespiratory distance between the upper and lower lids? arrest in a supermarket. He receives prompt car- Assuming the absence of normal variation 52 4 Neurologic Examination

Fig. 4.4 Papilledema. There are blurred disk margins and hemorrhages adjacent to the disk Fig. 4.7 . Bright yellow cholesterol embolus lodged at a bifurcation of the superior retinal artery

Fig. 4.5 Central retinal artery occlusion and boxcarring of blood column in an inferior retinal artery Fig. 4.8 Subhyaloid (preretinal) hemorrhage. This is located under the preretinal membrane and above the nerve fiber layer. This is a good marker for a subarachnoid hemorrhage in the right clinical setting

(a judgment call), sagging ­eyelids in the elderly and ophthalmologic diseases or eye surgery which affect the eyelids, there are primarily four neurologic conditions that cause asymmetric palpebral fissures. These are diagnosed by ana- tomic criteria. 1. Ptosis due to a Horner’s syndrome associated with sympathetic nervous system pathology causing weakness of Muller’s muscle. 2. Ptosis due to a 3rd nucleus or nerve lesion

Fig. 4.6 . Blurred disk margins and hemor- which causes weakness of levator palpebrae rhage adjacent to the inferior portion of the optic disk superioris (LPS) Cranial Nerve Examination 53

Table 4.3 Ophthalmoscopic findings, visual field defects, and pupils with lesions of the Anatomic Funduscopic findings Character of visual impairment Pupils location Retinal disease Retinal hemorrhage, Central scotomas, paracentral, cecocentral Normal retinal edema and arcuate scotomas (nerve fiber bundle defects), ring scotomas (fusion of superior and inferior arcuate defects), altitudinal defects and wedge-shaped defects with the point of origin at the blind spot Optic nerve Normal appearance Central, paracentral, cecocentral scotomas, Afferent pupillary defect disease or there may be disk arcuate scotomas (nerve fiber bundle (Marcus-Gunn pupil) edema, disk atrophy, defects), ring scotomas (fusion of superior flame, or splinter and inferior arcuate defects), altitudinal and hemorrhage at disk quadrantic defects margins Optic chiasm Normal appearance Junctional scotomas, bitemporal hemianop- Normal except afferent or nerve fiber layer sias, or quadrantanopsias with or without pupillary defect when atrophy macular splitting, central scotomas, one optic nerve is bitemporal scotomas with intact peripheral preferentially involved fields Optic tract Normal appearance Contralateral homonymous hemianopsia, Afferent pupillary defect or bilateral segmental particularly incongruous, often macular is common atrophy splitting. Homonymous scotomata Lateral geniculate Normal or Homonymous hemianopsia, especially Normal nucleus bilateral atrophy incongruous and often with macular splitting (bow tie pallor) Optic radiations Parietal Normal Contralateral homonymous hemianopsia, Normal contralateral homonymous inferior quadrantanopsia (pie-on-the-floor), macular sparing or splitting Temporal Normal Contralateral homonymous hemianopsia, Normal contralateral homonymous superior quadrantanopsia (pie-in-the-sky), macular sparing or splitting Occipital lobe Normal Contralateral homonymous hemianopsia or Normal quadrantanopsia usually congruous and with macular sparing, temporal crescent defect (monocular), bilateral visual field defects such as checkerboard vision, bilateral altitudinal hemianopsias, homonymous scotomas, cortical blindness, denial of cortical blindness (Anton’s syndrome)

3. Ptosis due to LPS weakness associated with pected Horner’s syndrome should be examined pathology affecting the neuromuscular ­junction in a darkened room to augment the pupillary (myasthenia gravis) or muscle (ocular asymmetry. The diagnosis of a 3rd nerve or myopathy). nucleus lesion is supported by a larger, ipsilat- 4. Widened palpebral fissure due to weakness of eral, poorly reactive or nonreactive pupil, and/or orbicularis oculi. This is most often secondary extraocular muscle weakness involving only 3rd to neurogenic pathology, central or peripheral, nerve ­innervated muscles. Weakness of orbicu- affecting facial muscles. laris oculi is ­discerned at distance by observing a The diagnosis of Horner’s syndrome is slower eye blink on the involved side. Lesions secured by the simultaneous presence of miosis may be located in the contralateral cerebral with or without anhidrosis. Patients with a sus- hemisphere and corticobulbar pathway. This 54 4 Neurologic Examination Miscellaneous History of transient visual obscurations, unilateral or bilateral, lasting seconds Transient visual obscurations as Transient described above History of impaired color and light perception History of impaired color and light perception Fluorescein angiography used Fluorescein angiography from to differentiate papilledema Pupil Normal Normal unless there is secondary optic atrophy Afferent pupillary defect Afferent Afferent pupillary defect Afferent Normal Vision Normal except enlarged enlarged Normal except blind spots Constricted visual fields, blind spots enlarged Central, paracentral or cecocentral scotoma. Less frequently arcuate, altitudinal and quadrantic defects Central, paracentral or cecocentral scotomas. Less frequently arcuate, altitudinal and quadrantic defects Hyperopia common, normal vision with refraction Laterality Bilateral Bilateral Unilateral, rarely bilateral Unilateral, rarely bilateral Bilateral Fundus Blurred disk margins, absent Blurred disk margins, pulsations, frequent splinter venous or flame hemorrhages at disk disk hyperemia margins, Same as above, ± optic atrophy, ± optic atrophy, Same as above, and central cup is obliterated Normal appearance. Infrequently optic disk pallor with a preference for the temporal side Blurred disk margins, absent Blurred disk margins, pulsations, ± splinter or venous flame hemorrhages at disk margins Blurred disk margins, absent Blurred disk margins, pulsations, and no venous hemorrhage Differential diagnosis of optic nerve pathology diagnosis of optic nerve Differential Pathology Table 4.4 Papilledema Papilledema, chronic Papilledema, Retrobulbar neuritis Retrobulbar Optic neuritis Pseudopapilledema Cranial Nerve Examination 55

Table 4.5 Distinguishing optic nerve from macular disease Lesion location Fundus Laterality Vision Pupil Miscellaneous Macula Edema, hemorrhage, Unilateral Central scotomas, often Normal Abnormal or other pathology or bilateral positive (dark). Decreased photostress test, in the macula visual acuity metamorphopsia, micropsia, and photopsia Optic nerve Optic nerve pallor, Unilateral, Central, paracentral Afferent Normal edema, and rarely bilateral or cecocentral scotomas, pupillary photostress test. hemorrhage at disk arcuate and ring scotomas, defect Abnormal flight margins or normal altitudinal and quadrantic of colors defects. Decreased visual acuity

Table 4.6 Differential diagnosis of optic nerve and retinal pathology of vascular origin Pathology Fundus Laterality Vision Pupil Miscellaneous Central retinal Optic disk pallor, Unilateral Monocular, altitudinal Usually normal Commonly of artery disease arteriolar attenua- and quadrantic unless there is embolic origin, tion, segmented defects, blindness, an extensive occasionally appearance of decreased visual infarction which associated with blood in arterioles acuity rarely results in temporal (boxcar effect) an afferent arteritis or other pupillary defect vasculitides Anterior ischemic Pallid edema with Unilateral The most common Afferent Small cup-to- optic neuropathy hemorrhages at findings are altitudinal pupillary defect disk ratio (less disk margins and quadrantic visual than 0.2). field loss. Decreased Painless, sudden visual acuity, central, loss of vision paracentral, or cecocentral scotomas may occur Posterior ischemic Normal Unilateral Same as above Afferent Same as above optic neuropathy pupillary defect Central retinal Engorged veins, Unilateral Variable loss, more Normal Good prognosis vein occlusion numerous often mild and with hemorrhages distal peripheral scotomas to occlusion, ± disk edema Venous stasis Small dot and blot Unilateral Normal but frequently Normal Associated with hemorrhages in the associated with severe occlusive middle portion of episodes of amaurosis internal carotid the retina and fugax artery disease microaneurysms

pathway descends down the contralateral inter- nuclear ptosis due to lesions of the contralateral nal capsule, contralateral midbrain until the cerebral hemisphere. fibers cross in the upper pons to synapse in the ipsilateral facial nucleus, and thence to the ipsi- Ptosis may occur after Bell’s palsy due to lateral facial nerve. aberrant reinnervation. Ptosis may occur after Bell’s palsy due to aberrant reinnervation. A rare disorder is supra- 56 4 Neurologic Examination

Eyelid retraction is present when is seen Actions of with the Eye between the (located over the ) and in Primary Position (See Table 4.7) the lid margin. Eyelid retraction and lid lag on One oculomotor nucleus supplies the innervation downgaze suggest thyroid due to of the ipsilateral inferior rectus (IR), medial ­rectus pathologic shortening of the LPS muscle associ- (MR), inferior oblique (IO), and the contralateral ated with inflammation and fibrosis. Dorsal superior rectus (SR) muscle. The nerve fibers des- supranuclear mesencephalic lesions involving the tined to supply the SR decussate at the level of the nucleus of the posterior commissure cause eyelid nucleus. There is a single midline nucleus that retraction on direct forward gaze and upgaze but supplies both levator palpebrae superioris (LPS) not downgaze. muscles. The oculomotor nucleus is in the mid- brain and lies ventral to the Sylvian aqueduct. The Apraxias of eyelid opening and closing may axons cross the red nuclei and exit the midbrain just occur especially with nondominant hemi- medial to the cerebral peduncle. sphere lesions and extrapyramidal disorders. One oculomotor nucleus supplies the innerva- Apraxias of eyelid opening and closing may tion of inferior rectus, medial rectus, inferior occur especially with nondominant hemisphere oblique, and the contralateral superior rectus lesions and with Parkinson’s disease and Parkinson- muscles. plus syndromes. For instance, the patient may be requested to close his eyes and keep them closed, The oculomotor nerve passes between the supe- but does so only for 1 or 2 s in extreme cases. This rior cerebellar and posterior cerebral arteries, runs has also been termed motor impersistence in near the medial aspect of the uncus of the temporal which case it should be evident on other requests lobe, and enters the cavernous sinus. The nerve such as “raise your arm and maintain the posi- separates into the superior division and inferior divi- tion” which the patient will not do. sion in the anterior portion of the cavernous sinus as Blepharospasm is an involuntary, symmetri- each passes through the superior orbital fissure. The cal, and strong contraction of the orbicularis superior division supplies the LPS and the SR mus- oculi. It is most often seen as an isolated phenom- cles. The inferior division innervates the IR, MR, IO enon and considered a focal dystonia. Otherwise, muscles, and the sphincter pupillae. it may accompany other extrapyramidal diseases, The trochlear nucleus provides fibers for the particularly Parkinson’s disease. contralateral 4th nerve as the fibers completely decussate in the anterior medullary velum located Extraocular Muscles (See Fig. 4.9) in the dorsal region of the midbrain. The 4th A brief review of pertinent anatomy should be nerve is the only completely crossed cranial helpful. The organization of the nuclei, oculomo- nerve. The fibers emerge from the dorsal mid- tor, trochlear, and abducens is complex, the brain near the midline, travel through the adja- nerves less so. cent cisterns, enter the cavernous sinus, pass

Table 4.7 Actions of extraocular muscles with the eye in primary position Muscle Primary action Secondary action Lateral rectus Abduction – Medial rectus Adduction – Superior rectus Elevation Intorsion Inferior rectus Depression Extorsion Superior oblique Intorsion Depression Fig. 4.9 Eye muscle function Inferior oblique Extorsion Elevation Cranial Nerve Examination 57

Fig. 4.10 Bielschowsky test for a left trochlear (4th) nerve lesion. The diplopia increases with head tilt to the left which is ipsilateral to the 4th nerve lesion. The left eye is hypertropic in the neutral position through the superior orbital fissure, and innervate Isolated 4th nerve palsies are not rare. Hence, the superior oblique (SO) muscle. the SO muscles may be the only extraocular mus- cle involved when a patient complains of diplo- The 4th nerve is the only completely crossed pia. Its function depends on the position of the cranial nerve. eye in the . When there is a total 3rd nerve palsy, the involved eye is deviated laterally and The 6th nucleus supplies fibers for the ipsilat- usually down. When the patient is asked to look eral 6th nerve which innervates the lateral rectus. down, the eye intorts. If this does not occur, the The 6th nerve also travels through the cavernous patient has a combined 3rd and 4th nerve lesion sinus and enters the orbit through the superior indicating pathology in the cavernous sinus where orbital fissure. There are interneurons in the 6th the nerves are adjacent to each other. When the nucleus which supply fibers for the contralateral eyes are in midposition, 4th nerve function is a medial longitudinal fasciculus (MLF). mixture of intorsion and lowering. When the eye The examination begins with asking the is adducted, the SO lowers the eye without any patient to look up, down, right, and left. This is rotation. A lesion of the 4th nerve usually results followed by a request to look up and down when in an upward deviation () in primary the eyes are deviated right and left. When the eye position presumably due to unopposed action of is adducted, the oblique muscles elevate (IO) and the IO muscle. The head tilts away from the side lower (SO) the eye. When the eye is abducted, of the 4th nerve lesion to reduce the diplopia. The the SR and IR are active. Therefore, on left Bielschowsky head-tilt test involves tilting the upward gaze, the IO and MR are active OD and head toward the side of the weak SO which the SR and LR are the prime movers OS. increases the separation of the images (see Fig. 4.10). 4th nerve function depends on the position of the eye in the orbit. A mass lesion compressing the 3rd nerve usu- ally affects pupillary fibers because they are superficial and on the dorsal surface of the nerve. 58 4 Neurologic Examination

An isolated lesion of one 3rd nerve nucleus, upgaze, the color of the light coming from the although rare, provides a unique opportunity to uppermost image arises from the weak muscle. understand the anatomic organization. The Specifically, if there is a left superior rectus mus- ­consequences are a complete ipsilateral 3rd nerve cle weakness and the left eye is covered by the palsy and contralateral SR weakness and ptosis. red filter, the red light is higher. If the patient has A mass lesion compressing the 3rd nerve usually a left lateral rectus weakness, diplopia is elicited affects pupillary fibers first since they are superfi- on left lateral gaze. If the red filter is placed over cial and on the dorsal surface of the nerve. A 3rd the right eye (MR function), the farthest image to nerve lesion sparing the pupil is most common the left is white which arises from the left eye with an ischemic infarction affecting the central indicating a left lateral rectus paresis. portion of the nerve such as occurs with the small vessel disease of diabetes. Rarely, an internal With the red glass test the distal image comes carotid artery aneurysm within the cavernous from the weak muscle. sinus is the etiology. A lesion of the 6th nucleus affects both the To summarize, on right lateral gaze the color neurons of the 6th nerve and interneurons which of the image (white or red) farthest to the right send fibers to the contralateral MLF. There are a designates the weak muscle. On downgaze, the few case reports which describe conjugate gaze color of the lowest image (white or red) desig- paresis with such lesions. nates the weak muscle. Thus, the test can be applied to assess any muscle by requesting the Gaze palsies due to brainstem pathology are patient to look in the direction of the function of caused by lesions of the paramedian pontine that eye muscle. reticular formation (PPRF) and rarely 6th nucleus lesions. Saccades and Pursuit Testing both saccades and pursuit may seem oner- For the most part, however, gaze palsies due ous or excessive, but it can be easily accomplished to brainstem pathology are caused by lesions of in 20–30 s. Testing saccades is commonly omitted, the paramedian pontine reticular formation but useful information is often obtained in patients (PPRF). A 6th nerve lesion which results in ipsi- with central nervous system (CNS) pathology. lateral lateral rectus weakness is often accompa- nied by a head turn to the side of the lesion. Saccades are examined by requesting the Additionally, horizontal diplopia at distance is a patient to look quickly from one stationary common symptom and sign of 6th nerve palsies point to another. since the eyes must diverge at distance. Lateral rectus weakness due to a 6th nerve lesion is a Saccades are examined by requesting the common early sign of increased intracranial patient to look quickly from one stationary point ­pressure due to its ascending pathway along the to another. The test may elicit evidence of a ­clivus which makes it vulnerable to any move- ­specific ocular muscle paresis. For example, from ment of the brainstem occurring due to mass or central fixation to far right lateral fixation, the pressure effect. test may uncover a right LR or left MR weakness. Weakness of only the left MR muscle may indi- Red glass test for diplopia cate pathology involving the muscle, neuromus- When the patient complains of diplopia or there cular junction, 3rd nerve, 3rd nucleus, or MLF. appears to be an ocular paresis, the red glass test Weakness of only the right LR may indicate can identify the weak muscle. A translucent red pathology involving the muscle, neuromuscular filter is placed over one eye. The light is shined junction, 6th nerve, or 6th nucleus. directly at the pupil and the patient focuses on the light. If, for example, the patient has diplopia on Saccades from either a lateral or vertical fixa- tion point to central fixation may be inaccu- rate and disclose significant pathology. Cranial Nerve Examination 59

Saccades from either a lateral or vertical fixa- The essential element of an INO is an ipsilat- tion point to central fixation may be inaccurate. eral adduction defect. This is often an undershoot, a hypometric sac- cade, followed by a small corrective saccade in 1. Horizontal dysconjugate gaze. This is typified the same direction. This is a normal phenomenon. by the syndrome of the MLF or internuclear The eyes may overshoot, a hypermetric saccade, ophthalmoplegia (INO). As previously out- then slide back, a glissade, to the central point of lined, the pathway connects the PPRF with the fixation. This is called ocular . The eyes contralateral oculomotor nucleus via interven- may also return to central fixation with another ing synapses in the ipsilateral abducens saccade. Both phenomena, ocular dysmetria and nucleus before crossing the midline. It is pri- a corrective saccade in the opposite direction to marily a quick system pathway and thus best reach the target, are indicative of cerebellar sys- elicited by testing with saccades. The main tem pathology. finding is ipsilateral paresis of adduction or an The saccade velocity to right and left and up adductor lag, a slow saccade of the medial rec- and down should be roughly compared. If, for tus as compared to the lateral rectus. A second instance, saccades are slower or incomplete to element is nystagmus in the contralateral the right, there must be a lesion in the oculomotor abducting eye. The etiology of this component pathway that generates eye movements to the is speculative, but it depends on the detectable right. This would be the left frontoparietal eye presence of ipsilateral MR paresis. A third and fields, left anterior limb of internal capsule or variable feature is preservation of conver- thalamus, left mesencephalic reticular formation, gence. When absent, it implies a lesion in the and right PPRF. This is the most clinically appli- midbrain near the termination of the MLF. cable of several pathways which also involve 2. Bilateral INO. This is associated with abnor- basal ganglia and superior colliculus. malities in the vertical plane, especially down- Pursuit is examined by requesting the patient beat and/or upbeat nystagmus. Occasionally, to follow the examiner’s slowly moving finger in this is unilateral with contralateral rotatory all directions of gaze. This is the usual method of nystagmus. See-saw nystagmus may occur. examination and uncovers any obvious ocular Vestibular fibers in the MLF originating from paresis. Nystagmus may be elicited at the termi- the posterior semicircular canal may explain nation point. If there is a defect of the pursuit the torsional elements. Skew deviation, a vertical system, the patient’s eyes may not be able to keep separation of the eyes, can be present with up with a moving target and saccades may be the higher eye ipsilateral to the lesion. The substituted to catch up with it. This “saccadic WEBINO syndrome, wall-eyed bilateral inter- pursuit” is especially common in patients with nuclear ophthalmoplegia, is self-explanatory and extrapyramidal disorders such as Parkinson’s has been well described. The “one-and-a-half” disease, but has no precise localizing significance. syndrome indicates involvement of the PPRF It is not unusual in the normal elderly population. and MLF. For example, a left-sided lesion results Saccadic pursuit occurs when the patient is in an ipsilateral gaze paresis (PPRF) and ipsilat- unable to keep up with a moving target using eral MR paresis (MLF). Thus the ipsilateral eye the pursuit system and saccades are has no horizontal movement. A common asso- substituted. ciated finding is an ipsilateral facial weakness.

Dysconjugate Gaze and Abnormal The “one-and-a-half” syndrome indicates Eye Positions involvement of both PPRF and MLF.

3. Vertical dysconjugate eye movements are The MLF is primarily a quick system pathway; infrequent. The vertical one-and-a-half thus, it is best examined for with saccades. ­syndrome is manifested by upgaze paresis and 60 4 Neurologic Examination

monocular paresis of downgaze. This occurs 2. Upgaze paresis is usually due to a lesion of the with thalamo-mesencephalic infarctions. posterior commissure. Monocular elevation paresis occurs with con- 3. Downgaze paresis is usually due to a lesion of tralateral pretectal lesions which interrupt path- the rostral interstitial nucleus of the medial ways from the rostral interstitial nucleus of the longitudinal fasciculus (riMLF). This is a MLF (riMLF) to the SR and IO subnuclei. structure separate from the MLF.

Skew deviation is a vertical misalignment Eye Deviations of the eyes which occurs with brainstem, 1. Eye deviation left, for example, indicates a 8th nerve, and neuromuscular junction lesion above the oculomotor decussation on lesions. the left or a right PPRF lesion.

4. Skew deviation, a vertical misalignment, has Downward eye deviation occurs with thal- previously been attributed to CNS lesions amic hemorrhage, metabolic, and hypoxic- only, such as the MLF. It may be present, how- ischemic encephalopathies. ever, with peripheral vestibular disease and certainly in some patients with ocular myas- 2. Downward eye deviation is less specific. It thenia gravis. It occurs in CNS lesions when occurs with thalamic hemorrhage, metabolic, otolith connections are interrupted. Peripheral and hypoxic-ischemic encephalopathies. vestibular disease may produce a contralateral 3. Upward eye deviation, if persistent, occurs hypertropia as opposed to the ipsilateral hyper- almost always with hypoxic-ischemic enceph- tropia with an MLF lesion. alopathy. Oculogyric crises associated with 5. The ocular tilt reaction is manifested by an neuroleptics is rarely the etiology. ipsilateral head tilt, conjugate eye torsion, and hypotropia. Lesions producing this triad Ocular Oscillations: Nystagmus have been discovered in the ipsilateral laby- rinth, 8th nerve, vestibular nucleus, and con- Nystagmus is best observed by looking at the tralateral meso-diencephalon involving the blood vessels on the sclera since rotatory (tor- interstitial nucleus of Cajal. This reaction sional) elements are often otherwise missed. may be persistent (tonic) or intermittent (phasic). 1. Examination. Nystagmus is usually of the “jerk” type. There is a quick phase analo- Gaze Paresis gous to a saccade and a slow phase analo- Horizontal gaze paresis occurs with lesions of gous to pursuit. The quick phase determines the oculomotor pathways for horizontal gaze. its direction. Nystagmus is best observed by looking at the blood vessels on the sclera 1. Horizontal gaze paresis occurs with lesions of since rotatory (torsional) elements are often the oculomotor pathways for horizontal gaze. otherwise missed. A comment such as “There Briefly, gaze paresis to the right (gaze prefer- is nystagmus on upgaze” has no meaning ence left) indicates a lesion above the oculo- without a notation of the quick phase. motor decussation on the left or below it in the Lesions of the central or peripheral nervous right PPRF. The oculomotor decussation is system (PNS) can provoke some form of located at the pontomesencephalic junction. nystagmus in any direction of gaze. Conversely, a note which states “There is upward beating nystagmus on upgaze” indi- Upgaze paresis is usually due to a lesion of cates brainstem pathology or toxic effect of the posterior commissure. Downgaze pare- drugs. Infrequently, low amplitude nystagmus sis is associated with riMLF lesions. is discovered on funduscopic examination Cranial Nerve Examination 61

and not with unaided visual inspection. The eye rolling behind lightly closed lids. Proof of retinal movement is opposite to the move- nystagmus behind closed lids can be obtained ment of the globes and thus upbeat retinal through electronystagmography, which uses nystagmus is observed as downbeat nystag- the corneoretinal potential to record eye move- mus. Only an astute examiner identifies this ments or at the bedside using Frenzel glasses. phenomenon which can be dismissed as These are glasses which have thick lenses poor cooperation. Pendular nystagmus is which prevent visual fixation and magnify the manifested by rapid horizontal eye move- eyes allowing for more accurate observation ments of equal velocity in both directions. by the examiner. Occasionally, there is a mixed jerk and pen- dular nystagmus. This type of nystagmus is Horizontal gaze-evoked nystagmus indicates most often congenital in origin. Acquired drug effect or a brainstem/cerebellar lesion. forms have been described. 2. Horizontal nystagmus of vestibular origin. 3. Gaze-evoked nystagmus. This is nystagmus This is due to lesions of the peripheral vestibu- provoked by looking up, down, right, or left. lar apparatus, 8th nerve, or vestibular nuclei in Horizontal gaze-evoked nystagmus means the brainstem. The nystagmus is usually pres- nystagmus to the right on right lateral gaze ent on direct forward gaze and increases in and nystagmus to the left on left lateral gaze. intensity when the eyes are deviated in the It is the most common form of nystagmus direction of the quick phase (Alexander’s law). encountered. The most likely etiology is drug It is horizontal or combined horizontal rota- effect, sedatives, and especially anticonvul- tory. The rotation is clockwise when the quick sants. Brainstem/cerebellar lesions may cause phase is to the left and counterclockwise when gaze-evoked nystagmus. This is not seen with it is to the right. When the nystagmus is direc- peripheral vestibular disease. Gaze-evoked tion-fixed such as always to the right whether nystagmus may be upbeat which is either due the eyes are deviated right, up, down, or left, to toxic effect of drugs or brainstem/cerebellar peripheral vestibular origin is suspected. system pathology. Downbeat gaze-evoked Clearly, the absence of brainstem/cerebellar nystagmus is rare and due to brainstem lesions. system symptoms and signs is required before 4. Specific types of nystagmus. a peripheral etiology can be diagnosed. The • Endpoint nystagmus. This is a physiologi- physiology of the vestibular system will be cal form of nystagmus if the eyes are discussed under the section on the “8th Cranial ­deviated more than 30° to either side. It is Nerve.” occasionally sustained when the eyes are deviated more than 40° to either side. Direction-fixed nystagmus to the contralat- • Torsional (pure rotatory) nystagmus. This eral side is a common finding with periph- is seen with brainstem disease, especially eral vestibular disease. the medulla, and has been described with Wallenberg’s syndrome. This syndrome is most commonly seen with vertebral artery Since visual fixation inhibits nystagmus of disease resulting in ischemia in the poste- vestibular origin, vertigo may be present rior inferior cerebellar artery distribution. without visible abnormalities, an indica- tion for electronystagmography. An INO commonly causes dissociated nystagmus. Visual fixation suppresses nystagmus of peripheral origin. Consequently, with eyes closed the patient is often more symptomatic. • Dissociated nystagmus. Internuclear oph- Nystagmus may sometimes be perceived as thalmoplegia is the best representative of 62 4 Neurologic Examination

this form of nystagmus. A lesion of the The lesion is most common at the cervical- left MLF, for instance, commonly results medullary junction and often involves the in an ipsilateral paresis of adduction (left cerebellar nodulus and flocculus. The most medial rectus weakness) and horizontal common causes are Chiari I malformation, jerk nystagmus to the right of the contral- multiple sclerosis, and spinocerebellar ateral eye. Between ages 15 and 50, it degeneration. nearly always points to multiple sclerosis. • Upbeat nystagmus. This is a primary posi- In childhood, a pontine glioma, and over tion nystagmus with quick phase beating age 50, a brainstem infarction due to basi- upward and more intense on upgaze. lar artery disease are primary diagnostic Brainstem lesions, especially involving the considerations. perihypoglossal nuclei in the medulla, have • Periodic alternating nystagmus. This is been discovered in some of these patients. horizontal jerk nystagmus that changes Stroke, multiple sclerosis, and toxic effect directions every 90 s after about a 10-s of drugs are the most common etiologies. interlude of stability. Thus, for example, • See-saw nystagmus. This is characterized there is horizontal jerk nystagmus to the by an intorsion of the elevating eye and right for 90 s, then 10 s of stability, and extorsion of the opposite descending eye. It then 90 s of horizontal nystagmus to the is most often pendular and has been left. The pattern persists indefinitely. This described with parasellar neoplasms and form of nystagmus is commonly unrecog- thalamic-midbrain lesions. There are con- nized since it requires prolonged observa- genital forms which have distinctive fea- tion to diagnose. The location of the tures; the intorting eye falls and the lesion is at the cervical-medullary junc- extorting eye rises, just opposite to the tion and specific etiologies to consider pathologic form described above. include Chiari I malformation and multi- • Bruns’ nystagmus. This type of nystagmus is ple sclerosis. often associated with neoplasm, especially schwannomas of the 8th nerve. There is rapid Periodic alternating nystagmus is due to contralateral nystagmus of small amplitude lesions at the cervical-medullary junction. and slower, larger amplitude ipsilateral nys- tagmus due to brainstem compression. • Convergence-retractory nystagmus. This nystagmus is provoked by upgaze or sliding Seizures produce horizontal nystagmus an optokinetic tape downwards. It is mani- with quick phase directed contralateral fested by adducting saccades and retraction to the epileptic focus. of the eyes into the orbit due to simultane- ous activation of all extraocular muscles. • Seizures. Seizures produce horizontal nys- Lesions are in the dorsal midbrain. tagmus with quick phase directed contral- Etiologies include pineal neoplasm, multi- ateral to the epileptic focus. The nystagmus ple sclerosis, and stroke. is ordinarily brief, seconds to 2 or 3 min, the usual duration of seizures. Downbeat nystagmus is associated with • Positional nystagmus. Direction-changing lesions of the cerebellar nodulus and positional nystagmus (DCPN) has two flocculus, located near the cervical- forms, geotropic and ageotropic. Geotropic medullary junction. indicates that, when lying supine with head turned to the right, the quick phase beats • Downbeat nystagmus. The quick phase right and with head turned to the left the beats downward in primary position, but it quick phase beats left. In other words, nys- is usually more prominent on lateral gaze. tagmus is always directed towards the Cranial Nerve Examination 63

ground when the head is turned. Ageotropic back to central gaze. Usually, the eye move- is quick phase to the left when the patient is ments are asymmetric due to different supine with head right and vice versa when degrees of weakness in the contracting mus- the head is turned left. Both are most often cles. Pseudointernuclear ophthalmoplegia is observed with peripheral vestibular dis- a well-known feature of myasthenia gravis. ease. Rarely, the ageotropic type has been 5. Guidelines. described with brainstem/cerebellar system pathology. There remains some contro- The direction of nystagmus is specified versy about the localizing value of these by the direction of the quick phase. forms of nystagmus. Benign paroxysmal positional vertigo • Always designate the direction of nystag- (BPPV) will be discussed below under spe- mus by the direction of the quick phase and cial tests. focus on the sclera for both detection of torsional elements and accuracy. Congenital nystagmus is usually binoc- • Evaluate all directions of gaze. ular, pendular, decreased by conver- • Nystagmus which has more than one direc- gence, increased by visual fixation, and tion is of either central origin or due to drug has a null point. effect. A rare exception is DCPN, just described. • Congenital nystagmus. This is typically a • Pure torsional (rotatory) nystagmus indicates pendular nystagmus, although there may brainstem/cerebellar system pathology. be jerk elements. It is binocular, ordinarily • Horizontal direction-fixed nystagmus horizontal, decreased by convergence and which may be present in some or all direc- increased by visual fixation. There is often tions of gaze is commonly of peripheral a null point, a specific eye position where vestibular origin. This includes lesions of the patient has no nystagmus. It is associ- the 8th nerve. ated with albinism and • Purely positional nystagmus is usually of (absence of color perception). peripheral origin, but there are important Latent nystagmus is binocular nystag- exceptions especially if there is no fatiga- mus present only when one eye is covered. bility. Posterior fossa mass lesions have It is most often discovered during the fun- rarely presented in this fashion. duscopic examination. Hence, it is com- monly dismissed as inability to focus or Special Tests noncooperation. • Voluntary nystagmus. This is actually rapid The Dix-Hallpike test is mandatory when back-to-back saccadic eye movements there is a complaint of dizziness or vertigo which can be induced at will and is likely associated with a change of position. to be of genetic origin. It is important to recognize as nonpathologic since it may be 1. The Dix-Hallpike test for BPPV. employed by malingerers. This test is mandatory when there is a com- plaint of dizziness or vertigo associated with a Myasthenic nystagmus occurs because change of position, sitting up, lying down, or paretic muscles fatigue and the eye turning over in bed. Rarely, elderly patients drifts back to central gaze. complain of only a sense of instability and thus warrant testing if there is no other obvi- • Myasthenic nystagmus. Paretic muscles ous pathology. The seated patient’s head is fatigue easily and hence there is a slow drift turned to the examiner and the patient is 64 4 Neurologic Examination

moved rapidly to the supine position with the This is a useful test for patients in the Intensive head extended backwards as shown in the dia- Care Unit with altered levels of consciousness gram. Involvement of the posterior semicircu- and will be discussed in the chapter on exam- lar canal is most common. This results in ining the poorly responsive patient. rotatory, upbeat, vertical nystagmus directed to the undermost ear after a latency of up to The most important ocular are 20 s which ordinarily dissipates within 30 s. ocular dysmetria, ocular flutter, and opso- Habituation occurs with repeat testing as the clonus. These are associated with cerebellar nystagmus becomes more difficult to elicit. system pathology. Mechanical treatment with the canalith repo- sitioning maneuver is usually curative (see Ocular Oscillations: Dyskinesias Fig. 10.1). Medical treatment is useless. These are saccadic system abnormalities associ- ated with cerebellar system dysfunction, less Induced nystagmus, optokinetic or caloric, often brainstem. may uncover a focal lesion. 1. Ocular dysmetria, discussed earlier, is the most common . It is examined by 2. Induced nystagmus. Optokinetic nystagmus. asking the patient to look from a lateral or ver- Using a striped or dotted cloth, the patient is tical point of fixation to a central point. There requested to focus on the stripes or dots as the is an overshoot with slow return (glissade) cloth is moved slowly from the patient’s left to which defines ocular dysmetria. right and vice versa. When the patient’s eyes 2. Ocular flutter is several back-to-back saccades follow the cloth movement to the right, a sac- without an intersaccadic interval in the hori- cade to the left is elicited to return the eyes to zontal plane when the eyes are following a central fixation. This is compared with the moving target. opposite direction of movement. Significant 3. Opsoclonus is defined by chaotic saccades with- asymmetries are noted and correlated with out an intersaccadic interval in all planes. Some other neurologic findings. Poor quick phases to etiologies are brainstem encephalitis, multiple the right might imply a lesion involving the left sclerosis, and paraneoplastic ­syndromes, espe- oculomotor pathway above the oculomotor cially associated with neuroblastoma. decussation, which is located at the pontomes- 4. Square wave jerks, macrosquare wave jerks, encephalic junction or an ipsilateral lesion and macrosaccadic oscillations are additional below this level in the pons (PPRF). ocular dyskinesias. Further discussion of these disorders is beyond the scope of this book. Caloric nystagmus requires raising the 5. Ocular-palatal myoclonus. These are rhythmic head 30° which brings the horizontal canal oscillations of the eyes combined with simulta- vertical enabling a maximum response neous oscillations of one or more of the follow- with cold water irrigation. ing structures, soft palate, tongue, mouth, larynx, and diaphragm. The frequency is 1–3 cps. The 3. Caloric nystagmus. The head is raised 30° to lesion location is in the Guillain-Mollaret tri- bring the horizontal canal vertical which elic- angle. This triangle includes the red nucleus, its a maximum response with cold water irri- inferior olivary nucleus, and the dentate nucleus gation. After an inspection showing an intact plus their connections. The abnormal neuronal tympanic membrane, the ear is irrigated with discharge is located in the inferior olivary cold water. This should elicit contralateral nucleus. Stroke is the most common cause. quick phases. Warm water irrigation produces ipsilateral quick phases, but the temperature Pupils: Observations of warm water must be precise and therefore, 1. Pupillary size. The normal pupil is usually for practical purposes, only cold water is used. between 2 and 7 mm. In infants, the pupils are Cranial Nerve Examination 65

small and gradually increase to normal adult , ex anopsia, and macu- size by about the age of 8. Otherwise, pupils lar disease do not impair pupillary reac- are larger in youth and smaller in the elderly. tions to light. Fluctuation of pupillary size is common and is called “pupillary play.” Large fluctuations are called “hippus.” And, although quite notable When the pupils are tested, the patient must on inspection, it has no definite pathologic fixate on a distant point at least 6 ft. away. association. Pupils constrict in sleep and with forced eye closure. 2. Pupillary shape and position should be noted. Technique is critical. The test is best per- The pupils may be oval, irregular and, in rare formed in a dimly lit room. The patient must instances, eccentric in position. fixate on a distant point at least 6 ft. away. It is best to shine the light from below the horizon- The pupillary reaction to both light and tal meridian which should prevent contamina- near are graded 1+ (slow) to 4+ (brisk). tion with a near response. Firstly, each eye is tested individually to grade the pupillary response, 1+ to 4+, and whether it fatigues. 3. The pupillary reaction to both light and near are Assuming a questionable difference or a pos- graded 1+ (slow) to 4+ (brisk). A slower reac- sible optic nerve lesion, a bright light is shined tion on one side could be due to impaired con- into the good eye for 3–5 s and quickly striction of the sphincter pupillae (3rd nerve) or switched to the suspected involved eye. This partial interruption of the afferent pathway may be repeated a few times. The first well- (optic nerve). Assuming there is a brisk reac- defined movement of the pupil is key to the tion to light in the opposite eye, a persistent diagnosis as there is often a small fluctuation sluggish consensual response indicates an in pupillary size in normal individuals. The impaired efferent pathway or 3rd nerve lesion. abnormal side will dilate. A brisk consensual reaction means a poor affer- ent pathway, thus an optic nerve lesion. 4. Afferent pupillary defect (Marcus-Gunn When an afferent pupillary defect is sus- pupil). This test is designed to diagnose optic pected, the primary question is: Which nerve disease. It may occur with optic chiasm response is better, direct or consensual? lesions since some involvement of the optic nerve is common; it is a rare phenomenon The primary question to answer is: Which with optic tract disease. Even though vision response is better, direct or consensual? A may be seriously impaired due to a dense cata- reverse afferent pupillary defect is another ract, amblyopia ex anopsia, and macular dis- approach. If one focuses on the suspected ease, the test remains valid since these abnormal eye, a consensual response may pro- disorders do not impair pupillary reactions to voke an obvious constriction when the light is light. Macular disease almost never produces switched to the other eye, thus securing the an afferent pupillary defect. If so, it would diagnosis of optic nerve disease. require obliteration of most of the retina. Optic 5. Light-near dissociation. The near is nerve disease with a visual acuity of 20/25 in composed of three elements – convergence, one eye and a cataract with 20/400 in the other , and miosis. If the pupillary eye may nevertheless produce an afferent light reflexes are 4+ bilaterally, there is no need pupillary defect in the eye with 20/25 vision. to check the near response since the only Furthermore, an optic nerve lesion causing known dissociation is a poor light response poor color perception yet having 20/20 visual with a good near reaction. The most reliable acuity may still yield an afferent pupillary method for checking the near response is to defect. ask the patient to follow his own finger as it 66 4 Neurologic Examination

moves towards his nose. Guiding the patient’s • Parasympathetic involvement. This is arm may be necessary. Severe visual loss or clearly present when the larger pupil reacts even blindness does not obviate the test since sluggishly to both direct and consensual patients can imagine the location of their light or not at all. The asymmetry is best finger. noted in a well-lit room since the sphincter pupillae is then preferentially stimulated Argyll-Robertson pupils are bilateral and and the normal side responds well. miotic, whereas Adie’s pupils are usually • Sympathetic involvement. The pupils are unilateral and large. briskly reactive. In a darkened room, after 15–20 s have elapsed, the asymmetry • Argyll-Robertson pupils. Features include increases since the dilator pupillae is maxi- minimal-to-absent light response and intact mally stimulated and the involved side near response. There does not need to be a functions poorly. totally absent response to light. The pupils • Central . This is usually less than are bilateral, miotic, irregular, and often 0.5 mm. Both pupils react normally and the asymmetric. Vision should be grossly nor- asymmetry is the same in light and dark. mal to make this diagnosis. • Adie’s syndrome (tonic pupil). These pupils Pupils are equal when there is amauro- are unilateral and large in bright light. They sis due to optic nerve disease. do become bilateral at a slow rate of 4% per year. There is a poor-to-absent light reac- 7. Amaurotic pupil. Unilateral amaurosis due to tion, slow constriction to prolonged near optic nerve disease results in equal pupillary response, and slow redilation. Vermiform size because of an intact consensual response movements of the iris are observed on slit- from light stimulation of the normal eye. A light lamp examination. There is denervation stimulus to the amaurotic eye yields no response hypersensitivity as only the involved pupil in the normal eye. An afferent pupillary defect reacts to dilute pilocarpine 0.125%. Ankle is obvious. Bilateral amaurosis due to optic reflexes are decreased or absent. The lesion nerve or chiasm involvement will produce is in the . light-near dissociation, as described above. • Midbrain pupils. These are midposition, irregular, nonreactive to light, and may Trigeminal Nerve have a good reaction to near. • Bilateral optic nerve or chiasm disease. 1. Background anatomy. When the afferent loop of the pupillary The trigeminal nerve has sensory and motor reflex has a prominent lesion, the near components. The motor nucleus is located in the reflex usually exceeds that of the light midpons and receives fibers from both cerebral response. hemispheres, but mainly contralateral. The decus- 6. Asymmetric pupillary size in the absence of sation is in the pons, just above the nucleus. The eye disease or prior eye surgery which could motor root (portio minor) joins the mandibular affect the response. division (V3). This root supplies the muscles of mastication; these are the masseter, temporalis, If there is no local ophthalmic pathol- medial, and lateral pterygoids. ogy, asymmetric pupils are due to a parasympathetic or sympathetic lesion The motor root of the trigeminal nerve joins when there is a difference in pupillary the mandibular division (V3) which supplies size of greater than 0.5 mm. the muscles of mastication. Cranial Nerve Examination 67

The sensory portion arises from three nuclear nose, lower cornea, hard and soft palate, upper complexes, the nucleus of the spinal tract of V, gums and teeth, maxillary sinus, and nasal mucus the main sensory nucleus, and the mesencephalic membrane. The mandibular division mediates nucleus. The spinal tract of V descends from the sensory information from the anterior two thirds pons down to C3–C4 of the cervical cord. Axons of the tongue (except taste), lower gums and from the tract terminate in the adjacent nucleus of teeth, chin, lower lip, tympanic membrane, upper the spinal tract of V. These fibers mediate mainly ear, and auditory meatus. It contains the motor pain and temperature. The fibers cross and ascend root which innervates the muscles of mastica- to the ventral posteromedial nucleus of the thala- tion. The muscles which close and clench the mus (VPM). jaw are the temporalis, masseter, and medial The main sensory nucleus is located in the pterygoids. The lateral pterygoids protrude and rostral part of the pons just lateral to the motor move the jaws laterally. Additionally, the trigem- nucleus. It mediates touch and proprioception. It inal nerve innervates the meninges and the arter- sends crossed and uncrossed fibers to the VPM ies of the circle of Willis. Thus, it mediates pain via the quintothalamic and trigeminothalamic due to meningitis, subarachnoid hemorrhage, pathways, respectively. and migraine. The mesencephalic nucleus is located in the caudal midbrain and mediates proprioception The trigeminal nerve innervates the meninges from the muscles of mastication. and the arteries of the circle of Willis. The sensory root and motor root expand to form the gasserian ganglion which sits in 2. Examination of the trigeminal nerve: Meckel’s cave. The ganglion gives rise to the V1 Pinprick sensibility is checked in all three (ophthalmic), V2 (maxillary), and V3 (mandibu- divisions. Additional testing of gums, buccal lar) divisions. mucosa, and tongue can be added if clinically rel- The trigeminal nerve passes through the cere- evant. Sensory loss may occasionally be in an bellopontine angle. The ophthalmic and maxil- “onion-skin” pattern since there is an extensive lary divisions travel through the cavernous sinus longitudinal distribution of the spinal tract of V. along with cranial nerves 3, 4, 6 and the internal Midline structures, nose, and mouth are repre- carotid artery which carries sympathetic fibers in sented in the rostral part of the nucleus and lateral its sheath. The sympathetic fibers merge with the fibers in the caudal part. ophthalmic division as it passes through the supe- rior orbital fissure. The maxillary division travels The corneal reflex is tested by using a wisp of a short distance in the inferolateral portion of the cotton to touch the cornea which is over the cavernous sinus before it exits the skull via the iris, not the sclera. foramen rotundum.

The trigeminal nerve passes through the cer- The corneal reflex (see Fig.4 .11) can be con- ebellopontine angle. The ophthalmic and sidered an optional test depending on the his- maxillary divisions (V1 and V2) travel through tory. A wisp of cotton can be used to touch the the cavernous sinus along with cranial nerves cornea over the iris. The sclera is insensitive. An 3, 4, and 6 and the internal carotid artery. absent reflex with good sensation indicates a weak orbicularis oculi. A good contralateral The ophthalmic division receives sensory blink is expected with a unilateral 7th nerve information from the upper lid, forehead, ante- lesion. This is a useful test in an unresponsive rior scalp, upper half of cornea and iris, top and patient. side of nose, frontal sinus and carries sympa- Jaw jerk (masseter reflex): When the lower thetic fibers to the dilator pupillae via the long jaw is tapped, there is contraction of the masseter ciliary nerves. The maxillary division is purely and temporalis muscles. The afferent and efferent sensory and supplies the lower eyelid, lateral limb run through the mandibular branch (V3). 68 4 Neurologic Examination

Parasympathetic and sensory function is medi- ated by the nervus intermedius. The parasympa- thetic origin is in the superior salivatory nucleus in the pontine tegmentum. The adjacent lacrimal nucleus supplies nerve fibers to the lacrimal gland via the greater superficial petrosal nerve. Thus, a facial nerve lesion proximal to this nerve may cause a dry eye. Excessive tearing () is associated with orbicularis oculi weakness. Parasympathetic fibers to the nervus intermedius also supply the submaxillary, submandibular, and Fig. 4.11 Corneal reflex. The correct method for testing the corneal reflex is performed O.D. There is no response sublingual glands via the chorda tympani. when the sclera is touched, O.S., since it has minimal innervation The most important sensory function of the facial nerve is taste which is mediated by the Bilateral supranuclear lesions cause an increased chorda tympani which supplies the anterior reflex which can be a useful finding. This reflex is two thirds of the tongue. often absent in normal individuals.

The most important sensory function of the 7th Cranial Nerve (Facial) facial nerve is taste which is mediated by the chorda tympani which supplies the anterior two 1. Background anatomy. thirds of the tongue. These nerve fibers travel The 7th nerve nucleus is located in the caudal through the nervus intermedius and terminate in pontine tegmentum. It receives corticobulbar the nucleus of the solitary tract in the medulla. fibers which originate from the lower third of the Other sensory fibers in the 7th nerve supply the precentral gyrus; these fibers decussate in the mucosa of the nose, pharynx, palate, pinna, mas- upper pons. There are both ipsilateral and con- toid, and skin of the external auditory meatus. tralateral fibers which supply the upper facial 2. Examination of the facial nerve: muscles, mainly frontalis, but primarily contral- (a) Observations. ateral fibers which innervate the lower facial muscles. The 7th nerve roots loop around the 6th Asymmetry of palpebral fissures, the dis- nucleus, then exit the ventrolateral portion of the tance between the upper and lower eyelids, pons, pass through the cerebellopontine angle is quite common with a central facial nerve adjacent to the 5th and 8th nerves, and then enter paresis. the internal auditory meatus. The nervus interme- dius lies between the 7th and 8th nerves and Because of the anatomy, discussed mediates both parasympathetic and sensory func- above, a central 7th paresis may involve tion (taste). The facial nerve motor fibers inner- only the lower facial muscles, whereas a vate frontalis, orbicularis oculi, orbicularis oris, peripheral 7th lesion usually involves all stapedius, and platysma musculature. facial muscles. However, the well-trained neurologist is quite aware that asymmetry Upper facial muscles are innervated by both of the palpebral fissures, the distance ipsilateral and contralateral fibers, whereas between the upper and lower eyelids, is lower facial muscles are supplied primarily by quite common with a central 7th nerve contralateral fibers, thus the sparing of fronta- paresis. The wider palpebral fissure indi- lis musculature occurs with most supranuclear cates orbicularis oculi weakness. Beginners lesions. mistakenly believe that the opposite eye is Cranial Nerve Examination 69

ptotic. A flattened nasolabial fold may be (c) Abnormal involuntary movements. evident on the weak side, but note should be made that older individuals often have Facial synkinesias are movements that asymmetries without indicating pathology. occur after Bell’s palsy when there is The frontalis muscle is rarely affected with aberrant reinnervation. facial weakness due to a CNS lesion. Ultimately, one has to depend on clinical • Facial synkinesias. These are move- judgment as to whether the asymmetry is ments that occur after Bell’s palsy when abnormal. there is aberrant reinnervation. For example, nerve fibers that were to be Facial asymmetry may be evident only directed to the orbicularis oris innervate with spontaneous expressions, especially the orbicularis oculi instead. Thus, smiling, and is called “mimetic” facial movement of the lips results in closure weakness. of the ipsilateral eye. The opposite may occur such that eye closure causes Facial asymmetry may be evident only movements of orbicularis oris and plat- with spontaneous expressions, especially ysma. Crocodile tears are a form of syn- smiling, and is called “mimetic” facial kinesia due to aberrant reinnervation of weakness. This may be associated with the lacrimal gland when the patient is frontal lobe lesions as a separate pathway using ipsilateral facial muscles. probably mediates this response. Frontalis asymmetry, fewer wrinkles on one side, Crocodile tears are a form of synki- can be discerned on upward gaze. nesia due to aberrant reinnervation of Asymmetric eye blinks, slower on the weak the lacrimal gland when the patient is side, are a good clue for a 7th nerve using ipsilateral facial muscles. paresis. (b) Voluntary movements. Forced eye closure may be incomplete, • Dyskinesias and dystonia. These evident by seeing the on the include an array of especially lower weak side, whereas they are “buried” on facial movements such as pouting and the normal side. The examiner should pursing of the lips as well as grimacing compare eyelid strength by attempting to and grinning. They are commonly asso- pull the lids open with his or her fingers on ciated with involuntary tongue move- both sides, simultaneously. A facial gri- ments. Tardive dyskinesias are a mace or a request to “show me your teeth” prototypical example; these occur as a will assess strength of orbicularis oris. complication of long-term treatment Retracting the lower jaw with a strong with neuroleptic drugs. grimace tests platysma function. Puffing • Facial . These are worm-like out the cheek tests the buccinator muscle. movements associated with lesions of Bell’s phenomenon may occur on forced the pons probably involving the facial eye closure as the eye moves up and later- nucleus. This must be differentiated ally. This is a normal response which is from fasciculations which are twitches visible if the orbicularis oculi is weak on of individual muscle fibers. this side. Ten percent of normal patients Hemifacial spasms are gross spas- will not have this movement. Consequently, modic movements believed to be an asymmetry may possibly have clinical caused by compression of the facial significance by indicating a unilateral nerve by an arterial loop. upgaze paresis. 70 4 Neurologic Examination

• Hemifacial . These are gross diagnostic method to differentiate facial spasmodic movements of a few or all of weakness of CNS disease, cerebral, or the facial muscles on one side. They brainstem, from peripheral disease such occur spontaneously and are believed as Bell’s palsy. The chorda tympani to be caused by compression of the which arises from the nervus interme- facial nerve by an arterial loop, typi- dius mediates this function. Taste is cally the anterior inferior cerebellar tested by placing a supersaturated sugar artery, which is the artery located in the solution on the anterior two thirds of the cerebellopontine angle. protruded tongue, sequentially. Usually, • Focal seizures. Simple partial focal a Q-tip is used to brush on the solution, motor seizures may affect only the face first on the suspected abnormal side and with gross jerky movements ordinarily then on the normal side. The patient is of relatively short duration such as requested to nod his or her head when 2 min or less. They occur in a regular taste is perceived since the tongue must cadence, whereas hemifacial spasm is remain protruded for the duration of the irregular and spasmodic with no test to prevent moving the solution to reprieve throughout the day. the posterior portion of the tongue • Tics. These are sudden contractions of which is innervated by the glossopha- a group of muscles which may occur in ryngeal (9th) cranial nerve. a focal fashion or be part of a general- • Stapedius muscle. A branch of the facial ized syndrome, Tourette’s disease. nerve innervates the stapedius muscle which, if nonfunctional, results in magni­ Apraxia of eyelid opening or closing fication of sound, hyperacusis. This can may occur with lesions of the non- be a major complaint in some patients dominant hemisphere as well as with Bell’s palsy. The stapedius muscle extrapyramidal disorders. tightens the ossicular chain which pro- tects the cochlea from excessively • Apraxia of eyelid opening or closing. loud sound. Apraxia of eyelid closing is often called “motor impersistence.” When The corneal reflex assesses facial the patient with this disorder is asked nerve as well as the trigeminal nerve. to close his eyes, he often does so momentarily. Maintenance of eye clo- (e) Reflexes. sure is impossible. The opposite occurs Facial nerve reflexes are tested through with apraxia of eyelid opening. True the corneal reflex. A delayed or slow eye motor impersistence must affect other blink with good sensation supports the motor functions and commonly occurs presence of facial nerve involvement. with lesions of the nondominant Glabellar and snout reflexes also test facial hemisphere. muscles and will be described under the (d) Sensory function. section on reflexes.

The primary test of sensory function of the facial nerve is taste which is lost 8th Cranial Nerve (Vestibulocochlear) only with peripheral facial nerve involvement. Cochlear Nerve: Background Anatomy The auditory pathway begins in the hair cells of • The primary test of sensory function of the organ of Corti in the cochlea. The cell bodies the facial nerve is taste. This is the best are in the spiral ganglion. The nerve fibers which Cranial Nerve Examination 71 emanate from these first order neurons make up deaf ear. With sensorineural hearing loss, the the cochlear portion of the 8th nerve which termi- sound is heard louder in the normal ear. nates in the dorsal and ventral cochlear nuclei located near the pontomedullary junction. There Sensorineural hearing loss is diagnosed by are ipsilateral and commissural pathways. The Weber’s test lateralizing to the normal ear and latter include the trapezoid body and pathways the Rinne’s test which is positive. connecting both inferior colliculi before they ascend to the medial geniculate nucleus and then In summary, conductive hearing loss is associ- to the auditory cortex in the superior temporal ated with Weber’s test lateralizing to the abnormal gyrus. The primary clinical significance of this ear accompanied by a negative Rinne’s test. anatomy is the sparing of hearing when there is a Sensorineural hearing loss is manifested by unilateral lesion which destroys one superior Weber’s test lateralizing to the normal ear and a temporal gyrus. Rinne’s test which is positive (normal).

There is no clinically significant hearing loss Vestibular Nerve: Background Anatomy when one superior temporal gyrus (auditory The vestibular system begins in the labyrinth cortex) is destroyed because of both ipsilat- which is composed of the otolith organs (utricle eral and commissural pathways. and saccule) and the semicircular canals (poste- rior, anterior, and horizontal). Linear accelera- Cochlear Nerve: Clinical Evaluation tion is mediated by the utricle and saccule, Hearing loss is either conductive or sensorineu- angular acceleration by the semicircular canals. ral. Conductive hearing loss occurs when there is Stimu­lation of either unit results in a volley of a lesion between the environment and the organ afferent impulses which terminate in the neu- of Corti. Sensorineural hearing loss occurs with rons within Scarpa’s ganglion which is located lesions of the cochlea, 8th nerve, and central in the internal auditory meatus. Nerve fibers auditory pathways. Testing is done with a 512 Hz emanating from this ganglia comprise the ves- tuning fork. tibular portion of the 8th nerve which enters the medulla and terminates in one of the four ves- Conductive hearing loss is diagnosed with the tibular nuclei on each side, the superior, medial, Weber’s test lateralizing to the abnormal ear inferior, and lateral. These are situated in the associated with a negative Rinne’s test. rostral medulla and caudal pons. Additional pathways project via the inferior cerebellar peduncle to the vestibulocerebellum. The latter Rinne’s test is performed by placing the stem structures are the flocculonodular lobe, uvula, of the tuning fork on the mastoid process. When and fastigial nuclei. Projections from the ves- the vibration is no longer heard, the tines of the tibular nuclei form part of the MLF which plays tuning fork are moved to within 1–2 in. from the a role in eye, head, and neck movements. The external auditory meatus. A normal Rinne’s test medial and lateral vestibulospinal tracts contain equals continued perception of vibration and is fibers which affect muscular tone. Decerebrate called positive. An abnormal Rinne’s test means posturing, for example, requires an intact ves- no perception of vibration and is designated tibulospinal system. negative. Weber’s test is performed by placing the stem Vestibular Nerve: History of the tuning fork on the midline of the skull or forehead. When the vibration is perceived equally The vestibular triad is manifested by vertigo, in both ears, the test is normal. With conductive nausea, and diaphoresis. hearing loss, the vibration is heard louder in the 72 4 Neurologic Examination

The vestibular triad is a useful term denoting the Vestibular Nerve: Examination symptoms of vertigo, nausea, and diaphoresis. Direction-fixed contralateral nystagmus is the This triad occurs with a focal unilateral 8th nerve hallmark of an 8th nerve lesion although it may lesion or semicircular canal pathology. Additional also occur with CNS disease. Direction-fixed common complaints include increased vertigo nystagmus indicates contralateral nystagmus in all with eyes closed, staggering, a sense of rotation directions of gaze. There is a slight rotatory ele- opposite to the side of the lesion, and . ment which is the summation of the vectors result- The latter symptom is simply a perceived oscilla- ing from involvement of all three semicircular tion of the environment simultaneous with the canals. When the nystagmus is directed to the left observed nystagmus. Additional auditory mani- side, there is a clockwise element and, when festations may occur depending on the underly- directed to the right, the rotatory part is counter- ing pathology. clockwise. The nystagmus conforms to Alexander’s Law. When the observed nystagmus is anything Vestibular Nerve: Physiology but direction-fixed, CNS disease or drug toxicity are the primary diagnostic considerations. The vestibular portion of the 8th nerve gov- erns slow phase movement of the eyes. Contralateral nystagmus is the hallmark of an 8th nerve lesion, but it is not a pathognomonic The vestibular portion of the 8th nerve governs sign. slow phase eye movements. In other words, stim- ulation (function) of the left 8th nerve results in The head thrust test, if abnormal, is virtually contralateral slow phases. This is followed by an diagnostic of a peripheral vestibular lesion (see ipsilateral quick phase mediated by the PPRF. Fig. 4.12). The patient is instructed to look Thus, if there is a left 8th nerve lesion, for exam- straight ahead at the examiner’s nose. Then, for ple, the unopposed function of the normal right example, the patient’s head is turned to the left 8th nerve generates slow eye movements to the while maintaining fixation on the examiner’s left (side of the lesion) with a quick return to the nose. This requires the patient to maintain right right. Consequently, a left 8th nerve lesion will lateral gaze. With a rapid head turn to a central cause nystagmus to the right (opposite to the side position, the patient must keep his eyes straight of the lesion). This contralateral nystagmus is ahead. Consequently, the eyes must move from a present on right lateral gaze, direct forward gaze, right lateral to a central position. This movement upgaze, downgaze, and is much smaller on left which is part of the vestibular reflex arc requires lateral gaze, if present at all. One can explain the the function of the right 8th nerve to generate the absence or near-absence of contralateral nystag- requisite eye movement from the right lateral mus with ipsilateral gaze by the inability to gen- position to the central position. If the patient has erate an ipsilateral slow phase since the eyes are a right 8th nerve lesion, slow movements to the already fully deviated to the left side. This is the left are impaired and, as the patient’s head stops, basis of there is a corrective saccadic refixation from right Alexander’s Law; the amplitude of nystagmus to left. Thus, the head thrust test indicates a right is greatest when the eyes are deviated in the direc- peripheral 8th nerve lesion. tion of the quick phase. It is important to note that there is no persistent eye deviation with 8th nerve The head thrust test, if abnormal, is virtually lesions as visual fixation overrides the slow phase diagnostic of a peripheral vestibular lesion. imbalance.

The dynamic visual acuity test is performed The amplitude of nystagmus is greatest when by first instructing the patient to read a visual the eyes are deviated in the direction of the acuity card with or without his glasses. The quick phase (Alexander’s Law). visual acuity is noted. He is then asked to read Cranial Nerve Examination 73

Fig. 4.12 Head thrust test Normal response A. The patient’s head is turned to the right about 20° with her eyes fixed on the nose of the examiner who is standing directly in front of her B. The patient’s head is then rapidly turned from right to left while she is maintaining focus on the examiner’s nose. The patient’s eyes have moved from left to right at the same speed as the head movement right to left as there is no adjust- ment of eye position Abnormal response (right 8th nerve lesion) A. The patient’s head is turned to the left about 20° with her eyes fixed on the nose of the examiner who is standing directly in front of her B. The patient’s head is quickly turned from left to right but her eyes cannot move with sufficient velocity from right to left to maintain fixation on the examiner’s nose. Thus there is a refixation saccade from the patient’s right to left (shown by arrows) C. The refixation saccade, right to left, has restored fixation on the examiner’s nose

the same card while shaking his head at 2 Hz, The dynamic visual acuity test is a simple, use- usually aided by the examiner. The visual acuity ful method of evaluating vestibular function. is again noted. If there is a decline of visual acu- ity by more than two lines of the acuity card, there is vestibular dysfunction as the patient is Past-pointing is a test performed with the not able to generate the required slow phases to patient’s eyes closed. The patient is instructed to maintain central fixation. fully extend his arm in a vertical direction and make 74 4 Neurologic Examination repeated movements through a downward arc to touch the examiner’s finger. When this test is abnor- mal, the patient gradually moves towards the side of the lesion when the latter is peripheral. If there is CNS disease, the past-pointing is nonlocalizing. Other tests that are particularly useful to diag- nose vestibular disorders are the Romberg test and the stepping test. These tests are commonly abnormal with vestibular disorders. They will be discussed under the section of “Gait and Station Examination.”

The Romberg and the stepping test are useful for evaluation of vestibular disorders.

Vestibular testing in the comatose patient is particularly important. This includes the vestibu- loocular reflexes also known as the oculocephalic maneuver and caloric testing. The Doll’s eye test is an older, less acceptable term. Caloric testing is seldom done in the clinic setting. It can be per- formed with either air or water and is part of the electronystagmogram. This neurophysiologic test is critically important to perform when the patient has recurrent vertigo and a normal examination. Since visual fixation frequently eliminates visible nystagmus with peripheral vestibular disease, examination of the patient with eyes closed is necessary. This is done using electronystagmog- Fig. 4.13 Dix-Hallpike maneuver. The patient sits up raphy which records eye movements using the straight on the examining table with her legs on the table corneoretinal potential. Frenzel glasses can be in front of her and her hands in her lap. The hand position prevents her from holding onto the table edges which employed. These are high diopter lenses elimi- would inhibit rapid movement to the supine position. The nating visual fixation and magnifying the eyes to examiner stands at the patient’s right side. The patient is improve visualization for the examiner. then instructed to turn her head to the right, approxi- mately 45°, and keep her eyes open throughout the The electronystagmogram is a neurophysio- maneuver. She is then quickly moved but not jerked to the supine position with her head extended below the level of logic test which is mandatory in a patient with the table. The patient may look in any direction while her vertigo and a normal neurologic examination. eyes are inspected for up to 45 s or until the nystagmus has abated. She is then brought up quickly to the sitting position and observed briefly for rebound nystagmus, a The Dix-Hallpike test is best explained by the reversal of the rotation noted in the previous position. diagram (Fig. 4.13). The patient is moved from a This is an inconsistent finding. The procedure is then sitting to a supine position with the head turned repeated for the left side in the same manner approximately 45° to one side when sitting and extended below the level of the table. Patients directed to the undermost ear which is the patho- who have BPPV exhibit nystagmus almost imme- logic side. The most common type of nystagmus diately, although there may be a latency of up to is torsional, upbeating nystagmus with a counter- 20 s, rarely 40 s. The nystagmus usually lasts clockwise rotation if the head is turned right and from a few seconds up to 20 s, rarely 30, and is a clockwise rotation if the head is turned left. Cranial Nerve Examination 75

The etiology is dislodgment of otoconia which located in the nucleus ambiguus in the medulla. are calcium carbonate crystals from the tips of The nerve fibers emerge close together from the hair cells located on the macula of the utricle. the medulla and exit through the jugular fora- These most often fall into the posterior semicir- men. The motor component innervates pha- cular canals and cause this form of nystagmus ryngeal constrictors and the stylopharyngeus through deformation of the cupula. Additional muscle which elevates the pharynx. The sen- features of this nystagmus include rebound nystag- sory portion receives afferents from the poste- mus on sitting up; this is torsional nystagmus in rior one third of the tongue, tonsils, soft palate, the reverse direction which is usually very brief, and tympanic membrane. Jacobson’s nerve if present. There is habituation with repeat testing supplies the tympanic membrane. The central as the nystagmus gradually subsides. Otoconia connection is the solitary tract. It carries particles may less often be deposited in the hori- chemoreceptor fibers from the carotid body zontal semicircular canal (10%) and rarely in the and baroreceptor fibers from the carotid sinus. anterior canal (2%). Nystagmus resulting from these occurrences is quite different. The sensory portion of the glossopharyn- geal nerve (9th) receives afferents from the The Dix-Hallpike test elicits torsional nystag- posterior one third of the tongue, tonsils, mus in the patient with benign paroxysmal soft palate, and tympanic membrane. positional vertigo. 2. Examination. Horizontal canal BPPV produces DCPN. The (a) The soft palate is observed for asymmetry. geotropic form is present when nystagmus beats Sagging on one side may occur. towards the ground. With the patient supine, head (b) Check the gag reflex on both sides when turned to the left, nystagmus beats left, and with there is a history of dysphagia or a suspi- the patient supine, head turned to the right, nys- cion of multiple cranial neuropathies. tagmus beats right. Ageotropic nystagmus is the (c) Check perception of touch on the soft palate, converse. With the patient supine, head right, uvula, pharynx, and posterior third of the nystagmus beats left, and with the patient supine, tongue. head left, the nystagmus beats to the right. 3. Clinical correlations. Geotropic nystagmus occurs with canalo- (a) Dysphagia. lithiasis, free-floating otoconia debris and the (b) Transient hypertension after carotid endar- ageotropic form occurs with cupulolithiasis, terectomy because of denervation of the when otoconia adheres to the cupula. Nystagmus carotid sinus. The contralateral carotid from anterior canal BPPV is downbeat and sinus assumes control either immediately torsional or within several hours.

Transient hypertension after carotid endar- Glossopharyngeal Nerve terectomy may occur if there is denerva- (9th Cranial Nerve) tion of the carotid sinus.

1. Background anatomy. (c) Glossopharyngeal neuralgia. Sharp throat and ear pain often provoked by swallowing. The nucleus ambiguus in the medulla con- (d) Jugular foramen syndrome. Paresis of 9th, tains the cells of origin of the 9th, 10th, and 10th, and 11th cranial nerves usually due bulbar portion of the 11th cranial nerves. to neoplasm at the foramen. (e) Decrease or increase in salivary secretions The cells of origin of the 9th, 10th, and bul- as the nerve innervates the otic ganglion bar portion of the 11th cranial nerves are and parotid gland. 76 4 Neurologic Examination

Vagus Nerve (10th Cranial Nerve) mal side. With bilateral lesions of the vagus nerve, the palate does not move and dysphagia 1. Background anatomy. occurs, particularly with liquids. Motor function arises from the nucleus ambiguus in the medulla, leaves the skull via the jugular foramen, forms the pharyngeal Spinal Accessory Nerve plexus with the glossopharyngeal nerve, and (11th Cranial Nerve) provides motor fibers to the pharynx and soft palate. Branches of the vagus nerve in the 1. Background anatomy. neck include the cardiac rami which follow the carotid arteries down to the aorta and The 11th cranial nerve has both cranial and eventually to the cardiac plexus. At the base of spinal contributions. The spinal contribution the neck, the recurrent laryngeal nerves sup- is located in the 1st through 5th cervical ply all the muscles of the larynx. A lesion of cord segments. this nerve is often a complication of surgery and results in paralysis of the vocal cord on The cranial portion originates in the nucleus that side. This causes a hoarse, raspy voice. ambiguus in the medulla. The spinal part is located in the 1st through 5th cervical cord The vagus nerve forms the pharyngeal segments. C1–C2 innervate the ipsilateral plexus and mediates motor function of the sternocleidomastoid muscles and C3–C4 soft palate. innervate the ipsilateral trapezius muscles. The cranial and spinal roots unite and the The parasympathetic fibers arise from the entire 11th cranial nerve exits through the jug- dorsal motor nucleus of the vagus which is ular foramen with the 9th and 10th cranial located lateral to the hypoglossal nucleus. nerves. Supranuclear innervation of the stern- These parasympathetic branches innervate the ocleidomastoid and trapezius musculature is pharynx, esophagus, trachea, bronchi, lungs, still debatable. Briefly, the right cerebral hemi- heart, intestines, liver, and pancreas. sphere initiates movements of the head to the opposite side. This view is corroborated by The parasympathetic branches of the dorsal observations of focal seizure activity mani- motor nucleus of the vagus nerve innervate fested by head jerk to the left due to a right the pharynx, esophagus, trachea, bronchi, cerebral epileptic focus. Consequently, the lungs, heart, intestines, liver, and pancreas. ipsilateral sternocleidomastoid muscle is innervated mainly by the ipsilateral cerebral hemisphere. Sensory fibers carry taste from the epiglottis, hard and soft palate, and pharynx. These taste fibers run in the solitary tract along with afferent The sternocleidomastoid muscle flexes the glossopharyngeal fibers. Visceral sensations head when both sides are activated simul- from the pharynx, chest, and abdomen also taneously. The trapezius muscle retracts project through the nucleus of the solitary tract. the head and the scapula. 2. Examination. The palate and uvula are examined at rest The sternocleidomastoid muscle flexes the and with phonation. With phonation, the pal- head when both sides are activated simultane- ate should elevate in a symmetrical fashion ously. If the right sternocleidomastoid muscle with no deviation of the uvula. With unilateral is activated, the head is drawn down to the lesions, there is ipsilateral flattening of the ipsilateral shoulder and the occiput is pulled soft palate and phonation will not elevate the down on the same side. This rotates the head soft palate. The uvula is deviated to the nor- to the left. The trapezius muscle retracts the Motor Examination 77

head and the scapula as well as raising the down to the cervical medullary junction. abducted arm above the horizontal plane. The nerve roots are medial to the 9th, 10th, 2. Examination. and 11th cranial nerves and they pass through The right sternocleidomastoid muscle is the hypoglossal canal. The nerve fibers descend examined by having the patient turn his head to through the neck to the angle of the mandible the left against resistance with simultaneous near the internal carotid artery and the internal palpation of the musculature and vice versa. jugular vein. They supply the intrinsic and Flexing the head against resistance evaluates extrinsic muscles of the tongue. The supranu- both sternocleidomastoid muscles. The trape- clear control of the tongue is mediated by cor- zius muscle retracts the head and the scapula. It ticobulbar fibers. They originate in the lower assists the abducting arm to elevate above the part of the precentral gyrus. The corticolingual horizontal plane. It is examined by having the fibers from each cerebral hemisphere supply patient shrug his shoulders against resistance. both sides of the tongue with the exception of 3. Clinical-anatomic correlation. the genioglossus muscle which receives con- tralateral innervation. The pontomedullary Weakness of head flexion occurs in many junction is the location of the crossing fibers. patients with myopathy, myasthenia gravis, 2. Examination. and chronic inflammatory demyelinating The tongue should be observed for sym- polyneuropathy (CIDP). metry, atrophy, and abnormal involuntary movements. The latter includes dyskinesias, Weakness of head flexion, bilateral sterno- , fasciculations, myoclonus, and cleidomastoid weakness, occurs in many tremor. patients with myopathy, myasthenia gravis, and The patient is then requested to protrude chronic inflammatory demyelinating polyneu- his tongue and move it to either side. The ropathy (CIDP). Less often, the same diseases tongue may deviate to the hemiparetic side if weaken trapezius muscles and, consequently, the lesion is above the level of the decussation head extension. Neck surgery in the posterior of fibers at the pontomedullary junction. The cervical triangle region can easily injure the normal, ipsilateral genioglossus pushes the C3–C4 roots as they are located superficially. tongue to the opposite side. Differentiating This results in a unilateral drooping shoulder central from peripheral 12th nerve lesions is and may affect muscles in the thoracic outlet. often difficult and requires a careful history and neurologic examination.

Hypoglossal Nerve (12th Cranial Nerve) Motor Examination 1. Background anatomy. There are three motor systems, corticospinal, The corticolingual fibers from each cere- extrapyramidal, and cerebellar. A practical bral hemisphere supply both sides of the approach is to assess all of them together, in one tongue with the exception of the genioglos- category, particularly since there are overlapping sus muscle. The genioglossus muscle functions. The motor examination can be broken pushes the tongue to the opposite side and down into five categories. thus may deviate to the hemiparetic side. There are three motor systems, corticospinal, The 12th cranial nerve has purely motor extrapyramidal, and cerebellar. function. The nerve fibers arise from the hypo- glossal nucleus which is a column of cells 1. General observations. extending from the pontomedullary junction 2. Close inspection. 78 4 Neurologic Examination

3. Coordination. patient is asked to extend his arms and spread his 4. Strength. fingers. A tremor, if present, is usually bilateral, 5. Tone. rapid (greater than 10 Hz), and of small ampli- tude. It may be more prominent on one side. As the severity increases, the amplitude becomes General Observations larger. An action tremor occurs when making a voluntary movement. It is most often perpendicu- Among the first observations made on entering lar to the direction of movement and is thus best the examining room are those of the patient’s appreciated on finger-nose-finger test. It is the posture and movements. Is the head in a neutral most disabling form since it interferes with using position, turned to either side, flexed, or extended? eating utensils or drinking from a cup. Action Are the arms and hands resting comfortably in and postural are virtually always present the patient’s lap or are they in a forced abnormal together, although one may be much more promi- position? Are the legs extended, flexed, or turned nent. Having the patient draw a spiral at each visit inward? Is the trunk flexed, extended, or twisted? is a practical way of evaluating the success of These abnormal findings commonly have more treatment. The most common cause of action/ than one interpretation and must be evaluated in postural tremor is essential tremor which is famil- the context of the entire neurologic examination. ial in two thirds of cases. Infrequently, a resting tremor may be present which may baffle the Abnormal movements are divided into hypoki- examiner. Occasionally, patients with Parkinson’s netic or hyperkinetic. disease exhibit an action/postural tremor, but to a lesser degree than a resting tremor. Recent patho- logical studies suggest that it is of cerebellar ori- gin. Physiological tremor associated with fear, Bradykinesia is often mistakenly attributed anxiety, and fatigue has similar characteristics. to age. Adverse effects of drugs and hyperthyroidism produce the same type of tremor. It should be Abnormal movements are divided into hypoki- noted that pure essential tremor may also affect netic or hyperkinetic. The prototype of a hypoki- the head and voice. netic is Parkinson’s disease. Bradykinesia is the hallmark, a slowing of all Action and postural tremors are virtually spontaneous movements. It is easily spied with always present together. any movement such as when the patient takes off or puts on his glasses. In elderly patients, bradyki- nesia is often mistakenly attributed to age. Getting Akathisia is a severe form of motor restless- out of a chair, turning in bed, or even sitting down ness that affects the legs. Patients have an inner can be challenging tasks. The Parkinson patient sense of restlessness and are often pacing the often misjudges the location of the seat and may floor or marching in place. This is nearly always sit on the armrest. Hypomimia or masked facies, a due to the use of neuroleptic medication (dop- loss of facial mobility, is common. There may be amine blockers). It begins at the onset of treat- diminished eye blinks and, quite often, apraxia of ment or with an increased dose of medication. It eye opening and closing. The latter has been called resolves when treatment is discontinued. the “reptilian stare.” Blepharospasm commonly prevents reading or watching television. Writing Akathisia is a severe form of motor restless- becomes laborious and micrographia is common. ness that affects the legs. Hyperkinetic movement disorders are numer- ous. Tremor is the most common. It can be Resting tremor is of lower frequency, about divided simply into three types, postural, action, 2–6 Hz, and is commonly unilateral and a proto- and resting. To evaluate postural tremor, the typical sign of Parkinson’s disease. It is frequently Motor Examination 79 intermittent, observed by family members. Thus Athetosis is manifested by slow, writhing it may be absent during a brief examination. It movements with a predilection for distal can be more obvious when the patient walks at musculature. which time there is a “pill-rolling” character. Pill- rolling refers to movement of the thumb on the first two fingers. This type of tremor may affect Dystonia presents with a myriad of character- the lips, tongue, chin, jaw, hands, and legs, but istics. Dystonic movements can be generalized or not the head. Likewise, it increases with excite- focal and tend to occur in the same location. The ment. Tremor with sustained posture may also be duration may last seconds to hours. Dystonic present, but to a much lesser extent and ordinarily spasms or myoclonus last less than 1 s. Dystonic is eliminated with movement. A resting tremor movements may last several seconds such as may begin in a single digit. It also increases with athetotic dystonia. Dystonic postures last minutes to excitement and is absent during sleep. hours. Contractures may occur when the posture lasts days or weeks. Some examples are torticollis Patients with Parkinson’s disease often exhibit (cervical dystonia), tortipelvis (scoliosis), lordosis, a pill-rolling tremor when walking. and inversion of feet and hands.

Chorea is manifested by quick, jerky move- Dystonic movements can be generalized or ments with a predilection for distal musculature. focal and tend to occur in the same location. The movements are present at rest, but increase The duration varies from seconds to hours. with stress and activity. Patients often find it impossible to maintain a fixed position. Hence, There are primary dystonias of childhood the term “milkmaid’s grip” as the patient repeat- onset, some of which are probably of genetic origin edly grips and releases the doctor’s fingers. (autosomal dominant). Dopa-responsive dystonia Additionally, the patient may be unable to maintain of childhood should be recognized since it is treat- tongue protrusion. Hypotonia is evident and the able. There are dystonic forms of Parkinson’s dis- movements disappear in sleep. Some etiologies ease such as Lubag’s disease, a genetic form include Huntington’s chorea, chorea gravidarum which occurs in the Philippine islands. of pregnancy, Sydenham’s chorea, systemic lupus Focal dystonias include cervical dystonia, oro- erythematosus, and hyperthyroidism. mandibular dystonia (Meige’s syndrome), writer’s , spasmodic dysphonia, and musician’s Chorea is manifested by quick, jerky move- cramp. Spasmodic dysphonia is dystonia of the ments with a predilection for distal muscu­ vocal cords which produces a tremulous, breathy, lature. low volume speech. , forced upward/lateral eye deviation, is probably dystonia Athetosis is manifested by much slower, larger, of eye muscles that occurs with encephalitis and almost constant writhing movements with a lethargica and is now rarely observed in patients predilection for distal musculature. The move- with the neuroleptic malignant syndrome, ments may also affect facial and truncal muscles Wilson’s disease, and those taking neuroleptics. as well as the extremities. Voluntary movements are severely impaired as attempts to move usually Cervical dystonia is the most common form increase athetotic movements. The etiology is of focal dystonia. almost invariably cerebral palsy with injury to the basal ganglia. Other etiologies are degenerative Cervical dystonia is the most common form of diseases such as Wilson’s disease and kernicterus. focal dystonia and has a few manifestations Sleep eliminates athetotic movements. including torticollis (twisting to one side usually 80 4 Neurologic Examination with head tilt), anterocollis (forced flexion), and the same as with tardive dyskinesias, namely retrocollis (forced extension). Patients may some- neuroleptics and metoclopramide. These dystonic times be able to resume a normal posture using a movements may occur during treatment or shortly sensory stimulus (trick), a light pressure to coun- after treatment has been discontinued. teract the abnormal contraction. Hemiballismus is a unilateral, flinging move- Tardive dystonia is manifested by more sus- ment of an arm or leg generated at a proximal tained movements, particularly with facial joint. It is nearly always due to a stroke, hemor- grimacing, jaw deviation, and protrusion. rhagic or ischemic, affecting the contralateral subthalamic nucleus. Thus, it is associated with Myoclonus is manifested by jerky, sudden, posterior circulation disease affecting the basilar twitch-like movements of any body part. They and posterior cerebral arteries. may be part of an epileptic syndrome such as juvenile myoclonic epilepsy. When they are mul- Hemiballismus is a unilateral, flinging move- tifocal, one suspects an underlying metabolic ment of an arm or leg generated at a proximal encephalopathy. The movements are quicker than joint. those of chorea. Opsoclonus is a term that refers to the same type of movement which selectively Tardive dyskinesias are relatively common affects the eyes. Opsoclonus is due to cerebellar and are most often involuntary oral, buccal, and system or brainstem dysfunction and is mani- lingual movements. These are manifested by fested by chaotic saccades in all planes. chewing and/or lip movements, and writhing of the tongue when protruded or even at rest. Less Asterixis, a sudden dropping movement of often, there may be respiratory dyskinesias noted the hands, may occur with any metabolic as sporadic chest excursions as well as truncal encephalopathy. and pelvic movements. Tardive dyskinesia is often accompanied by akathisia, an inner rest- Asterixis refers to sudden dropping move- lessness that may provoke the patient to stand and ments of the hands when the arms are extended continually move his legs. Tardive dyskinesias and the hands dorsiflexed at the wrists. There is a are commonly associated with neuroleptics (dop- sudden loss of muscular tone which has been amine-blocking agents) usually after 3 months of referred to as negative myoclonus. It is almost treatment or after treatment has been discontin- always a manifestation of a metabolic encephal- ued. Metoclopramide, a common medicine used opathy unless it is unilateral, a rare phenomenon for gastroparesis, is often the culprit. after stroke involving corticospinal pathways. It is best known as an associated finding with hepatic Tardive dyskinesias are relatively common encephalopathy, but perhaps more common in and are most often oral, buccal, and lingual acute hypercapnia. Any metabolic derangement movements. can cause asterixis.

Orofacial dyskinesias may occur as a common Tics are motor and vocal. Each type can be complication of treatment for Parkinson’s dis- divided into simple and complex. ease, levodopa. The movements are similar to tardive dyskinesias. Other etiologies are Tics are motor and vocal. Each type can be Huntington’s and Sydenham’s chorea and divided into simple and complex. A simple motor Wilson’s disease. tic is a sudden jerky, involuntary movement often Tardive dystonia is manifested by more sus- similar to a myoclonic jerk or chorea. Simple tained movements, particularly with facial grimac- tics, however, are more likely to be repetitive. ing, jaw deviation, and protrusion. The etiology is Complex motor tics are a series of simple tics Motor Examination 81 often in the same sequence. Sometimes there is a Fasciculations are quick twitches due to coordinated pattern of movement. Simple vocal contraction of muscle fibers. They are never tics are characteristically a sniff, grunt, cough, or strong enough to produce movement of a limb. clearing of the throat. Complex vocal tics would Fasciculations are the hallmark findings in dis- be verbalization such as coprolalia, obscene lan- eases affecting the anterior horn cell such as guage. When both motor and vocal tics are pres- amyotrophic lateral sclerosis. The most common ent in childhood, the diagnosis is Tourette’s cause of localized fasciculations, however, is a syndrome. Motor tics alone may be simply a radiculopathy due most often to a herniated cer- transient habit spasm. vical or lumbar disk. Fasciculations do not occur with neuromuscular junction or muscle diseases and rarely with peripheral nerve disease. When Close Inspection amyotrophic lateral sclerosis is suspected, the chest and back of the patient should be carefully Atrophy is a result of diseases affecting the inspected under good illumination for fascicula- anterior horn cell, roots, and nerves. tions. Fasciculations persist during sleep.

Observations should be made regarding mus- Fasciculations are primarily due to anterior cular atrophy or hypertrophy, muscle bulk, horn cell disease and radiculopathy. spasms, fasciculations, and myokymia. To assess bulk, it is useful to inspect the arms in the pronated Myokymia is an unusual phenomenon charac- and supinated position to evaluate extensors and terized by prolonged vermiform movements flexors of the forearm, respectively. Measuring which are most often seen on the face, especially the circumference of a limb is best done at its involving the orbicularis oculi muscles. They greatest apparent girth. The dominant limb may have been observed in patients who have lesions be 1 cm larger in the thigh or calf and 0.5 cm of the facial nucleus. Myokymia of limb muscles larger at the forearm and upper arm. Atrophy is a have been reported after radiation damage to the result of diseases affecting the anterior horn cell, brachial or lumbosacral plexuses. roots, and nerves. Disuse atrophy and wasting of muscles are noted in patients who are cachectic, deconditioned due to prolonged inactivity or with Coordination chronic myopathies. Hypertrophied muscles or pseudohypertrophy typically involve calf muscu- Testing of coordination often yields more diag- lature and are noted rarely in myopathies such as nostic information than other parts of the motor Duchenne’s muscular dystrophy or Thomsen’s examination. The earliest deficit of lesions of the disease (myotonia congenita). motor systems, whether corticospinal, extrapyra- Spasms or cramps are muscular or neurogenic midal, or cerebellar, are first detected by these in origin and often referred to by patients as “char- examinations. leyhorses.” These are sudden tonic contractions of muscles due to a sensory stimulus. Nocturnal Testing of coordination often yields more cramps are quite common and may provoke the diagnostic information than other parts of the patient to get out of bed and walk around. Should motor examination. Impaired rapid alternating this be, the case a presumptive diagnosis of rest- movements occur with cerebellar, corticospinal, less legs syndrome can be made and treatment and extrapyramidal disorders. initiated, often with excellent results.

The most useful is impairment of rapid alter- Fasciculations are quick twitches due to con- nating movements. Abnormalities, called dysdia- traction of muscle fibers, the hallmark finding dochokinesis – a convoluted term – have been in diseases affecting the anterior horn cell. defined as a purely cerebellar disorder in the past. 82 4 Neurologic Examination

Although it is quite characteristic of cerebellar Failure of check – the rebound phenomenon system pathology, it is not pathognomonic, a – is a useful sign of cerebellar dysfunction. The critical distinction. It is often the earliest sign of patient is requested to adduct his arm at the shoul- a corticospinal tract lesion and certainly a common der and flex his arm about 90° at the elbow using abnormality found in patients with extrapyramidal his biceps muscle. The examiner pulls strongly at disorders such as Parkinson’s disease. One differ- the wrist while simultaneously placing his fore- ence with cerebellar dysfunction may be the arm close to the patient’s cheek between his fist irregular rhythm and rate. This can be difficult to and face. When the examiner suddenly lets go, ascertain. The test may be performed in different the patient must check the flexion movement or ways. Alternating pronation and supination of the his arm will strike the examiner’s forearm. Failure hands as rapidly as possible on the leg is the most to check the movement is a distinctly cerebellar common method. Having the patient do the system sign. movements simultaneously with both hands and with partially extended arms may be the quickest and most sensitive. Rapid finger and foot tapping Strength are equally sensitive and are often abnormal before physical weakness becomes apparent as There are different methods of evaluating well as the last to recover from a corticospinal strength. One method is to have the patient hold a tract lesion. position and resist the examiner’s attempt to move the limb. The opposing technique is the More specific signs of cerebellar system dys- reverse. The examiner resists the patient’s attempt function include the finger-to-nose and heel- to move. When the examiner attempts to move to-shin tests. the limb from a fixed position, it can be done with a sudden exertion of force or steadily increasing strength. The latter method is more precise, espe- More specific signs of cerebellar system dys- cially when assessing distal musculature. function include the finger-to-nose and heel-to- shin tests. The former is performed such that the There are numerous confounding factors when patient’s arm is fully extended when touching the evaluating strength such as fatigue, somno- examiner’s finger. This is more likely to elicit lence, conversion reaction, and malingering. ataxia or tremor. Heel-to-shin testing is com- monly difficult for the obese patient. Testing the patient in the supine position is sometimes easier. There are numerous confounding factors when Many patients place the heel to the side of the evaluating strength. These include fatigue and shin which damps the oscillations that might somnolence which result in a wide variability of occur if the heel was placed precisely on the edge effort. Conversion reactions and malingering of the shin. The initial placement of the heel on often produce a ratchety, “give-way” weakness the knee or the edge of the shin without any rather than smooth steady weakening. A positive movement may elicit the side-to-side oscillations. Hoover sign is an additional finding to support The movement downwards may be jerky – either of these diagnoses. This sign is a useful decomposition of movement – since fluid, coor- finding when leg weakness is claimed. In the dinated movements are interrupted. It is important supine position, the weak leg is evaluated for to be aware of the fact that severe position sense strength of hip flexion while the examiner’s hand loss may give rise to similar abnormal signs. is placed underneath the heel of the good leg. Normally, there is downward pressure exerted by the normal leg. The absence of this pressure sug- Failure of check – the rebound phenomenon – gests unsatisfactory effort. Impaired comprehen- is a useful sign of cerebellar dysfunction. sion also obviates an adequate assessment and Motor Examination 83 persistent efforts to explain the movement are quadriceps, hamstrings, anterior tibialis, and often fruitless and are best abandoned after three gastrocnemius. Depending on the patient’s symp- or four attempts. The syndrome of motor imper- toms and suspected diagnosis, additional muscles sistence, the inability to maintain a fixed position may be added. For example, a history of back pain despite adequate strength, is a well-known or possible radicular pain requires evaluation of although uncommon manifestation of nondomi- the extensor hallucis longus, a commonly affected nant hemisphere lesions which impedes the muscle with lumbar radiculopathy. examination. If myopathy or myasthenia gravis are sus- Examination of strength begins with the pected, proximal muscles should be the focus assessment of arm drift. of the examination, especially head flexion and extension. Examination of strength begins with the assess- If myopathy or myasthenia gravis are sus- ment of arm drift. The patient is instructed to fully pected, proximal muscles should be the focus of extend both arms with the palms facing upwards. the examination, especially head flexion and A downward drift with pronation is a typical early extension. Repeated checking of deltoid strength, sign of a corticospinal tract lesion, cerebral, brain- usually 15 attempts, evaluates for fatigue and is stem, or spinal cord. A drift upward is noted with an especially useful evaluation for myasthenia proprioceptive deficits and thus is called a “pari- gravis. Climbing up one or two flights of stairs is etal drift.” Clearly, position sense loss of any ana- a useful method of eliciting fatigue. tomic origin, central or PNS, may cause the Neuropathies require a more detailed assess- identical finding. An unusual drift is the one that ment of distal musculature. Suspected radiculop- moves the arm laterally and this may occur with athies and plexopathies require careful assessment cerebellar system lesions. It is essential to exclude of all muscles that could be affected, virtually all the presence of shoulder pathology which often muscles of an affected limb. causes downward drift with pronation. In summary, examination of muscular strength A general survey of arm and leg strength must will vary from patient to patient depending on the include proximal and distal musculature. A stan- history and the suspected diagnosis. What must dard method of evaluation is noted in the adja- always be kept in mind is to compare manual cent table (Table 4.8). strength with functional strength. A patient with Pluses and minuses can be added to the numbers weak iliopsoas muscles should not be able to get on the scale to provide more options. Selected up quickly from a chair. A patient who has severe muscles in the arms usually include interossei, weakness of ankle dorsiflexion should not be able extensor carpi radialis, biceps, triceps, and deltoids. to perform heel-walking. A major principle, Leg muscles usually examined are the iliopsoas, therefore, is that functional strength nearly always takes precedence over the results of manual Table 4.8 The Medical Research Council rating of strength testing. ­muscle strength Scale Muscle strength 0 No contraction Tone 1 Flicker or trace of contraction 2 Active movement with gravity eliminated There are three common types of abnormal 3 Active movement against gravity muscular tone, spasticity, cogwheel rigidity, 4 Active movement against gravity and resistance and paratonic rigidity (Gegenhalten). 5 Normal power From Medical Research Council: Aids to the Examination of the Peripheral Nervous System. Memorandum No. 45. There are three common types of abnormal London: Crown; 1976 muscular tone, spasticity, cogwheel rigidity, and 84 4 Neurologic Examination paratonic rigidity (Gegenhalten). Spasticity of Paratonia (Gegenhalten) is the active resistance the arm can be most easily evaluated by quick to any movement by the patient. Despite clear supination movements (biceps) of the patient’s requests by the examiner to relax, the patient relaxed arm. Flexor tone is particularly increased actively resists the examiner’s attempt to flex or in the arm. There is a free unopposed interval, extend the limb, especially at the elbow and knee. followed by variable resistance and then a sudden When the physician increases the speed of move- giving-way (clasp knife reaction). In the legs, ment, the patient’s resistance is often augmented. spasticity is most prominent in extensor muscles A common misinterpretation of this sign is “the and is best evaluated at the knees. Rapid flexor patient is poorly cooperative” or actively movement with the patient sitting and the legs “obstructs” the examination. Paratonia indicates swinging freely may elicit increased tone; but bilateral cerebral dysfunction, whether metabolic, perhaps even better is to evaluate the patient when toxic, vascular, degenerative, or of infectious origin. supine. When a relaxed leg is abruptly and Dementing diseases and metabolic encephalopathy momentarily elevated at the knee, there is a are probably the most frequent etiologies. delayed return to a fully extended position. The leg “hangs up,” exhibiting a “spastic catch.” Myotonia is characterized by impaired relax- Adductor tone is also especially increased. This ation of muscle. causes a “scissoring,” a tendency to adduct the paretic leg which crosses over the normal leg. It Myotonia is characterized by impaired relax- is easiest to detect when the patient walks. ation of muscle. A tap on the thenar eminence, for instance, induces a muscle contraction which Cogwheel rigidity is a characteristic sign of dis- adducts the thumb. This often persists for several eases affecting the extrapyramidal system, but seconds. The patient may complain of inability to it is not pathognomonic of Parkinson’s disease. release a firm grasp quickly even when shaking hands with someone. The edge of a tongue blade Cogwheel rigidity is a characteristic sign of can be placed on the midline of the tongue fol- diseases affecting the extrapyramidal system. It lowed by lightly tapping the upper edge with a is a typical feature of Parkinson’s disease, reflex hammer. This often produces curling of Parkinson-plus syndromes such as multiple sys- the tongue muscles. The two most common tem atrophy, progressive supranuclear palsy, cor- causes of myotonia are myotonic dystrophy and tical basal ganglionic degeneration, striatonigral Thomsen’s disease. degeneration, Creutzfeldt-Jacob disease, Lewy Other less common abnormalities are hypoto- body disease, and adverse effects of neuroleptic nia and flaccidity. Hypotonia is most easily and drugs which are dopamine blockers, especially commonly recognized in infants, especially the the phenothiazines. It is commonly detected in neonate. It occurs with both central and periph- patients with essential tremor which diminishes eral diseases. Cerebellar pathology is an etiology its diagnostic significance. The combination of of hypotonia, but it can be difficult to recognize. cogwheel rigidity with bradykinesia or resting A sudden attack of decreased tone occurs in cata- tremor, however, strongly supports a diagnosis of plexy, a component of narcolepsy which will be Parkinson’s disease. There is a ratchety move- discussed in a subsequent section. Momentary ment when the examiner very slowly moves the loss of tone also occurs with akinetic seizures and arm, typically rotating at the wrist or flexing and asterixis. Flaccidity is prominent with acute spi- extending the arm at the elbow. Agonist and nal cord injuries causing . antagonist muscles have similar tone. Finally, there is a rare disorder called stiff- person (stiff-man) syndrome characterized by paroxysmal, often painful muscular rigidity, and Paratonia is the active resistance to any move- spasms of the axial and limb musculature. Muscular ment by the patient. tone is normal between these paroxysms which Gait and Station Examination 85 can result in abrupt falls. The etiology is antibodies Common causes of an apraxic gait include to glutamic acid decarboxylase, an enzyme which Parkinson’s disease, normal pressure hydro- breaks down glutamic acid to gamma-aminobu- cephalus, and bilateral frontal lobe pathology. tyric acid (GABA), an inhibitory transmitter. This disorder can be a paraneoplastic syndrome. Common causes of an apraxic gait include Parkinson’s disease, normal pressure hydrocephalus, and bilateral frontal lobe Gait and Station Examination pathology.

Gait can be affected by abnormalities arising 2. Parkinson gait. from all anatomic sites. The abnormal walk, how- The patient with Parkinson’s disease has a ever, is not always pathognomonic for a specific gait similar, if not sometimes identical, to an anatomic locus of disease. There are overlapping apraxic gait. Initiation of the first step is hesi- findings. tant, may be followed by shuffling and then Observations should include: turning en bloc taking a few extra steps. 1. Initiation. Turning en bloc is manifested by a rigid pos- 2. Base. ture and an absence of truncal or head move- 3. Stride. ments. The patient may have camptocormia, a 4. Turning. flexed spine, is usually stooped, and has a 5. Arm movements. diminished arm swing. In addition to the flexed 6. Speed. spine, the knees may be flexed. The gait may 7. Center of gravity. be festinating. This is an uncontrollable accel- 8. Involuntary movements. eration of gait. The patient usually hesitates 9. Body posture. when going through doorways. Retropulsion 10. Balance. when attempting to stand and misjudging both distances and the turning circle in the process of sitting down are common. Verbal cues may be helpful when the patient freezes such as, Common Gait Disorders “Now lift your left leg.” 3. . 1. Apraxic gait. Sensory ataxia is due to impaired proprio­ An apraxic gait is manifested by inability to ception. take that first step, followed by shuffling. Sensory ataxia is due to impaired proprio- An apraxic gait is manifested by inability ception. The gait tends to be slower, associ- to take that first step. Initiation of movement is ated with a wide base and may be “slapping.” markedly impaired. This is typically followed There is gross ataxia with a tendency to fall to by shuffling or a very short stride and taking either side. The patient focuses on the floor several steps when turning. The base may be since the absence of visual cues will aggravate wide or narrow. The feet seem stuck to the the ataxia. Neuropathy is the most likely etiol- floor and there is often freezing of movements, ogy. Myelopathies, such as subacute combined especially when going through narrow spaces degeneration which is associated with poste-

such as a doorway. “Freezing” in doorways is rior column demyelination due to B12 defi- typical of Parkinson’s disease. While seated, ciency, are much less common. however, the patient can move his legs well to 4. . initiate stepping or bicycling. Retropulsion is This gait is wide-based, slow, and often common and may inhibit these movements. associated with a sensory ataxia. The patient 86 4 Neurologic Examination

often takes high steps and throws his foot out. This gait disorder usually implies a spastic It is frequently an audible gait because of the paraparesis. The patient walks slowly with a “slapping” sound due to weakness of ankle narrow base, adducted legs, and drags his feet. dorsiflexion (anterior tibialis). The etiology is The legs are stiff and extended at the knees. usually a neuropathy. The legs tend to cross over each other or “scis- 5. Cerebellar system dysfunction. sor.” A myelopathy is almost always the ana- tomic basis if there are no other neurologic Cerebellar diseases may cause a lurching, signs. Other very unlikely considerations staggering gait. would include a parasagittal neoplasm, bilat- eral infarctions involving the internal capsules This patient has a lurching, staggering gait. and, least likely, brainstem pathology. The base is wide and foot placement is irregu- The most common etiologies, assuming a lar. There may be titubation, severe rhythmic myelopathy, would be multiple sclerosis, cer- truncal, and head tremor. With a unilateral vical cord compression due to spinal stenosis lesion, there is usually a drift to that side. or a herniated disk and, lastly, neoplasm. When a patient is instructed to walk a few steps forwards then backwards a few times with the eyes closed, deviation to the side of a Uncommon Gait Disorders lesion occurs. This is likely to lead to a pattern of movement which is “star-shaped.” When 1. Vestibular gait. the brainstem is involved, there may be drop attacks due to a sudden loss of tone, “negative A vestibular gait is typified by a wide base myoclonus.” and drifting to the side of the lesion. 6. Hemiparetic gait. The patient walks with a wide base and The hemiparetic gait is manifested by a drifts to the side of the lesion. There may be a wide base, extension of the knee, circum- rigid head position to reduce oscillopsia, the duction of the leg, and with a tendency to perception that the environment is oscillating, drag the foot. that occurs due to an impaired vestibuloocular reflex. This is associated with an abnormal stepping test which will be described below. The hemiparetic gait is manifested by a 2. Cautious gait. wide base, extension of the knee, circumduc- This is a gait that may occur in elderly indi- tion of the leg, and with a tendency to drag the viduals who have previously injured them- foot. The arm is adducted, flexed at the elbow, selves in a fall and are fearful of repeated pronated, and flexed at the wrist. injury. The patient walks slowly with a wide Ipsilateral loss of arm swing is occasion- base, short stride, and often reaches out for ally the only finding in the arm when the support. The arms are frequently abducted and patient has a spastic leg. The etiology is a con- the turning is en bloc. tralateral cerebral or brainstem lesion or an 3. Gait associated with choreoathetosis and ipsilateral spinal cord lesion which involves dystonia. the corticospinal tract. The patient with choreoathetosis often has 7. Paraparetic gait. a “dancing” gait which is wide-based and manifested by irregular steps. There are asso- The paraparetic gait is associated with a ciated abnormal involuntary movements in the slow walk with a narrow base, adducted upper extremities. Occasionally, there is a legs, and dragging of the feet. spastic element with circumduction of the Gait and Station Examination 87

legs. In the patient who has dystonia, there is a good heel-walk certainly has psychogenic sustained abnormal posture. ­factors which play a significant role in his 4. Waddling gait. neurologic presentation. The same compari- This is due to proximal leg weakness. The son can be made with a toe-walk as compared patients may walk on their toes and often have to weakness of gastrocnemius muscles. Lastly, a wide base. A lumbar lordosis is common. this test may pick up weakness that is not 5. Antalgic gait. detectable by manual testing. This is an abnormal gait provoked by 9. Romberg test. pain. The patient avoids weightbearing, for example, on an injured leg or diseased, pain- The Romberg test is an assessment for ful joint. Occasionally, the appearance simu- abnormal position sense and vestibular lates a gait due to a neurologic disorder dysfunction. rather than a simple . Thus, referrals to a neurologist for an antalgic gait are not rare The Romberg test compares standing with and care must be taken to avoid unnecessary feet together, eyes open followed by eyes investigations. closed. When the patient is stable with eyes 6. Astasia-abasia. open and eye closure causes a major sway or an extra step to maintain balance, the test is Astasia-abasia is typically found in the positive. This is a test for abnormal position patient with a conversion reaction who sense and vestibular dysfunction. Interpretation makes lurching irregular steps. of abnormal sway is often controversial and hence an abnormal test often depends on the This is typically a patient who has a con- examiner. A minimal sway is clearly negative. version reaction and makes lurching irregular Unsteadiness with eyes open and closed is steps. The patient appears to be on the verge of also a negative test. a near-collapse. Associated symptoms are The most common cause of a positive test often a give-way weakness. The patient may is a sensory neuropathy. Myelopathies involv- manifest a Hoover sign when supine. ing the posterior columns will cause the same 7. Tandem gait. finding, but are quite rare. Major diagnostic The patient attempts to walk a straight line considerations in that instance would be mul- with his or her heel touching the tip of the big tiple sclerosis and subacute combined degen- toe. This impairment is common with any of eration due to B12 deficiency. Cervical spinal the above-noted gait disorders. Its significance stenosis and cord compression due to a herni- is particularly relevant when it is the only ated disk usually cause motor dysfunction as abnormal finding and thus would suggest they first affect the anterior portion of the spi- either cerebellar system disease or a vestibular nal cord. disorder. Normal elderly patients, over 70 years, Vestibular dysfunction, especially vestibular may be unable to perform tandem gait. neuritis, is manifested by falling toward the side of the involved 8th nerve. Semicircular canal Normal elderly patients, over 70 years, pathology due to drug toxicity or Ménière’s dis- may be unable to perform tandem gait. ease are additional etiologies. Eighth nerve damage due to drug toxicity is another distinct 8. Heel and toe-walking. etiology. Whether a positive Romberg can be An inability to get up on heels and take a caused by cerebellar system dysfunction is con- few steps without a slight footdrop is an excel- troversial. This does occur occasionally, in my lent functional test. A patient who has give- opinion, and is probably due to involvement of way weakness of ankle dorsiflexion and a vestibulocerebellar pathways. 88 4 Neurologic Examination

10. Stepping test (Fukuda test). The reflex examination is susceptible to both misinterpretation and faulty technique making The stepping test is useful for evaluating reflex findings often less valuable than func- vestibular function. tional testing. This is a good test for vestibular dysfunc- tion. The patient is requested to take 60 steps while standing in place with the arms out- stretched directly in front of him. Turning Method of Examination more than 45° to one side indicates vestibu- lar dysfunction; usually the turn is towards Reflexes are obtained by tapping near the the side of a peripheral lesion. ­tendon insertions and should avoid hitting the 11. Pull test. muscle itself which can always generate a The examiner stands behind the patient and reflex. pulls the patient’s shoulders back quickly with moderate force. The patient with Parkinson’s Patients are best examined sitting in relaxed disease commonly takes a few steps backwards fashion with the arms flexed at the elbows and or may even fall into the examiner’s arms. hands in the lap. The angle between the forearm and upper arm should approach 90°. Reflexes are obtained by tapping near the tendon inser- Reflex Examination tions and should avoid hitting the muscle itself which can always generate a reflex. Reflexes routinely evaluated are the biceps, triceps, bra- The major purpose of the reflex examination is to chioradialis, Hoffmann, patellar (knee), Achilles distinguish between CNS and PNS disease. If (ankle), and plantar (Babinski reflex). Biceps CNS disease is present, the reflex examination and brachioradialis are innervated by C5-6 can help to determine the presence or absence of roots, triceps by C6-7, Hoffmann C8, patellar unilateral disease. If there is PNS disease, local- L3-4, and Achilles S1. ized hyporeflexia may help to differentiate plexus, focal root, and nerve pathology. For many neu- Hoffmann sign is elicited by hyperextending rologists, minor asymmetries can be of great the third finger and flicking the nail down- diagnostic significance or simply insignificant. ward; a positive sign is quick flexion of the Furthermore, the technique used to elicit reflexes distal phalanx of the thumb. Only an asym- may vary between examiners. Consequently, this metry is abnormal. section of the examination is quite susceptible to misinterpretation which makes reflex findings The biceps reflex is elicited by the examiner often less valuable than functional testing. The placing her thumb on the biceps tendon and tap- latter includes rapid alternating movements, fin- ping it. Triceps and brachioradialis reflexes are ger and foot tapping, finger-to-nose testing, heel- usually more easily elicited (see Fig. 4.14). A dif- to-shin testing, and heel and toe-walking. ferent position can be used for the triceps reflex; Examination of manual strength is particularly the arm can be held up in the abducted position at useful for PNS/muscular disease, but less so for the shoulder without the patient’s assistance CNS disease when compared to functional tests. before tapping the tendon. Hoffmann sign is elic- ited by hyperextending the third finger and flick- A major purpose of the reflex examination is ing its nail downward. A positive Hoffmann is a to distinguish between central nervous system quick flexion of the distal phalanx of the thumb (CNS) and peripheral nervous system (PNS) (see Fig. 4.15). Only an asymmetry is definitely disease. abnormal. Bilaterally positive Hoffmann signs Reflex Examination 89

Fig. 4.14 Brachioradialis reflex. The arm is held in flexion at the elbow a little less than 90° and pronated halfway. The tendon is struck just above the styloid process

Fig. 4.15 Hoffmann sign. The patient’s hand is fixed in the pronated position and the middle finger is hyperex- tended. The patient’s nail is then strongly and firmly flicked downward by the examiner’s thumb. A positive response is flexion of the terminal phalanx of the patient’s thumb are commonly found in normal individuals. When examining knee and ankle reflexes, it is often best for the patient to be sitting with good body sup- port, the popliteal fossa just at the edge of the examining table. When sitting far forward, many Fig. 4.16 Ankle reflex. This is best obtained with the patients have a tendency to flex the knees and patient seated at the table edge with his legs dangling. The foot is partially dorsiflexed by placing upward pressure on thus inhibit free extensor movement. The ankle the ball of the foot. The Achilles tendon is then firmly- reflex requires both hands of the examiner, one to-strongly tapped for the reflex hammer and the other to put slight upward pressure on the ball of the foot (see of trace, 1+, 2+, 3+, or 4+ with 4+ indicating Fig. 4.16). ­clonus. Some neurologists use only 1+ to 4+ thus eliminating the option of trace. Clonus is a series Reflexes are judged by velocity of movement, of repetitive jerks, usually three or more, elicited amplitude, and relaxing phase in descending by a single tap. Clonus is considered sustained if order of importance. it does not stop at a maintained position held by the examiner. Sustained clonus is usually elicited Reflexes are judged by velocity of movement, at the ankles and ordinarily requires persistent amplitude, and relaxing phase in descending order mild upward pressure on the ball of the foot. of importance. Velocity of movement is most Sustained clonus is always abnormal. Unsustained sensitive. They can be graded by using a system clonus is abnormal only if asymmetric. 90 4 Neurologic Examination

Enhancement Technique ited by scratching the abdomen, in an oblique fashion, in each of the four quadrants. This reflex, When a reflex cannot be obtained, enhance- which is multisynaptic, pulls the umbilicus ment techniques can be useful. towards the stimulus. The side of the absent reflex is abnormal and indicates ipsilateral pathology When a reflex cannot be obtained, enhance- affecting the corticospinal tract usually in the spi- ment techniques can be useful. If unobtainable in nal cord. If only the upper abdominal reflexes are the arms, the patient can be requested to grit his present, there may be a lesion at T10 or T11 sup- teeth. If knee or ankle reflexes cannot be elicited, pressing the lower abdominal reflexes. Beevor gritting the teeth, making a fist, or the Jendrassik sign may then be present; head flexion causes the maneuver can be utilized. The Jendrassik maneu- umbilicus to move upward. It is important to real- ver is interlocking the fingers of both hands and ize that normal individuals often have absent pulling on them. One or two seconds after the abdominal reflexes. The cremasteric reflex is maneuver is made, the reflex should be assessed. elicited by scratching the inner thigh which Occasionally, the originally absent reflex on both causes the cremasteric muscle to contract, thereby sides becomes asymmetric such as present on elevating the testicle on that side. Unilateral one side, absent on the other, or 2+ on one side, absence may indicate ipsilateral pathology in the and 1+ on the other thus indicating localizable corticospinal tract. pathology. Two other reflexes are infrequently useful. The superficial anal reflex (S2 through S5) is manifested by contraction of the anal sphincter Reflex Aberrations when pricking the mucus membrane in the peria- nal area. The bulbocavernosus reflex (S3–S4) is a Reflex spread should be noted. For example, a tap contraction of the anal sphincter detected by a on the brachioradialis tendon may generate both gloved finger in the rectum when the glans penis the intended reflex and finger flexion. The knee is pinched. reflex may induce adduction at the hip. These responses are common with corticospinal tract lesions, but definitely significant only if asym- Abnormal Reflexes metric since some normal individuals exhibit reflex spread. An inverted reflex is an absent The Babinski sign indicates corticospinal tract reflex associated with reflex spread. For instance, disease, located in cerebrum, brainstem, and tapping the triceps tendon produces flexion rather spinal cord. than extension at the elbow. One possible expla- nation could be a C7 root lesion associated with a myelopathy. Slow relaxing phases of reflexes, Testing the plantar response may elicit the especially the ankle jerk, have been found to be a Babinski sign, the prototypical finding of corti- good predictor of hypothyroidism. cospinal tract disease (see Fig. 4.17). Therefore, it is present only with lesions affecting the motor portions of the cerebral hemispheres, brainstem, Superficial Reflexes and spinal cord. Strictly localized lesions of other parts of the CNS such as cerebellum, hypothal- The superficial abdominal reflex is abnormal amus, occipital lobes, etc., will not produce a on the absent side. Bilateral absence is not an Babinski sign. abnormal sign. The elicitation of this sign should begin with a brief explanation to the patient as to what the The abdominal and cremasteric reflexes are stimulus will feel like. Demonstrating it first on the two most commonly used. The former is elic- the patient’s hand can put the patient at ease. Reflex Examination 91

on one side, and no response (mute) on the other may have diagnostic import.

The Babinski sign is determined by the first movement of the big toe; when this movement is definite, continuing the stimulus is unnecessary.

There are other methods of obtaining an abnormal plantar response, extensor movement of the big toe. The two most commonly used when there is an equivocal Babinski sign are the Chaddock and Oppenheim methods (see Fig. 4.17). The Chaddock is probably the most Fig. 4.17 Extensor plantar responses. The Babinski, reliable second choice. It is performed by using Oppenheim, and Chaddock signs are most useful. The the same stimulus as that used for the Babinski Babinski sign is elicited with slow firm pressure beginning at the lateral part of the heel and moving upward as shown sign applied along the lateral surface of the foot above. The first definitive movement of the big toe should beginning near the heel and extending up to the end the test. This is often obtained within 2 in. of begin- fifth toe. The stimulus is a slow deep pressure. ning the stimulus. The same movement may be obtained Oppenheim sign is produced by the examiner with Chaddock and Oppenheim tests, as demonstrated dragging the knuckles slowly and firmly down the tibia from knee to ankle and can be combined A simple statement might be, “This is a reflex test with the plantar and Chaddock methods. An to check the spinal cord and brain.” Otherwise, advantage of both the Oppenheim and Chaddock many patients believe it is a test to determine method is a reduced chance of eliciting a with- whether they feel the stimulus and often evokes drawal response. immediate distrust and dislike of the examiner. The patient’s leg should be extended at the The grasp reflex is elicited by placing two fin- knee. A firm pressure with a key, broken tongue gers in the palm of the patient’s hand and blade, or other similar object is applied to the drawing the fingers out between the thumb lateral portion of the sole of the foot near the heel and the index finger. and slowly moved up to the base of the fifth toe. It is an uncomfortable stimulus, but does not have The grasp reflex is elicited by placing two to be painful and should not break the skin. fingers in the palm of the patient’s hand and Infrequently, when there is no response, the stim- drawing the fingers out between the thumb and ulus can be turned and continued along the base the index finger. When the patient involuntarily of the other toes. The first movement of the big grasps the examiner’s fingers, the reflex is pres- toe determines the reflex. Once this movement is ent. This is a normal reflex in the first few months clearly visible, continuing this stimulus is of no of life, but reappears most commonly with cere- value. Upward movement is abnormal and is bral degenerative diseases. A unilateral grasp usually obtained within 2–3 in. of beginning the reflex usually indicates contralateral frontal lobe stimulus. Occasionally, it may be difficult to dis- disease. tinguish between a withdrawal response and a The glabellar reflex is obtained by repeatedly Babinski sign. Fanning or abduction of the toes is tapping the bridge of a patient’s nose. A positive not required and its presence without big toe response, also known as Myerson sign in extension has no diagnostic significance. Parkinson’s disease, is manifested by repeated eye Asymmetry of the plantar response, strong ­flexion blinks, whereas a normal response is usually no 92 4 Neurologic Examination more than two or three blinks. This is also com- Table 4.9 Anatomic site of lesion/reflex change monly present in cerebral degenerative diseases. Site of lesion Reflexes Cerebral (cortical Increased The glabellar reflex is obtained by repeatedly and subcortical) tapping the bridge of a patient’s nose and a Extrapyramidal Unchanged positive sign (Myerson’s sign) is manifested Brainstem Increased by repeated eye blinks. Cerebellum Unchanged or pendular Spinal cord *Increaseda The sucking reflex can be elicited by stimulat- Root Decreased ing the perioral regions. A rooting reflex may be Plexus Decreased Nerve Decreased obtained by tactile stimulation just lateral to the Neuromuscular junction Unchanged mouth. The response is a turn of the lips, mouth, Muscle Unchanged and sometimes the head toward the stimulus. a Exceptions: Anterior horn cell involvement such as These reflexes are observed primarily as a late occurs with amyotrophic lateral sclerosis will decrease development in cerebral degenerative diseases. reflexes

1. Reflexes are commonly normal with diseases Reflexes of Questionable Significance involving the CNS or PNS. 2. Generalized symmetrical is The snout reflex is elicited by tapping the upper lip not, as an only sign, pathologic. of the patient. The response is a prompt pouting 3. Generalized symmetrical hyporeflexia or even and protrusion of the lips. In essence, it is hyper- areflexia is not, as an only sign, pathologic. reflexia of the orbicularis oris muscles and thus 4. Unilateral hyperactive reflexes are abnormal commonly present with lesions affecting frontal on the side that is hyperactive and indicates lobes and corticobulbar pathways in subcortical pathology involving the corticospinal tract in and midbrain regions. However, it has been occa- cerebrum, brainstem, or spinal cord. sionally well documented in healthy individuals 5. Asymmetric reflexes affecting only one limb and thus of no definite pathologic significance. may indicate a CNS lesion involving the The palmomental reflex is obtained by scratch- ­corticospinal tract affecting the hyperactive ing the palm of a patient’s hand. This may cause side or a plexus/root/nerve lesion on the ipsilateral contraction of the mentalis and, less hypoactive side. The decision will hinge on a often, the orbicularis oris muscles. As it is often good history. present in normal individuals, it is of no clear diagnostic significance. This reflex is probably Babinski signs, unilateral or bilateral, and more common in patients who have cerebral ­sustained clonus are always abnormal. degenerative diseases. 6. Babinski signs, unilateral or bilateral, are always abnormal indicating pathology affect- Synopsis ing the corticospinal tract in cerebrum, brain- stem, or spinal cord. When unilateral, the See Table 4.9 lesion is in the ipsilateral cord or the contral- ateral cerebral hemisphere and brainstem. Major Principles 7. Hoffmann signs, when unilateral, usually indicate pathology affecting the corti- Reflexes are commonly normal with diseases cospinal tract in the ipsilateral spinal cord involving the central nervous system or the or contralateral cerebral hemisphere and peripheral nervous system. brainstem. Sensory Examination 93

8. Asymmetry of reflexes in arms vs. legs, The examiner must take all of the above-noted ­particularly hyperactive only in the legs, is influences into consideration before reaching a suggestive of thoracic myelopathy. final conclusion about the sensory examination. 9. Sustained clonus is always abnormal. An additional cautionary note is to avoid prema- Unsustained clonus is only abnormal if ture dismissal of the results of the exam as indic- asymmetric. ative of psychogenic causes or malingering. 10. Absence of a single reflex is most common Numerous embarrassing and dangerous errors with a root or focal nerve lesion. have been made with such impetuous diagnoses. It is safer to err on the side of organicity. The purpose of the sensory examination is to Sensory Examination uncover unilateral deficits, distal sensory loss, and to search for specific patterns of sensory loss The sensory examination of one patient by two depending on the history. Checking for sensory physicians performed hours apart is seldom iden- levels when spinal cord disease is suspected is tical unless the lesion is acute and severe. A pin critical. Sacral sensation, perianal especially, may or vibration stimulus is not likely to be given with be particularly involved with conus medullaris the same force. Testing position sense can be and cauda equina lesions. A neurogenic bladder done with different degrees of movement and at mandates this assessment. Root, plexus, or focal different speeds. nerve pathology require a specific outline of the distribution of sensory loss. The sensory exam may be affected by fatigue, There are four primary sensory modalities, somnolence, fear, and drug effect. pain/temperature, vibration, position sense, and touch. Large myelinated fibers, fast conducting, There are confounding factors which may carry sensations of vibration, position sense, and alter the patient’s responses such as fatigue, som- touch. Small unmyelinated, slower conducting nolence, fear, and drug effect. Oppositional and fibers mediate pain and temperature. pessimistic attitudes must be detected and taken into account. The patient’s demeanor and verbal Small unmyelinated, slower conducting fibers responses should be a tip off. mediate pain and temperature. Large myeli- Delayed responses may be due to bradyphrenia nated, quick conducting fibers mediate vibra- (slow thinking) associated with dementia, mental tion, position, and touch sensitivity. retardation, or depression. An obsessive, indecisive individual or one who is somatically preoccupied may cogitate over each answer and magnify minor Nerve fibers conducting pain and temperature sensory differences. One method which may enter the spinal cord via the dorsal horn, usually compensate for these responses is to ask the patient; descend one or two segments and synapse there. “If the good side is worth $1.00, what is the bad Then the fibers cross immediately via the anterior side worth?” Many answers are in the range of commissure, ascend in the lateral spinothalamic 70–95 cents and can be construed as unimportant. tract, synapse in the thalamus (ventral posterior There are patients who wish to control the results lateral (VPL) nucleus), and subsequent axons ter- of the examination and willfully or erroneously minate in the parietal lobe. Sensory testing is deny perception of a stimulus. Willful denial occurs done with pin and/or cold. The latter is most sim- with patients who are malingering or have a con- ply done with the tines of the tuning fork, although version reaction. Distinguishing between these two they may become warm quickly with prolonged diagnoses can be quite vexing. Much less often are or repeated use. Patients are occasionally fright- those patients who erroneously report perception ened of pins, and then, temperature testing is an of a stimulus, perhaps to reduce the chances of excellent substitute. Either stimulus is applied at discovering and facing a serious illness. the toes first and slowly moved upward to find a 94 4 Neurologic Examination sensory level. When the patient’s responses to pin gait, positive Romberg, finger-to-nose and heel- are inconsistent, observing the patient’s expres- to-shin ataxia. Pseudoathetosis, wandering move- sion or wince is more reliable. Distal sensory loss, ments of the limb, usually an arm, may occur. sensory levels, dermatomal, and nerve distribu- tion patterns are sought for. Severe proprioception impairment may result in an ataxic gait, positive Romberg, finger-to- When the patient’s responses to pin are incon- nose, and heel-to-shin ataxia. sistent, observing the patient’s expression or wince is more reliable. Loss of light touch perception on the toes is a Vibration, proprioception, and most touch strong confirmatory finding of a significant fibers are mediated by large myelinated, rapid con- neuropathy. ducting axons. They enter the dorsal horn, ascend in the posterior (dorsal) columns, decussate in the The sensation of light touch is mediated by medial lemniscus (medulla), ascend to the VPL both large myelinated fast-conducting fibers which nucleus of the thalamus where they synapse, and ascend in the posterior columns and to a limited subsequent axons terminate in the parietal cortex. extent small unmyelinated fibers which ascend in Vibration sense perception depends at least par- the ventromedial part of the spinothalamic tract. tially on the intensity of the stimulus which is best Since touch fibers are widely distributed, the kept approximately uniform with each test. Loss assessment of light touch seldom yields diagnostic of vibration sense at the toe joints mandates a information. One exception is the occasional loss search for a vibration sensory level at ankle, knee, of light touch on the toes which is a strong con- hip, and spine in that order. In addition to immedi- firmatory finding of a significant neuropathy. ate perception of vibration, the stimulus should be kept in place for several seconds. There are patients who perceive the stimulus for only 1 or 2 s, known Sensory Abnormalities According as rapid adaptation. This is probably an abnormal to Lesion Site finding in patients under age 70, although there are no established criteria. Vibration sense is usually 1. Neuropathy. lost before position sense for lesions involving Large fiber neuropathies are first detected by thalamus and below. testing of vibration. Initially, there is usually rapid adaptation to the stimulus which may be Loss of vibration sense at the toe joints man- perceived for only a few seconds. Position dates a search for a vibration sensory level at sense loss is a much later development. Small ankle, knee, hip, and spine in that order. fiber neuropathies are manifested by distal loss of pain and temperature perception, the latter Proprioception loss (position sense) is exam- usually being more sensitive. The legs are ined with either of two methods. These are the invariably involved first. Loss of light touch direction of movement or whether the resultant usually occurs after pain and temperature sen- position is above or below the resting position. sitivity is impaired. However, when impaired, Clearly, the patient must be given the appropriate the diagnosis of neuropathy is definite. instructions. Slow movements are harder to 2. Plexopathy. detect, so the examiner needs to have a uniform approach. The exam begins with the metatarsal- There is a predilection for motor dysfunc- phalangeal joint of the big toe and a distal inter- tion in plexopathies even though sensory phalangeal joint of a finger. If either is abnormal, symptoms, especially pain, are prominent. a sensory level is sought for at wrist, elbow, and shoulder and/or ankle, knee and hip. Severe pro- There is a predilection for motor dysfunc- prioception impairment may result in an ataxic tion in plexopathies even though sensory Sensory Examination 95

symptoms, especially pain, are prominent. the hand and arm from T1 through C5. C4 and There may be patchy loss to pin. An example higher will be evident on the shoulder, neck, is the lumbosacral plexopathy which occurs and occipital region. with diabetes. This causes excruciating pain, Vibration and position sense loss is ipsilat- disabling weakness of quadriceps, and iliop- eral with vibration loss occurring first. Sensory soas musculature, yet just minimal sensory dissociation is a term which refers to preserva- loss along the medial calf (saphenous nerve). tion of vibration and position sense and The saphenous nerve is a branch of the femo- impairment of perception of pin and tempera- ral nerve. ture. This is a common phenomenon in patients 3. Radiculopathy. with intramedullary (within cord) lesions such as syringomyelia or astrocytoma. Most patients with radiculopathy have ­little or no sensory impairment even if sensory Sensory dissociation is a term which refers symptoms such as pain and paresthesias to preservation of vibration and position are prominent. sense and impairment of perception of pin and temperature. Most patients with radiculopathy have little or no sensory impairment even if sen- Light touch is usually preserved because it sory symptoms such as pain and paresthesias is mediated by both ipsilateral (dorsal column) are prominent. A dermatomal pattern of sen- and contralateral (spinothalamic) pathways. It sory loss to pinprick and, infrequently, light should be noted that rapid adaptation to a touch may be present. However, a meticu- stimulus, either pin or vibration, may occur lous search for touch and pinprick impair- prior to total loss of perception. ment is seldom worth the time investment. Sacral sensation should be evaluated since Vibration and position sense are nearly intramedullary lesions (inside spinal cord) always intact, presumably because multiple may spare sacral fibers (“sacral sparing”) roots mediate these sensory functions. which are situated in the lateral portion of the Multiple extensive radiculopathies and dorsal spinothalamic tract. Conversely, cauda equina root ganglionopathies will impair all sensory and conus medullaris lesions typically cause modalities. asymmetric sacral sensory loss along with the 4. Myelopathy. expected bowel, bladder, and sexual dysfunc- tion and motor involvement of sacral inner- Myelopathies cause sensory loss to pin and vated musculature. temperature contralateral to the lesion, typ- ically a sensory level one or two segments Sacral sensation should be evaluated since below the level of the lesion. intramedullary lesions (inside spinal cord) may spare sacral fibers (“sacral sparing”). Myelopathies cause sensory loss to pin and temperature contralateral to the lesion, typi- 5. Brainstem. cally a sensory level one or two segments Brainstem lesions cause contralateral hemi- below the level of the lesion. The sensory level hypesthesia to pain/temperature, vibration, is often considerably below the lesion level and position sense, the latter two provided the which thus mandates neuroimaging of the lesion is above the decussation of the medial entire spinal cord. The rough anatomic land- lemniscus in the medulla. After partial recovery mark levels are L1 at the groin, T10 at the from a brainstem lesion, typically an infarc- umbilicus, T6–T7 at the edge of the ribcage, tion, sensory levels may be present sometimes and T4 at the nipple. A higher level on the sparing one leg or only exhibiting segmental chest requires a search for a sensory level on sensory loss. The recognition of this residual 96 4 Neurologic Examination

deficit is important as it obviates the need for using, for example, a key, rubber band, and additional neuroimaging. Painful paresthesias cotton. Differentiating hard from soft may be and burning sensations may be residual symp- possible but identification of substance or toms which require treatment. object may be missed. Two-point discrimina- 6. Thalamus. tion is usually assessed at the fingertips where asymmetries are most easily demonstrated. A Thalamic pain syndromes may produce normal detectable separation is 2–4 mm. excruciatingly painful paresthesias, burning Agraphesthesia is the inability to accurately sensations, hyperesthesia, and hyperpathia. detect numbers written on the palmar surface of the hand. The test is done quickly using the Thalamic lesions involving the VPL blunt end of a safety pin or a Q-tip. nucleus may cause dense sensory loss involv- Double simultaneous stimulation with ing all primary sensory modalities. touch may result in perception of the stimulus Concomitant or subsequent thalamic pain syn- on only one side (extinction). Sensory hierar- dromes manifested by excruciatingly painful chy can be employed to elicit the abnormality. paresthesias, burning sensations, hyperesthe- This refers to the dominance of sensory per- sia, and hyperpathia are not uncommon ception in descending order, face-chest-leg- residua. Thalamic stroke, infarction or hemor- hand. Thus, right face, right chest, or right leg rhage, is the usual etiology. Posterior cerebral touch simultaneous with a left hand touch may artery branches irrigate this region. result in the absence of perception of the touch 7. Cerebral. on the left hand. Yet bilateral hand touches may be easily perceived and is thus a less sen- Sensory cortex lesions have a predilection sitive method of examination. for involvement of the hand and position sense loss is preferentially involved. Nonorganic Sensory Loss Sensory cortex lesions, mainly parietal but also posterior frontal, are less likely to cause Nonorganic sensory loss should be suspected complete hemihypesthesia. There is a predi- when the findings do not follow an anatomic lection for involvement of the hand and position pattern, for example, sensory loss stopping sense is preferentially involved. There is seldom precisely at the midline since sensory fibers loss of vibratory sense. for pain overlap the midline.

Double simultaneous stimulation, stereog- nosis, graphesthesia, and two-point dis- The sensory findings do not follow an anatomic crimination are useful tests for cerebral pattern. There may be loss of all sensory modali- lesions. ties – vision, hearing, vibration, position, pin, touch, even smell – on one side. Sensory loss typi- More complex sensory abnormalities are cally stops precisely at the midline even though often present. The proviso for making that judg- sensory fibers for pain and temperature overlap ment is intact primary sensory modalities, pin, and thus the stimulus should be perceived prior to touch, and vibration. Such sensory findings as reaching the midline. On the face the overlap is described below may also be discovered in high minimal. Responses are commonly inconsistent if cervical cord lesions and thus the term cortical the examination is repeated the following day or sensory loss is not entirely appropriate. even a few hours later. Sensory responses may A common abnormality is astereognosis, occasionally be obtained, evident by a wince or the inability to detect form, shape, size, and withdrawal with pinprick stimulation of the palm texture. This is easy to demonstrate simply of the hand or the sole of the foot. Dermatome Patterns 97

Sensory Terminology 9. Paresthesias – Spontaneous sensations occur- ring without stimulation often described as 1. Allesthesia – Perception of a stimulus on the prickling, burning, crawling, and itching. contralateral limb in the same location. 2. Allodynia – Pain evoked by a nonpainful stimulus. Dermatome Patterns (Fig. 4.18) 3. Anesthesia – Absence of any sensation. 4. Anesthesia-dolorosa – Spontaneous pain A familiarity with dermatome patterns is useful located in an area of anesthesia. This is char- for the everyday examination of patients. The acteristic of herpes zoster. commonly used landmarks with frequent varia- 5. Dysesthesia – Unpleasant sensations provoked tions include: by an ordinary stimulus. C6, C7, C8 on the hand. The thumb is largely 6. Hyperesthesia – Acute sensitivity to a sensory C6; the index, third, and part of the fourth fin- stimulus. gers are C7; and part of the fourth plus fifth 7. Hypesthesia – Decreased perception of a finger is C8. stimulus. C4 is at the clavicle and just beneath it is T2. 8. Hyperpathia – Exaggerated response to a painful T4 is at the nipple. stimulus. T10 is at the umbilicus.

Fig. 4.18 Dermatomes 98 4 Neurologic Examination

L1 is at the groin. Rare etiologies to be cognizant of in puzzling L4-5, S1 are on the foot. L4 is the medial part clinical situations include incipient cerebellar of the big toe, L5 is the medial portion of the dor- tonsillar herniation and retropharyngeal abscess. sum and sole of the foot from the big toe to the Ultimately, clinical judgment based on per- fourth toe. sonal experience will be required to make the S1 is the lateral part of the foot, dorsum, and decision about performing a lumbar puncture. In sole including the fifth toe. critically ill patients, a normal CT (head) associ- ated with or without papilledema should ease the fears of provoking a herniation syndrome. In Mechanical Signs questionable cases, it is best to opt for perfor- mance of a lumbar puncture since possible etiolo- Meningeal Disease gies are meningitis, subarachnoid hemorrhage and, least likely, leptomeningeal carcinomatosis. Nuchal rigidity is the cardinal manifestation of meningeal irritability. The patient is unable to voluntarily put his chin on his chest. When supine and at rest, the patient’s neck cannot be fully Cervical and Lumbar Radiculopathy flexed by the examiner. There are variable degrees of resistance ranging from immediate, severe, Head extension is the most useful mechanical and rock hard to minimal, detectable only at the test for the diagnosis of cervical radiculopathy. final few centimeters, called “end nuchal rigidity.” Head extension is the most useful mechanical test Brudzinski sign is obtained by placing one for the diagnosis of cervical radiculopathy since hand firmly on the patient’s chest, then lifting the it narrows the intervertebral foramina and is back of the head. When positive, the patient will likely to augment compression of a nerve root. flex the hips and knees. Brief extension for a few seconds is commonly Kernig sign is described in a few ways, both in not diagnostic. Prolonged extension for 1 min is a sitting and supine positions. In the supine posi- very sensitive and specific sign which frequently tion, the hip and knee are both flexed to 90°. Then reproduces the patient’s symptoms of pain and/or the leg is extended at the knee another 45° which paresthesias. Head tilts, turns, and axial compres- results in resistance and pain. Straight leg raising sion are much less sensitive. produces similar results. This sign is rarely of clinical utility for meningeal disease. Head flexion which causes neck pain supports the presence of muscular etiology. The interpretation of nuchal rigidity is affected by limitations of head movement in other direc- Head flexion which causes neck pain supports tions and the presence or absence of fever. the presence of muscular etiology. It may also elicit Lhermitte sign, electricity-like sensations The significance of nuchal rigidity is - deter which radiate down the spine and legs. Although mined by the absence of other signs. These are classically associated with multiple sclerosis, it limitations of head tilt, head turn, and head may occur with any pathology affecting the spi- extension. These findings, when unaccompanied nal cord, especially degenerative diseases such as by fever or systemic illness, imply other etiolo- a herniated cervical disk or spinal stenosis. gies. They include cervical muscle strain, ante- When arm movements at one joint alone pro- rior interlocking osteophytosis, neck trauma, and voke radiating pain, even neck pain, the source is the severe rigidity of extrapyramidal disorders. generally in the involved painful joint, most com- Parkinson-plus syndromes such as progressive monly the shoulder. Etiologies include tendonitis, supranuclear palsy commonly produce nuchal rotator cuff injuries, capsulitis, and impingement rigidity. syndrome. Questions (True or False) 99

Thoracic outlet syndrome maneuvers are the Straight leg raising is positive if pain is pro- sternoclavicular, Adson’s, and hyperabduction. voked between the elevations of 30 and 70°. Unfortunately, false positive tests are common and the syndrome itself is rare. The sternoclavic- The straight leg raising test (Lasègue) is the ular test is performed by a retraction and down- primary test for lumbar root irritation. When the ward movement of both shoulders. Adson’s adds patient is supine, the extended leg is slowly ele- turning of the patient’s head to one side and tak- vated. If pain is elicited between 30 and 70°, lum- ing a deep breath. The hyperabduction maneuver bar root irritation is suspected. However, a tight adds to the Adson’s maneuver by abduction of sensation in hamstring muscles is relatively com- the shoulder with flexion at the elbow of the arm mon and not pathologic. If back pain or radicular opposite the direction of head turn. A positive pain below the popliteal fossa is elicited, then the test is reproduction of symptoms sometimes test is positive. The addition of ankle dorsiflexion accompanied by obliteration of the pulse and is a valuable provoking maneuver. production of a supraclavicular bruit on the side Crossed positive straight leg raising is a par- opposite the head turn. ticularly useful sign. When the patient is supine, elevation of the asymptomatic leg elicits radicu- lar pain in the symptomatic leg. This virtually Questions (True or False) secures a diagnosis of lumbar radiculopathy. Pain produced with elevation of the leg over 1. Orthostatic hypotension may be a cause of 70° is of questionable significance, quite doubt- loss of balance. ful if only in the posterior thigh. Pain provoked 2. Dysarthria is a good localizing sign. by less than 30° elevation is probably not organic. 3. Complete destruction of the superior tem- Pain which is worse with knee flexion or hip rota- poral gyrus causes contralateral hearing tion points to a source in the hip, muscle, or is loss. nonorganic. 4. Involuntary tongue movements may be due Straight leg raising in the sitting position is a to tardive dyskinesia. valuable additional test to assess for the presence 5. Asymmetric palpebral fissures may occur of lumbar radiculopathy. When the supine straight with cerebral lesions. leg raising test is questionable, this test, espe- 6. Light-near dissociation of pupillary reactions cially with the addition of ankle dorsiflexion and does not occur with optic nerve disease. head flexion, may evoke lumbar root pain and 7. The pupil with Horner’s syndrome reacts clinch the diagnosis. If the test is negative, the briskly to light. presence of lumbar root pathology is much less 8. Seizures may cause multidirectional likely. nystagmus. Tinel sign is produced by tapping over any 9. Peripheral vestibular disease causes horizon- peripheral nerve. Paresthesias are evoked in the tal gaze-evoked nystagmus. distribution of that nerve when the test is positive. 10. The hallmark of an internuclear ophthalmople- Phalen sign is produced by forced flexion of the gia is ipsilateral medial rectus weakness. wrist. This may evoke paresthesias in the distri- 11. Impaired rapid alternating movements are bution of the median nerve, especially when the pathognomonic of cerebellar system position is maintained for 30–60 s. Both tests are disease. commonly used to support a diagnosis of a carpal 12. Paratonic rigidity is a common sign in tunnel syndrome. patients with dementia. 13. Prolonged head extension is a useful diag- The thoracic outlet syndrome is rare and false nostic test for cervical radiculopathy as it can positive tests are common. provoke radicular pain. 100 4 Neurologic Examination

14. When swallowing provokes ear pain, 9. F glossopharyngeal neuralgia must be 10. T suspected. 11. F 15. Loss of position sense in an arm may cause 12. T an upward arm drift. 13. T 16. A positive Romberg occurs only with prop- 14. T rioception loss. 15. T 17. Reflexes are increased with a cerebellar 16. F hemisphere lesion. 17. F 18. Tonic downward eye deviation may occur 18. T with metabolic encephalopathies. 19. T 19. Naming is impaired in most aphasias. 20. T 20. Good repetition occurs with lesions outside 21. F the perisylvian region. 22. F 21. In a conduction aphasia repetition is normal. 23. T 22. Afferent pupillary defects occur with both 24. T retinal and optic nerve disease. 25. T 23. When disk margins are blurred, no hemor- 26. T rhages are present and venous pulsations are 27. F seen the patient does not have papilledema. 28. F 24. Gerstmann’s syndrome is manifested by finger 29. T agnosia, acalculia, agraphia, and right–left 30. T confusion. 25. Periodic alternating nystagmus occurs with lesions at the foramen magnum. Bibliography 26. Inability to imitate a gesture is a sign of an ideomotor apraxia. Bender MB, editor. The oculomotor system. Hoebers 27. Adie’s syndrome occurs with lesions of the medical division. New York: Harper & Row; 1964. Brazis PW, Masdeu JC, Biller J. Localization in clinical optic nerve. neurology. 5th ed. Philadelphia: Lippincott Williams 28. The Dix-Hallpike test is useful to diagnose & Wilkins; 2007. any peripheral vestibular disorder. Campbell WW. De Jong’s neurologic examination. 29. Corticolingual fibers supply both sides of the Philadelphia: Wolters Kluwer/Lippincott Williams & Wilkins; 2008. tongue. Chusid JG. Correlative neuroanatomy and functional neu- 30. Severe motor restlessness of the legs is called rology. 15th ed. Los Altos: Lange Medical Publications; akathisia. 1973. Critchley M. The parietal lobes. New York: Hafner; 1966. Digre KB, Corbett JJ. Practical viewing of the . Answers Burlington: Butterworth Heinemann; 2003. Furman JM, Cass SP. Balance disorders. A case study 1. T approach. Philadelphia: FA Davis; 1996. Glaser JS. Neuro-ophthalmology. 2nd ed. Philadelphia: 2. F J. B. Lippincott; 1999. 3. F Hillis AE. Aphasia. Progress in the last quarter of a 4. T century. Neurology. 2007;69:200–13. 5. T Ross RT. How to examine the nervous system. 4th ed. Totowa: Humana; 2006. 6. F Walsh TJ, editor. Visual fields. Examination and inter- 7. T pretation. San Francisco: American Academy of 8. F Ophthalmology; 1990. Evaluation of the Poorly Responsive Patient 5

Keywords Coma ¥ Encephalopathy ¥ Delirium ¥ Brain death ¥ Obtundation ¥ Stupor ¥ Herniation

Evaluation of the poorly responsive patient neously and to verbal, visual, and other sensory requires an organized approach. One aim of this stimuli. Nevertheless, it is worthwhile to review the chapter is to meticulously outline an examination most succinct descriptions as outlined in “Diagnosis technique, delineate the anatomic basis for the of Stupor and Coma” (Posner et al. 2007 ). observed abnormal neurologic signs, review perti- nent neurophysiology, and lastly, discuss the dif- ferential diagnosis. The differential diagnosis is Clouding of Consciousness formulated after answering a primary question. (Lethargy) Does this patient have a focal lesion, multifocal lesions, or global cerebral dysfunction? This seem- This state is manifested by reduced wakefulness ingly simple problem is often a formidable one. and awareness. The patient is inattentive, dis- tractible, and incompletely oriented. Nocturnal The primary question when evaluating a agitation is common. poorly responsive patient is distinguishing between a focal lesion, multifocal lesions, or global cerebral dysfunction. Delirium Common specifi c diagnostic problems of criti- The typical patient is disoriented, frightened, cally ill patients will be reviewed afterwards. These irritable, and often delusional. Hallucinations are include the encephalopathies of metabolic and common and typically visual. The anatomic basis hypoxic-ischemic origin. Herniation syndromes is bilateral cerebral dysfunction but, on rare due to focal lesions with mass effect are next. occasions, there is right posterior cerebral hemi- Lastly, the chronic disorders of consciousness and sphere pathology. Sepsis, especially urosepsis, brain death will be discussed. metabolic and toxic encephalopathies are the The defi nition of the levels of impaired con- most common etiologies of delirium. sciousness has been a source of debate for many years. Employing specifi c terms is fraught with haz- Defi ned alterations of consciousness include ard since there are often differing views as to what lethargy, delirium, obtundation, stupor, and term is most appropriate. The best approach is to coma. simply outline what the patient is able to do sponta-

J.N. Alpert, The Neurologic Diagnosis: A Practical Bedside Approach, 101 DOI 10.1007/978-1-4419-6724-4_5, © Springer Science+Business Media, LLC 2012 102 5 Evaluation of the Poorly Responsive Patient

of the limbs, head and eyes? For example, eye Obtundation deviation to the right may indicate a right cerebral, right subcortical, right midbrain or a left pontine The obtunded patient is somnolent, shows little lesion. Are there abnormal involuntary movements to no interest in the environment and displays or postures? For example, nystagmus or nystag- bradyphrenia (slowed thinking) when roused. moid jerks only to the right may indicate focal seizures emanating from the left cerebral cortex, especially the frontoparietal eye fi elds. A leg exter- Stupor nally rotated may be a sign of a hemiparesis or a hip fracture. Other abnormal movements or Only vigorous stimuli can arouse a stuporous postures to search for include myoclonus, sei- patient. The stimulus must be maintained as a zures, tremor, dystonia, and dyskinesias. brief interruption results in a prompt return to an unresponsive state. Respiration (see Fig. 5.1 )

What Is the Respiratory Pattern? Coma Inspection of the patient must include obser- The comatose patient is unarousable with eyes vation of the respiratory pattern. closed. In light coma pain may produce facial grimacing, posturing or withdrawal responses. Post hyperventilation apnea is induced by asking There is no localization of pain demonstrated by the patient to take fi ve deep breaths. Patients with the absence of defensive movements. In deep bilateral frontal lobe pathology may have a period coma there is no response to pain. of apnea that lasts more than 10 s. Apnea less than 10 s may be seen in normal individuals. Cheyne-Stokes respiration is hyperpnea fol- Neurologic Examination lowed by apnea in a crescendo-decrescendo fash- ion with apnea ranging from 10 to 20 s. It may Inspection occur with bilateral subcortical and diencephalic lesions and indicates preserved brainstem func- The examination begins with a brief inspection of tion. It is most common in the absence of neuro- the patient lying in bed. What are the positions logic disease in patients with severe congestive

Abnormal Respiratory Patterns in Central Nervous System Disease

Cheyne-Stokes Respiration Diencephalic disease Central transtentorial herniation Metabolic encephalopathies

Central neurogenic hyperventilation Midbrain neoplasm Metabolic acidosis Respiratory alkalosis

Fig. 5.1 Abnormal Apneustic breathing respiratory patterns in Bilateral lesions in pons central nervous system disease or metabolic encephalopathies Ataxic breathing Neurologic Examination 103 heart failure because of slowed velocity of aerated limit. An intubated patient should be requested blood from the weak left ventricle to chemorecep- to nod yes or no to these simple commands. To tors in the carotid body. determine if the responses are accurate, he is Central neurogenic hyperventilation is a rare given a few questions in succession such as, phenomenon that may occur with upper brain- “Are you at home? Are you in the hospital? stem lesions, especially neoplasms. The neo- Are you in a drug store?” The patient is plasm is believed to cause localized lactic acidosis requested to nod yes or no to each option. The which provokes a compensatory respiratory response to a question such as, “Do you know drive. Additionally, the diagnosis of pure central where you are?” has no reliable meaning, neurogenic hyperventilation requires an increased especially if the response is a nod yes.

p O2 , decreased p CO 2 and an increased pH. This 2. If the patient does not speak, check for the diagnosis implies the absence of more common ability to follow commands, one through four causes of hyperventilation such as respiratory steps. A one-step or simple command is “open compensation for metabolic acidosis, neurogenic your mouth.” A four-step command would be, pulmonary edema, sepsis, hypoxia, hepatic fail- for example, “with your right hand touch ure, psychogenic hyperventilation, and toxic fac- your left ear.” Each underlined word is a sin- tors such as salicylate poisoning. Psychogenic gle step of identifi cation. If rightÐleft confu- hyperventilation disappears in sleep. sion is suspected, nonlateralizing commands Apneustic breathing is apnea for 2Ð3 s after tak- should be employed such as, “Open your ing a deep inspiration. There may also be a similar, mouth, touch your ear, cross your legs, or brief apneic event after full expiration. Apneustic close your eyes.” If the patient is deaf, large breathing is rare and may occur with pontine lesions written commands on a blank sheet of paper or very severe metabolic encephalopathies. may be required. If the patient cannot follow Ataxic breathing could be called agonal respi- any commands, can he imitate gestures? This rations because of the irregular pattern and gasp- disparity may be present in patients with ing quality. It is ordinarily a preterminal event aphasia. due to medullary damage and typically occurs 3. If the patient does not respond to verbal or with herniation of the cerebellar tonsils through visual stimuli, a painful stimulus should be the foramen magnum. Ondine’s curse may result given. There are four common types of pain from bilateral lesions of the medulla involving stimuli, supraorbital pressure, nail-bed pres- chemosensory structures. The result is the absence sure, bilateral temporomandibular joint pres- of automatic respiratory drive. Yawning, hiccup- sure, and sternal rub. Supraorbital pressure ing, and vomiting are associated features of is effi cacious, the least aggressive or violent lesions of the medulla. and is therefore preferable. The sternal rub produced by grinding one’s knuckles on the pectoralis major can be the most potent Mental Status Examination stimulus but it may be disturbing to onlookers and should be a last resort. This examination can be performed whether or not the patient is intubated. Supraorbital pressure is a preferred method of assessing the response to pain.

Intubated patients must be evaluated for orientation. 4. Reactions to pain in an otherwise unrespon- sive patient are often the most revealing ele- 1. Greet the patient. If the patient can be roused ments of the examination. The aim is to elicit to speak, immediately check orientation to the presence of a focal lesion which obviously person, place, and date. Continue with the has a differential diagnosis that is distinct from mental status examination to the patient’s nonfocal dysfunction, an encephalopathy. 104 5 Evaluation of the Poorly Responsive Patient

Subhyaloid hemorrhages occur with subarach- The reaction to pain often elicits focal noid hemorrhage. Bilateral blurred disk mar- signs. gins with adjacent fl ame and/or splinter hemorrhages indicate papilledema. Venous sta- Some responses indicating a focal lesion sis retinopathy, dot and blot hemorrhages in the include: mid portion of the retina, occurs with common (a) Production of an asymmetric facial or internal carotid artery high grade stenoses or grimace. occlusions. The retinal stigmata of longstand- (b) Production of asymmetric movements of ing hypertension and diabetes may aid in mak- extremities thus uncovering the presence ing the diagnosis. of one or more paretic limbs. The visual fi eld examination can be tried using (c) Attempting to remove a stimulus may double simultaneous stimulation of fi nger motion if unmask a paresis and/or demonstrate the patient is arousable and can respond verbally or some awareness of self not previously by pointing to the fi ngers that move. It must be appreciated. remembered that vigorous motion may be per- (d) Decorticate posturing (see Fig. 5.2 ) is ceived in a hemianopic fi eld (Riddoch’s phenome- manifested by slow fl exion of the arms non). Finger counting is often more reliable as and extension of the legs. The arms are patients whose speech is impaired can mimic the adducted at the shoulders, fl exed at the number of fi ngers fl ashed in the intact fi eld. The use elbows, wrists, and fi ngers; the legs are of visual threat by slowly bringing the fi nger into extended, internally rotated, and plantar the patient’s visual fi eld from the periphery may fl exed. The lesions which produce decor- elicit an attempted ipsilateral and/or contralateral ticate posturing are subcortical, involve eye blink. The eyelid on the side being examined the diencephalon and may extend to the may have to be held open gently. The presence of a midbrain. Fragments of this response have blink only on one side indicates weakness of the the same diagnostic signifi cance. orbicularis oculi on the side of the absent blink.

Decorticate posturing is fl exion of the arms Visual fi elds can be assessed by using visual at the elbows and leg extension while threat. decerebrate posturing is extension of all limbs. Vestibuloocular refl exes are composed of the oculocephalic maneuver and caloric testing. The (e) Decerebrate posturing (see Fig. 5.2 ) is former involves moving the head quickly in characterized by extensor posturing of all one direction to determine whether the eyes move extremities. Additionally, the arms are completely to the limbus in the contralateral direc- adducted and pronated. Although this tion. The normal alert patient does not display this posture is usually provoked by a stimulus, refl ex since normal random saccades will be inter- especially a painful one, it may occur posed. A poorly responsive patient with an intact spontaneously. Lesions which produce oculomotor system will move his eyes to the left decerebrate posturing typically involve when his head is quickly moved to the right and midbrain and upper pons, thus sparing the vice versa. This can also be assessed in the vertical vestibulospinal tracts. plane as quickly moving the head upwards elicits down eye movements and vice versa. Thus, one purpose of the oculocephalic maneuver is to Cranial Nerves expose paretic eye movements due to a 6th nerve, medial longitudinal fasciculus, 3rd nerve or PPRF Funduscopic examination is mandatory even lesion in the horizontal plane and a 3rd nerve, though the diagnostic yield is relatively small. interstitial nucleus of Cajal, posterior commissure, Neurologic Examination 105

Fig. 5.2 Movement responses to painful stimuli (supraorbital pressure) at diencephalic, midbrain-upper pons and lower pons-medullary levels and riMLF lesion in the vertical plane. Bilateral fi ciently in one direction to maintain complete lesions of the riMLF will eliminate both upward adduction whence down eye movements are and downward saccades but only a lesion of the strictly mediated by the superior oblique muscle. interstitial nucleus of Cajal is likely to eliminate In deep coma no eye movements can be induced. the vertical oculocephalic response as it is a relay An important purpose of oculocephalic and station for vestibular projections. Unmasking a 4th caloric testing is the exposure of an eye move- nerve lesion is diffi cult as it requires a vertical ment paresis. oculocephalic maneuver with the head turned suf- 106 5 Evaluation of the Poorly Responsive Patient

Caloric testing with 20Ð30 cc of ice water can provided the patient has an intact peripheral be done at the bedside. Hot water irrigation vestibular apparatus. Thus it is used to diag- requires careful temperature control and is there- nose malingering or a conversion reaction. fore not practical. The head of the bed is elevated 30¡ to bring the horizontal canal into a vertical Caloric testing is an infallible method of position for a maximum response. Secondly, the diagnosing psychogenic unresponsiveness ears are checked for cerumen, which should be assuming an intact peripheral vestibular removed if completely blocking the canal. The apparatus. irrigation is done slowly and steadily, usually over 2Ð3 min and can be halted when an obvious 3. Caloric testing can uncover a medial rectus response is elicited. An emesis basin placed under paresis (3rd nerve or MLF lesion), gaze pare- the ear will catch the water draining out. The sis (lesion of the oculomotor pathway, espe- opposite ear should be examined after a 5-min cially the PPRF), and lateral rectus paresis delay to eliminate a residual response from the (6th nerve lesion). fi rst irrigation. Both ears must be examined since, 4. Bilateral, simultaneous cold water irrigation otherwise, a focal lesion affecting the lateral rec- produces downward eye deviation with upbeat tus and medial rectus muscles or a gaze paresis nystagmus. The absence of upbeat nystagmus can be missed. indicates a posterior commissure or bilateral pretectal lesion. The absence of down devia- Utility of Caloric Testing tion supports the presence of pathology affect- ing the riMLF.

Caloric testing may be utilized to assess level of consciousness. Spontaneous Eye Movements in a Poorly Responsive Patient 1. Assessing depth of unresponsiveness. For the patient with an altered level of con- 1. Aside from drug toxicity, all forms of nystag- sciousness and without extraocular muscle pare- mus suggest brainstem pathology, less often sis, there are four caloric responses to cold water purely cerebellar lesions. irrigation that assess the degree of unresponsive- ness. This can be done in serial fashion to mea- In the unresponsive patient, suspect seizure sure either a decline or improvement in the when horizontal, unilateral jerk nystagmus neurologic status. is seen. The fi rst response is contralateral nystagmus which occurs in a normal alert person. 2. Horizontal, unilateral jerk nystagmus is always The second response is ipsilateral deviation a suspected manifestation of seizure. with contralateral nystagmus. This response is 3. Specifi c types of abnormal eye movements. common when the patient is lethargic or (a) Ocular bobbing. Rapid, conjugate, down- obtunded. ward eye movement with a slow return to The third response is ipsilateral deviation only. primary position at a rate of 3Ð6 per minute. This response occurs in a patient who is obtunded, (b) Reverse ocular bobbing. Rapid, conjugate stuporous, or in light coma. upward eye movement with a slow return The fourth response is no eye movement in a to primary position. patient who is in deep coma. (c) Ocular dipping. Slow downward and rapid 2. Psychogenic unresponsiveness. up movement to primary position. Caloric testing is an infallible method of (d) Reverse ocular dipping. Slow up and a checking for the depth of unresponsiveness, rapid down movement to primary position. Neurologic Examination 107

(e) Ping-pong gaze. Rapid alternating gaze Pupils deviation every few seconds. (f) Periodic alternating gaze deviation. Each Pupils are discussed in detail in the chapter on the deviation lasts approximately 2 min. autonomic nervous system. Pupillary size, shape, 4. All of the above specifi c eye movement abnor- and reactivity are assessed. Signifi cant asymme- malities may be due to metabolic, toxic, try in light is greater than 0.5 mm. Parasympathetic hypoxic, or ischemic encephalopathies. Ocular or sympathetic nervous system pathology should bobbing, however, is a characteristic fi nding explain the asymmetry in the absence of con- after pontine strokes. Periodic alternating gaze founding ocular pathology such as prior surgery, deviation is relatively common in the vegeta- ocular trauma, or diseases of the iris. It should be tive state (to be discussed). remembered that unilateral, optic nerve lesions are associated with equal pupils of normal size but asymmetric reactivity. Eye Deviations In the absence of local eye pathology pupil- Conjugate eye deviations have both anatomic lary asymmetry greater than 0.5 mm indicates and etiologic corollaries. For example, conju- impaired sympathetic or parasympathetic gate eye deviation to the left indicates a lesion function. above the oculomotor decussation (pontomes- encephalic junction) on the left or below it on In the unresponsive patient, the following the right (PPRF). Conjugate downward eye fi ndings have anatomic or etiologic signifi cance. deviation has been reported with thalamic hem- Diencephalic lesions and metabolic encephal- orrhage, thus implicating compression of the opathy: 2Ð3 mm reactive pupils. posterior commissure, but most often it is sec- Midbrain lesions: 4Ð5 mm irregular nonreac- ondary to metabolic or hypoxic-ischemic tive pupils. encephalopathy. Conjugate upward deviation is Pretectal lesions: 5Ð6 mm round nonreactive almost invariably due to hypoxic-ischemic pupils. encephalopathy. Pontine lesions Ð pinpoint, reactive pupils. Skew deviation means the vertical separation Medullary lesions Ð Horner’s syndrome. of the eyes. The term could be applied to hyper- tropia of 4th nerve lesions, hypotropia of 3rd nerve lesions or even patients with myasthenia Corneal Reflexes gravis. However, it is a term usually restricted to central or peripheral lesions affecting otolith The corneal refl ex tests the trigeminal-facial- input. Lesions of vestibular nuclei, 8th nerve oculomotor refl ex connections. A unilateral cor- and vestibulocerebellar pathways may cause neal stimulus using a wisp of cotton to touch the ipsilateral hypotropia and MLF lesions, ipsilat- cornea (over the iris) causes a bilateral blink eral hypertropia. The ocular tilt reaction, mani- response and elevation of the eyes (Bell’s phe- fested by head tilt, eye torsion and hypotropia nomenon). Thus, when the left cornea is touched, causes skew deviation ipsilateral to a vestibular an absent or slow blink on the right with intact left system lesion or contralateral to a mesodien- blink indicates right orbicularis oculi weakness or cephalic lesion. A detailed discussion of the a right 7th nerve, right 7th nucleus, or left supra- ocular tilt reaction is beyond the scope of this nuclear pathway lesion. An absent or slow ipsilat- text. eral response has the same meaning with opposite lateralization. Unilateral absence of Bell’s phe- nomenon, upward deviation of the eye when it Skew deviation, a vertical separation of the closes, usually indicates a superior rectus paresis eyes, has localizing value. and possible 3rd nerve lesion. Bilateral absence of 108 5 Evaluation of the Poorly Responsive Patient

Bell’s phenomenon is not necessarily pathologic Motor Function since it occurs in some normal individuals. Are there asymmetric spontaneous movements The corneal refl ex requires touching the cornea, which indicate focal disease? Is there decorticate located over the iris, with a wisp of cotton. or decerebrate posturing which is spontaneous or provoked by a painful stimulus? Multifocal myo- The corneomandibular refl ex is manifested by clonus is common in metabolic, toxic, septic, or contralateral jaw deviation and bilateral eye hypoxic-ischemic encephalopathies. Jerking of blink. It may occur with upper brainstem lesions. one limb or both arm and leg on one side should Conversely, the corneomental refl ex, an ipsilat- be presumed to be seizure activity. eral twitch of the mentalis muscle when the cor- nea is touched, is a common fi nding both in The patient must be constantly observed degenerative diseases of the nervous system and, for asym metric movements and abnormal less often, in the normal elderly population. posturing.

Paratonic rigidity (Gegenhalten) is a nonspe- Eyelids cifi c, frequent abnormality of muscular tone found in patients with metabolic encephalopathy. It is manifested by an active resistance to passive The eyes are closed in a comatose patient. movements and often misinterpreted as opposi- Incomplete closure of one eye indicates orbicu- tional or noncooperative behavior. Spasticity is laris oculi weakness and possibly a facial paresis. best elicited in the arms by quick pronation and Unilateral orbicularis oculi weakness can be dem- supination movements of the forearms. With one onstrated via the corneal refl ex and sometimes a or both hands under the popliteal fossa, a sudden weak or slow response (asymmetric) to visual high amplitude jerk upward may elicit a “hang threat. The examiner has to gently open both eyes up,” a slow fall of the leg indicating a spastic to check for that response. The blink response to catch. Spasticity indicates pathology affecting a sound stimulus may be asymmetric thus the corticospinal tract. If both legs are held up unmasking a weak orbicularis oculi on one side. under the popliteal fossa and suddenly released, a

more rapid fall can signify focal weakness and The eyes are closed in coma and open in a thus, in the absence of peripheral pathology, vegetative state. structural disease probably involving the corti- cospinal tract. Consequently, metabolic enceph- Patients in a vegetative state, which develops alopathy cannot be the sole cause of the altered after at least 2 weeks in coma, have normal sleepÐ mental status. Similarly, holding both arms up at wake cycles which result in an eyes open unre- the same level and suddenly removing support sponsive state during the awake period. Ptosis may disclose focal weakness on the side which may be observed in which case there may be falls more quickly. levator palpebrae superioris (3rd nerve lesion) or

Müller’s muscle weakness (Horner’s syndrome). Paratonic rigidity (Gegenhalten) is especially common in patients with metabolic encephal- opathies. Palate

An absent gag response is a necessary fi nding in Reflexes suspected brain death assuming the patient is not intubated. Unilateral loss indicates a focal lesion Discovering refl ex asymmetry between the left of the ipsilateral 9th and/or 10th cranial nerves or and right side is essential. In the patient with ipsilateral nucleus ambiguus in the medulla. cerebral or brainstem disease, the hyperactive Metabolic and Hypoxic-Ischemic Encephalopathies 109 side is the abnormal one indicating a lesion affect- in any direction, one must consider local pathol- ing the contralateral corticospinal tract. ogy affecting bone, tendon, or muscle. Symmetrical brisk or absent refl exes are seldom signifi cant in unresponsive patients. Sustained clonus, unilateral Hoffmann sign, unilateral or Metabolic and Hypoxic-Ischemic bilateral Babinski signs are always abnormal and Encephalopathies indicate a lesion involving the corticospinal tract. The presence of these abnormalities is not com- Metabolic Encephalopathy patible with a purely metabolic encephalopathy. Metabolic encephalopathy is the most common cause of altered mental status among hospitalized The refl ex exam focuses on discovering asym- patients. It is relatively simple to discover the eti- metrical and abnormal refl exes. ology in most instances. On occasion, abnormal neurologic signs are misinterpreted as indicating a primary neurologic process, and conversely, a Sensory neurologic disease may be dismissed as an expected metabolic alteration. Therefore, an A patient, whether or not intubated, who can be analysis of the of these dis- roused, may cooperate with vibration, position, orders is worth reviewing. and pinprick testing. In an intubated patient, The cardinal manifestations of central nervous these modalities can be examined provided the system complications of metabolic derangements patient is suffi ciently alert. The patient may ges- can be separated into four categories. These are ture yes or no by head shake as well as raising or alterations of the mental status, abnormal respira- lowering his fi nger in response to movement of tory patterns, characteristic oculomotor and pupil- his digits by the examiner. The obtunded or stu- lary fi ndings, and typical motor phenomena. porous patient is tested by noting the facial gri- Delirium is usually the fi rst manifestation of mace or limb withdrawal after pinprick an overt metabolic encephalopathy. The one stimulation of each limb. If unilateral hypesthe- exception is the rare occurrence of delirium asso- sia is suspected, repeated checking with pin- ciated with acute nondominant hemisphere pricks on the trunk beginning on the presumed lesions. The recognition that delirium has abnormal side and moving horizontally across occurred is thus very useful when questions arise the abdomen or chest may elicit a response such about the differential diagnosis, particularly if a as a facial grimace or an attempt to remove the primary neurologic etiology is suspected. stimulus as it approaches but not yet reaches the midline. This is due to nerve fi bers which over- Delirium is usually the fi rst manifestation of lap the midline. an overt metabolic encephalopathy.

If the patient is arousable, a complete sensory Delirium is characterized by an altered level of examination can be performed. consciousness, acute onset with fl uctuating course, impaired attention, and disorganized thinking. There are delusions and primarily visual, Meningeal Signs not auditory hallucinations. Abnormalities of the sleepÐwake cycle are almost invariably present. Is the patient’s neck suffi ciently supple to bring Confusion and hyperactivity (“sundowning”) the chin to rest on the chest? Nuchal rigidity may occur at night. be obvious or detectable only at the termination The major alterations of the mental status are of head fl exion, so-called end nuchal rigidity. disorders of arousal. These can be divided into a Testing for Brudzinski and Kernig signs may be hypervigilant and hypovigilant state. The hyper- necessary. If the patient’s head cannot be moved vigilant state is distinguished by distractibility and 110 5 Evaluation of the Poorly Responsive Patient accompanied by purposeless motor activity known with encephalopathies. Errors in spelling words as punding. An example would be fi ddling with or backwards and doing serial subtractions are com- manipulating the bedcovers. With requests to mon but may be due to the primary problem of name the months of the year the patient may begin attention rather than a true cognitive disorder. accurately, then stop suddenly to discuss another topic. Perseveration of responses is common. Visual hallucinations, not auditory hallucina- The hypovigilant patient requires constant tions, are common. sensory stimulation to obtain responses. Otherwise, the patient is immobile, apathetic, and withdrawn. Thus there are features which may Respiratory Disorders even suggest akinetic mutism. Patients who have Abnormalities of respiratory rates are a nearly metabolic encephalopathy often fl uctuate between invariable development in metabolic encephal- the hypovigilant and hypervigilant states. opathies. Cheyne-Stokes respirations are charac- It is important to differentiate dementia from a teristic and presumably due to impairment of metabolic encephalopathy. The former is marked frontal lobe function on respiratory drive assum- by impaired cognition fi rst; much later there is a ing that congestive heart failure is not present. change in alertness. The conspicuous elements of a metabolic encephalopathy are early changes in Cheyne-Stokes respirations are common in alertness and impaired attention. Disorientation patients with metabolic encephalopathies. and loss of short-term recall occur in both states. A determination of respiratory function at the Dementia begins with impaired cognition. bedside usually focuses on the respiratory rate. The early signs of metabolic encephalopathy Consequently, a review of disorders of respira- are impaired alertness and attention. tion will focus on the respiratory rate and the etiology of both hyperventilation and hypoven- Another important clinical problem is the dif- tilation. This will be a brief summary which ferentiation of encephalopathies from psycho- emphasizes the most common and important genic disorders. Disorientation to self is nearly etiologies. always psychogenic in origin and quite often Hyperventilation may be due to neurologic there is no simultaneous disorder of alertness disease or a respiratory attempt to compensate and no prior sign of abnormal cognition. If the for metabolic acidosis. Structural pathology must patient is fully responsive and disoriented to self, be excluded. Some examples follow. caloric testing can be utilized to accurately ascer- 1. Hyperventilation causing respiratory alkalosis tain the level of consciousness. An electroen- due to primary increased drive includes: psycho- cephalogram (EEG) is less reliable since normal genic hyperventilation, hepatic encephalopathy, appearing records may occur in patients who are sepsis, hypoxia, pulmonary disease, neurogenic in coma. pulmonary edema, central neurogenic hyper- ventilation, and drug toxicities Ð especially sali- Disorientation to self is nearly always psycho- cylates. Psychogenic hyperventilation is easily genic in origin. diagnosed as it disappears in sleep. 2. Hyperventilation as a respiratory compensa- Other features of encephalopathy include tion for metabolic acidosis includes: uremia, inappropriate behavior, paranoid ideation, and diabetic ketoacidosis (Kussmaul breathing), perceptual disorders. The latter are manifested by treatment with metformin, lactic acidosis both illusions (distortion of a sensory perception) (especially due to hypoxia), diarrhea, and drug and visual hallucinations. Visual hallucinations toxicity (especially salicylates). can be combined with auditory hallucinations Hypoventilation may be due to neurologic but auditory hallucinations alone do not occur disease and a respiratory attempt to compen- Metabolic and Hypoxic-Ischemic Encephalopathies 111

sate for metabolic alkalosis. Structural pathol- the right side above the oculomotor decussa- ogy must be excluded. Some examples follow. tion at the pontomesencephalic junction or 1. Hypoventilation causing respiratory acidosis below it near or in the left PPRF. Skew devia- due to a decreased respiratory drive includes: tion, a vertical separation of the globes, and central nervous system disease due to head dysconjugate eye movements also establish trauma, sedatives, stroke, and infection. the presence of structural pathology. Restriction of chest expansion by structural Nystagmus, vertical or horizontal, indicates abnormalities or weak muscles may cause drug toxicity or structural disease most often respiratory acidosis. Some examples are affecting the brainstem and less likely the cer- severe kyphoscoliosis, chest injury, myasthe- ebellum. Rotatory nystagmus (torsional) indi- nia gravis, and Guillain–Barré syndrome. cates a brainstem lesion, probably affecting Intrinsic lung disease such as chronic obstruc- vestibular nuclei. Vestibuloocular examina- tive pulmonary disease should be considered. tions are composed of the oculocephalic 2. Hypoventilation as a respiratory compensa- maneuver and caloric testing. The oculocepha- tion for metabolic alkalosis includes: vomit- lic refl ex generates conjugate horizontal or ing, gastric drainage, acute and chronic vertical eye movements with brisk horizontal pulmonary disease, volume contraction, or vertical movements of the head, respec- diuretic therapy, potassium defi ciency associ- tively. Caloric testing is a stronger stimulus ated with acid shift into the cell, Cushing’s which produces a more varied and reliable syndrome, and primary aldosteronism. response. The details have been discussed in Section “Utility of Caloric Testing.” Eye Signs 1. Pupils Vertical eye deviations may occur with Patients with metabolic encephalopathy metabolic/hypoxic-ischemic disease or have pupils which are usually 2Ð3 mm, equal focal brainstem-thalamic lesions. and normally reactive to light. A signifi cant size Horizontal gaze deviation always indi- asymmetry (>0.5 mm) or abnormal reactivity cates a focal lesion. indicates the presence of focal disease and thus metabolic factors may not be the sole or pri- mary etiology. Nonreactive pupils are due to Abnormalities of Motor Function other neurologic diseases or caused by anticho- Generalized and focal motor seizures may occur. linergic drugs, prior eye surgery, ocular trauma The focal seizures usually shift locations and or eye diseases that affect the pupil. These latter may shift sides. Multifocal myoclonus is com- conditions should be excluded after review of mon and manifested by migrating twitching of the history. muscle groups in random fashion. It is particu- larly prominent in facial musculature. This occurs Patients with metabolic encephalopathy in severe encephalopathy and typically is not have pupils which are usually 2Ð3 mm, recorded on electroencephalographic tracings. equal and normally reactive to light. Generalized and focal motor seizures may 2. Eye movements occur with metabolic encephalopathies. Downward and rarely upward eye devia- tion may occur with metabolic, ischemic, and Tremor of the hands is a frequent accompaniment. hypoxic encephalopathies. Lateral eye devia- It is of moderate to high amplitude, asynchronous, tion does not; it nearly always indicates a postural, and action in type. The rate is about 8Ð10 Hz. structural focal central nervous system disease It may involve the tongue, face, and legs. It is dis- process. If the eyes are deviated to the right tressing to the patient as it usually interferes with pur- there is a lesion in the oculomotor pathway on poseful activity. 112 5 Evaluation of the Poorly Responsive Patient

Asterixis is elicited in an awake or lethargic fi nding, a lumbar puncture is done with fl uid patient by extending the arms with the palms face sent for cell count, protein, glucose, Gram stain, down and hands and fi ngers dorsifl exed by the fungal smear, India ink prep, cryptococcal anti- patient. This is a position as if one is pushing gen, VDRL, routine and fungal cultures. against a wall. A quick bilateral fl apping, asyn- Polymerase chain reaction studies may be con- chronous movement downward occurs, usually sidered. An extra sample should be obtained and fi ngers fi rst and commonly with a latency of a saved for possible additional testing. Finally, an few to several seconds. It will be repeated con- EEG may be necessary to exclude nonconvul- tinuously as long as the position is maintained. sive status epilepticus and to grade the severity The fl ap frequency is variable. Dorsifl exing the of the encephalopathy. feet with the legs extended may elicit the same response. The etiology was once thought to be Differential Diagnosis of Metabolic specifi c for hepatic encephalopathy but is now Disorders known to occur in practically all metabolic In order to avoid overlooking an important imme- derangements and is perhaps most common with diate etiology, it may be useful to think anatomi- hypercapnia. cally. Once a metabolic encephalopathy is diagnosed, the disease process causing the meta- Asterixis may occur with any metabolic bolic abnormality must be identifi ed and treated. encephalopathy. 1. Neck (a) Hypothyroidism or hyperthyroidism. Paratonic rigidity is a hallmark fi nding. Its fea- (b) Parathyroid disease. Hypercalcemia and tures are active resistance by the patient in response hypocalcemia. to the examiner’s attempt to move his arm usually 2. Chest at the elbow. This occurs despite the examiner’s (a) Pulmonary disease. Hypoxia and admonition to “relax” or “don’t resist.” hypercapnia. Decerebrate rigidity and rarely decorticate (b) Cardiac disease. Severe congestive heart rigidity may occur with severe metabolic enceph- failure with low cardiac output. alopathies. There may be severe sudden extensor 3. Abdomen spasms, especially of the legs. These postures (a) Liver Ð hepatic encephalopathy. quite naturally raise fears of an unexpected mass (b) Pancreas Ð hyperglycemia, hypoglycemia, lesion causing central herniation. The presence of pancreatic failure. normally reactive, symmetrical pupils should (c) Adrenal Ð Addison’s disease, Cushing’s ameliorate these fears. Any severe metabolic dis- disease. order may cause spontaneous decerebrate (d) Kidney Ð renal failure. posturing. 4. Electrolyte and pH disorders (a) Hyponatremia, hypernatremia. Decerebrate rigidity may occur with severe (b) Metabolic acidosis, metabolic alkalosis, metabolic encephalopathies. respiratory acidosis, respiratory alkalosis.

The emergency workup for an unresponsive Specific Diseases Which Mimic the patient with a suspected metabolic encephalop- Confusion of Metabolic Encephalopathy athy includes: complete blood count, electro- Most of these disorders are uncommon with lytes, glucose, BUN, creatinine, PT/INR, PTT, the exception of pure fl uent aphasias as the liver functions, calcium, phosphorus, arterial only manifestation of an acute illness. blood gases, TSH, blood cultures, drug screen, Nevertheless, the fi rst three listed below require urinalysis and urine culture. A stat noncontrast prompt treatment to avert catastrophic CT scan follows. In the absence of a diagnostic consequences. Metabolic and Hypoxic-Ischemic Encephalopathies 113

These include: forgotten in this era of sophisticated neuroimag- 1. Fluent aphasias, especially Wernicke’s and ing. Furthermore, an EEG is often diagnostic of transcortical sensory, due to acute middle herpes simplex encephalitis. Wernicke’s enceph- cerebral artery (MCA) distribution stroke or alopathy is manifested by the triad of an altered herpes simplex encephalitis. sensorium, eye signs such as ocular pareses and 2. Complex partial status epilepticus (noncon- nystagmus, and limb or gait ataxia. All of these vulsive status epilepticus). diagnoses necessitate immediate recognition and 3. Wernicke’s encephalopathy. treatment. 4. Hashimoto’s encephalitis. Hashimoto’s encephalitis is rare but usually 5. Acute encephalopathy of systemic lupus ery- preceded by thyroid dysfunction, especially thematosus (SLE). hypothyroidism. There are elevated antithyro- 6. Encephalitis, especially herpes simplex and globulin and antithyroperoxidase antibodies. human herpes virus 6 (HHV-6), which occurs Encephalitis must always be considered when an after bone marrow transplants. overt metabolic etiology is not apparent. Fever 7. CreutzfeldtÐJakob disease. and signs of meningeal disease are often absent. 8. Progressive multifocal leukoencephalopathy. A lumbar puncture, therefore, is mandatory. 9. Opportunistic infections, especially fungal CreutzfeldtÐJakob disease usually manifests disease. other signs such as seizures, myoclonus, extrapy- 10. Human immunodefi ciency virus (HIV) ramidal fi ndings and prior dementia. Cerebrospinal encephalopathy. fl uid may be sent for 14-3-3 protein, but PCR 11. Vascular diseases. testing may soon take precedence. An EEG com- (a) Multiple cerebral infarctions of any monly exhibits diagnostic features. Progressive etiology. multifocal leukoencephalopathy caused by the (b) Antiphospholipid antibody syndrome. JC virus occurs in the immunosuppressed patient (c) Disseminated intravascular coagulation and ordinarily presents with multifocal signs; but (DIC). initially the diagnosis is often obscure. (d) Thrombotic thrombocytopenic purpura Opportunistic infections, primarily fungal dis- (TTP). ease, must always be suspected in patients on (e) Nonbacterial thrombotic endocarditis long-term steroid treatment or those who are (marantic endocarditis). immunosuppressed for various reasons. HIV (f) MELAS. Mitochondrial encephalopathy encephalopathy is well-described but a rare ini- with lactic acidosis and stroke-like tial neurologic presentation. episodes. The vascular diseases listed above are essential 12. Acute demyelinating encephalomyelitis to recognize. Multiple small infarctions of any (ADEM). etiology can cause an encephalopathy. A few spe- Fluent aphasias, complex partial status epi- cifi c ones include the antiphospholipid antibody lepticus and Wernicke’s encephalopathy require syndrome which may lead to arterial thromboses an immediate diagnosis. Acute fl uent aphasias and less often venous sinus thromboses. TTP, customarily occur with an MCA distribution which may be a complication of clopidogrel treat- stroke, ischemic or hemorrhagic. An acute isch- ment, causes microvascular thrombi. Associated emic origin may not be readily diagnosed by CT features include hemolytic anemia and renal fail- imaging; thus, it mandates careful attention to ure. DIC should be suspected in patients with sep- language function, especially the presence of sis and malignancies. Nonbacterial thrombotic paraphasias. Herpes simplex encephalitis may endocarditis is associated with platelet fi brin veg- also present in this fashion since unilateral tem- etations in patients with underlying adenocarci- poral lobe involvement is characteristic. Complex nomas and hematologic malignancies. Multiple partial status epilepticus (nonconvulsive status systemic emboli are common but neurologic epilepticus) requires an urgent EEG often manifestations predominate. 114 5 Evaluation of the Poorly Responsive Patient

Hypoxic-Ischemic Encephalopathy Clinical correlates of subcortical infarctions, involving the superfi cial and deep branches of Although the most common cause of hypoxic- the MCA, invariably include hemiparesis and ischemic encephalopathy is cardiac arrest, there commonly aphasias of various types in dominant are numerous instances when oxygenation is hemisphere lesions. maintained but hypotension produces severe cere- Pure hypoxic encephalopathies do not cause bral ischemia. Conversely, cardiac function and watershed infarctions, but rather neuronal dam- blood pressure may remain normal, yet hypoxia is age in specifi c populations of neurons which are severe. It is important, therefore, to be precise in particularly sensitive to hypoxia. These are located these defi nitions. Ischemic encephalopathy is in the striatum (caudate and putamen nuclei), often more devastating than hypoxic encephal- cerebral cortex, CA1 region of the hippocampus opathy. One need only look at intrepid mountain and the cerebellum, especially Purkinje cells. climbers who summit Himalayan peaks and func- Loss of Purkinje cells may be responsible for tion well despite p O 2’s in the 30 mmHg range. action myoclonus, a debilitating, often prolonged disorder known as the Lance-Adams syndrome.

Ischemic encephalopathy is often more dev- Pure hypoxic encephalopathies do not cause astating than hypoxic encephalopathy. watershed infarctions.

The sequelae of ischemic and hypoxic-isch- emic encephalopathy are watershed infarctions. These are located in the border zones between the Prognostic Factors in Comatose MCA and anterior cerebral artery (ACA), MCA Survivors After CPR and posterior cerebral artery (PCA), and the bor- der zone between the deep and superfi cial branches of the MCA. Circumstances surrounding a cardiac arrest, including the etiology, cannot be relied upon The sequelae of ischemic and hypoxic-ischemic for prognosis. encephalopathy are watershed infarctions. 1. Circumstances surrounding CPR such as time Clinical correlates of anterior infarctions (ACA- of anoxia, duration of CPR, etiology of cardiac MCA) include crural paresis (leg greater than arm) arrest cannot be relied upon for prognosis and myoclonic jerks of leg or arm. Dominant (Wijdicks 2006 ). hemisphere lesions result in transient mutism, dys- 2. Hyperthermia is associated with a poor out- nomia, and especially, transcortical motor aphasia come but is also not suffi ciently accurate. (decreased output, good comprehension and good 3. Features of the neurologic examination which repetition). Nondominant hemisphere lesions can indicate poor prognosis are: be manifested by apathy and abulia. (a) Absent pupillary reactions 3 days after Clinical correlates of posterior infarctions cardiac arrest. (MCA-PCA) are homonymous hemianopsia or (b) Absent corneal refl exes 3 days after car- inferior quadrantanopsia, hemisensory defi cits diac arrest. such as sensory extinction with double simultane- (c) Absent or extensor motor responses 3 ous stimulation, astereognosis and agraphesthe- days after cardiac arrest. sia. Dominant hemisphere lesions are associated (d) Myoclonic status epilepticus within 24 h with transcortical sensory aphasia manifested by of cardiorespiratory arrest. paraphasias, fl uent speech, good repetition, and 4. Electrophysiologic studies. poor comprehension. Nondominant infarctions (a) Somatosensory evoked potentials. cause anosognosia and . Bilateral absence of cortical SSEPs (N20 Prognostic Factors in Comatose Survivors After CPR 115

Fig. 5.3 Estimating prognosis in nontraumatic coma. All state; Sev Disab severe disability; Mod Disab moderate patients surviving various early intervals after onset of disability; Good Recov good recovery; Mot motor coma are categorized on the basis of sequential criteria responses; Ext extensor; Flex fl exor; Spont Eye Movt relating to their clinical examinations. Best levels of spontaneous eye movements; Nl normal. Nonreactive recovery within 1 year are given for each of the prognos- motor responses means the absence of any motor response tic groups. No Recov no recovery; Veg State vegetative to pain. Levy et al. (1981 ) , with permission

response) 1Ð3 days post arrest is a strong Quantitative brain MRI diffusion- indicator of a poor prognosis. weighted imaging during the fi rst week (b) Electroencephalography patterns. Burst post cardiac arrest is currently being stud- suppression or generalized epileptiform dis- ied and may be useful. charges are associated with poor outcomes (d) Biochemical markers. Serum neuron-specifi c but not with good prognostic accuracy. enolase levels of greater than 33 mg/L show (c) Neuroimaging. To date there are insuffi - good predictive value for poor prognosis. cient data to assess prognostic value. 5. Figure 5.3 is an excellent summary. 116 5 Evaluation of the Poorly Responsive Patient

given. This usually progresses into central Herniation Syndromes transtentorial herniation. The clinical signs of falcine herniation are those which accompany Mass lesions of any etiology which affect only the mass lesion itself. one cerebral hemisphere will not ordinarily alter the level of consciousness. If the mass effect from the lesion alone or secondary edema causes a Falcine herniation may be followed by shift of midline structures to affect the contralat- central transtentorial herniation. eral hemisphere, diencephalon (hypothalamus, thalamus, and subthalamus), or midbrain, the 2. Central transtentorial herniation. patient becomes poorly responsive. Central transtentorial herniation can be divided into diencephalic, midbrain-upper Mass lesions of any etiology which affect only pons and lower pons-upper medulla stages. one cerebral hemisphere will not alter the level of consciousness. Central transtentorial herniation can be divided into diencephalic, midbrain-upper If there is a mass lesion in the posterior fossa pons, and lower pons-upper medulla stages. such as a tumor or hemorrhage within the cere- bellum, the patient will remain alert unless there The diencephalic stage occurs when down- is compression and secondary dysfunction of the ward displacement of the cerebral hemispheres brainstem, especially the midbrain reticular forces the diencephalon through the tentorial formation. notch. The etiologies are mass lesions affect- Because of the different mechanisms which ing or within the frontal, parietal, and occipital are involved, herniations can be divided into lobes. There is traction on the pituitary stalk supratentorial and infratentorial. It is essential to and buckling of the diencephalon against the recognize their clinical signs at the bedside since midbrain. Subsequently, there is further down- they convey the ominous signals of impending ward movement associated with clinical signs demise. of pontine and medullary dysfunction. There are three supratentorial herniations, fal- Clinical signs of the diencephalic stage in cine (cingulate), central transtentorial, and uncal its early development are drowsiness and (see Fig. 5.4 ). The clinical signs of all of them rarely agitation. Respirations are normal or can be divided into the effects on level of con- Cheyne-Stokes. The pupils are 1Ð3 mm, equal sciousness, respiration, pupils, eye movements, and show a normal reaction to light except for and motor function. All descriptions will focus the uncommon instance of a brief Horner’s on these fi ve elements. An emphasis will be syndrome due to unilateral hypothalamic placed on eye signs which are often the most reli- injury. Vestibuloocular refl exes (oculocephal- able diagnostic features (see Table 5.1 ). ics and calorics) are intact, showing full hori- zontal eye movements. Ice water calorics There are three supratentorial herniations: induce ipsilateral eye deviation and occasion- falcine, central transtentorial, and uncal. ally horizontal contralateral nystagmus. Spontaneous eye movements may be roving. 1. Falcine herniations. Patients exhibit paratonic rigidity and often Falcine herniations are associated with her- attempt to remove a noxious stimulus. niation of the cingulate gyrus under the falx In the later diencephalic stage, Cheyne- cerebri. This results in compression of Stokes respirations are present and caloric branches of the ACA (pericallosal and callo- irrigation elicits ipsilateral eye deviation with- somarginal), which supply the medial portion out contralateral nystagmus. A painful stimu- of the cerebral hemisphere causing edema and lus provokes decorticate posturing, which may thus increasing the shift if no treatment is develop asymmetrically. Herniation Syndromes 117

Fig. 5.4 Supratentorial herniation. (a ) When mass effect transtentorial herniation of the superior vermis of the is symmetrical both central and uncal herniation are likely cerebellum. The lower adjacent arrow discloses mass to occur. (b ) Mass lesion causing falcine herniation, effect of a cerebellar neoplasm causing upward bulging midline shift, central and uncal herniation. Obtained with of the tentorium cerebelli. The lowest arrow indicates permission from Plum and Posner (2007 ) . Infratentorial herniation of the cerebellar tonsils herniation. (c ) The uppermost arrow shows ascending 118 5 Evaluation of the Poorly Responsive Patient

Table 5.1 Eye signs in herniation syndromes Pupil (size and light Oculovestibular response reaction) Oculocephalic Refl ex (cold calories) Central transtentorial Herniation Diencephalic 1Ð3 mm, reactive NL Ipsilateral deviation with or without contralateral nystagmus Midbrain/upper pons Midposition, 4Ð5 mm, Sluggish, may be Sluggish, ipsilateral deviation, fi xed dysconjugate may be dysconjugate Lower pons/upper Midposition, 4Ð5 mm, Absent Absent medulla fi xed Uncal herniation Early 3rd nerve 5Ð7 mm, sluggish Normal or medial rectus Ipsilateral deviation with or reaction, ipsilateral paresis without contralateral nystagmus and medial rectus paresis Late 3rd nerve 5Ð7 mm, no reaction, Medial rectus paresis Ipsilateral deviation, no nystag- ipsilateral mus and medial rectus paresis Transtentorial 4Ð6 mm, nonreactive Vertical oculocephalics, no With bilateral cold caloric upward Herniation upgaze stimulation, no upbeat nystagmus

The midbrain-pons stage carries ominous tentorium cerebelli (Kernohan’s notch) pro- sequelae as complete recovery from this stage ducing an ipsilateral hemiparesis. The Sylvian is rare even if the offending lesion is com- aqueduct is commonly compressed causing pletely removed. Diabetes insipidus and fever ventricular enlargement. The subsequent are common due to traction on the pituitary increase in supratentorial volume causes fur- stalk and hypothalamus. Central neurogenic ther downward displacement of the dienceph- hyperventilation supervenes, the pupils alon. Herniation produces brainstem ischemia become midposition (4Ð5 mm), irregular, and and stretching of the medial perforating fi xed. Vestibuloocular refl exes elicit horizon- branches of the basilar artery which is attached tal eye movements only with maximal stimu- to the circle of Willis. The results are midline lation. Decerebrate rigidity replaces decorticate hemorrhages in the midbrain and pons (Duret’s posturing when painful stimuli are applied. hemorrhages). Low pontine-medullary stage clinical signs are deep coma and shallow respirations, which Uncal herniation occurs with temporal lobe eventually become ataxic. Pupils are midposi- lesions or any laterally placed lesion. tion, fi xed to light and vestibuloocular refl exes An ipsilateral hemiparesis, due to are absent. There is no response to pain other Kernohan’s notch, is caused by compres- than occasional fl exor movements at the knees sion of the opposite cerebral peduncle. with plantar stimulation. 3. Uncal herniation. Uncal herniation can be divided into the Uncal herniation occurs with temporal early 3rd nerve, late 3rd nerve, and midbrain- lobe lesions or any laterally placed lesion. The upper pons stages. hippocampus and uncus bulge over the edge The early 3rd nerve stage is an ipsilateral of the tentorium trapping the 3rd nerve against enlargement of the pupil with a sluggish light it. The midbrain is fl attened and the PCA reaction. Breathing is normal. Oculocephalic which surrounds it is compressed. This may and caloric testing are normal except for occa- cause an ipsilateral occipital infarction with sionally demonstrable ipsilateral medial rectus associated edema which aggravates the herni- weakness due to the 3rd nerve compression. ation process. The opposite cerebral peduncle Painful stimuli provoke an attempt to remove the can be compressed against the incisura of the stimulus. Paratonic rigidity in the contralateral Chronic Disorders of Consciousness 119

limb may be present. Contralateral hemiparesis, the tentorial notch. Aqueductal compression hyperrefl exia, and Babinski signs are usual but usually occurs with secondary ventricular infrequently there is an ipsilateral hemiparesis enlargement. The superior cerebellar artery is due to Kernohan’s notch. compressed causing cerebellar infarctions, The late 3rd nerve stage is manifested by a secondary edema and augmented pressure rapid decline into coma. Respirations are aggravating the upward herniation. The veins hyperpneic (central neurogenic hyperventila- of Rosenthal and Galen are compressed caus- tion), less likely Cheyne-Stokes. There is an ing venous congestion and further increase in ipsilateral fi xed large pupil and medial rectus intracranial pressure. paresis is evident with vestibuloocular refl ex testing. Posturing occurs with painful stim- Transtentorial upward herniation causes uli, bilateral decerebrate or asymmetric, pretectal and posterior commissure com- decorticate on one side and decerebrate on pression with bilateral large, fi xed pupils the other. and upgaze paresis, respectively. The midbrain-upper pontine stage reveals a deeply comatose patient with fi xed midposi- Clinical signs are impairment of upward gaze tion (4Ð5 mm) pupils and absent vestibuloocu- and enlarged round fi xed pupils (5Ð6 mm) due to lar refl exes. Decerebrate posturing prevails. pressure on the posterior commissure and adja- The subsequent decline follows the pattern of cent pretectum, respectively. There may be cen- central herniation described in the foregoing tral neurogenic hyperventilation and occasionally section. downward eye deviation. Retractory nystagmus may occur due to simultaneous contraction of all Infratentorial herniations are tonsillar and eye muscles which pulls the eyes back into the upward transtentorial. orbit. With suffi cient pressure on the midbrain Tonsillar herniation results in medullary impaired function of the mesencephalic reticular infarctions and respiratory arrest. formation will cause coma.

4. Infratentorial herniations. Infratentorial herniations are twofold, ton- Chronic Disorders of Consciousness sillar and upward transtentorial. Tonsillar herniation is caused by mass effect The conscious state depends on two characteris- in the posterior fossa which shoves the cerebel- tics, wakefulness and awareness (Rosenberg lar tonsils through the foramen magnum. 2009). The reticular formation, primarily in the Normally the tonsils can be as low as 5 mm thalamus, is the substrate required for wakeful- below the foramen magnum. When they descend ness. Awareness depends on additional structures, below this critical level, they may be com- the cerebral cortex, and subcortical white matter pressed against the edge of the foramen mag- connections with the thalamus. num along with the medulla and high cervical cord. These structures become infarcted and the The conscious state depends on two charac- medullary centers for respiratory and circula- teristics, wakefulness and awareness. tory control are destroyed. An early clinical sign of tonsillar herniation may be a slightly stiff 1. Coma neck. This augurs an impending rapid respira- Coma is a state of unresponsiveness and tory and circulatory collapse. unarousability with closed eyes. Thus, the retic- 5. Transtentorial upward herniation. ular formation is not functioning. In deep coma Transtentorial upward herniation occurs there is no withdrawal or posturing to painful with posterior fossa mass lesions which push stimuli. In light coma painful stimuli will the cerebellum and midbrain upward through produce posturing and withdrawal responses. 120 5 Evaluation of the Poorly Responsive Patient

After a few weeks, the comatose patient transi- 3. Akinetic mutism tions into a vegetative state if treatment efforts This is a self-defi ning term. The patient have failed. Any etiology can lead to a vegeta- does not speak, is immobile or nearly so, but tive state, be it hypoxic-ischemic, infectious, appears hypervigilant. The semblance of atten- metabolic, toxic, or structural pathology. tiveness derives from the presence of visual tracking with conjugate eye movements. In coma, the reticular formation is not Additional features of this state which differ- functioning, the patient is unarousable, and entiate it from the vegetative state is the pos- the eyes are closed. sibility of an acute or subacute origin as well as prompt recovery if due to a mass lesion which 2. Vegetative state can be surgically treated. Rupture of an ante- The vegetative state, awake but unaware, rior communicating aneurysm may cause this indicates preservation of the reticular forma- state and it may become permanent. It is caused tion alone. It can be associated with a fl at EEG by bilateral lesions of the anterior cingulate (electrocerebral silence) or a severely abnor- and medial prefrontal cortex as well as struc- mal EEG. The vegetative state is characterized tural lesions involving the dorsomedial nucleus by the absence of meaningful interaction with of the thalamus and midbrain tegmentum. people or environmental stimuli. The patient has normal sleepÐwake cycles and therefore Akinetic mutism differs from the vegetative the eyes are open for long periods. There is state since it may be of acute origin and spontaneous breathing, eye blinks, and roving resolve quickly if the lesion is treatable. eye movements. Pain may provoke facial gri- macing, decorticate posturing and limb with- 4. Minimally conscious state drawal. The patient may utter sounds but never The minimally conscious state can be words; he may yawn, chew, and swallow defi ned as a near vegetative state (Bernat 2010 saliva. Cranial nerve refl exes are intact. There and Giacino et al. 2002 ). The presence of frag- is fecal and urinary incontinence. Because the ments of awareness of self and environment patient is awake and may have roving eye are infrequently or rarely witnessed. This is movements, both medical personnel and non- determined by an appropriate but often incon- medical individuals are often seduced into a sistent verbal or gestural response to yesÐno belief that the patient is aware. The vegetative questions, ability to follow simple commands state becomes clearly evident at 1 month. It and purposeful behavior. Rarely there is intel- becomes permanent at 3 months if the etiol- ligible verbalization. The patient may reach ogy is nontraumatic, 12 months if traumatic. for objects and demonstrate pursuit eye move- Although absolute certainty is not reachable, ments to moving stimuli. The diagnosis the veracity of the diagnosis is supported by depends on accurate observations by caregiv- numerous neuroimaging, neurophysiologic, ers who will likely need coaching on reliable and neuropathologic correlations. methods of communication.

In the vegetative state the reticular forma- The minimally conscious state is a near tion is functioning and the eyes are open. vegetative state as there are rare meaning- There are normal sleepÐwake cycles. ful responses to verbal commands and/or purposeful behavior. Additional terminology applied to these patients includes the persistent vegetative The patient’s physician will commonly not state, permanent vegetative state, akinetic be present when behavior indicating awareness mutism, and apallic state. Of these terms aki- is exhibited. Neurologic examinations are noto- netic mutism requires a separate discussion as rious for variability which may be related to it differs from the vegetative state. time of day, fatigue, fever, or other physiologic Brain Death 121

and environmental changes. Consequently, ordinarily hypoxic-ischemic encephalopa- repeated careful examinations are essential to thy, have a near zero chance of a meaningful provide an accurate diagnosis. recovery. At 1 month the prognosis is Attention must be paid to the patient’s fam- extremely poor, probably less than 1%. ily and their observations. Even though unre- Neuroimaging studies, especially MRI, alistic assumptions and conclusions are made, functional MRI, and PET scan have demon- a grain of an accurate observation may emerge strated abnormalities that may be useful for which changes the diagnosis from vegetative estimating prognosis. Details of these fi ndings state to minimally conscious state, a critical are beyond the scope of this text. distinction. An inaccurate diagnosis of a veg- 5. Locked-in state etative state instead of minimally conscious Although this is not a disorder of con- state probably ranges up to 40%. About 50% sciousness it must be distinguished from the of patients in the minimally conscious state vegetative state and akinetic mutism. This regain independent function at 1 year. state is usually due to a pontine lesion causing quadriplegia, paralysis of horizontal eye Attention must be paid to the patient’s fam- movements but preservation of downward eye ily and their observations since physicians movements. Upward eye movements above are unlikely to be present when responses the horizontal plane are usually not retained. are witnessed. The etiology is ordinarily a stroke, ischemic, or hemorrhagic, with destruction of the basis The etiology of a vegetative state or mini- pontis and PPRF. A rare etiology is central mally conscious state is most often hypoxic pontine myelinolysis, which is demyelination and/or ischemic encephalopathy due to car- of the basis pontis commonly due to an exces- diac arrest, respiratory failure, prolonged sively rapid correction of hyponatremia. hypotension, or head trauma. The pathology Communication with the patients via down- of nontraumatic cases especially involves cor- ward eye movement and eye blinks is occa- tical and thalamic neurons but less so of sub- sionally possible; however, the prognosis for cortical white matter tracts, hypothalamus, signifi cant recovery is poor. and brainstem. The pathology of head injury is typically manifested by diffuse axonal Brain Death injury, white matter greater than gray matter. In view of the pathology described above, There are four clinical settings in which the the prognosis of the patient in a traumatic veg- determination of death is made. First is the etative or minimally conscious state is far bet- total collapse of cardiac and pulmonary func- ter than nontraumatic etiology. Observation tion in the patient without mechanical support. for at least 1 year is required before a reason- Second is the collapse of cardiac function in ably accurate prognostic statement can be the intubated ventilated patient. Third is pul- made, especially for young patients. monary failure despite maximum oxygenation Approximately 50% of patients in a minimally and ventilator support. The fourth is brain conscious state due to trauma will be able to death in the intubated patient with adequate function independently at 1 year. artifi cial ventilation and sustained cardiac

function. There is general acceptance of the The prognosis of the patient in a traumatic principle that absence of brain function is vegetative or minimally conscious state is absence of life. far better than a nontraumatic etiology,

usually hypoxic-ischemic encephalopathy. There is general acceptance of the principle that absence of brain function is absence of Patients who are in a vegetative state for life. 3 months due to a nontraumatic etiology, 122 5 Evaluation of the Poorly Responsive Patient

The most common situation is the total col- No deviation of the eyes to irriga- lapse of cardiac and pulmonary function which tion in each ear with 50 mL of cold nowadays occurs primarily out of hospital. water (allow 1 min after injection Emergency code blue teams in the hospital have and at least 5 min between testing been extraordinarily successful in resuscitating on each side). patients after cardiac and/or respiratory arrest. Facial sensation and facial motor But the majority of recipients have serious response. hypoxic-ischemic sequelae. This clinical scenario No corneal refl ex to touch with a requires careful and accurate defi nitions of brain cotton swab. dysfunction. Furthermore, organ transplantation No jaw refl ex. mandates precision. No grimacing to deep painful Every hospital has its own protocol for brain stimuli. death, although the differences between them are Pharyngeal and tracheal refl exes. now negligible. A typical brain death protocol is No response after stimulation of described below: the posterior pharynx with tongue 1. Diagnostic criteria for clinical diagnosis of blade. brain death (for patients older than 18 years) No cough response to bronchial (Wijdicks 2010 ). suctioning. (a) Prerequisites. Brain death is the absence (iii) Apnea Ð testing performed as of clinical brain function when the proxi- follows: mate cause is known and demonstrably Prerequisites. irreversible. Core temperature ³ 36.5¡C or (i) Clinical fi ndings with or without 97¡F. neuroimaging evidence of an acute Systolic blood pressure CNS catastrophe that are compatible ³ 90 mmHg. with the clinical diagnosis of brain Euvolemia. Option : positive fl uid death. balance in the previous 6 h.

(ii) Exclusion of complicating medical Normal p CO2 . Option : arterial

conditions that may confound clinical p CO 2 ³ 40 mmHg.

assessment (no severe electrolyte, Normal p O2 . Option : preoxygen-

acidÐbase, or endocrine disturbance). ation to obtain arterial p O2 ³ (iii) No drug intoxication or poisoning. 200 mmHg. (iv) Core temperature ³ 32¡C (90¡F). Connect a pulse oximeter and dis- (b) The three cardinal fi ndings in brain death connect the ventilator.

are coma or unresponsiveness, absence of Deliver 100% O2 , 6 L/min into the brainstem refl exes, and apnea. trachea. Option: place a cannula at (i) Coma or unresponsiveness Ð no motor the level of the carina. response of any extremity to deep Look closely for respiratory move- painful stimuli. ments (abdominal or chest excur- (ii) Absence of brainstem refl exes. sions that produce adequate tidal Pupils. volumes).

No response to bright light. Measure p O2 , p CO 2, and pH after Size: midposition (4 mm) to dilated approximately 8 min and reconnect (9 mm). the ventilator. Ocular movement. If respiratory movements are absent

No oculocephalic refl ex (testing and arterial p CO2 is ³ 60 mmHg

only when no fracture or instability ( option: 20 mmHg increase in p CO2

of the cervical spine is apparent). over a baseline normal p CO 2 ), the Brain Death 123

apnea test result is positive (i.e., it (c) Respiratory-like movements (shoulder supports the diagnosis of brain elevation and adduction, back arching, death). intercostal expansion without signifi cant If respiratory movements are tidal volumes). observed, the apnea test result is (d) Sweating, blushing, tachycardia. negative (i.e., it does not support the (e) Normal blood pressure without pharma- clinical diagnosis of brain death) and cologic support or sudden increases in the test should be repeated after a blood pressure. suitable interval. (f) Absence of diabetes insipidus. Connect the ventilator if, during test- (g) Deep tendon refl exes; superfi cial abdomi- ing, the systolic blood pressure nal refl exes; triple fl exion response. becomes £ 90 mmHg or the pulse (h) Babinski sign. oximeter indicates signifi cant oxy- (i) All of these fi ndings are due to preserved gen desaturation and cardiac arrhyth- function of spinal cord pathways and mias are present; immediately draw neurons.

an arterial blood sample. If p CO2 is 4. Confi rmatory laboratory tests (options )

³60 mmHg or p CO2 ³ 20 mmHg over Brain death is a clinical diagnosis. A repeat

baseline normal p CO2 , the apnea test clinical evaluation 6 h later is recommended, result is positive (it supports the clin- but this interval is arbitrary. A confi rmatory ical diagnosis of brain death); if test is not mandatory but is desirable in patients

p CO2 is <60 mmHg or p CO2 increase in whom specifi c components of clinical test- is <20 mmHg over baseline normal ing cannot be reliably performed or evaluated.

p CO2 , the result is indeterminate, It should be emphasized that any of the sug- and an additional confi rmatory test gested confi rmatory tests may produce similar can be considered. results in patients with catastrophic brain dam- 2. Pitfalls in the diagnosis of brain death age, who do not (yet) fulfi ll the clinical criteria The following conditions may interfere of brain death. The following confi rmatory with the clinical diagnosis of brain death, so test fi ndings are listed in the order of the most that the diagnosis cannot be made with cer- sensitive test fi rst. Consensus criteria are iden- tainty on clinical grounds alone. Confi rmatory tifi ed by individual tests. tests are recommended. (a) Technetium-99m hexamethylpropyle- (a) Severe facial trauma. neamineoxime brain scan. No uptake of (b) Preexisting pupillary abnormalities. isotope in brain parenchyma (“hollow (c) Toxic levels of any sedative drugs, amino- skull phenomenon”). glycosides, tricyclic antidepressants, anti- (b) Electroencephalography. No electrical cholinergics, antiepileptic drugs, activity during at least 30 min of record- chemotherapeutic agents, or neuromuscu- ing that adheres to the minimal technical lar blocking agents. criteria for EEG recording in suspected (d) Sleep apnea or severe pulmonary disease brain death as adopted by the American

resulting in chronic retention of CO2 . Electroencephalographic Society, includ- 3. Clinical observations compatible with diagno- ing 16-channel EEG instruments. sis of brain death (Ropper 1984 ) (c) Transcranial Doppler ultrasonography. (a) Spontaneous movements of limbs other (i) 10% of patients may not have tempo- than pathologic fl exion or extension ral insonation windows. Therefore, response. the initial absence of Doppler signals (b) Lazarus sign: slow body movements pro- cannot be interpreted as consistent ducing fl exion at the waist. with brain death. 124 5 Evaluation of the Poorly Responsive Patient

(ii) Small systolic peaks in early systole 11. Paratonic rigidity is a common abnormality without diastolic fl ow or reverberat- in patients with metabolic encephalopathy. ing fl ow indicate very high vascular 12. The sensory examination of an obtunded resistance associated with greatly patient may yield focal signs. increased intracranial pressure and 13. Acute delirium usually indicates a metabolic support the diagnosis of brain death. or toxic encephalopathy. 5. Medical record documentation (standard ) 14. Disorientation to self is frequently observed (a) Etiology and irreversibility of condition. at the onset of a toxic encephalopathy. (b) Absence of brainstem refl exes. 15. Hyperventilation causes a respiratory acidosis. (c) Absence of motor response to pain. 16. Skew deviation is a horizontal divergence of (d) Absence of respiration with the globes.

p CO2 ³ 60 mmHg. 17. Multifocal myoclonus is a hallmark sign of a (e) Justifi cation for confi rmatory test and metabolic encephalopathy. result of confi rmatory test. 18. Asterixis is pathognomonic of hepatic (f) Repeat neurologic examination. Option : encephalopathy. the interval is arbitrary, but a 6-h period is 19. Circumstances surrounding a cardiac arrest reasonable. are a good prognostic indicator. 20. Falcine herniations may compress the ante- rior cerebral artery causing cerebral edema. Questions (True or False) 21. Cheyne-Stokes respirations may occur with both central transtentorial herniation and 1. The most important question to solve in an congestive heart failure. unresponsive patient is whether a focal 22. Upward transtentorial herniation due to a lesion, multifocal lesions, or diffuse dysfunc- cerebellar mass lesion is likely to compress tion are present. the posterior commissure to produce paresis 2. Cheyne-Stokes respiration is sustained of upward gaze. hyperventilation. 23. An intact thalamic reticular formation is 3. Apneustic breathing is due to a pontine required to preserve wakefulness. lesion. 24. In coma the eyes are closed; in a vegetative 4. An intubated patient cannot have a satisfac- state they open periodically due to preserved tory assessment of his mental status for diag- wakefulness. nostic purposes. 25. A vegetative state due to trauma is perma- 5. Decorticate posturing includes fl exion of the nent at 3 months. legs. 26. Akinetic mutism may not occur acutely. 6. Patients with papilledema may have hemor- 27. A minimally conscious state commonly rhages which are located at the disk depends on observations by nurses who may margins. need instructions about recording their 7. Caloric testing is a more powerful stimulus observations. than the oculocephalic maneuver. 28. A patient with a locked-in state is unable to 8. Caloric testing is the best method to detect communicate. psychogenic unresponsiveness. 29. Assessment for brain death does not include 9. Horizontal eye deviation generally indicates caloric testing. a focal lesion. Eye deviation in the vertical 30. The corneal refl ex is part of the examination plane is common with metabolic or hypoxic for brain death. ischemic encephalopathy. 31. Deep tendon refl exes may be present in brain 10. Pupil abnormalities due to Edinger-Westphal death. nucleus lesions are larger than pretectal 32. Kernohan’s notch is associated with an ipsi- lesions. lateral hemiparesis. References 125

26. F Answers 27. T 28. F 1 . T 29. F 2 . F 30. T 3 . T 31. T 4 . F 32. T 5 . F 6 . T 7 . T References 8 . T 9 . T Bernat JL. Current controversies in states of chronic 10. F unconsciousness. Neurol Clin Pract. 2010;75 Suppl 11. T 1:S33Ð8. 12. T Giacino JT, Ashwal S, Childs N, et al. The minimally con- scious state. Neurology. 2002;58:349Ð53. 13. T Levy DE, Bates D, Caronna JJ, et al. Prognosis in non- 14. F traumatic coma. Ann Intern Med. 1981;94:293Ð301. 15. F Posner J, Saper C, Schiff N, Plum F. Diagnosis of stupor 16. F and coma. New York: Oxford University Press; 2007. 17. T Ropper AH. Unusual spontaneous movements in brain- 18. F dead patients. Neurology. 1984;34:1089Ð92. 19. F Rosenberg RN. Consciousness, coma and brain death- 20. T 2009. JAMA. 2009;301:1172Ð4. Wijdicks EFM, Hidra AH, Young GB. Practice parameter: 21. T prediction of outcome in comatose survivors after car- 22. T diopulmonary resuscitation (an evidence-based 23. T review). Neurology. 2006;67:203Ð10. 24. T Wijdicks EFM, Varelas PN, Gronseth GS, Greer DM. Evidence-based guideline update: determining brain 25. F death in adults. Neurology. 2010;74:1911Ð18. The Six Major Anatomic Decussations with Clinical 6 Correlation

Keywords Anatomic decussations ¥ Pursuit ¥ Corticospinal ¥ Visual pathway ¥ Corticobulbar ¥ Sensory systems ¥ Oculomotor pathway ¥ Saccades

Initially, the most confounding elements of junction. The fi bers then descend in the contralateral neuroanatomy for many medical students are corticospinal tract located in the lateral columns likely to be the decussations of the neural path- and terminate at lower motor neurons in the ante- ways. There are six major and partial decussa- rior horn of the spinal cord. Lamination of the tions; most are simple, but they must be committed fi bers from medial to lateral are cervical, thoracic, to memory as they are the kernel of neuroana- lumbar, and sacral. tomic localization. These are the corticospinal tract, corticobulbar tract, visual pathway, oculo- The corticospinal tract arises from both pre- motor decussation with its associated pathways, central (frontal) and postcentral (parietal) lateral spinothalamic tract, and medial lemniscus. cortical areas. This chapter will attempt to summarize these anatomic facts in a concise, proximate fashion to facilitate an easy review when required. 80–90% of corticospinal tract fi bers cross in the decussation of the pyramids located at the cervical-medullary junction. The Corticospinal Tract (Fig. 6.1 ) The neurologic defi cits arising from lesions of There are three motor systems in the brain: corti- the corticospinal tract include physical weakness, cospinal, extrapyramidal, and cerebellar. The poor dexterity, abnormal muscular tone, and corticospinal tract is the primary motor system abnormal refl exes. The weakness has a predilec- pathway. It arises from both precentral (frontal) tion for distal musculature. Poor dexterity, which (60%) and postcentral (parietal) (40%) cortical is tested by fi nger tap, foot tap, and rapid alternat- areas. The fi bers descend in the ipsilateral corona ing movements, frequently occurs fi rst. Muscular radiata, posterior limb of the internal capsule, tone may be spastic and refl exes can be increased. central portion of the cerebral peduncle, basis The prototypical abnormal refl ex is the Babinski pontis, medullary pyramids and 80Ð90% cross in sign. Any one of these fi ndings can be noted in the pyramidal decussation at the cervical-medullary isolation or in different combinations.

J.N. Alpert, The Neurologic Diagnosis: A Practical Bedside Approach, 127 DOI 10.1007/978-1-4419-6724-4_6, © Springer Science+Business Media, LLC 2012 128 6 The Six Major Anatomic Decussations with Clinical Correlation

Fig. 6.1 Corticospinal tract pathways. Obtained with permission from Blumenfeld (2002 )

1. Clinical summary: monoparesis, spasticity, impaired dexterity, (a) Cerebral (cortical or subcortical) and brain- increased refl exes, and Babinski sign. stem lesions which affect the corticospinal tract may cause one or more of these fi nd- Cerebral and brainstem lesions cause ings: contralateral hemiparesis or mono- contralateral hemiparesis and spinal paresis, spasticity, impaired dexterity, cord lesions, ipsilateral hemiparesis. increased refl exes, and Babinski sign. (b) Unilateral cervical spinal cord lesions (c) Unilateral thoracic spinal cord lesions which affect the corticospinal tract in the which involve the corticospinal tract may lateral columns may cause one or more of cause one or more of these fi ndings: ipsi- these fi ndings: ipsilateral hemiparesis or lateral leg weakness, spasticity, impairment The Corticobulbar Pathways 129

of rapid foot tapping, increased refl exes, and Babinski sign. The Corticobulbar Pathways (d) Lesions at the cervical-medullary junc- (Fig. 6.2 ) tion may cause a triparesis, quadriparesis, cruciate paresis (such as involvement 1. Clinical correlation: of left arm and right leg), and rarely (a) Contralateral facial weakness may occur bilateral arm paralyses (man-in-a-barrel with lesions affecting the corticobulbar syndrome). pathways. Nerve fi bers which control (e) Most spinal cord lesions produce bilateral facial muscles originate from neurons in abnormalities of the corticospinal tracts. the lower 3rd of the cortical motor fi elds,

Fig. 6.2 Corticobulbar pathways. Adapted from Brazis et al. (2007 ) 130 6 The Six Major Anatomic Decussations with Clinical Correlation

descend in the corona radiata, genu of the through the corona radiata, internal internal capsule, medial portion of the capsule, medial portion of the cerebral cerebral peduncle, basis pontis, and decus- peduncle, and decussate in the pons to sate just above the 7th nerve nuclei. There synapse in the motor nucleus of V. is some bilateral innervation in the 7th Although there is bilateral control of nerve nuclei. The frontalis muscle has motor function, there is primarily contral- bilateral innervation and is usually, but ateral innervation of the masseter muscle. not always, spared with contralateral Thus jaw deviation to the contralateral lesions above the decussation. Emotional, side of the lesion may rarely occur. involuntary facial movements are likely (d) There is bilateral innervation of the to be innervated by a separate pathway nucleus ambiguus (9th through 11th cranial since voluntary movements and strength nerve nuclei). Consequently, dysphagia is may be normal; but a brief laugh may generally not a symptom of an acute expose an overt facial droop. A separate unilateral cerebral lesion unless it is exten- pathway to explain this disparity is likely sive. An unusual exception is dysphagia as lesions of the frontal lobe, just anterior associated with nonfl uent aphasias due to to the precentral gyrus, have been found an apraxia of tongue movements. to be associated with these clinical fi ndings. Because of bilateral innervation of the nucleus ambiguus, dysphagia is seldom Corticobulbar lesions often cause caused by an acute unilateral cerebral contralateral facial weakness. lesion.

(e) Corticolingual fi bers to the tongue arise Emotional, involuntary facial move- from neurons in the lower portion of the ments are likely to be innervated by a precentral gyrus. These muscles are inner- separate pathway since voluntary move- vated bilaterally except for the genioglossus ments and strength may be normal. which is solely supplied by the contralat- eral cerebral hemisphere. Consequently, (b) Cerebral, thalamic, and midbrain lesions the tongue may deviate to the side of the may cause contralateral facial numbness hemiparesis. Dysarthria appears most and, infrequently, a decreased corneal often when corticolingual pathways are refl ex. The ventral posteromedial nucleus interrupted on either side. The lesions can (VPM) of the thalamus receives crossed be located in either cerebral hemisphere fi bers from the main sensory nucleus of or in the brainstem. the trigeminal nerve. The fi bers exit from the latter nucleus, cross the midline, and Dysarthria appears most often ascend in the quintothalamic tract. Fibers when corticolingual pathways are ascend from the VPM to ipsilateral pari- interrupted. etal lobe neurons. (f) The corticobulbar pathway to the sterno- Cerebral, thalamic and midbrain cleidomastoid muscles probably involves lesions may cause contralateral facial a double decussation. The trapezius mus- numbness. cle on the side of the hemiplegia is paretic. The sternocleidomastoid muscle, however, (c) Nerve fi bers which control jaw muscles shows ipsilateral weakness. Consequently, via the motor root of the 5th nerve originate the head is turned to the side of the lesion. in the lower frontal motor cortex, descend This conception of the anatomic pathway The Oculomotor Decussation and Associated Pathways 131

is supported by the presence of focal seizures which cause contralateral jerking The Oculomotor Decussation of the head (adversive seizure). It has been and Associated Pathways (Fig. 6.3 ) suggested that the pathway for sterno- mastoid control crosses the midline to the 1. Saccadic system for horizontal eye opposite side of the pons and then crosses movements. back to the ipsilateral spinal cord via the Saccades (quick eye movements), which are pyramidal decussation (a second decussa- both voluntary and refl exive, are triggered from tion). Thus, hemiplegia with contralateral both frontal and parietal cortex eye fi elds (FEF head deviation implies a lesion between and PEF). FEF and PEF are located around the fi rst and the second decussations. the lateral part of the precentral gyrus. There are descending multisynaptic pathways from the FEF and PEF to the brainstem. Some pass The head is turned to the side of a cor- through the thalamus and down to the midbrain ticobulbar lesion because of ipsilateral reticular formation; others may descend through sternocleidomastoid weakness. the anterior limb of the internal capsule and the medial portion of the cerebral peduncle. The 2. Clinical summary: decussation of these fi bers is at the pontomes- (a) Unilateral cerebral lesions, right or left, may encephalic junction and the oculomotor path- cause one or more of the following fi ndings: way terminates at the contralateral PPRF. The contralateral facial numbness with or with- FEF and PEF also provide major input to the out a decreased corneal refl ex, contralateral superior colliculus which is a critical part of facial weakness, dysarthria due primarily to the saccadic system contributing additional involvement of the corticolingual pathway, innervation to the PPRF. ipsilateral head deviation, and contralateral tongue deviation. Rarely, there is contralat- eral jaw deviation due to masseter weakness Voluntary and refl exive saccades are trig- which is supplied mostly by the contralat- gered from FEF and PEF. eral cerebral hemisphere, but still with sig- nifi cant bilateral innervation. The PPRF sends fi bers to the ipsilateral 6th (b) Midbrain and rostral pontine lesions may nerve nucleus to synapse with neurons whose cause the same clinical fi ndings. axons form the 6th nerve and interneurons (c) Medulla lesions may result in contralat- whose axons immediately cross the midline eral head deviation and contralateral to constitute part of the medial longitudinal tongue deviation. fasciculus (MLF). The MLF ascends to the (d) Lesions immediately below the decussa- 3rd nerve nucleus where its axons synapse. tion of the pyramids may cause ipsilateral head deviation because it is below the sec- The 6th nucleus contains interneurons which ond decussation of the pathway to the send their axons to the contralateral MLF. sternomastoid muscle. 3. Synopsis: Thus, for example, the right cerebral eye Unilateral lesions on either side of the cor- fi elds (FEF and PEF) initiate gaze to the left. ticobulbar pathway, cerebral or brainstem, The pathway travels downward through the commonly cause one or more of the following right thalamus and anterior limb of the inter- clinical symptoms or signs: (1) dysarthria, (2) nal capsule, right midbrain in the region of the contralateral facial numbness, (3) contralat- rostral interstitial nucleus of the MLF (riMLF), eral facial weakness, (4) contralateral tongue crosses the midline at the pontomesencephalic deviation, and (5) ipsilateral or contralateral junction and synapses in the left PPRF. The head deviation. left PPRF neurons send their axons to neurons 132 6 The Six Major Anatomic Decussations with Clinical Correlation

Fig. 6.3 Oculomotor pathway

and interneurons in the left 6th nerve nucleus. ipsilateral medial rectus weakness or an These neurons are the source of the left 6th adductor lag with saccadic eye movements nerve and, the interneurons, a source of fi bers in the direction of ipsilateral medial rectus which cross the midline to form part of the function. There is frequent nystagmus in right MLF which ascends to the right 3rd the contralateral abducting eye. The etiol- nerve nucleus. The right 3rd nerve arises from ogy of this phenomenon remains specula- the right 3rd nerve nucleus to innervate the tive. Despite paresis of the medial rectus right medial rectus. muscle, convergence is often preserved. 2. Clinical correlation: Saccadic system (f) Lesions of the abducens nucleus cause pathways. ipsilateral lateral rectus weakness and, in (a) Lesions of frontoparietal eye fi elds cause rarely documented cases, an ipsilateral ipsilateral eye deviation. gaze paresis. (b) Lesions of the thalamus and the anterior limb of the internal capsule may cause The PPRF generates ipsilateral conju- ipsilateral eye deviation. Thalamic lesions gate gaze; a lesion causes contralateral have also rarely been associated with eye deviation. “wrong-way eye deviation,” that is toward the hemiparetic side, an inexplicable observation. The MLF generates ipsilateral adduc- (c) Lesions of the midbrain cause ipsilateral tion; a lesion produces adduction eye deviation. paresis. (d) Lesions of PPRF cause contralateral eye deviation. 3. Pursuit system for horizontal eye movements. (e) Lesions of the MLF cause an internuclear Pursuit (slow) eye movements are usually ophthalmoplegia which is manifested by the only examined eye movement system at The Oculomotor Decussation and Associated Pathways 133

the bedside. This ignores the above-described 5. Vertical eye movements. saccadic system pathway lesions of which (a) Saccadic system. generate important clinical data. Since the Vertical saccades are generated from pursuit system is complex, its description will neurons in the riMLF located in the mid- be condensed to the most important proposed brain. The neurons for upward saccades elements. are located in the lateral portion and Ipsilateral pursuit eye movements are gen- downward saccades, medial. These neu- erated from the occipitotemporal region. rons send fi bers to the interstitial nucleus Fibers from this area project to the ipsilateral of Cajal (INC) which projects to the dorsomedial frontal cortex. Lesions involving posterior commissure. these loci impair ipsilateral pursuit. Thus the left oculomotor pathway generates ipsilateral The rostral interstitial nucleus of the pursuit eye movements and vice versa. The riMLF generates vertical saccades. pathway may descend through the thalamus and crosses twice in the brainstem. Pontine (b) Pursuit system. nuclei project ipsilateral fi bers via the middle The INC located in the midbrain medi- cerebellar peduncle to the cerebellum where ates vertical smooth pursuit. It receives there are multiple synapses. Ultimately, input from the cerebellum via the superior Purkinje cells send inhibitory fi bers to the cerebellar peduncle, lower brainstem, and medial vestibular nuclei (MVN). There are the medial longitudinal fasciculus. excitatory projections from the MVN to the Projections to oculomotor nuclei from the contralateral 6th nucleus. Smooth pursuit may riMLF and INC for up movements decus- not be initiated in the PPRF. sate in the posterior commissure. 6. Clinical correlation: Vertical eye movements. The cerebral hemisphere governs ipsilateral (a) Vertical palsies fi rst affect saccades, as ocular pursuit. pursuit eye movements and vestibulo-ocu- lar refl exes are often initially spared. 4. Clinical correlation: Pursuit pathway. (b) Upgaze paresis is more common than com- (a) Bilateral occipital lobe lesions completely bined up and down palsies which are more eliminate smooth pursuit. common than isolated downgaze palsies. (b) Brainstem lesions may cause ipsilateral or (c) Paresis of upgaze is caused by lesions of contralateral pursuit defects due to the dou- the posterior commissure or nucleus of the ble decussation. posterior commissure. There are rare (c) Cerebellar lesions, especially fl occulus and instances of upgaze paresis due to lesions vermis, interfere with pursuit eye move- of the median raphe in the pons. ments in either direction, most often ipsilateral. Paresis of upward gaze is most often (d) Vestibulo-ocular refl exes are often enhanced due to lesions of the posterior in patients with impaired ocular pursuit. commissure. (e) Saccadic pursuit is the substitution of sac- cades to catch up with a moving target. (d) Paresis of downgaze saccades occurs with It is the most commonly observed abnor- bilateral dorsomedial lesions of the riMLF mality with lesions involving pursuit system and, if the lesions extend laterally, upgaze pathways. saccades are also impaired.

Saccadic pursuit is the substitution of Combined paresis of downgaze and saccades to catch up with a moving upgaze saccades result from bilateral target. riMLF lesions. 134 6 The Six Major Anatomic Decussations with Clinical Correlation

(e) Paresis of downgaze saccades, pursuit, and The optic nerve has four sections. They are the oculocephalic eye movements have been optic nerve head (1 mm), intraorbital (25 mm), observed with bilateral INC and posterior intracanalicular (9 mm), and intracranial segment commissure lesions. Downgaze paralysis (4Ð16 mm). The total length of the optic nerve of all eye movements, saccades, pursuit, averages about 45Ð50 mm. The optic nerve fi bers and oculocephalics result from bilateral have a topical arrangement. INC or posterior commissure lesions. Those optic nerve fi bers which receive visual input from the temporal fi elds cross to the contral- ateral side of the chiasm. Optic nerve fi bers receiv- The Visual Pathways (Fig. 6.4 ) ing input from the nasal fi elds remain uncrossed. Macular fi bers are both crossed and uncrossed, and believed to comprise approximately 80% of The fi rst neuronal elements are rods and cones nerve fi bers at the chiasm. The anteriorÐposterior which contain pigment that, when exposed to diameter of the chiasm is approximately 1 cm and light, produces electrical activity. At the posterior it varies from several millimeter to 1.5 cm above pole is the macula and, nasal to it, the optic nerve. the diaphragma sellae. Cones, which perceive color, are in the macula and the rods are located elsewhere. Both rods and cones convey activity to the ganglion cells, which Macular fi bers are believed to comprise provide the axons that comprise the optic nerve, approximately 80% of the nerve fi bers at the optic chiasm, and optic tract. These synapse in chiasm. the lateral geniculate nucleus. A prefi xed chiasm is located over the anterior Retinal ganglion cells provide the axons that portion of the sella and postfi xed behind the dor- comprise the optic nerve, optic chiasm, and sum sella. About 80% of optic chiasms are just optic tract. above the dorsum sella. Fibers from the inferior part of the nasal retina are in the ventral part of the chiasm. Fibers from the superior nasal retina remain dorsal in the chiasm. Consequently, pitu- itary neoplasms are more likely to affect superior temporal quadrants and lesions originating from above the chiasm, such as the craniopharyn- gioma, are more likely to cause inferior temporal quadrant defects. The optic tract (2.5 cm) extends from the chi- asm to the lateral geniculate nucleus of the thal- amus. This nucleus receives the axons from the retinal ganglion cells. Neurons in the lateral geniculate nucleus provide axons which form the optic radiations. Meyer’s loop is a portion of the radiations which run lateral to the temporal horn of the lateral ventricle about 5 cm behind the tip of the temporal lobe. The superior fi bers correspond to the superior retina and, conse- quently, lesions in the parietal lobe may produce inferior homonymous (both eyes) quadrantano- psias. The inferior portion contains fi bers from the inferior retina and, consequently, temporal Fig. 6.4 Visual pathways lobe lesions may produce superior homonymous The Visual Pathways 135 quadrantanopsias. The central portion of the defects and lesions from above result in optic radiations contains fi bers from the macula inferior visual fi eld loss. which are both crossed and uncrossed. Thus, (c) Optic tract, lateral geniculate (rare), optic unilateral lesions of the optic radiations usually radiation, and occipital lobe lesions usu- spare central vision. ally cause a contralateral homonymous hemianopsia. Neurons in the lateral geniculate nucleus pro- (d) Incongruous (different amount of visual vide axons which form the optic radiations. loss in each eye) homonymous hemiano- psias or quadrantanopsias are most com- monly noted with optic tract lesions, Parietal lobe lesions may cause inferior hom- infrequently lateral geniculate lesions. onymous quadrantanopsias, and temporal (e) Congruous (identical amount of visual loss lobe lesions may produce superior homony- in each eye) homonymous hemianopsias or mous quadrantanopsias. quadrantanopsias may be observed with any lesion behind the chiasm. The visual cortex is located on the superior and inferior edges of the calcarine fi ssure in the Visual pathway lesions behind the occipital lobe. The right side, for example, con- optic chiasm cause contralateral tains fi bers from the right temporal retina and left homonymous hemianopsias. nasal retina. Thus, right occipital lesions cause a contralateral homonymous (both eyes) hemiano- (f) Parietal lobe lesions which involve the psia. The occipital pole receives macular fi bers optic radiations are likely to produce infe- from both eyes such that central visual loss ordi- rior homonymous quadrantanopsias (pie- narily occurs from bilateral involvement only. on-the-fl oor). The term “striate cortex” is so named because the (g) Temporal lobe lesions are likely to cause line of Gennari (a thick white band) is easily seen superior homonymous quadrantanopsias in the visual cortex without magnifi cation. The (pie-in-the-sky). striate cortex extends onto the convexity of the (h) Occipital lobe lesions may produce hom- occipital lobe. onymous hemianopsias, quadrantanopsias 1. Clinical correlation: Visual impairments and, infrequently, homonymous scotomas occurring with lesions of the visual pathways. which are easily overlooked on bedside examination. Yet they commonly cause Optic nerve lesions cause central scotomas disabling symptoms. fi rst and decreased acuity second. (i) Medial occipital lobe lesions may cause a temporal crescent visual fi eld loss (a) Optic nerve lesions result in central scoto- affecting only one eye contralateral to the mas fi rst and decreased visual acuity sec- lesion. This defect violates the rule of ond. Unilateral altitudinal or quadrantic homonymous visual fi eld loss with visual fi eld loss occurs most often with involvement of visual pathways behind vascular pathology. the chiasm. (b) Optic chiasm lesions usually produce cen- tral scotomas since approximately 80% of Medial occipital lobe lesions may chiasm fi bers represent macular vision. cause a temporal crescent visual fi eld With progressive damage, bitemporal loss affecting only one eye contralat- hemianopsias or unilateral temporal quad- eral to the lesion. rantic fi eld defects may occur. Lesions from below, such as pituitary neoplasms, (j) Bilateral occipital lobe lesions are usually typically produce superior visual fi eld due to vascular disease involving the basilar 136 6 The Six Major Anatomic Decussations with Clinical Correlation

artery system. Visual fi eld defects include 3. Temporal crescent defect, contralateral. superior or inferior altitudinal hemianop- 4. Altitudinal homonymous hemianop- sias which have inverted representations. sias, inferior or superior. The upper banks receive innervation from 5. Bilateral incomplete homonymous the inferior visual fi elds and vice versa. hemianopsias or quadrantanopsias Consequently, bilateral superior or infe- including checkerboard vision and rior altitudinal fi eld defects of abrupt cortical blindness. onset are ordinarily due to basilar artery disease or cardioembolism to the basilar artery bifurcation. The Sensory Systems Altitudinal homonymous hemianop- sias are most often due to bilateral 1. Decussation of the medial lemniscus (medulla) occipital lobe infarctions. (Fig. 6.5 ) .

2. Synopsis: Visual impairments with visual The posterior (dorsal) columns mediate pathway lesions. ipsilateral position and vibration sense. (a) Optic nerve lesions: Central scotomas, Lamination of its fi bers from medial to lat- unilateral altitudinal or quadrantic fi eld eral are sacral, lumbar, thoracic, and defects, impaired visual acuity. cervical. (b) Optic chiasm lesions: Central scotomas, bitemporal hemianopsia, unilateral tempo- This pathway primarily mediates position ral quadrant visual fi eld loss, or combina- and vibration sense. Secondarily, with regard tions of central scotoma with temporal to clinical importance, are fi bers for pressure visual fi eld loss. and touch. (c) Optic tract lesions: Contralateral homony- Large myelinated fi bers exit from the dor- mous hemianopsia, usually incongruous, sal horn to ascend in the ipsilateral posterior rarely central scotomas, and homonymous columns, the fasciculus gracilis (medial), and scotomas. cuneatus (lateral). There is a somatotopic (d) Lateral geniculate lesions (rare): arrangement as sacral fi bers are medially Contralateral homonymous hemianopsia placed and subsequent fi bers are gradually or quadrantanopsia, congruous or layered in adjacent fashion, leg next and arm incongruous. lateral, as the tract moves rostrally to synapse (e) Optic radiation lesions (temporal lobe): in the nucleus gracilis and cuneatus in the Congruous, contralateral homonymous medulla. Axons of cells in these nuclei decus- hemianopsia, or superior quadrantanopsia. sate in the medial lemniscus which is midline Incongruity is uncommon. in the medulla. They largely maintain the (f) Optic radiation lesions (parietal lobe): somatotopic arrangement as they ascend to Congruous, contralateral homonymous the ventral posterolateral (VPL) nucleus of the hemianopsia, or inferior quadrantanopsia. thalamus. Axons from this thalamic nucleus Incongruity is uncommon. project to the postcentral gyrus in the parietal (g) Occipital lobe lesions: lobe. The calf and foot are represented on the 1. Contralateral, congruous homonymous medial surface of the cerebral hemisphere fol- quadrantanopsias and hemianopsias, lowed by the thigh, abdomen, and chest. The macular sparing, and with infrequent shoulder neurons are on the tip of the con- macular involvement. vexity. The arm, hand, digits, and face are rep- 2. Homonymous scotomata, contralateral. resented successively over the convexities. The Sensory Systems 137

Fig. 6.5 Posterior column-medial lemniscus pathway. With permission from Blumenfeld (2002 )

The three latter anatomic structures comprise (c) Thalamic lesions (VPL nucleus): the majority of the surface of the parietal lobe. Contralateral vibration and loss of posi- tion sense. Lesions above the medial lemniscus decus- (d) Parietal lesions: Contralateral loss of posi- sation produce contralateral loss of posi- tion sense much more prominent than tion and vibration sense. vibration sense. Vibration perception is commonly preserved. 2. Clinical correlation: (a) Spinal cord lesions: Ipsilateral impair- Vibration sense is lost before posi- ment of vibration and position sense. tion sense with any lesion of the cen- Vibration sense is lost fi rst. tral or peripheral nervous system, (b) Brainstem lesions: Contralateral impair- except for those in the cerebral ment of vibration and position sense. hemisphere. Vibration sense loss is more prominent. 138 6 The Six Major Anatomic Decussations with Clinical Correlation

3. Decussation of the anterior commissure of the from the VPL then ascend to the postcentral spinal cord (Fig. 6.6 ) . gyrus of the parietal lobe and are distributed Small, unmyelinated fi bers enter the spi- in the same fashion as those mediating posi- nal cord, descend one or two segments in the tion and vibration sense. zone of Lissauer and synapse in the dorsal horn. Axons from these neurons decussate in The lateral spinothalamic tract ascends and the anterior commissure and ascend in the synapses within the VPL nucleus of the ventrolateral portion of the spinal cord. There thalamus. is a somatotopic arrangement with sacral fi bers laterally placed followed by lumbar, 4. Clinical correlation: thoracic, and cervical. The fi bers are gradu- (a) Spinal cord lesions may cause contralat- ally layered, adjacent and medial, as the tract eral loss of pain and temperature produc- moves rostrally. This arrangement is ing a sensory level, commonly one or two unchanged as the tract ascends and synapses dermatomes below the level of the lesion. in the VPL nucleus of the thalamus. Axons In clinical practice, it is not unusual to

Fig. 6.6 Lateral spinothalamic pathway. With permission from Blumenfeld (2002 ) Bibliography 139

fi nd sensory levels much below the location 16. Incongruous homonymous hemianopsias are of the lesion. often due to optic tract lesions. 17. Monocular visual fi eld defects are always Spinal cord lesions may cause contral- due to lesions anterior to the chiasm. ateral loss of pain and temperature 18. Neurons of the VPL nucleus of the thalamus producing a sensory level. receive input from the lateral spinothalamic tract and the medial lemniscus. (b) Cerebral, thalamic, and brainstem lesions 19. Heavily myelinated fi bers mediating pain may cause contralateral loss of pain and and temperature cross in the anterior temperature sensation. commissure. 20. Bilateral occipital lobe lesions may cause altitudinal homonymous hemianopsias. Questions (True or False)

1. The corticospinal tract arises solely from the Answers frontal lobe. 2. Corticospinal tract lesions in the brainstem 1 . F cause a contralateral hemiparesis. 2 . T 3. Bilateral arm paralysis may be due to lesions 3 . T of the high cervical cord. 4 . T 4. Contralateral facial weakness can be caused 5 . F by midbrain lesions. 6 . F 5. It is not possible to observe central facial 7 . T weakness, unless there is a lesion of the cor- 8 . F ticobulbar pathway. 9 . T 6. The masseter muscle has equal bilateral 10. F innervation. 11. T 7. Dysphagia rarely occurs with an acute cere- 12. T bral lesion. 13. F 8. Dysarthria has localizing signifi cance. 14. T 9. The oculomotor pathway decussates between 15. F the midbrain and the pons. 16. T 10. The medial longitudinal fasciculus initiates 17. F conjugate gaze. 18. T 11. The right PPRF sends fi bers to interneurons 19. F in the right 6th nucleus which provide axons 20. T forming part of the left MLF. 12. Lesions of the posterior commissure produce upgaze paralysis. 13. The majority of nerve fi bers in the optic Bibliography nerve come from the temporal fi elds. 14. The central scotoma is a hallmark fi nding Blumenfeld H. Neuroanatomy through clinical cases. with optic nerve disease. Sunderland: Sinauer Associates; 2002. Brazis PW, Masdeu JC, Biller J. Localization in clinical 15. The earliest sign of an optic chiasm lesion is neurology. 5th ed. Philadelphia: Lippincott Williams a bitemporal hemianopsia. & Wilkins; 2007. Cerebrovascular Anatomy with Clinical Correlation 7

Keywords Aneurysm • Vascular malformation • Carotid • Vertebrobasilar • Hemorrhage • Ischemia • Stroke

Cerebrovascular disease ordinarily comprises about 5–15% of the outpatient practice of a gen- Ischemic Stroke (Cerebral Infarction) eral neurologist, but it is the predominant disor- and Transient Ischemic Attack der in the hospital. Thus, the medical student experience is skewed toward evaluation and treat- Cerebrovascular accident (CVA) is an unac- ment of stroke, a critically important fi eld, but ceptable term since it does not distinguish not necessarily representative of the spectrum of between hemorrhage and infarction. neurology. Adequate preparation to make the most of this experience requires a familiarity with Terminology must be clarifi ed fi rst. Strokes are vascular anatomy prior to beginning the manda- either ischemic or hemorrhagic. An ischemic tory rotation, presumably a minimum of 1 month. stroke is a cerebral infarction caused by an arterial This will be summarized to emphasize salient occlusion. A hemorrhagic stroke is an intracere- features. Discussion of pathology is essential bral hematoma or a subarachnoid hemorrhage since ischemic strokes (85%) and hemorrhagic (SAH). Cerebrovascular accident (CVA) is an strokes (15%) have distinctive clinical and ana- unacceptable diagnostic term since it neither dis- tomic patterns. Treatment implications will be tinguishes between infarction and hemorrhage nor briefl y reviewed. Case reports will be added to is “accident” an accurate description of the pathol- illustrate the importance of the history and neuro- ogy. Moreover, is a “right-sided CVA” a right cere- logic examination to determine both the anatomic bral infarction, a right cerebral hemorrhage, or a and pathologic diagnosis, the evaluation indi- right hemiparesis? Does it include brainstem cated, and some current treatment approaches. infarctions or hemorrhage? The use of the acronym

J.N. Alpert, The Neurologic Diagnosis: A Practical Bedside Approach, 141 DOI 10.1007/978-1-4419-6724-4_7, © Springer Science+Business Media, LLC 2012 142 7 Cerebrovascular Anatomy with Clinical Correlation

CVA is analogous to a medical diagnosis of heart By the time the patient arrived in the Emergency attack instead of myocardial infarction. Department, he was normal. A carotid Doppler showed a smooth 60% left The anatomic source of stroke, cardiac, internal carotid artery stenosis and a CAT scan of extracranial, or intracranial, determines treat- the brain was normal. Vascular surgery consulta- ment and should be part of the diagnosis. tion was obtained and a left carotid endarterec- tomy was recommended. A neurologic The anatomic source of cerebral infarction or consultation was then requested. transient ischemic attack (TIA) determines the The neurology consultant queried the patient selection of treatment. The exception is tissue about specifi c additional neurologic symptoms. plasminogen activator (TPA) which is used within The patient had both auditory and visual symp- 4.5 h of the onset of an acute ischemic stroke irre- tomatology. He now recollects that he heard noises spective of etiology. The overwhelming majority in one ear and saw colored spots in the left visual of cerebral infarctions are due to either athero- fi eld for 1–2 min. An MRA (head and neck) was sclerotic disease of intracranial arteries, extracra- obtained and disclosed a moderate-to-severe basi- nial arteries, or cardioembolism. Treatment of lar artery stenosis. arterial atherosclerotic disease currently requires Final diagnosis : the use of antiplatelet agents, stents, or endart- TIA in the vertebrobasilar distribution due to a erectomy. Cardioembolism is prevented by anti- basilar artery stenosis (Idicula and Joseph 2007 ). coagulation using warfarin, heparin or dabigatran. A diagnosis of “cerebral infarction due to throm- Lesson No. 1 : boembolism,” a not infrequent diagnosis, is not Most TIAs last less than 5 min, rarely over suffi cient to make treatment recommendations. 30 min. The vascular source must be identifi ed. Lesson No. 2 : Dysarthria is a nonlocalizing symptom and must Vertebrobasilar or carotid distribution must be be distinguished from aphasia, a language disor- stipulated as the source of transient ischemic der which presumes left cerebral hemisphere dis- attack or stroke. ease even if the patient is left-handed. A left-handed patient has only about a 50% chance that he or she A second anatomic defi nition is to distinguish is right hemisphere dominant. between vertebrobasilar and carotid distribution stroke. The history and neurologic examination Dysarthria must be distinguished from aphasia. should take precedence. Complete dependence Dysarthria is nonlocalizing whereas aphasia on neuroimaging may lead to erroneous diagno- indicates dominant hemisphere involvement. ses since the presence of multiple lesions is not infrequent. Treatment decisions depend on a pre- Lesson No. 3 : cise defi nition of the pathologic arterial supply. A neurologic history must include a complete Case 1 A 45-year-old man arrives in the review of all neurologic symptoms. Patients often Emergency Department with a history of slurred need prompting to recall symptoms of diagnostic speech and right-sided weakness. He has coro- signifi cance such as in this case. nary artery disease and smokes one pack of ciga- rettes per day. A fourth-year medical student on A complete review of all neurologic symptoms the cardiology service carefully questions the is essential in all neurologic evaluations. patient about his symptoms. For 5 min the patient’s speech was barely intelligible according Lesson No. 4 : to witnesses. His words were spoken correctly Hemiparesis is not a localizing symptom or fi nd- but with marked slurring. Weakness was greater ing other than usually indicating central nervous in the arm than the leg and walking was unstable. system but not peripheral nervous system disease. Ischemic Stroke (Cerebral Infarction) and Transient Ischemic Attack 143

Lesson No. 5 : out a left internal carotid origin of the ischemic Auditory symptoms may occur with posterior symptoms. circulation ischemic events but rarely with cere- Neurovascular anatomy begins with the aortic bral hemisphere ischemia. arch and great vessels (see Figs. 7.1 and 7.2 ). Clearly, cardioembolism or, infrequently, embo- Lesson No. 6 : lism from the aortic arch, is most likely to enter the Seeing colored spots, unformed visual hallucina- carotid circulation, left more than right, because of tions, are characteristic of occipital lobe ischemia the direct large vessel route. Approximately 70% or neuronal dysfunction of nonvascular origin. of cardioembolism events are in the carotid artery Lesson No. 7 : distribution, primarily middle cerebral artery. Of The vascular supply of the occipital lobe is the these, 7% are to the anterior cerebral artery (ACA). posterior cerebral artery (PCA) which arises from About 23% are in the vertebrobasilar system, 5% the tip of the basilar artery. An infrequent excep- multiple areas, and 2% in watershed zones (border tion is an anatomic variant whereby the PCA zone between vascular territories). arises directly from the internal carotid artery. The vertebral arteries arise from the subcla- vian arteries and infrequently the aortic arch. The Lesson No. 8 : right vertebral artery may originate from the The left visual fi eld location of the colored spots is innominate artery rather than the subclavian contralateral to the right hemiparesis, thus ruling artery. The vertebral arteries are frequently

Fig. 7.1 Aortic arch and branches 144 7 Cerebrovascular Anatomy with Clinical Correlation

intracranial vertebral arteries immediately below the origin of the basilar artery. It extends from the medulla where it supplies its ventral surface, travels down the anterior sulcus of the spinal cord all the way to the conus medullaris and cauda equina. There are radicular arteries arising from the aorta at lower cervical and upper tho- racic levels that provide additional vascular sup- port. The anterior two-thirds of the spinal cord are supplied by the anterior spinal arteries and its branches. The origin of the posterior spinal artery is usu- ally the vertebral arteries and these supply the posterior one-third of the spinal cord. There are posterior radicular arteries which provide addi- tional fl ow to the posterior spinal artery. The thoracic cord between T3 and T8 has lim- ited vascular supply and is particularly vulnerable to transient interruption of blood fl ow such as that may occur with repair of a thoracic or abdom- Fig. 7.2 Magnetic resonance angiography of the aortic inal aortic aneurysm. The artery of Adamkiewicz arch arises from the abdominal aorta and enters the spinal canal between T9 and L2. It is a large, asymmetric and the left is most often larger. The major contributor to the anterior spinal artery vertebral artery enters the transverse foramen at vasculature. Its protection during abdominal sur- C6 and exits at C2 curving around the atlanto- gery is essential to prevent paraplegia due to a occipital junction. It then enters the skull through thoracic spinal cord infarction. the foramen magnum. The thoracic cord between T3 and T8 has a Vertebral arteries are frequently asymmetric limited vascular supply and is particularly vul- and the left is most often larger. nerable to transient interruption of blood fl ow.

The posterior inferior cerebellar artery (PICA) The two vertebral arteries merge and form usually originates from the vertebral artery sev- the basilar artery at about the pontomedullary eral millimeters above the foramen magnum, junction (see Figs. 7.3 and 7.4 ). The long lateral circles the medulla, and supplies the dorsolateral circumferential, short lateral circumferential, medulla. A medial trunk irrigates the cerebellar and median arteries arise from the basilar artery. vermis and adjacent cerebellar hemisphere. A lat- The classical crossed syndromes such as Weber’s eral trunk supplies the cortical surface of the ton- and Benedikt’s are, in most instances, due to sils and inferior cerebellar hemisphere. pathology in the small, median, penetrating ves- sels arising from the basilar artery. These syn- The posterior inferior cerebellar artery (PICA) dromes are defi ned in Chap. 12. The pathology usually originates from the vertebral artery in these arteries is either microatheroma or lipohy- above the foramen magnum and supplies the alinosis, the subintimal collection of hyaline dorsolateral medulla. material which stains for fat. The latter can lead to vessel occlusion and produce lacunar infarctions. The anterior spinal artery is formed by an These are small and also commonly found in anastomosis of two branches derived from both deep subcortical cerebral white matter. Ischemic Stroke (Cerebral Infarction) and Transient Ischemic Attack 145

Fig. 7.3 Intracranial vasculature, anterior–posterior view

Fig. 7.4 MRA of intracranial vasculature, anterior–posterior view 146 7 Cerebrovascular Anatomy with Clinical Correlation

and vestibular apparatus as well as the facial The two vertebral arteries merge and form the nerves. The SCA arises from the distal portion of basilar artery at about the pontomedullary the basilar artery just below the basilar tip and junction. close to the pontomesencephalic junction. It cir- cles around the midbrain, supplies the upper pons There are three long lateral circumferential and midbrain tegmentum, tentorial surface of the arteries which arise from the basilar artery (see cerebellum, superior and lateral cerebellar hemi- Fig. 7.5 ). From below upward they are the ante- spheres, and superior cerebellar peduncle rior inferior cerebellar artery (AICA), superior (brachium conjunctivum). The posterior cerebral cerebellar artery (SCA), and PCA. A fourth, arteries originate from the basilar artery tip and the internal auditory artery, arises from either supply the occipital lobes, inferomedial portion the basilar artery or as a branch of AICA. AICA of the temporal lobes, midbrain, thalamus, chor- courses around the pons, travels through the oid plexus, and ependyma of the third and lateral cerebellopontine angle, supplies the lateral teg- ventricles. The fi rst branches off the PCA are the mentum of the lower two-thirds of the pons, small thalamoperforating arteries that supply brachium pontis, and the ventrolateral cerebellum. the midbrain, thalamus, and lateral geniculate The internal auditory artery supplies the cochlear nucleus. The medial and lateral posterior choroi- dal arteries are the next branches which supply the thalamus in its posterior portion and the plexus. The anterior portion of the PCA gives rise to the inferior temporal arteries which supply the inferior portion of the temporal lobe. Terminal branches of the PCA supply the pari- eto-occipital regions and, most importantly, give rise to the calcarine artery which supplies the visual cortex.

The anterior inferior cerebellar artery (AICA) supplies the pons, cerebellum and, variably, the peripheral vestibular, and cochlear apparatus.

The superior cerebellar artery (SCA) supplies the upper pons, midbrain, and cerebellum.

The posterior cerebral arteries usually origi- nate from the basilar tip and supply the occip- ital lobes, inferomedial portion of the temporal lobes, midbrain, and thalamus.

The common carotid arteries originate from the aortic arch on the left and usually the innomi- Fig. 7.5 Vertebrobasilar system with identifi cation of nate artery on the right. The proximal common adjacent cranial nerves. PCA posterior cerebral artery; carotid artery is seldom the site of symptomatic SCA superior cerebellar artery; AICA anterior inferior cer- ebellar artery; PICA posterior inferior cerebellar artery. atherosclerotic disease. It bifurcates into the With permission of Dr. Eric Bershad internal and external carotid arteries. Ischemic Stroke (Cerebral Infarction) and Transient Ischemic Attack 147

The common carotid arteries originate from The ophthalmic artery, the fi rst branch of the the aortic arch on the left and usually the internal carotid artery, is the origin of the cen- innominate artery on the right tral retinal artery which supplies the retina and the posterior ciliary arteries which supply The external carotid artery has several clini- the optic nerve. cally important branches. It divides into the superfi cial temporal and internal maxillary artery. The next two branches of the internal carotid The superfi cial temporal artery begins within the artery are fi rst the posterior communicating artery parotid gland, ascends to just in front of the ear so followed by the anterior choroidal artery. Just that it is easily palpated, and extends to the fore- beyond these vessels the internal carotid artery head. Temporal arteritis frequently involves this bifurcates into the anterior cerebral and middle vessel and the internal maxillary artery which cerebral arteries. The posterior communicating supplies deep structures of the face including the artery supplies the genu and anterior one-third of muscles of mastication. Hence headache and jaw the internal capsule and the tuberothalamic claudication are common presenting symptoms. branch supplies the anterior nucleus of the thala- One branch of the internal maxillary artery is the mus. Via the posterior communicating artery the middle meningeal artery which supplies the dura. circle of Willis connects the anterior circulation Head trauma, especially associated with a skull system (internal carotid artery) with the posterior fracture, may sever this artery which then results circulation system (posterior cerebral artery). in an epidural hematoma. The occipital artery The anterior choroidal artery supplies the optic arises directly from the posterior part of the external tract, lateral geniculate nucleus, hippocampus, carotid artery and supplies the posterior scalp. optic radiations, and a large part of the posterior limb of the internal capsule. It is the most impor- The external carotid artery branches include tant supply of the internal capsule after the mid- the superfi cial temporal and internal maxil- dle cerebral artery branches. Memory impairment lary arteries; the latter is the origin of the and amnesia have been documented with anterior middle meningeal artery choroidal artery occlusions. Eventual recovery would be expected, however, because of only The lowest portion of the internal carotid unilateral involvement. As mentioned previously artery is the usual site of atherosclerotic plaques. severe memory loss occurs only with bilateral The high cervical internal carotid artery is often cerebral disease. the diseased site in patients who have received The ACA is a midline vessel which supplies radiotherapy, have had an internal carotid artery the medial surface of each cerebral hemisphere dissection or fi bromuscular dysplasia. The other and the upper border of the frontal and parietal parts of the internal carotid artery are the petrous, lobes. The ACA’s A1 segment begins at the bifur- cavernous, and supraclinoid portions. The inter- cation of the internal carotid artery and ends at nal carotid artery in the cavernous sinus is adja- the level of the anterior communicating artery cent to the 3rd, 4th, fi rst and second divisions of (ACoA) and ACA junction. The A2 segment the 5th, and 6th cranial nerves. The fi rst major begins at this junction and continues to the genu branch off the supraclinoid portion of the internal of the corpus callosum. Penetrating branches carotid artery is the ophthalmic artery. This ves- from the A1 segment supply the optic nerve, sel enters the orbit via the optic canal and gives optic chiasm, optic tract, hypothalamus, inferior rise to the central retinal and posterior ciliary frontal lobe, and suprachiasmatic region. The A2 arteries. The single central retinal artery supplies segment’s penetrating branches supply the hypo- the entire retina. The posterior ciliary arteries thalamus and anterior striatum. The artery of supply the optic nerve. Heubner is commonly the fi rst branch of the A2 148 7 Cerebrovascular Anatomy with Clinical Correlation segment and supplies the anterior limb of the pattern is most common and provides a superior internal capsule, anterior putamen, anterior glo- and inferior division. The superior division gives bus pallidus, head of the caudate nucleus, hypo- rise to the orbitofrontal, precentral, central, and thalamus, and olfactory regions. anterior parietal arteries. The inferior division gives off the posterior parietal, posterior tempo- The anterior cerebral artery (ACA) is a mid- ral, and angular arteries. The trifurcation pattern line vessel which supplies the medial surface has an upper division which supplies much of the of each cerebral hemisphere and the upper frontal lobe, a middle division which divides into border of the frontal and parietal lobes. the central, anterior parietal, and angular branch and an inferior division which supplies most of The two major branches of the ACA are the the temporal lobe and temporo-occipital region. pericallosal and callosomarginal arteries. The The lenticulostriate vessels arise from the M1 pericallosal travels just over the corpus callosum segment of the middle cerebral artery and and the callosomarginal, if present, runs near or supply most of the internal capsule in the cingulate sulcus; both are easily identifi ed in angiograms. These vessels supply the inferior Summary frontal lobe, medial surface of the cerebral hemi- spheres, corpus callosum, and the superior 2 cm 1. Major Branches of the Vertebrobasilar of the lateral convexity, frontal, and parietal. System. The very short ACoA, 1.5 mm in length sup- (a) Posterior inferior cerebellar artery. This plies the suprachiasmatic region, dorsal optic chi- vessel originates from the vertebral artery. It asm, inferior frontal lobe, fornix, anterior portion supplies the dorsolateral medulla, cerebellar of the corpus callosum, septal area, and anterior vermis, and adjacent cerebellar hemisphere. hypothalamus. Treatment of aneurysms of the (b) Anterior inferior cerebellar artery. This ves- ACoA is especially hazardous since ischemia or sel arises from the most proximal section of vasospasm in small branches of this artery may the basilar artery. It supplies the lateral and produce akinetic mutism. lower two-thirds of the pons, ventrolateral

cerebellum and travels through the cerebel- The anterior communicating artery is just lopontine angle. 1.5 mm in length. (c) Internal auditory artery. This artery emerges either from the AICA or directly The middle cerebral artery irrigates the lateral from the basilar artery. It supplies the ves- surface of the cerebral hemispheres which include tibular and cochlear apparatus. frontal, parietal, and temporal cortex along with (d) Superior cerebellar artery. This artery the insula. The proximal portion (M1 segment) originates from the distal basilar artery gives rise to the lenticulostriate vessels which sup- supplying the upper pons, midbrain teg- ply the corona radiata, external capsule, putamen, mentum, superior, and lateral cerebellum. portions of the globus pallidus, caudate, and most (e) Posterior cerebral artery. This vessel begins of the anterior and posterior internal capsules. at the tip of the basilar artery. It supplies the midbrain, thalamus, lateral geniculate The middle cerebral artery irrigates the lateral nucleus, inferior temporal lobe, parieto- surface of the cerebral hemispheres which occipital region, and occipital lobe. include frontal, parietal, and temporal cortex (f) Anterior spinal artery. It is formed by an including the insula. anastomosis of branches of both vertebral arteries. It irrigates the ventral medulla There may be a bifurcation or trifurcation of and anterior two-thirds of the spinal cord the middle cerebral artery stem. The bifurcation down to the cauda equina. Ischemic Stroke (Cerebral Infarction) and Transient Ischemic Attack 149

2. Major Branches of the Internal and External • Ophthalmic artery. This is the fi rst Carotid Arteries (Fig. 7.6 ). major branch of the internal carotid (a) External carotid artery begins at the com- artery in its supraclinoid portion. Its mon carotid bifurcation. Its branches are: branches are the posterior ciliary arter- • Superfi cial temporal artery. This vessel ies which supply the optic nerve and is palpable, located in front of the ear, the central retinal artery which irrigates and easily biopsied when temporal the retina. The vascular supply to the arteritis is suspected. eye warrants special attention (see • Internal maxillary artery. It supplies the Fig. 7.7 ). The ophthalmic artery sends muscles of mastication. perforating branches to the intraorbital • Middle meningeal artery. This vessel portion of the optic nerve. The short arises from the internal maxillary artery, ciliary arteries provide irrigation to the supplies the dura mater and, especially, arteriole anastomotic circle of Zinn- bone. Haller which supplies the optic disk. (b) Internal carotid artery begins at the com- The single central retinal artery sup- mon carotid bifurcation. Its branches are: plies the entire retina. Thus an embolus

Fig. 7.6 Internal and external carotid system, lateral angiographic view. With permission of Dr. Eric Bershad 150 7 Cerebrovascular Anatomy with Clinical Correlation

Circle of Zinn-Haller

Sclera Recurrent branches from circle of Zinn-Haller

Collateral branches Posterior ciliary arteries

Collateral branches

Central retinal artery

Ophthalmic artery

Optic canal

Optic nerve

Internal carotid artery

Fig. 7.7 Vascular supply of the . The ophthalmic retina and the posterior ciliary arteries which irrigate the artery, the fi rst major branch of the internal carotid artery, choroid and the optic disk by means of an anastomotic supplies the globe. It provides collateral branches to the channel of arterioles (the circle of Zinn-Haller). Adapted optic nerve. Its two major branches are the central retinal from: Digre KB, Corbett JJ. Practical viewing of the optic artery destined to supply only the inner surface of the disc. Burlington: Butterworth Heinemann; 2003. p. 272

to the central retinal artery causes (c) Anterior cerebral artery. This large artery blindness or severe visual loss but an arises from the internal carotid artery embolus to one posterior ciliary artery bifurcation. It is a midline vessel which may be asymptomatic because of the supplies the medial surface of the cerebral rich vascular anastomoses. Anterior hemispheres, corpus callosum, and the ischemic optic neuropathy, both arter- upper 2 cm of the frontal and parietal con- itic (temporal arteritis) or nonarteritic vexities. Other branches supply the optic involve the posterior ciliary arteries. chiasm, optic tract, and hypothalamus. • Posterior communicating artery. This • Anterior communicating artery. This second offshoot anastomoses with the artery averages 1.5 mm in length and PCA connecting the anterior and poste- connects both anterior cerebral arteries. rior circulation. It supplies the chiasm, septal region, • Anterior choroidal artery. This is the hypothalamus, and anterior parts of the third branch off the internal carotid fornix and corpus callosum. artery and is the second most important (d) Middle cerebral artery. This large artery vascular supply of the internal capsule. arises from the internal carotid artery It also supplies the optic tract, lateral bifurcation. It irrigates the lateral surface geniculate nucleus, optic radiations, of the cerebral hemispheres including and hippocampus. frontal, parietal, and temporal cortex as Ischemic Stroke (Cerebral Infarction) and Transient Ischemic Attack 151

well as the insula. Lenticulostriate branches Smoking is a powerful risk factor for internal from the horizontal M1 segment supply carotid artery stenosis. most of the anterior and posterior limbs of the internal capsule and basal ganglia. 6. Extracranial internal carotid artery stenosis Case 2 A 77-year-old woman reports an episode due to atherosclerosis accounts for approxi- of loss of vision of the right eye lasting 3 min. mately 10% of cerebral infarctions. The mech- She describes a black curtain descending over the anism is primarily artery-to-artery embolism. eye. Her past medical history includes severe Hemodynamic factors, impaired perfusion in chronic obstructive pulmonary disease (COPD) and the distribution of the severely stenotic vessel a 60-pack year smoking habit. Medications are (greater than 90%), are likely responsible for an Albuterol inhaler and prednisone 5 mg q.d. less than 5% of ischemic strokes due to Neurologic examination is normal other than extracranial internal carotid stenoses. Although bilateral carotid bruits heard only over the high this may seem surprising, internal carotid cervical region just under the mandible. artery occlusions often occur without produc- ing any neurologic symptoms. Good collateral Diagnosis : circulation occurs via the circle of Willis or Amaurosis fugax secondary to embolism from a external carotid-intracranial arterial anastomo- right internal carotid artery stenosis/plaque. ses which develop as a consequence of slowly Preliminary studies : progressive stenosis and eventual occlusion. MRI (head) shows multiple T2 hyperintensities in subcortical white matter. Carotid Doppler stud- Extracranial carotid artery stenosis accounts ies disclose 50–79% stenoses of both internal for about 10% of cerebral infarctions which carotid arteries. Cerebral angiography reveals a are primarily due to artery-to-artery severe right internal carotid artery stenosis (90%) embolism. with ulcerated plaque in the right internal carotid artery 0.5 cm above the bifurcation. Symptomatic carotid stenoses produce TIAs The patient is successfully treated by a carotid or ischemic infarctions most often in the distribu- stent since she is a high-risk surgical candidate tion of the major middle cerebral artery branches. because of severe COPD. Small subcortical infarctions, however, are well- Clinical points: documented occurrences. Consequently, treat- 1. Amaurosis fugax is commonly due to platelet/ ment selection will depend on the clinician’s fi brin emboli arising from the irregular surface judgment as to whether an infarction is due to of an ulcerated atherosclerotic plaque located small vessel intracranial disease requiring medi- near or at the common carotid bifurcation. cal management or extracranial stenosis for 2. The platelet/fi brin embolus passes through the which surgery or stent is commonly employed. ophthalmic artery, then the central retinal This dilemma arises most often when the extracra- artery where it has been observed in retinal nial internal carotid artery stenosis is of border- arterioles by ophthalmoscopy. It has a grayish line signifi cance (60–70%) and without evidence coloration. of an ulcerated plaque. 3. Smoking is a powerful risk factor for internal carotid artery stenosis. Hypertension and dia- Symptomatic carotid stenoses require stenting betes are additional major risk factors. or endarterectomy. 4. Carotid Doppler studies were unnecessary since, irrespective of the result, angiography Asymptomatic stenosis treatment decisions was indicated. require an understanding of the natural history of 5. Subsequent medical treatment is antiplatelet this lesion. To the uninformed physician a severe therapy since emboli are of platelet/fi brin internal carotid artery stenosis indicates a high composition. incidence of sudden catastrophic ischemic stroke 152 7 Cerebrovascular Anatomy with Clinical Correlation without warning. Careful investigations, however, 2. Ataxia is due to disruption of cerebellar have established that TIAs precede ischemic pathways in the brainstem. stroke due to extracranial internal carotid artery 3. The presence of both weakness and Babinski stenosis about 95–97% of the time. Thus, an alert, signs indicates involvement of the corticospi- reasonably intelligent patient, well-educated about nal tract. the usual symptoms of a TIA can be followed by 4. Although this patient has a pontine lacunar careful observation awaiting a TIA before decid- infarction, other possible locations of pathol- ing on treatment. Another option is to repeat non- ogy which may cause ataxic hemiparesis invasive testing such as serial carotid Doppler include the posterior limb of the internal cap- studies to observe for asymptomatic progression sule and the midbrain in the region of the red of a carotid stenosis before deciding on surgical nucleus. treatment or stenting. Conversely, if the morbidity 5. Ataxia is not a pathognomonic sign of cere- of the treatment by stent or endarterectomy is less bellar hemisphere disease. Neither weakness than 3%, aggressive treatment may be warranted. nor Babinski signs occur with cerebellar At the date of this writing endarterectomy is the pathology. preferable treatment if the patient is a good surgi- Lacunar infarctions range in size from 0.2 to cal candidate. Morbidity varies from hospital to 15 mm. They are primarily located in the basal hospital and between surgeons or those physicians ganglia, thalamus, internal capsule, pons, and who perform stenting. Publication of results of rarely in cortical or subcortical white matter such treatment by individual vascular surgeons or neu- as the centrum semiovale or corona radiata. Most rointerventionists, which must include a neurolo- of them are in the distribution of the lenticulostri- gist’s participation, is an ideal scenario but ate vessels, thalamoperforating branches off the unlikely to occur for obvious reasons. PCA, and paramedian branches of the basilar artery. The most common pathology is micro-

Transient ischemic attacks precede ischemic atheroma or lipohyalinosis, the latter a subintimal stroke due to extracranial internal carotid collection of hyaline material which stains posi- artery stenosis about 95–97% of the time. tive for fat.

Lacunar infarctions, 0.2 to 15 mm, are primar- Case 3 A 55-year-old man arrives in the ily located in basal ganglia, thalamus, internal Emergency Room at noon. On arising that morning capsule, and pons. he was unsteady and noted right leg weakness. He has a 5-year history of well-controlled hyper- tension. His only medication is lisinopril. The classical syndromes of lacunar infarctions include pure motor stroke, pure sensory stroke, Neurologic examination : hemiparesis with homolateral ataxia (ataxic Blood pressure is 150/100 and the pulse is regu- hemiparesis), clumsy hand-dysarthria syndrome, lar, 72 beats per minute. Abnormal fi ndings are a sensorimotor stroke, and basilar artery branch right arm , mild distal weakness of syndromes (see Table 7.1 ). right arm and right leg. He has a moderate right hemiataxia and right Babinski sign. Case 4 A 62-year-old woman is brought to the Emergency Room because of impaired vision. Diagnosis : While singing in the shower she forgot the words Left pons lacunar infarction disclosed by an MRI to a favorite song. Her husband heard her misuse with diffusion-weighted imaging. and mispronounce words. Simultaneously she Clinical points: noted a bright glare and near total loss of vision. 1. The patient manifests one of the characteristic Her speech returned to normal in 1 min. Vision syndromes of lacunar infarction, ataxic slowly improved but when looking straight ahead hemiparesis. she could not see the fl oor. Ischemic Stroke (Cerebral Infarction) and Transient Ischemic Attack 153

Table 7.1 Lacunar syndromes Name Location Pure motor stroke Lesions of corona radiata, posterior limb of internal capsule, cerebral peduncle, pons, and medulla. Internal capsule lesions are most common Ataxic hemiparesis Lesions in corona radiata, pons, and posterior limb of internal capsule Clumsy hand-dysarthria syndrome Pons and anterior limb of internal capsule Pure sensory stroke Thalamus and brainstem Sensorimotor stroke Thalamic nuclei (VPL/VPM) and adjacent internal capsule VPL ventral posterolateral nucleus which receives sensory information from the contralateral arm, trunk, and leg; VPM ventral posteromedial nucleus which receives sensory information from the contralateral face

Neurologic examination is normal except for vertebral arteries in the neck. Carotid Doppler an inferior homonymous altitudinal hemianopsia. studies are of no value.

Questions: 1. What does the history indicate? When symptoms or signs indicate verte- 2. Is the examination helpful? brobasilar disease, an MRA is indicated, not 3. What type of pathology occurs abruptly and carotid Dopplers. often resolves over a short time? 4. Is there a single artery that supplies both Test results : occipital cortices? The MRI shows bilateral occipital lobe infarc- 5. What laboratory studies visualize this artery? tions involving the upper banks of the calcarine fi ssure. MRA (brain and neck) are normal. A tran- Analysis: sthoracic echocardiogram is normal. A 24-h 1. Misuse and mispronunciation of words are Holter monitor discloses several prolonged bursts symptoms of aphasia, not dysarthria, and of atrial fi brillation. nearly always indicate left cerebral hemi- sphere dysfunction. Complete loss of vision, Diagnosis : particularly with brightness or “glare” is a Bilateral occipital lobe infarctions secondary to common symptom of occipital cortex isch- cardioembolism in the basilar-posterior cerebral emia or dysfunction. Darkness implies retinal artery distribution associated with paroxysmal hypoperfusion. atrial fi brillation. Treatment is initiated with war- farin and Cardiology is consulted. Brightness or “glare” is a common symp- Cardioembolism causes about 20–25% of tom of occipital cortex dysfunction. cerebral infarctions. As previously noted, about 70% are in the carotid artery distribution and 2. The abnormal visual fi elds are most character- 23% in the vertebrobasilar system. The remain- istic of bilateral, superior occipital cortex der are in watershed zones or multiple regions. ischemia or infarction. Theoretically lesions of the optic chiasm could be considered but Cardioembolism causes about 20–25% of this would be exceedingly rare. cerebral infarctions. 3. Vascular disease. 4. Yes, the basilar artery. This artery usually The most common etiology of cardioembo- gives rise to both posterior cerebral arteries at lism is atrial fi brillation, perhaps 45%. Other its tip, the distal termination. What explains common etiologies are acute myocardial infarc- the aphasia? Ischemia in the distribution of the tion with mural thrombus, ventricular aneurysm, inferotemporal branch of the left PCA. valvular heart disease, and prosthetic valves. 5. An MRA (magnetic resonance angiography) There has been a keen interest in paradoxical will demonstrate the basilar artery and the embolism via a patent foramen ovale (PFO). 154 7 Cerebrovascular Anatomy with Clinical Correlation

The importance remains controversial. The Answers: simultaneous presence of an atrial septal aneurysm, 1. Yes. however, does increase the risk of an embolic 2. Blood pressure should be taken in both arms. event. Other uncommon etiologies include sick Both radial pulses should be palpated sinus syndrome, cardiomyopathy, atrial myxoma, simultaneously. marantic endocarditis, Libman-Sacks endocarditis 3. Blood pressures were taken in the left arm. (associated with systemic lupus erythematosus), Blood pressure in the right arm is 150/90. The and bacterial endocarditis. patient’s blood pressure was considered labile It is important to note that perioperative stroke because it was taken in either right or left arm is nearly always cardioembolic in origin. It man- by the nurse without documentation of which dates a careful, thorough cardiac assessment side was taken. There is a pulse delay on the including transesophageal echocardiogram and left side when both pulses are palpated Holter monitor. Atrial fi brillation is the most simultaneously. common etiology. 4. The diagnosis is subclavian steal syndrome. Cerebral angiography reveals an occlusion Perioperative stroke is nearly always cardio- proximal to the origin of the dominant left embolic and most often due to atrial vertebral artery, much larger than the right, a fi brillation. common anatomic variation. When the left arm is physically active or when the blood Case 5 A 68-year-old man complains of a 1-year pressure drops, there is retrograde blood fl ow history of frequent dizziness when standing up. down the left vertebral artery to supply the It is a vague, lightheaded sensation sometimes needs of the left arm. This results in brainstem accompanied by oblique double vision and facial ischemia which, in this case, is manifested by tingling. diplopia and facial paresthesias. Past medical history includes a two-pack per 5. Zero. Disabling TIAs are common. day smoking habit for 50 years and labile hyper- 6. Stenting can be performed for a stenosis, sur- tension for 5 years. He takes no medication. gery for a subclavian occlusion. The surgical Neurologic examination : procedure is a left carotid-to-subclavian Blood pressure 95/60 sitting, 90/60 standing after bypass, distal to the vertebral artery origin. If 1 min, 88/58 standing at 2 min and 94/60 at 3 min. a patient is asymptomatic treatment is not The pulse varies from 80 to 84 beats per minute. required. He is asymptomatic. There are loud bruits over Case 6 A 48-year-old man is referred by an all neck vessels, no cardiac murmur, and the neu- ophthalmologist because of blurred vision O.D. rologic examination is normal. of presumed vascular origin. The patient reports Questions: an abrupt onset of painless loss of vision affect- 1. Can the diagnosis be made without ing his lower fi eld of vision 3 days ago. He has no neuroimaging? illnesses and takes no medicine. 2. What part of the medical examination should Neurologic examination reveals a blood pres- be reassessed in more detail? sure of 150/110. The funduscopic examination 3. What could it show? discloses a swollen, pale optic disk with two 4. If confi rmed, what is the diagnosis and the risk fl ame-shaped hemorrhages at the disk margin. of stroke? The retinal vasculature is normal. He has an inf- 5. What is the risk of stroke? eronasal quadrant fi eld loss and an afferent pupil- 6. What is the treatment? lary defect. Venous Sinus Disease 155

Questions: The superior sagittal sinus drains the cerebral 1. Where is the lesion? cortex and also the scalp by way of the emissary 2. What vascular supply is compromised? veins which pass through the skull. It contains 3. What is the etiology? most of the arachnoid granulations which absorb 4. What is the differential diagnosis? the cerebrospinal fl uid. Hence papilledema is a relatively common presenting sign of superior Answers: sagittal sinus thrombosis along with headache, 1. Unilateral quadrant visual fi eld loss and an altered mental status, hemiparesis, and hemisen- afferent pupillary defect indicate optic nerve sory syndromes. A classical but uncommon disease. The funduscopic appearance of pallid presentation is alternating hemiparesis. Seizures, edema with hemorrhages at the disk margin grand mal or partial, are frequent because infarc- and the abrupt onset indicate an ischemic tions are typically hemorrhagic. Hemorrhage is a event involving the optic nerve. Papilledema cortical irritant. Multiple hemorrhagic infarctions is bilateral and generally has no major effect point strongly to a venous sinus thrombosis. on vision other than enlarged blind spots.

2. Multiple posterior ciliary arteries which arise from the ophthalmic arteries supply the optic Superior sagittal sinus thrombosis frequently nerve. A central retinal artery occlusion may causes hemorrhagic infarctions which are cause disk pallor, narrowed arterioles, and epileptogenic. segmented blood fl ow called boxcarring. 3. The etiology of this patient’s visual loss is The lateral (transverse) sinus drains the brain- posterior ciliary artery disease associated with stem, cerebellum and posterior part of the cere- untreated hypertension. bral hemisphere. The sigmoid section is adjacent 4. The diagnosis is anterior ischemic optic neu- to the mastoid process. Therefore, mastoiditis ropathy, nonarteritic (Arnold 2003 ) . The major and otitis media may produce a septic thrombosis differential diagnosis is temporal arteritis of this sinus and papilledema with its attendant which involves the same vessels. Ordinarily manifestations of headache and transient visual there is associated headache and an increased obscurations, unilateral or bilateral, which often sedimentation rate with arteritis. last just seconds. Horizontal diplopia, an occa- sional development, is most often due to a 6th Ischemic stroke due to intracranial small ves- nerve palsy, a nonspecifi c feature associated with sel disease, extracranial vascular disease, and increased intracranial pressure. Thus all of the cardioembolism has been briefl y reviewed. There features of pseudotumor cerebri are produced. are numerous additional causes of ischemic stroke. In Chap. 11, Diagnostic Dilemmas, other Lateral sinus thrombosis is often septic and etiologies will be discussed. due to mastoiditis or otitis media.

The cavernous sinuses are located just lateral Venous Sinus Disease to the pituitary fossa (see Fig. 7.8 ). They drain the orbit and the base of the anterior parts of the Venous sinus disease is characterized by protean brain. Its contents include the internal carotid clinical manifestations (Biousse and Bousser artery in its medial portion. Adjacent and lateral 1999 ) . There is often a baffl ing array of signs and to it is the 6th nerve. Along the lateral wall, symptoms. Thus the diagnosis depends on an superior to inferior, are the 3rd nerve, 4th nerve, alert physician who is aware of the predisposing ophthalmic, and maxillary divisions of the illnesses. The three sinuses of greatest clinical trigeminal nerve. A cavernous sinus thrombosis importance are the superior sagittal sinus, lateral produces unique clinical signs and is therefore or transverse sinus, and the cavernous sinus. more easily recognized. There is usually a unilateral 156 7 Cerebrovascular Anatomy with Clinical Correlation

Fig. 7.8 Internal structure of the cavernous sinus

Table 7.2 Etiology of venous sinus disease Disease category Diseases Sepsis Otitis media; mastoiditis; facial, scalp, and dental infections; sinusitis; bacterial endocarditis; meningitis, or other intracranial infections Coagulopathies Factor V Leiden, proteins C and S defi ciencies, disseminated intravascular coagulation, lupus anticoagulant, and heparin or heparinoid-induced thrombocytopenia Other hematologic disorders Polycythemia, sickle cell anemia, paroxysmal nocturnal hemoglobinuria, and thrombocythemia Neoplasm Meningiomas, metastases, any visceral carcinoma, lymphoma, leukemia, carcinoid, and glomus jugulare tumors Gynecologic-obstetric Oral contraceptives, puerperium, and pregnancy Connective tissue diseases Systemic lupus erythematosus, temporal arteritis, and Wegener’s granulomatosus Trauma Neurosurgical procedures, head injury with or without fracture, and jugular vein catheters Medical illnesses Cardiac Congestive heart failure and congenital heart disease Gastrointestinal Cirrhosis, Crohn’s disease, and ulcerative colitis Medicines Androgen, l -asparaginase Miscellaneous Dehydration of any etiology, Behcet’s disease, nephrotic syndrome, and sarcoidosis ophthalmoplegia with variable involvement of The most important etiologic considerations the above-noted cranial nerves. Horner’s syn- of venous sinus disease are noted in the adjacent drome may occur as the sympathetic fi bers travel table (Table 7.2 ). through the sheath of the internal carotid artery. Unilateral or bilateral proptosis and chemosis are frequent concomitant signs. Intracerebral Hemorrhage

Cranial nerves 3rd, 4th, 5th-1 and 2, 6th, and About 15% of all strokes are due to intracranial sympathetic fi bers pass through the cavernous hemorrhage. Intracerebral hematomas account for sinus. a little more than half of this number. The typical Intracerebral Hemorrhage 157 locations (frequencies) are putamen (40%), lobar Amyloid angiopathy comprises about 20% (22%), thalamus (15%), pons (8%), cerebellum of intracerebral hematomas in patients over (8%), and caudate (8%). age 70. The most common risk factor is hypertension which underlies 70–80% of hemorrhagic strokes. Complications of intracerebral hematomas Additional, additive risk factors are age, low include mass effect with a herniation syndrome, cholesterol level (less than 160 mg/dL), heavy intraventricular rupture, and hydrocephalus. alcohol use, smoking, and liver disease with Seizures occur primarily with lobar hematomas. thrombocytopenia. Pregnancy in the third trimes- Because of extension of the hematoma or associ- ter is a risk factor for hemorrhage but ordinarily ated edema into adjacent structures a neat dis- this is associated with eclampsia and rupture tinction into well-defi ned clinical syndromes is of either an arteriovenous malformation (AVM) not possible. There are, however, some useful or aneurysm. diagnostic patterns (see Table 7.3 ).

Hypertension is the predominant factor in Case 7 A 57-year-old man is seen in a neurology 70–80% of intracerebral hematomas. clinic 2 days after suffering a severe occipital headache followed by feeling dizzy. He denies vertigo but says he was unsteady. The symptoms Chronic hypertension leads to fi brinoid necro- occurred toward the end of losing a tennis match. sis with weakening of arterial walls and then He has had similar symptoms in the past, mainly aneurysmal development known as Charcot- after a loss at tennis but this time his dizziness Bouchard aneurysms. These are found in lenticu- was more prolonged as it lasted the entire day. lostriate vessels originating from both anterior His headache resolved by the next morning. His and middle cerebral arteries, thalamogeniculate past medical history is normal other than hyper- branches off the posterior cerebral arteries, thal- tension controlled with ramipril. amoperforating vessels arising from both poste- Neurologic examination reveals a blood pres- rior communicating and posterior cerebral sure of 150/100, moderate right heel-to-shin arteries, and basilar perforating arteries. ataxia and an unsteady tandem gait. There are no Nonhypertensive etiologies include aneu- other abnormal fi ndings. rysms which may rupture into cerebral paren- chyma as well as the subarachnoid space, AVMs, Scenario 1 : amyloid angiopathy, coagulopathies, hemorrhage A stat noncontrast CAT scan shows a 2 cm right into an acute infarction, hemorrhage into malig- cerebellar hemisphere hematoma. Routine blood nant neoplasms, rarely cavernous angiomas, and chemistries, CBC with platelet count, PT, and PTT drugs. Warfarin and TPA are common offending are normal. Because the hematoma is less than 3 cm medicines. Amphetamines and over-the-counter and the abnormal neurologic fi ndings are minimal, medicines such as phenylpropanolamine and conservative management is selected. The patient pseudoephedrine can be the culprits. is initially monitored in the Neurology ICU for Amyloid angiopathy is a relatively common 2 days. Hypertension is treated, serial neurologic etiology of hemorrhage in the elderly (Rosand examination discloses resolution of the ataxia after and Greenberg 2000 ). It comprises about 20% of 1 week. During this time two additional CAT scans intracerebral hematomas in patients over age 70. of the brain show beginning resolution of the hema- The hematomas are generally lobar and often toma. The patient is discharged at 1 week. recur in different locations. Infrequently, they may be multiple at fi rst diagnosis. The differen- Cerebellar hematomas, when less than 3 cm, tial diagnosis then includes venous sinus throm- often resolve spontaneously and do not require bosis, hemorrhagic metastases, and leukemia. surgery. 158 7 Cerebrovascular Anatomy with Clinical Correlation

Table 7.3 Variable presenting manifestations of intracerebral hematomas Location Clinical presentations Putamen (40%) Aphasia (D), neglect-denial (ND), eye deviation, homonymous hemianopsia, apraxia, and hemiparesis Lobar (22%) Frontal – abulia, aphasia (D), hemiparesis, denial or neglect (ND), and eye deviation Parietal – aphasia (D), neglect-denial (ND), hemiparesis, hemisensory loss, apraxia, eye deviation, and homonymous hemianopsia Temporal – aphasia (D), neglect-denial (ND), and homonymous hemianopsia Occipital – homonymous hemianopsia Thalamus (15%) Hemisensory loss greater than hemiparesis, convergence-retractory nystagmus, downward eye deviation, paresis of upgaze, “wrong-way” eye deviation or gaze palsy, skew deviation, and light-near dissociation Pons (8%) Hemiparesis, quadriparesis, gaze palsy (unilateral or bilateral), decerebrate rigidity, locked-in- syndrome, pinpoint pupils, ocular bobbing or dipping, apneustic breathing, 1–1/2 syndrome (unilateral involvement of MLF and PPRF), ipsilateral or contralateral ataxia with contralateral hemiparesis, and hemisensory loss Cerebellum (8%) Ipsilateral gaze preference with gaze paretic nystagmus, ipsilateral ataxia, and is uncommon since hemorrhage in the vermis of the cerebellum is rare Caudate (8%) Confusional state, transient gaze paresis, hemiparesis, hemisensory loss, and ipsilateral Horner’s syndrome D dominant hemisphere; ND non-dominant hemisphere

Scenario 2 : Lesson No. 3 : CAT scan and laboratory data are as described Hemorrhages usually occur during physical above. Two days after admission the patient com- activity. One-third are maximum at onset and plains of dizziness. Examination shows paresis of two-thirds by 30 min. About one-third progress upgaze and the pupils enlarge from 3 to 5 mm. afterwards. The pupillary reactions are now 1+/4. The patient Lesson No. 4 : remains alert, oriented and has a normal screen- Large lesions of the cerebellum often produce ing mental status examination. An MRI discloses few abnormal signs. an enlarged hematoma with rostral displacement of the tip (iter) of the Sylvian aqueduct and the pons. Neurosurgical consultation is obtained. Large lesions of the cerebellum often produce Immediately prior to surgery the patient is diffi - few abnormal signs. cult to rouse. The hematoma is evacuated and postoperatively the patient is alert, pupils are Scenario 2 analysis : 4 mm, equal and with a 3+/4 reaction to light. Upgaze is normal. The patient is able to perform Lesson No. 1 : tandem gait 3 days after surgery. Serial neurologic examinations which focus on the function of adjacent anatomic structures pre- Scenario 1 analysis : dict subsequent neurologic developments. Lesson No. 1 : Right heel-to-shin ataxia is a discrete focal fi nd- An enlarging cerebellar hematoma usually ing supporting the presence of a localized lesion. requires neurosurgical intervention Only a complete neurologic examination would elicit this sign. Lesson No. 2 : Lesson No. 2 : Upgaze paresis indicates posterior commissure Cerebellar system lesions usually cause ipsilat- compression and the large sluggishly reacting eral abnormal signs. pupils point to pretectal involvement. Subarachnoid Hemorrhage 159

Lesson No. 3: 6. What may explain the episode of loss of Lethargy implies dysfunction of the mesenceph- coordination of the right hand for 2 min? alic reticular formation. These fi ndings occur 7. Does the pupillary abnormality have diagnostic with transtentorial upward herniation through the signifi cance? tentorial notch. 8. Does a normal CAT scan (head) rule out hemorrhage? Lesson No. 4 : Prompt evacuation of the hematoma and a large Answers: amount of the cerebellar parenchyma does not 1. Unilateral headache, common in migraine, interfere with a good recovery. does not generally occur with SAH. Nausea and vomiting, photophobia, and phonophobia occur with both migraine and SAH. Absence Subarachnoid Hemorrhage of a positive family history for migraine is usual in patients with SAH. 2. Low-grade fever is common with SAH, but Case 8 A 36-year-old woman arrives in the not migraine. A positive Brudzinski sign indi- Emergency Room because of a severe bilateral cates meningeal irritation. pounding headache of 12 h duration. Associated 3. In optic nerve disease the pupils are equal in symptoms include nausea, vomiting, photophobia, size but there is a decreased response to light and phonophobia. She has had two similar head- on the involved side. In this case the pupils are aches in the past 3 years prompting Emergency equal in size but have a different reactivity to Room visits. Standard treatment for migraine on light. The direct and consensual responses are those occasions, ketorolac, and prochlorperazine the same O.D. and hence the afferent loop I.V., terminated the headaches in 1 h. O.S. is intact. The direct and consensual The patient adds an additional history of tran- response O.S. is the same thus indicating a sient loss of coordination of the right hand for defect in pupillary constriction or a partial 3rd 2–3 min about 2 months ago. nerve lesion on the left side. Her past medical history is negative. There is 4. SAH or meningitis. Carcinomatous meningi- no history of migraine in the family. tis seldom begins acutely. General physical and neurologic examinations 5. The prior headaches may have been “sentinel” reveal a temperature of 100°F, blood pressure headaches, minor leaks from an aneurysm 140/100, slight end nuchal rigidity, and positive which may precede SAH. Brudzinski’s sign (see section on neurologic 6. This is likely to have been a TIA which may examination for explanation). Pupils are 5 mm, occur in patients who have an aneurysm equal and with 3+/4 reaction to light O.D. and because of embolism from a clot within the 2+/4 O.S. Consensual responses are identical. aneurysm. CAT scan (head) is normal. 7. Yes. A 3rd nerve lesion suggests a posterior Questions: communicating aneurysm which frequently 1. What common feature of migraine headache compresses this nerve. seldom occurs with SAH? 8. No. CAT scans are normal in 5–10% of 2. What physical signs do not occur with patients with SAH. migraine? The next step is a lumbar puncture.

3. The pupils show asymmetric reactivity to The opening pressure is high at 230 mm H2 O, light. Is this due to a 3rd nerve or optic nerve as the normal is equal to or less than 200 mm

lesion? H2 O. The fl uid is bloody and, after centrifuging, 4. What is the differential diagnosis of acute the supernatant is yellow (xanthochromic) indi- headache, fever, and meningeal irritation? cating prior hemorrhage. Xanthochromia is a 5. What is the importance of the two prior result of bilirubin which becomes apparent about headaches? 10–12 h after hemorrhage and may persist for 160 7 Cerebrovascular Anatomy with Clinical Correlation

2–4 weeks. Testing for xanthochromia is the most no injury. She has occasionally been incontinent reliable test for detecting hemorrhage. of urine without awareness until she noted that Cerebrospinal fl uid obtained within just a few she was wet. She complains of forgetfulness. hours is typically pink due to oxyhemoglobin Neurologic examination : but the supernatant is clear. The RBC count is The patient exhibits bradyphrenia (slowed think- 10,000 and wbc count is 1,000, or a 10 to 1 ratio. ing) and makes errors with short-term recall. Peripheral blood has a ratio of 700 to 1. Thus When seated on the examining table she leans there is an increased wbc count, characteristic of backward and requires support. She walks with a an infl ammatory response to hemorrhage. Coun- short stride, takes a few shuffl ing steps when ting red blood cells in the fi rst and fourth tubes turning, and has a positive pull test. can be useful but is much less reliable than the other methods just described. Analysis : Truncal instability usually implies dysfunction in Followup examination : centrally located regions such as that which occurs Funduscopic examination is repeated on the night with Parkinson’s disease, lesions of the cerebellar of admission and shows a subhyaloid hemorrhage vermis, hydrocephalus, and a mass lesion with mid- O.S (Fig. 4.8). The hemorrhage has a half-moon line shift. Her gait is apraxic and she has a history of appearance with a horizontal fl at side located urinary incontinence. Hence, she manifests the syn- superiorly. There should be a careful search for drome of normal pressure hydrocephalus (NPH), this fi nding which is characteristic of SAH. which is gait apraxia, memory loss, and urinary Angiography : incontinence. She has communicating hydrocepha- The patient has an 8 mm left posterior communi- lus due to obstruction of cerebrospinal fl uid fl ow cating aneurysm. Treatment is surgical clipping around the convexities, a relatively common sequela or coiling. The latter method is selected and is of SAH and meningitis. Adhe sions in the subarach- successful. noid space or surrounding the arachnoid villae may be the etiology. These are two well-known etiologies Hospital course : of the NPH syndrome which usually begins with a Four days after admission and 3 days after the gait apraxia. In this case a ventriculo-peritoneal aneurysm is coiled the patient is noted to misuse shunt returns the patient to normal. words. She is heard to say “ted” instead of “bed,” SAH, due to ruptured cerebral aneurysms, “book” instead of “television”; thus she exhibits accounts for 6–8% of all strokes (Suarez et al. phonemic and semantic paraphasias, respectively. 2006) . Thirty-four percent arise from the ACoA, Vasospasm involving the left middle cerebral artery 30% from the internal carotid-posterior commu- is suspected and proven by transcranial Doppler nicating arteries, 22% from the middle cerebral study. The patient is treated by hypervolemia and artery, and about 8% from the vertebrobasilar induced hypertension. The following day language system. Multiple aneurysms have been found in function returns to normal. The patient is dis- 20–30% of patients with SAH. The remainders of charged on the eighth hospital day feeling well. SAHs are due to coagulopathies such as throm-

bocytopenia, meningeal neoplasms, amyloid Vasospasm, which causes focal neurologic angiopathy, and use of sympathomimetic drugs signs, and communicating hydrocephalus are such as methamphetamines and cocaine. common complications of subarachnoid hemorrhage. Subarachnoid hemorrhage due to ruptured cerebral aneurysms accounts for 6–8% of all Outpatient visit 1 month later : strokes. The patient returns with a complaint of a tendency to fall backward. Two days prior to this visit she The location of the aneurysm can sometimes be fell off a kitchen stool onto her back but suffered predicted by the neurologic examination. Clinical Subarachnoid Hemorrhage 161

Table 7.4 Clinical correlations Table 7.5 Hunt and Hess clinical grading scale Location of aneurysm Manifestations Group Condition Anterior communicating Abulia and leg weakness 0 Unruptured aneurysm aneurysms 1 Asymptomatic or minimal headache and slight Left middle cerebral artery Aphasia and right nuchal rigidity aneurysm hemiparesis 2 Moderate or severe headache, nuchal rigidity Right middle cerebral artery Anosognosia/neglect and and no neurologic defi cit other than cranial aneurysm left hemiparesis nerve palsy Posterior communicating- 3rd nerve palsy 3 Drowsiness, confusion, or mild focal defi cit internal carotid artery 4 Stupor, moderate-to-severe hemiparesis aneurysm 5 Deep coma, decerebrate posturing, and Basilar artery aneurysm Midbrain signs, much moribund appearance variability. correlation (Mohr 2004 ) is noted in Table 7.4 . A 4. Annual rupture rates are 0.5–2% depending standard grading system of the severity of the hem- on aneurysm size. Aneurysms, 10 mm or orrhage is the Hunt and Hess Scale (Table 7.5 ). larger, rupture at a higher rate. Complications: 1. Rebleeding (7%) occurs most often in the fi rst Annual rupture rates are 0.5–2% depending 2 weeks. on aneurysm size. 2. Vasospasm (46%) is associated with delayed narrowing of large arteries usually between Genetic factors: days 4 and 12 and is often symptomatic. 1. Autosomal dominant polycystic kidney disease. 3. Hydrocephalus (20%) may occur any time Five to ten percent of patients have aneurysms after SAH. on screening studies. 4. Seizures (25%). 2. Marfan’s syndrome. 5. Increased intracranial pressure with or with- 3. Ehler-Danlos syndrome. out 6th nerve palsy. 4. Neurofi bromatosis type I. 6. Hyponatremia (28%). 5. Seven to twenty percent of patients with aneu- (a) Syndrome of inappropriate ADH secretion rysmal SAH have a fi rst or second degree rela- (SIADH) which is associated with high tive with a confi rmed aneurysm. intravascular volume. 6. Fibromuscular dysplasia. (b) Cerebral salt wasting which is associated Environmental predisposing factors: with low intravascular volume. 1. Smoking. This decreases the effectiveness of 7. Cardiac arrhythmias (35%). alpha-1 antitrypsin which inhibits proteases 8. Pulmonary edema (23%). This may be neuro- such as elastase. genic or cardiogenic. 2. Hypertension. Epidemiology: 3. Hormones. Lack of hormone replacement in 1. Aneurysms are present in 1–6% in large post-menopausal women. autopsy studies in adults (Schievink 1997 ) . 4. Heavy ethanol consumption, especially binge drinking. Aneurysms are present in 1–6% in large Case 9 A 24-year-old man requests an evalua- autopsy studies in adults. tion for acute, severe, unilateral, and pounding headaches associated with nausea and photophobia. 2. Aneurysms are incidentally discovered in 0.5– The last one occurred yesterday. He has had several 1% among adults undergoing angiography. headaches a year over the past 3 years. For a few 3. Each year 2% of patients with aneurysms minutes prior to the headache his right hand develop new ones. tingles and feels clumsy. These latter symptoms 162 7 Cerebrovascular Anatomy with Clinical Correlation last from 3 to 4 h. He still has clumsiness of the Table 7.6 Spetzler-Martin grading scale right hand today. He has been diagnosed with the Feature Score International Headache Society (HIS) classifi cation Maximum dimension of 1.2.4 or migraine with prolonged aura. Past med- <3 cm 1 ical history includes rare grand mal seizures with a 3–6 cm 2 postictal right arm paresis lasting 1 h. This was >6 cm 3 diagnosed as a Todd’s phenomenon. They began Location at the age of 10 and have been well-controlled Non-eloquent 0 with valproate for the last 5 years because of In or adjacent to eloquent cortex 1 good compliance, previously a problem. Venous drainage Superfi cial only 0 Neurologic examination : Deep 1 This is normal other than dysdiadochokinesis (impaired rapid alternating movements) affecting the right arm. Seizures, migraine, and focal fi ndings are the Questions: hallmark manifestations of an arteriovenous 1. What part of the history indicates a focal malformation. lesion? 2. Does the examination support the same AVMs are congenital lesions of focal collec- localization? tions of dilated arteries and veins connected by 3. Where is the lesion and why? one or more fi stulas (Arteriovenous Malformations 4. What fi ndings by history and examination 1999) . The arteries do not have a smooth muscle suggest the underlying pathology? layer. They are located within brain parenchyma Answers: and may cause both intracerebral and SAH. 1. The headache history is typical of migraine. Complications are hemorrhage (2% of all Paresthesias are part of the classical migraine strokes), seizures, migraine, and hypoperfusion spectrum. The prototypical sensory phenomenon (ischemic steal) of adjacent structures. The ana- is a “march of paresthesias” beginning in the tomic features of the AVM are best described by hand, then arm and face. Clumsiness is not com- the Spetzler-Martin Grading Scale (Table 7.6 ). mon with migraine. It is a red fl ag indicating Epidemiology and natural history : focal dysfunction and a possible lesion in the Risk of hemorrhage is 2–4% per year and the left cerebral hemisphere. Migraine and clumsi- highest incidence of hemorrhage is between ages ness in one hand would be an unlikely manifes- 20 and 40. The annual risk of recurrent hemor- tation of a brainstem or cerebellar lesion. rhage in those treated conservatively is 5.9%. 2. Yes. Impaired rapid alternating movements Arterial aneurysms are often present within the may occur with lesions involving any of the malformation. Predictors of high risk for recur- three motor systems, corticospinal, extrapyra- rent hemorrhage include prior hemorrhage, deep midal, and cerebellar. venous drainage, and deep location. 3. A postictal right arm paresis or Todd’s phe- nomenon indicates left cerebral hemisphere Arteriovenous malformations carry a 2–4% pathology. Seizures are pathognomonic of per year risk of hemorrhage. cerebral hemisphere disease. Thus the patient has evidence of a left cerebral hemisphere lesion, probably frontal lobe. The sum of the scores equals the grade. Grades 4. The triad of migraine, seizures, and focal fi nd- 1–3 are considered treatable by embolization, ings is a common presentation with AVMs. surgical resection, and radiosurgery. Details of This patient has a left frontal AVM. treatment are beyond the scope of this text. Answers 163

19. The most reliable cerebrospinal fl uid test for Questions (True or False) hemorrhage is xanthochromia of the super- natant in a centrifuged CSF sample. 1. Seizures are a frequent occurrence with supe- 20. A single episode of migraine aura lasting rior sagittal sinus thrombosis. 10 h does not require neuroimaging. 2. The posterior inferior cerebellar artery arises 21. The most common sign of increased intrac- from the basilar artery. ranial pressure is a 3rd nerve palsy. 3. A major supply of the anterior spinal artery is 22. The lenticulostriate arteries arise from the the artery of Adamkiewicz. M2 segment of the middle cerebral artery. 4. Acute vertigo may occur with occlusion of 23. The clumsy hand-dysarthria syndrome is the anterior inferior cerebellar artery. specifi c for a pontine infarction. 5. The posterior cerebral artery supplies the 24. A common cause of a basilar artery occlu- occipital cortex but not deep nuclear sion at its termination is cardioembolism. structures. 25. When a patient is heard to say “fl ed” instead 6. Epidural hematomas can be caused by of “bed” after successful clipping of a left traumatic injury to the middle cerebral middle cerebral artery aneurysm, vasospasm artery. may be occurring and causing left temporal 7. The fi rst branch of the internal carotid artery lobe ischemia. is the ophthalmic artery which takes off from its supraclinoid portion. 8. The anterior communicating artery is a large Answers diameter vessel that connects the two ante- rior cerebral arteries. 1 . T 9. Pure sensory stroke may occur from an 2 . F infarction in the thalamus. 3 . T 10. Carotid atherosclerotic disease is the most 4 . T common cause of perioperative stroke after 5 . F coronary bypass surgery. 6 . F 11. High-grade extracranial internal carotid artery 7 . T stenosis usually causes stroke by embolism. 8 . F 12. Embolism associated with atherosclerotic 9 . T disease at the origin of the common carotid 10. F artery is a common source of cerebral 11. T infarction. 12. F 13. The posterior cerebral arteries provide impor- 13. T tant vascular supply to the temporal lobes. 14. T 14. Ischemia associated with anterior commu- 15. T nicating artery aneurysms may produce 16. F abulia. 17. T 15. Crohn’s disease has been associated with 18. F venous sinus disease. 19. T 16. Lobar intracerebral hematomas are most 20. F amenable to surgical treatment. 21. F 17. Arteriovenous malformations commonly 22. F cause seizures. 23. F 18. Intracerebral hematomas most likely to cause 24. T seizures are thalamic. 25. T 164 7 Cerebrovascular Anatomy with Clinical Correlation

Rosand J, Greenberg SM. Cerebral amyloid angiopathy. References Neurologist. 2000;6:315–25. Schievink W. Intracranial aneurysms. N Engl J Med. Arnold AC. Pathogenesis of nonarteritic anterior ischemic 1997;336:28–39. optic neuropathy. J Neuroophthalmol. 2003;23:157–63. Suarez JI, Tarr RW, Selman WR. Aneurysmal suba- Biousse V, Bousser MG. Cerebral venous thrombosis. rachnoid hemorrhage. N Engl J Med. 2006;354: Neurologist. 1999;5:326–49. 387–96. Idicula T, Joseph LN. Neurological complications and The Arteriovenous Malformation Study Group. aspects of basilar artery occlusive disease. Neurologist. Arteriovenous malformations of the brain in adults. 2007;13:363–8. N Engl J Med. 1999;340:1812–8. Mohr JP, Choi D, Grotta JC, Weir B, Wolf PA. Stroke. Pathophysiology, diagnosis and management. 2nd ed. New York: Churchill Livingstone; 2004. Autonomic Nervous System Anatomy with Clinical 8 Correlation

Keywords Blood pressure ¥ Genitourinary ¥ Parasympathetic ¥ Pupil ¥ Sympathetic

Familiarity with the anatomy of the autonomic optic tract, brachium of the superior colliculus, nervous system is critically important for the pretectum, Edinger-Westphal nucleus of the diagnosis of numerous neurologic diseases. 3rd cranial nerve and then the 3rd nerve which Abnormalities of the pupil, blood pressure, blad- exits the brainstem. The 3rd nerve travels der, bowel, and sexual function are frequent con- between the posterior cerebral and superior comitants of neurologic disorders and may cerebellar arteries, under the medial aspect of provide the sole clue for solving diagnostic puz- the uncus of the temporal lobe, into the cav- zles. The most clinically useful elements of the ernous sinus and exits through the superior autonomic nervous system will be reviewed in orbital fi ssure. Fibers then synapse at the ciliary this chapter. ganglion in the orbit. Small ciliary fi bers extend from the ciliary ganglion to innervate the sphincter pupillae. Pupil The three neurons in the parasympathetic In the absence of ophthalmologic pathology, pathway are located in the pretectum, there are just three possible explanations for Edinger-Westphal nucleus and ciliary pupillary asymmetry. These are a sympathetic ganglion. system lesion on the side of the small pupil, para- sympathetic lesion on the side of the large pupil, and physiologic (central) anisocoria. The pretectum is the pupillary center and is the location of the fi rst of three synapses. The

second synapse is at the Edinger-Westphal There are only three explanations for a pupil- nucleus and the third is at the ciliary ganglion. lary asymmetry in the absence of ocular Interference with the parasympathetic outfl ow, pathology, a sympathetic or parasympathetic beginning with the pretectum, may cause a lesion and normal variation. large, ipsilateral, poorly reactive pupil. Pretectal involvement usually results in round 5Ð6 mm 1. The parasympathetic pathway (see Fig. 8.1 ) is nonreactive pupils. Involvement of the Edinger- a three neuron arc which mediates the light Westphal nucleus ordinarily results in a nonre- refl ex. The light refl ex proceeds from retinal active, 4Ð5 mm, irregular pupil. A 3rd nerve ganglion cells, to the optic nerve, optic chiasm, lesion produces a large pupil of variable size

J.N. Alpert, The Neurologic Diagnosis: A Practical Bedside Approach, 165 DOI 10.1007/978-1-4419-6724-4_8, © Springer Science+Business Media, LLC 2012 166 8 Autonomic Nervous System Anatomy with Clinical Correlation

Fig. 8.1 Parasympathetic pathway of the light refl ex. With permission from: Blumenfeld (2002 ) , p. 541

which is poorly reactive or nonreactive to light. into a discrete bundle in the dorsolateral A ciliary ganglion lesion results in Adie’s syn- medulla. They pass into the spinal cord to drome (tonic pupil) (see page 168). synapse in the intermediolateral cell column 2. The sympathetic pathway contains three neu- at C8-T1-T2. The sympathetic fi bers exit with rons and two synapses (see Fig. 8.2 ). A lesion the C8-T1-T2 roots and travel upward in the causes a Horner’s syndrome which is miosis, sympathetic chain where the second synapse ptosis, occasionally inverse ptosis with eleva- occurs in the superior cervical ganglion. Most tion of the lower lid and anhidrosis. The pres- of the axons exiting the ganglion enter the ence of both ptosis and inverse ptosis gives the sheath of the internal carotid artery. A smaller impression of . The location and number of fi bers, the sudomotor fi bers, enter severity of the involvement of the sympathetic the sheath of the external carotid artery to system will determine whether all or just one or supply the sweat glands on the face. The sym- two of these fi ndings are present. The fi rst-order pathetic fi bers remain in the internal carotid neuron is in the hypothalamus and its axons artery sheath as it passes through the cavern- descend diffusely through the ipsilateral ous sinus, then exit with the fi rst division of midbrain and pons; but they eventually merge the trigeminal nerve forming the long ciliary Pupil 167

Fig. 8.2 The sympathetic pathway for innervation of the pupil. With permission from: Blumenfeld (2002 )

fi bers to innervate Muller’s muscle in the through looking at old pictures of the patient. upper and lower lids (superior and inferior Many patients have a slight asymmetry, tarsal muscles) and the dilator pupillae. however, and further detailed assessment is required only if there is clinical relevance. The three neurons of the sympathetic path-

way are located in the hypothalamus, inter- Physiologic (central) anisocoria is gener- mediolateral cell column in the spinal cord ally less than 0.5 mm. at C8-T1-T2 and the superior cervical ganglion. The most practical bedside method of diag- nosing Horner’s syndrome is to turn off the 3. Physiologic or central anisocoria is generally lights and see if the pupillary asymmetry less than 0.5 mm. The diagnosis is best made increases after 5 s and before 15 s have elapsed. 168 8 Autonomic Nervous System Anatomy with Clinical Correlation

This is because of the lag in pupillary dilatation optic nerve disease will interfere with the light for several seconds in the presence of a sympa- refl ex and result in better pupillary constric- thetic lesion. If, for example, a 0.5 mm asym- tion with the near response. Additionally, any metry in light becomes a 1 mm asymmetry in midbrain lesion may produce light-near dark at 5 s, the smaller pupil is evidently unable dissociation. to dilate well and hence a sympathetic system lesion, Horner’s syndrome, must be suspected. Argyll-Robertson pupils are bilateral, With a Horner’s syndrome the pupils respond miotic, and irregular. briskly to both direct and consensual testing. Conversely, if a bright light shown on each eye Adie’s pupil (tonic pupil) occurs typically independently produces a slower response on in young women aged 20Ð40, begins with only one side, then a parasympathetic lesion, , is unilateral and due to pathology in usually 3rd nerve, is the etiology. the ciliary ganglion or short ciliary nerve fi bers. It is often asymptomatic and discovered inci- dentally by an acquaintance or when putting When Horner’s syndrome is suspected the on makeup. Ankle or knee refl exes are usually pupils should be examined in the dark. absent. A comparison of Adie’s and Argyll- Robertson pupils is reviewed in Table 8.1 . 4. Light-near dissociation (retinomesencephalic- 5. An afferent pupillary defect (APD) must be occipitomesencephalic dissociation). This is distinguished from parasympathetic pathway the comparison between the response to light disruption. An APD is diagnostic of an optic and response to near. This discrepancy can be nerve lesion. In this case the direct response of easily missed if one simply looks for a contrac- the involved eye to light is suboptimal, but the tion to light rather than a disparity between the consensual response to light is normal because two reactions. A suspicion of light-near disso- the 3rd nerve innervating the involved eye is ciation begins with an assessment of the pupil- usually intact. In an uncertain case, a light is lary reaction to light. If it is 2+/4, for example, held on the involved eye for 2Ð3 s, then the then testing a near reaction is the next step. intact eye for 2Ð3 s, then back to the involved A suspicion of lues or midbrain disease would eye. There may be an inability of the involved also prompt such testing. Visual loss due to eye to constrict on this second attempt or a severe optic nerve or chiasm pathology may third attempt and, consequently, an APD is result in light-near dissociation. Even the blind present which is diagnostic of optic nerve dis- individual can imagine his own fi nger approach- ease. Another method of detecting an APD is ing his face which provokes the near-synkinesis to simply shine a light directly on the pupil ( accommodation, convergence of the eyes and measure the time it takes for the pupil to and pupillary constriction), a response which is begin to dilate. The pupil which dilates more independent of the visual system. quickly has the APD. The older term for an Severe optic chiasm, optic nerve and also APD is Marcus-Gunn pupil. midbrain lesions may result in light-near dissociation. An afferent pupillary defect occurs when the consensual pupillary response is better The prototypical patient with light-near than the direct one. dissociation is the patient who has neurosyph- ilis. These are Argyll-Robertson pupils 6. Chemical testing. For a questionable sympa- (Thompson and Kardon 2006). They are bilat- thetic lesion (Horner’s syndrome), a relatively eral, miotic, irregular and associated with iris new development is the use of apraclonidine atrophy. This diagnosis assumes normal vision 0.5% (Garibaldi et al. 2006 ). Apraclonidine is since, as noted above, optic chiasm or bilateral an alpha receptor agonist which has been used Case Reports 169

Table. 8.1 (comparison of Adie’s and Argyll-Robertson pupils) Argyll-Robertson pupil Adie’s pupil (tonic pupil) Laterality Bilateral, usually Unilateral, initially, but becomes bilateral about 4%/year Size Miotic, 1Ð3 mm commonly Mydriatic, 5Ð7 mm commonly Reactivity to light Nil to minimal Nil to minimal Response to dark Dilates poorly Dilates poorly Reactivity to near Good Slow constriction to prolonged near effort and slow redilation Vision Normal Normal Iris Variable degrees of atrophy Vermiform movement of the border region best seen with slit-lamp exam Refl exes Commonly absent at ankles since Commonly absent at the ankles and occasionally is frequently present at the knees Lesion location Probably midbrain Ciliary ganglion or short ciliary nerves Pharmacologic testing None Supersensitivity response to either pilocarpine 0.125 or 2.5% methacholine (Mecholyl) RPR, FTA-ABS Positive Negative

to reduce elevated intraocular pressure. stimulates release of norepinephrine from the Installation of one drop of a 0.5% solution in presynaptic terminal and, consequently, the both eyes usually affects only the involved eye involved eye does not dilate as well as the nor- restoring the pupil to normal size. Ptosis may mal eye. It is diffi cult to obtain and rarely also be reversed. The eyes are examined 1 h used. after installation. The basis for the pharmaco- 7. Unexplained mydriasis. Patients who have logical response is denervation supersensitivity transient enlargement of the pupil are occa- of alpha 1 receptors in the iris dilator and in sionally seen, a puzzling clinical observation. Muller’s muscles. The test is useful for fi rst, sec- Transient mydriasis has been described in ond and third order neurons. patients with migraine. In the unresponsive patient who is brought to the Emergency A 0.5% apraclonidine solution is used to Room, uncal herniation causing a “Hutchinson” confi rm a Horner’s syndrome. pupil is suspected. When neuroimaging stud- ies (MRI/MRA) are obtained, the frequent The cocaine 10% eyedrop test is the older absence of pathology is vexing for the attend- standard method. Cocaine will prevent the ing physician. Grand mal seizures have rarely reuptake of norepinephrine and the increased been noted to produce transient unilateral amount of norepinephrine in the synapse mydriasis and consequently this diagnosis results in maximal pupillary dilatation in nor- should be one of exclusion. Finally there is the mal eyes. Depletion of norepinephrine in the diagnosis of a “springing” pupil. This unilat- synapse which occurs in patients with sympa- eral mydriasis is frequently accompanied by thetic system denervation results in a poor vague abnormal sensations in the eye and, response. despite the absence of confi rming symptoma- tology, a migraine variant is suspected. A 10% cocaine solution is also used to confi rm a Horner’s syndrome. Case Reports The hydroxyamphetamine test is diagnos- tic of a third-order neuron lesion. A third-order Case 1 A 55-year-old man sees his internist lesion results in decreased norepinephrine in because of a chronic cough producing yellowish- the presynaptic terminal. Hydroxyamphetamine white sputum every morning for the last 3 months. 170 8 Autonomic Nervous System Anatomy with Clinical Correlation

He has chronic bronchitis and smokes two packs relevant. A limited review of systems is always of cigarettes per day. inadequate. Physical examination : Case 2 A 20-year-old woman complains of dif- Physical examination by his internist discloses fi culty running. She is on her college volleyball asymmetric pupils, 4 mm on the right and 3 mm team and has been unable to compete effectively on the left with brisk equal reactions. Palpebral for the last 3 months. fi ssures are slightly asymmetric with the right Neurologic examination : greater than the left. In a darkened room the There is a pupillary asymmetry with the right right pupil is 5.5 mm and the left is 4 mm. There pupil 4.5 mm and the left 4.0 mm. In dark the is no anhidrosis. Chest examination reveals scat- right pupil is 5.2 mm and the left 4.5 mm. There tered rhonchi and wheezes. is questionable anhidrosis on the left face, tested Laboratory tests : by checking the temperature on the face just The chest X-ray with apical lordotic views and a under and above the eye as the involved side may CAT scan of the chest disclose fi ndings of emphy- be warm because of the absence of sweating. The sema. An MRI (brain) and MRA (head and neck) palpebral fi ssures are equal. There is mild spas- are normal. Neurologic consultation is requested. ticity of the left leg. Plantar responses are neutral on the right and extensor on the left. Neurologic examination : What test should be performed which can The neurologist does a complete review of neuro- localize the lesion? What single neuroimaging logic symptomatology. Twenty years ago the study should then be ordered? patient recollects severe, unilateral, excruciating, A cocaine test is performed using a 10% solu- left-sided headaches lasting 20 min to 1 h. They tion. Two drops are placed in both eyes. The right occurred roughly three times a day occasionally pupil dilates to 6.5 mm and the left to 5 mm. An awakening him at night during a period of 1 month. MRI (cervical) is obtained and discloses an When they occurred he had to pace the fl oor. intramedullary cyst in the lower cervical and Diagnosis : upper thoracic cord on the left side. Cluster headache associated with a partial Diagnosis : Horner’s syndrome. Cervical myelopathy secondary to syringomyelia.

Cluster headache may be associated with a Main clinical points: Horner’s syndrome. 1. The left Babinski sign and mild spasticity of the left leg indicate involvement of the left Main clinical points: corticospinal tract below the pyramidal decus- 1. The absence of facial anhidrosis supports a sation or the right corticospinal tract above the third-order Horner’s syndrome which implies decussation. involvement of the sympathetic chain in the 2. The presence of a Horner’s syndrome is con- internal carotid artery sheath. In patients who fi rmed by the cocaine eye drop test. have cluster headache, angiography has dem- 3. A Horner’s syndrome on the left side fi ts onstrated changes in the carotid siphon mani- neatly with a left corticospinal tract lesion fested by distention of the carotid artery wall indicating pathology within the left side of the thus possibly compressing the sympathetic cervical spinal cord. A crossed syndrome, plexus lying within it. right Horner’s and left Babinski would indi- 2. The cardinal manifestations of cluster head- cate a brainstem lesion. ache are the short duration and the behavior of the patient who often paces back and forth. Spinal cord lesions which produce Horner’s 3. Although neuroimaging is essential, a com- syndrome are most often intramedullary. plete history yields the diagnosis and is more Case Reports 171

4. An MRI scan of the cervical cord reveals a Case 4 A 73-year-old woman is brought to the syrinx with maximum involvement at the emergency room by ambulance because of the C8-T1 level. sudden onset of a left hemiplegia after exercising on her stationary bicycle. Case 3 A 47-year-old man requests an evalua- tion because of an episode of severe vertigo 5 Neurologic examination : days prior to his appointment. One month ago he Blood pressure is 160/90 and pulse 100, irregu- was in a motor vehicle accident. His car was larly irregular. Her mental status testing is normal struck from behind when he was stopped at a red other than the fact that she denies any weakness. light. Because of constant neck pain he sought Her head and eyes are deviated to the right. When chiropractic treatment. One hour after his second she is asked to close her eyes she does so only for treatment he had an acute episode of vertigo asso- 2Ð3 s despite repeated requests. Eye movements to ciated with nausea and vomiting which lasted the left can be elicited only with the oculocephalic 5 min. This was followed by a staggering gait maneuver. She has left facial weakness and a left for 1/2 h. hemiparesis. She is aware of and identifi es her left arm and leg but states they have normal strength Neurologic examination : despite inability to lift them. She has a left hyper- The abnormal fi ndings are restricted to the pupils. refl exia, Hoffmann and Babinski signs. The right pupil is 5 mm with a 4+/4 reaction. The A CAT scan shows a questionable but large left is 4.5 mm with a 4+/4 reaction. The right face hypodense region in the right frontal, parietal, is slightly moist and cool. After 5 s in a darkened and temporal lobes in the middle cerebral artery room, the right pupil is 6 mm and the left is 5 mm distribution. CBC, chemistries, PT, and PTT are proving a partial left Horner’s syndrome. normal. The patient is treated with intravenous An MRI (brain) reveals a small infarction in tissue plasminogen activator (TPA) at 2 h after the left dorsolateral medulla. An MRA discloses the onset of symptoms. a left vertebral artery dissection which is likely to A second examination is performed 12 h later. be a complication of chiropractic manipulation of The patient is lethargic but arousable and then the neck. oriented. She still denies any weakness. She now Main clinical points: has a left homonymous hemianopsia to fi nger 1. Vertigo is the prototypical symptom of ves- counting. The right pupil is 4 mm with a 2+/4 tibular dysfunction. reaction to light and the left is 3 mm with a 4+/4 reaction. The left hemiparesis is unchanged, but Vertigo and a Horner’s syndrome suggest a there is paresis of the right leg with a right brainstem lesion. Babinski sign. Is there a possible connection between the his- 2. The vestibular system includes the peripheral tory and stroke mechanism? What physiologic vestibular apparatus (semicircular canals and process is occurring? Why does the patient have otolith organs), 8th nerve and the vestibular paresis of the right leg and a right Babinski sign? nuclei. There are additional pathways from What might explain the left homonymous hemi- the vestibular nuclei to other brainstem nuclei anopsia, a new fi nding? and cerebellum. Atrial fi brillation and cardioembolism may 3. A left Horner’s syndrome with a history of have been provoked by exercise. The embolic vertigo points to involvement of the sympa- event was followed by marked cerebral edema thetic pathway and vestibular nuclei. They are causing an uncal herniation syndrome. The uncus close to each other in the left dorsolateral and parahippocampal gyrus is compressing the medulla. The vascular supply of this region is posterior cerebral artery downward resulting in the left vertebral and posterior inferior cere- obstructed fl ow and ipsilateral occipital ischemia bellar artery. on the right side. This causes a left homonymous 172 8 Autonomic Nervous System Anatomy with Clinical Correlation hemianopsia. The sluggish right pupil indicates opaque vision, brightness, or simply blindness. right 3rd nerve compression and the pupil is These observations point towards occipital lobe selectively involved since these fi bers are on the ischemia. Infrequent complaints are neck and periphery of the 3rd nerve making them easily shoulder pain. These symptoms may be due to susceptible to a compressive lesion. The herni- ischemia of neck musculature. Orthostatic dysp- ated parahippocampal gyrus pushes the midbrain nea may occur presumably caused by impaired from right to left against the rigid tentorial edge vascular supply to the apices of the lungs. Lastly, on the left side. This indents the contralateral slowed mentation may accompany one of the (left) cerebral peduncle which is called Kernohan’s above-noted symptoms although it is rarely an notch and produces ipsilateral signs of pyramidal isolated complaint. Clearly, the review of neuro- tract injury. Ipsilateral means on the same side as logic symptoms, a critical step in taking a history, the mass effect. Consequently, there is a right leg is likely to yield additional information. paresis and Babinski sign. A brief review of the physiology of blood A CAT scan (noncontrast) shows the mass pressure control, through the baroreceptor refl ex effect and a few punctate hemorrhages in the (Benarroch 2008 ), will be elaborated. Baro- midbrain (Duret’s hemorrhages) caused by down- receptors are located in the carotid sinus and ward displacement of the midbrain and tearing of aortic arch. These are innervated by the glossopha- paramedian perforating vessels. ryngeal and vagus nerves, respectively. A drop in In summary, this patient had a cardioembolic blood pressure provokes a decrease in afferent event to the right middle cerebral artery caused neural discharges from these regions. The affer- by atrial fi brillation. This was followed by acute ent nerve fi bers of the 9th and 10th cranial nerves cerebral edema resulting in an uncal herniation enter the dorsomedial medulla to make up part of syndrome. TPA was neither benefi cial nor delete- the nucleus of the tractus solitarius. The decrease rious in view of the absence of a hematoma. in afferent input through this pathway results in decreased output from the nucleus ambiguus which contains the nuclei of the 9th, 10th and Blood Pressure 11th cranial nerves. The sinus node receives decreased vagal input which causes compensa- An essential element of every neurologic examina- tory increased heart rate. There is an increased tion is measurement of pulse and blood pressure. sympathetic output mediated by fi bers from the The absence of a history of hypertension even in a nucleus tractus solitarius to the caudal ventrolat- 30-year-old patient with footdrop does not obviate eral medulla, which is an excitatory pathway. an obligation to participate in preventive care. Noradrenergic connections from this area inner- Furthermore, hypotension or hypertension is often vate the paraventricular nucleus and supraoptic relevant to many neurologic disorders. nucleus of the hypothalamus. This stimulates vasopressin release from the vasopressin-synthe- Blood pressure and pulse should be taken sizing neurons located in these structures. in every patient irrespective of the chief Vasopressin causes smooth muscle constriction complaint. in arterial walls resulting in an increased blood pressure. Sympathetic fi bers also descend from There are manifold symptoms of hypotension the medulla down to the intermediolateral cell which most often demand neurologic attention. column in the spinal cord where they synapse Most are obvious and include brief or prolonged between T1 and L2. The preganglionic fi bers dizziness when standing up, blurred vision, then exit at their respective levels to form the fatigue, and weakness. Blurry vision is bilateral sympathetic chain which is between T1 and L2. and is most often described as “darkening” due to T1 through T4 supply the arms and heart, T5 impaired blood supply to both . Much less through T12 supply the adrenal medulla, and T11 often is absence of vision or such complaints as through L2 the genitourinary system. Blood Pressure 173

Lessons: Baroreceptors are located in the carotid sinus 1. The room turning black indicates retinal and aortic arch. These are innervated by the ischemia. When unilateral and brief it is called 9th and 10th cranial nerves, respectively. amaurosis fugax; when bilateral, hypotension is the likely etiology. 2. Dyspnea suggests ischemia to the apices of Hypotension decreases afferent input from the the lungs. 9th and 10th cranial nerves. 3. The blood pressure must be taken supine, sit- ting, and 3 min after standing unless there is a

The result is increased sympathetic output major drop of blood pressure in the interim. from the caudal ventrolateral medulla and The defi nition of “orthostatic hypotension” is decreased vagal input to the sinus node. a reduction in systolic blood pressure of at least 20 mmHg or a reduction in diastolic blood pressure of at least 10 mmHg during the Norepinephrine is released from postgangli- fi rst 3 min of standing. This is a classic fi nding onic axons and, as noted above, from central ner- of sympathetic failure of vasoconstriction. vous system noradrenergic axons. Catecholamines The lack of a compensatory increase in heart are released from the adrenal medulla into the rate is common. Orthostatic hypotension circulation through the adrenal vein. These secre- among elderly people is very common (Lipsitz tions result in a compensatory return of blood 1989 ). Dehydration, medications and decon- pressure to normal levels. ditioning are the most likely etiologies. Case 5 A 68-year-old retired male engineer complains of an episode of loss of vision associ- Orthostatic hypotension is defi ned as a sys- ated with shortness of breath. He was playing tolic blood pressure reduction of 20 mmHg with his grandchild on the fl oor for approximately or diastolic blood pressure reduction of 10 min, stood up and was immediately short of 10 mmHg during the fi rst 3 min of breath. Simultaneously, he lost his vision. He standing. describes the room turning black. He lay down on the sofa and recovered over 2 min. On further Orthostatic hypotension among elderly specifi c questioning the patient reports an people living in long-term care facilities is unsteady gait and mild memory impairment very common with a reported incidence of developing over the past year. 54 to 68%. Examination : Blood pressure when supine is 140/90 with a 4. Orthostatic hypotension is a common mani- pulse of 80 and regular. Blood pressure upon festation of neurologic diseases with auto- standing, taken immediately, is 122/82 with a nomic dysfunction such as multiple system pulse of 84 and regular. After standing for 3 min atrophy, Lewy body dementia, Parkinson’s the patient feels faint, the blood pressure is 80/60 disease and pure autonomic failure. Multiple and the pulse remains 84. The neurologic exami- system atrophy encompasses conditions that nation discloses impaired short-term memory as were described separately in the past such as he recalls 1 of 3 words after 3 min have elapsed. olivopontocerebellar atrophy, ShyÐDrager He is unable to add 14 + 19 or reverse 5-letter syndrome, and striatonigral degeneration. words. He walks with a wide-based gait and is Case 6 A 50-year-old woman complains of short- unable to perform tandem gait. He exhibits mod- ness of breath when getting up from a squatting erate heel-to-shin ataxia. position. This is sometimes accompanied by Diagnosis : blurred vision which she describes as brightness Multiple system atrophy. associated with multicolored spots. When that 174 8 Autonomic Nervous System Anatomy with Clinical Correlation happens she has no functional vision. These symp- medical evaluation has disclosed no organic pathol- toms last less than 30 s and have been increasing ogy. The student is under considerable stress trying in frequency over the last 3 months. She denies other to excel in her premedical courses at the university. neurologic complaints. On thorough questioning She reluctantly acknowledges being chronically she acknowledges other symptoms which include anxious since her symptoms have been repeat- burning, prickly sensations in both feet. edly dismissed as psychogenic. Treatment with alprazolam and fl uoxetine have been ineffective. Neurologic examination : Neurologic examination reveals a blood pressure Examination : of 140/80 in the supine position with a heart rate Blood pressure supine is 110/80 with a heart rate of 80 and blood pressure immediately on stand- of 90. Upon standing, blood pressure is 105/85 ing is 100/74 with a heart rate of 82. The patient with a heart rate of 110. The patient is then complains of being slightly dizzy, but after 2 min requested to walk up and down the corridor for the blood pressure rises to 120/80 with a heart 3 min after which she becomes fatigued. Her rate of 84 and she feels well. Nevertheless, a fi nal blood pressure is 100/60 and her heart rate is 130 blood pressure is taken at 3 min at which time the and regular. She complains of anxiety. The remain- patient complains of shortness of breath and dim- der of the neurologic examination is normal. ming of her vision. Her blood pressure is 85/60 Impression : and heart rate is 88. Paroxysmal orthostatic tachycardia syndrome Neurologic examination discloses absent (POTS) (Jacob et al. 2000 ). ankle refl exes, absent vibratory perception at toes and ankles and distal sensory loss to pin in all Lessons: extremities. This is apparent in the hands up to 1. One must never assume psychiatric explana- the wrist and in the feet up to the ankles. tions for a patient’s symptoms without a thor- ough examination. Impression : 2. POTS is a well-established cause of chronic Autonomic neuropathy associated with a sensory fatigue and is manifested by a tachycardia, neuropathy. Diabetes is suspected and proven. with minimal exertion, of at least 30 points higher than the resting rate. Lessons:

1. The history of burning paresthesias in the feet is common with neuropathy. The addition of POTS is a well-established cause of chronic orthostatic hypotension strongly suggests an fatigue producing tachycardia, with mini- autonomic neuropathy. mal exertion, of at least 30 points higher 2. The blood pressure must be taken after 3 min than the resting rate. have elapsed if earlier blood pressures are nondiagnostic. 3. Deconditioning may be a signifi cant factor in its development. Consequently, this is a treat- The blood pressure must be taken after able form of chronic fatigue. 3 min have elapsed if earlier blood pres- sures are nondiagnostic. Genitourinary System 3. The most common cause of orthostatic hypotension due to an autonomic neuropathy Anatomic and Physiologic Overview is diabetes; but it occurs more commonly as an adverse effect of antihypertensive medication. The bladder receives input from parasympathetic Amyloidosis is a rare etiology. and sympathetic systems. Micturition depends on coordinated function of the detrusor muscle in Case 7 A 20-year-old female college student has the bladder wall, the bladder neck, and the external been referred because of chronic fatigue. A general sphincter (Benarroch 2010 ). Genitourinary System 175

The detrusor is innervated by the parasympa- ascending in the lateral spinothalamic tract. thetic system. The sacral parasympathetic nuclei Micturition is then initiated in the medial frontal are located in gray matter lateral to the central micturition centers which activate voiding (detru- canal (intermediolateral cell column) in the S2-S4 sor refl ex). The detrusor refl ex is regulated by segments. Efferent axons travel through the pelvic pontine centers and mediated by spinal cord path- nerves to ganglion cells in the bladder wall. ways. The refl ex is initiated by voluntary relax- ation of the external urethral sphincter and pelvic The detrusor is innervated by the parasympa- fl oor muscles. Onuf’s nucleus (S2-S4) contains thetic system and the bladder neck receives the somatic neurons which give rise to axons sympathetic innervation. which mediate this response. Then sympathetic inhibition to relax the bladder neck occurs fol- The bladder neck receives sympathetic inner- lowed by activation of parasympathetics to cause vation originating in the intermediolateral cell detrusor muscle contraction. column of the spinal cord at T11, T12 and L1. Postganglionic neurons in the sympathetic chain Neurogenic Bladder give rise to the hypogastric nerve. Sympathetic with Clinical Correlation activation contracts the internal sphincter which 1 . Hyperrefl exic ( spastic ) neurogenic bladder . allows storage of urine. This is caused by detrusor-sphincter dyssyner- The external sphincter receives somatic gia, i.e., incoordinated function. The internal innervation originating from Onuf’s nucleus, sphincter becomes hypertonic but not suffi - S2-S4, located in the ventrolateral gray matter cient to withstand the detrusor contraction of the spinal cord. The axons are carried via the thus resulting in urinary incontinence. The pudendal nerves. The external sphincter relaxes bladder capacity is reduced and there are fre- during micturition. Additionally, there is quent uninhibited contractions. This may somatic innervation of pelvic fl oor muscles occur with any chronic central nervous system which also relax the perineal and levator ani disorder affecting the cerebral hemispheres, musculature. brainstem, or spinal cord. 2 . Flaccid neurogenic bladder . This is an The external sphincter receives somatic inner- arefl exic atonic bladder which results in over- vation originating from Onuf’s nucleus, fl ow incontinence and/or stress incontinence. S2-S4, located in the ventrolateral gray matter The bladder capacity is markedly increased of the spinal cord. and the voiding stream is poor. This is due to disease affecting dorsal roots, cauda equina, Sensory systems of the bladder include sym- the lumbosacral plexus, and acute CNS pathetics, T11-L1 roots, which mediate pain. lesions such as large cerebral hemisphere Sensory fi bers also travel through the parasympa- or brainstem infarctions and spinal cord thetic system and mediate refl ex bladder activity. trauma. The somatic afferents from the external sphincter and urethra travel in pudendal nerves and stretch Sexual Function receptors in the pacinian corpuscles in the blad- der wall lead to the sensation of bladder fi lling. Sexual arousal occurs through multiple sensory Impulses that give rise to a sensation of fullness modalities including psychogenic factors. Three of the bladder, mediated by somatic afferents, neural pathways innervate the genital organs. pass into the lateral spinothalamic tract and These are the sacral parasympathetics (pelvic ascend to the sensory cortex. nerves), thoracolumbar sympathetics (hypogas- Summary : The sense of bladder fullness reaches tric and thoracolumbar sympathetic chain), and the sensory cortex mainly through somatic afferents somatic nerves (pudendal nerve). 176 8 Autonomic Nervous System Anatomy with Clinical Correlation

Sacral parasympathetic nerves are primarily A lax anal sphincter implies lesions of sensory responsible for erection. roots, conus medullaris, motor roots (S3-4) or peripheral nerves. Sacral parasympathetic nerves are primarily responsible for erection by eliciting vasodilation of penile arteries and relaxation of venous sinu- soids in erectile tissue. In the female, parasympa- Questions (True or False) thetics cause secretion of mucus by Bartholin’s glands. There is a less potent role for the sympa- 1. The pupillary center is the Edinger-Westphal thetic system with regard to secretions, but it does nucleus. increase vaginal blood fl ow. There are descend- 2. The ciliary ganglion is the location of the ing erectile pathways from the cerebral hemi- third synapse of the sympathetic system. spheres through the lateral columns of the spinal 3. Intramedullary lesions of the spinal cord may cord. Consequently, psychogenic erection may cause Horner’s syndrome. occur via sympathetic pathways when refl ex 4. Physiologic anisocoria is less than 1 mm. erection is abolished due to sacral lesions. 5. The best method of testing for a parasympa- Ejaculation is mediated by the sympathetic sys- thetic lesion is a brightly lit room. tem at T12-L1 and the somatic nerves. Semen is 6. Light-near dissociation fi ndings are most emitted into the urethra by sympathetically medi- likely to be observed in a brightly lit room. ated smooth muscle contraction in the seminal 7. Argyll-Robertson pupils are mydriatic. vesicles, vas deferens, prostate and bladder neck. 8. Symmetrical pupils occur with severe unilat- The somatic nerves originate in the anterior horn eral optic nerve lesions. cell in Onuf’s nuclei in the S2-S4 spinal cord seg- 9. There is depletion of norepinephrine in both ments. These fi bers travel in the pudendal nerves the synapse and presynaptic terminal irre- and provide excitatory input to striated muscles, spective of the lesion site in patients with bulbocavernosus and bulbospongiosus muscles in Horner’s syndrome. the male, which are responsible for ejaculation and 10. Horner’s syndrome can be associated with provide for rhythmic perineal contraction in cluster headache. females during orgasm. 11. Sympathetic fi bers to the pupil remain unilateral. 12. Horner’s syndrome may occur with lesions Ejaculation is mediated by both sympathetic of the dorsolateral medulla. and somatic nerves. 13. Pupillary fi bers are located in the center of the 3rd nerve. 14. Shortness of breath and neck pain may occur Bowel Function with hypotension. 15. Baroreceptors are located in the carotid sinus The anal sphincter mechanism includes the inter- and innervated by the vagus nerve. nal anal sphincter, external anal sphincter, and 16. The thalamus is critically involved in blood puborectalis muscles. The latter two muscles act pressure control. as the voluntary sphincter as they are innervated 17. Darkening of vision in both eyes is called by pudendal nerves (somatic). Receptors in the amaurosis fugax. pelvic fl oor detect the presence of stool. Thus 18. An acute frontal lobe lesion may produce an some neuropathies, especially secondary to dia- atonic bladder. betes, may result in diarrhea with fecal inconti- 19. Sacral sympathetic fi bers are responsible for nence. A lax anal sphincter implies lesions of erections. sensory roots, conus medullaris, motor roots 20. Diabetic neuropathies may cause diarrhea (S3-4), or peripheral nerves. with fecal incontinence. References 177

18. T Answers 19. F 20. T 1 . F 2 . F 3 . T References 4 . F 5 . T Benarroch EE. Neural control of the bladder. Neurology. 6 . T 2010;75(20):1839Ð46. 7 . F Benarroch EE. The arterial barorefl ex. Neurology. 2008; 8 . T 71:1733Ð8. Blumenfeld H. Neuroanatomy through clinical cases. 9 . F Sunderland: Sinauer Associates; 2002. 10. T Garibaldi DC, Hindman HB, et al. Effect of apraclonidine 11. T on ptosis in Horners syndrome. Ophthal Plast Reconstr 12. T Surg. 2006;22:53Ð5. Jacob G, Costa F, Shannon JR, et al. The neuropathic pos- 13. F tural tachycardia syndrome. N Engl J Med. 2000;343: 14. T 1008Ð15. 15. F Lipsitz LA. Orthostatic hypotension in the elderly. N Engl 16. F J Med. 1989;321:952Ð7. Thompson HS, Kardon RH. The Argyll-Robertson pupil. 17. F J Neuroophthalmol. 2006;26:134Ð8. Neuromuscular Diseases: Neuroanatomic and Differential 9 Diagnoses

Keywords Myopathy • Neuromuscular junction • Myotonia • Plexus • Demyelination • Neuropathy • Radiculopathy

The neuromuscular diseases can be defi ned as sensory, acquired or genetic, part of a systemic those which affect the lower motor neuron disorder, and is there concomitant autonomic (AHC), roots and root ganglia, plexus, nerves, nervous system involvement? An additional piece neuromuscular junction and muscle. This covers of information may also be vital, the results of an immense array of illnesses: degenerative, electromyography and nerve conduction studies genetic, immune, infectious, metabolic, neoplas- (EMG-NCV). Since analysis of this physiologic tic, nutritional, paraneoplastic, traumatic, toxic, study is beyond the scope of this text, accommo- and vasculitic. Clearly, a thorough review is nei- dations will be made for the absence of data ther possible nor intended. An approach to derived from this examination or its interpreta- patients with these disorders will be outlined with tion will be provided. the major emphasis placed on the history and Myopathies can be subdivided into several neurologic examination. The method used herein categories which include the dystrophies will be case presentations followed by case (genetic), infl ammatory, metabolic and endo- analysis. crine, infectious, nutritional, paraneoplastic, The fi rst task for the physician is to distinguish toxic, and vasculitic. Differentiating between between central nervous system and neuromus- these disorders can be an arduous task. The ana- cular disease. There is an overlap at the AHC, but tomic distribution of weakness commonly pro- pathology which affects these lower motor neu- vides the diagnostic clue and this element will be rons will be included. This fi rst hurdle is a famil- emphasized. iar one, often simple, but occasionally baffl ing Motor neuron, root and root ganglia, plexus even to seasoned neurologists. Careful attention and neuromuscular junction diseases make up the is always required for this fi rst step. The second remainder of the illnesses to be discussed. Their task is to select the site of pathology within the characteristic features will be reviewed. neuromuscular system. An accurate selection will There will be 34 case presentations followed allow for a well-reasoned differential diagnosis. by a case analysis. At the end of the chapter, the Neuropathies will be examined in a special case numbers of each anatomic site will be listed manner. Several distinctions must be made. Is the to facilitate a review of all reports pertaining to neuropathy acute or chronic, primarily motor or that disease location.

J.N. Alpert, The Neurologic Diagnosis: A Practical Bedside Approach, 179 DOI 10.1007/978-1-4419-6724-4_9, © Springer Science+Business Media, LLC 2012 180 9 Neuromuscular Diseases: Neuroanatomic and Differential Diagnoses

failure of conservative treatment. An EMG is Cases utilized for radiculopathy primarily for local- ization purposes or when the anatomic diagno- Case 1 A 55-year-old woman complains of left sis is unclear. It is commonly normal in arm pain for 2 weeks. She describes an intermit- radiculopathies and should be performed, if tent, severe ache affecting the upper arm and necessary, after at least 2 weeks and preferably forearm muscles reaching an intensity of 8/10. It 3 weeks have elapsed from the onset of symp- can be provoked by working on a computer and is toms as signs of denervation are not found aggravated at night. When queried about her earlier. sleeping position, she reports that she sleeps on her back using a neck support which she has used Case 2 A 77-year-old man complains of the for years to prevent neck strains. rapid onset over 2 days of unprovoked, severe, right thigh and hip pain. As a result, he has fallen Neurologic Examination : three times and requires assistance to walk. He Blood pressure is 120/70; pulse is 80 and has a history of hypertension and coronary artery regular. disease. There is 4+/5 strength of left triceps. Extension of the head for 50 s while seated provokes pain in Neurologic Examination : the left triceps muscle. Isolated arm movement Blood pressure is 140/80; pulse is 64 and produces no pain. regular. Strength of right iliopsoas is 3/5, quadriceps Questions : 4/5 and extensor hallucis longus 4/5. The right 1. Why is the pain provoked when the patient knee jerk is absent. There is hypesthesia to pin works on her computer? Why is it worse at over the right medial calf. night? Questions : 2. Why does head extension produce pain? 1. Where is the lesion? 3. Where is the lesion? 2. What is the differential diagnosis? 4. Is the character and location of the pain typi- 3. What specifi c test is indicated? What is the cal for this condition? What is the differential prognosis? diagnosis? What test should be ordered? Case 2 Analysis : Case 1 Analysis : 1. The lumbosacral plexus. A herniated disk at 1. Working on a computer commonly involves L3–L4 does not affect the extensor hallucis leaning forward which extends the head. The longus (L5 root) nor does it commonly cause patient’s neck support has the same effect. medial calf numbness. The numbness is due to 2. Head extension narrows the intervertebral involvement of the saphenous nerve, a branch foramina aggravating any compression of the of the femoral nerve. exiting nerve root. 2. Diabetes mellitus is the primary consideration 3. Cervical radiculopathy, probably at C6–C7 as this may be its fi rst manifestation. The most because the triceps muscle is primarily inner- common etiology is idiopathic. Additional vated by the C7 root. possibilities are malignant pelvic neoplasm, 4. Yes. The differential diagnosis is primarily radiation plexopathy, psoas hemorrhage, and herniated disk or spondylosis. Cervical spine an immune or viral origin. fi lms with oblique views may disclose focal 3. Hemoglobin A1C and 2-h glucose tolerance narrowing of an intervertebral foramen due to test. An EMG-NCV could be useful after spondylosis and should be ordered. This 2 weeks have elapsed. Clinical judgment will patient’s diagnosis is cervical radiculopathy determine whether a pelvic MRI is indicated. due to spondylosis at C6–C7. A magnetic reso- The prognosis is excellent for recovery over nance imaging (MRI) scan should only follow 6 months if it is of diabetic or idiopathic type, Cases 181

the most usual etiologies. Speculation about 3. No. A positive Romberg is common with the idiopathic variety includes viral, immune, position sense loss. Loss of vibration percep- and microvasculitic pathogenesis. tion exceeds position sense loss in patients with neuropathy. Case 3 A 35-year-old man complains of rapid 4. Nocturnal paresthesias suggest superimposed progression of his diabetic neuropathy. He was carpal tunnel syndromes. diagnosed with type I diabetes at age 24 and has a 5. Yes. This patient has hypothyroid neuropathy 5-year history of burning pain with paresthesias superimposed on his diabetic neuropathy and affecting feet greater than hands. Over the last 3–4 bilateral carpal tunnel syndromes, common weeks he has become fatigued, drowsy, developed with hypothyroidism. The symptoms of a hoarse voice and notes progressive weakness of fatigue, drowsiness, and hoarseness indicate both feet. Paresthesias in both hands have mark- additional pathology best explained by this edly increased and are especially troublesome at disorder. night. His diabetic control is excellent. Case 4 A 78-year-old woman complains of dif- Neurologic Examination : fi culty climbing stairs. She has noticed progres- Blood pressure is 130/90; pulse is 100 and sive leg weakness of over 2 years. This past year regular. she has become aware of weakness of both hands. The patient is obese (BMI = 31), lethargic, and She has lost her “equilibrium” in the shower has a hoarse voice. Strength of anterior tibialis is when washing her hair and has fallen twice. 4/5 bilaterally and gastrocnemius 4+/5 bilater- ally. Refl exes are absent in the legs and “slow” at History Analysis : brachioradialis, bilaterally. Vibration perception 1. Weakness climbing stairs usually indicates is absent at the toes, ankles, and knees. Position proximal leg weakness, primarily quadriceps, sense is absent at the toes. There is subjective dis- and iliopsoas. This could indicate a myopathy tal sensory loss to pin and temperature in all or neuromuscular junction disorder, i.e., myas- extremities. The Romberg test is positive. thenia gravis. 2. Weakness of intrinsic hand musculature, distal Questions : weakness militates against myopathy but not 1. Are the symptoms, signs, and progression the neuromuscular junction. compatible with a diabetic neuropathy? What 3. Washing one’s hair is usually done with eyes features are unexpected? closed. Losing her balance while washing her 2. What part of the refl ex examination should be hair implies eye closure and thus she depends clarifi ed? on visual cues to maintain balance. When 3. Is a positive Romberg unusual in these cir- vision is required to maintain balance there is cumstances? Is it usual to fi nd severe vibration either vestibular dysfunction or a propriocep- sense loss and only minimal position sense tive abnormality. impairment? 4. Weakness does not occur with vestibular dis- 4. Why are paresthesias in the hands much more ease. Hence, the patient must have signifi cant severe? sensory loss. Furthermore, myopathy and a 5. Should a new diagnosis be considered? disorder of the neuromuscular junction are not likely since neither are accompanied by an Case 3 Analysis : abnormal sensory examination. 1. No. Fatigue, drowsiness, hoarseness, and rapid progression are red fl ags. Well-controlled Neurologic Examination : diabetes is not compatible with rapid neuro- Blood pressure is 110/60; pulse is 72 and logic deterioration. regular. 2. The brachioradialis refl ex has a slow relaxing The patient has a quadriparesis with mild prox- phase, common with hypothyroidism. imal and moderate distal weakness. Head fl exion 182 9 Neuromuscular Diseases: Neuroanatomic and Differential Diagnoses is moderately weak. Refl exes are unobtainable cerebrospinal fl uid protein elevation and demy- except for 1+ triceps and 1+ knee jerks, but only elinating features of neuropathy by EMG-NCV. with reinforcement. Vibration perception is absent Case 5 A 65-year-old retired watchman com- at the toes, ankles, and knees. Position sense is plains of tingling in both arms and hands for 6 absent at the toes and a few errors are made at the months. Occasionally there is associated chest ankles. Mild pinprick stimulation evokes severe pain. Initially the tingling was intermittent, last- pain, much more than expected. The Romberg ing just minutes and affecting either arm test is positive. independently. Now it is constant with periodic Questions : exacerbations occasionally awakening him at 1. Where is the lesion? Could this patient have a night with or without chest pain. The hands are myelopathy? particularly affected and have become weak. 2. Does the fi nding of proximal weakness, espe- Past medical history includes osteoarthritis cially involving head fl exion, help with the and hypertension. His only medicine is diagnosis? hydrochlorothiazide. 3. Is a positive Romberg test a precisely localiz- Neurologic Examination : ing fi nding? Blood pressure is 140/90; pulse is 78 with an 4. Does the disparity between vibration and posi- occasional irregular beat. tion sense loss have value in the differential There is atrophy of the right thenar eminence. diagnosis? What is the name of the exagger- Strength of right opponens pollicis (OP) and left ated response to pin? abductor pollicis brevis (APB) is 4/5. Leg strength 5. What is the diagnosis and what laboratory is normal. Refl exes are normal at 2+ and sym- fi ndings confi rm it? metrical. Sensory examination is normal. There are deformities of the left proximal interphalan- Case 4 Analysis : geal joints. 1. This patient has a neuropathy, not a myelopa- thy. The absence of refl exes supports this Questions : diagnosis. Hyperrefl exia and Babinski signs 1. What additional bedside tests would be useful are characteristic of myelopathy. Pure AHC to make a diagnosis? pathology is ruled out by sensory loss. 2. Could tingling in both arms and hands be an Polyradiculopathy is a consideration but it early sign of neuropathy? What is the signifi - ordinarily produces severe pain. cance of nocturnal paresthesias? Why are 2. Most neuropathies produce distal weakness. myelopathy and radiculopathy unlikely? Proximal weakness indicates nonselective 3. What could explain the patient’s chest pain? pathology such as an immune disorder or 4. What is the diagnosis? vasculitis. 5. What is the underlying basis for the fi nal 3. No. Positive Romberg tests are primarily due diagnosis? to a proprioceptive or vestibular disorder, the Case 5 Analysis : former most often due to neuropathy, rarely 1. Prolonged head extension, for up to 1 min, often myelopathy. aggravates cervical root pain. The test is negative. 4. Vibration loss nearly always exceeds position Phalen’s maneuver (forced fl exion at the wrist) is sense loss with the exception of cerebral dis- performed and provokes paresthesias after main- ease. The disparity between these modalities taining the position for 30 s. Tinel’s sign is pres- is expected and, consequently, not especially ent as paresthesias are elicited from the 2nd–4th helpful. An exaggerated response to pin is fi ngers of both hands. A Tinel’s sign is elicited by called hyperpathia. tapping the volar surface of the wrist over the 5 . Diagnosis: Chronic infl ammatory demyelinat- median nerve. These positive signs support but ing polyneuropathy. This is associated with do not confi rm the diagnosis. Tingling in both Cases 183

arms and hands may indicate a compressive is slightly weaker. Right iliopsoas, quadriceps, and neuropathy and nocturnal paresthesias are virtu- anterior tibialis are 3/5, 4/5, and 4+/5, respectively. ally diagnostic of carpal tunnel syndrome. The left side is 3+/5, 4+/5, and 4/5, respectively. The 2. Polyneuropathy begins in the legs and thus can patient walks with a short stride and bilateral foot- be rejected. Examination shows no signs of a drop. Refl exes are 1+ and symmetrical, plantars are myelopathy such as hyperrefl exia, spasticity, fl exor and sensory testing is normal. or Babinski signs. Muscular atrophy may Questions : occur with AHC disease, radiculopathy or 1. Where is the lesion? neuropathy. Cervical radiculopathy is a con- 2. What is the differential diagnosis? sideration but the typical fi ndings of radicular 3. What fi ndings confi rm the diagnosis? pain and asymmetric refl exes are not present. Sensory symptoms exclude AHC disease. Case 6 Analysis : 3. Chest pain may occur with cervical radicul- 1. The patient has a painless, pure motor system opathy and, in rare instances, carpal tunnel disorder. He does not have concomitant signs syndromes. of cerebral disease such as aphasia, impaired 4. Carpal tunnel syndrome. Prior repetitive pro- cognition, visual fi eld defects or a history of voking activities such as typing or sewing are seizures. There are no signs of brainstem not required for the diagnosis. pathology such as crossed fi ndings or eye 5. The patient has arthritis affecting the proximal movement abnormalities. Typical fi ndings of interphalangeal joints, thus rheumatoid arthri- myelopathy such as spasticity, hyperrefl exia, tis is a consideration as it may cause carpal tun- and Babinski signs are not present. The cardi- nel syndrome. Other unusual specifi c etiologies nal symptom of polyradiculopathy, pain, is include hypothyroidism and amyloidosis. absent. There are no sensory symptoms or signs which usually accompany neuropathy. Case 6 A 45-year-old retired professional basket- Thus, the lesion is either in the neuromuscular ball player complains of progressive leg weakness junction or muscle. A purely motor neuropa- of over 3 years. For the past one year, he has not thy is a remote consideration. been able to run or play basketball with his two 2. The differential diagnosis must take into sons because he has been dragging his feet. For 6 account the duration of the illness, the tempo months he has had diffi culty making his overhead of progression and, particularly, the anatomic shots at the three-point line as they are falling short distribution of the pathology. of the basket. Morning and evening strength is the The initial diagnostic considerations are same. Climbing stairs requires extreme effort. Over inclusion body myositis (IBM), dermatomyo- the last few weeks, swallowing has been diffi cult. sitis (DM), polymyositis (PM), muscular dys- History Analysis : trophy (limb-girdle), and multifocal motor Dragging his feet implies weak anterior tibialis neuropathy with conduction block. Myasthenia musculature. Wrist fl exor weakness is suggested gravis is a remote possibility since fl uctuating since making overhead shots requires good strength especially related to time of day is strength of these muscles. Inability to climb stairs more typical. signifi es quadriceps/iliopsoas weakness and rais- 3 . Diagnosis : IBM. The diagnostic features ing the arms requires good deltoid strength. Thus include the distribution of weakness, proximal the illness begins with symptomatic distal weak- and distal, which are characteristic of IBM. ness but becomes generalized. Swallowing is often impaired. The CK is 800 U/L which rules out a motor neuropathy. Neurologic Examination : The EMG shows signs of an infl ammatory Blood pressure is 140/82; pulse is 90 and regular. myopathy and repetitive nerve stimulation Strength of deltoids, triceps, wrist fl exors, and reveals no abnormalities. This test may dis- arm pronation are 4/5 in both arms but the right side close evidence for myasthenia gravis. 184 9 Neuromuscular Diseases: Neuroanatomic and Differential Diagnoses

The muscle biopsy reveals intracellular amyloid 3. Can diabetes be the only underlying pathology? deposits, vacuoles in muscle fi bers and infl am- What word is used to explain the patient’s per- mation with CD8 lymphocytes. The diagnosis of ception of a vibration stimulus as painful? IBM is confi rmed. What disease is it associated with? Can the sensory signs (not symptoms) be caused by Brief Review : lumbar radiculopathy? DM. The CK is elevated up to 50 times normal. 4. Can one neurologic disease explain all of the It is associated with malignancy, vasculitis, car- fi ndings? diac involvement, and interstitial lung disease. Dysphagia, fever, and rash occur. The rash is a Case 7 Analysis : “heliotrope” rash with periorbital edema, pur- 1. No. The patient has normal peripheral pulses. plish discoloration of eyelids, sun sensitive rash, Additionally, paresthesias are not caused by and papular erythematous lesions over the vascular disease. knuckles. 2. Unilateral calf pain suggests lumbar radicul- PM. There is no rash. Dysphagia is common opathy and does not occur with neuropathy. and polyarthritis may occur. The CK is markedly Neuropathy typically causes bilateral foot dis- elevated up to 50 times normal. comfort such as paresthesias and burning sen- Vasculitis can occur with either DM or PM sations. Lumbar radiculopathy due to a and the disorders include Sjögren’s disease, scle- herniated disk usually causes pain which is roderma, rheumatoid arthritis, systemic lupus aggravated by prolonged sitting. When root erythematosus (SLE), and mixed connective tis- pain is provoked by walking, the term applied sue disease. is neurogenic claudication and it is character- istic of lumbar spinal stenosis. Normal straight Case 7 A 77-year-old man complains of severe leg raising is a common fi nding. right calf pain after walking one block. This has 3. No. Allodynia. This word is used to explain developed gradually over 6 weeks. There is no the evocation of pain with a nonpainful stimu- weakness but he has paresthesias in both feet. He lus. Allodynia is most often associated with has no calf pain when sitting or lying down. neuropathy. Sensory signs are caused by neu- Past medical history: The patient has a 50 ropathy, not lumbar radiculopathy. pack-year history of smoking, hypertension, and 4. No. The MRI shows severe lumbar spinal type II diabetes. Current medications are met- stenosis at L4–L5, right greater than left. This formin and lisinopril. explains the calf pain with exercise. Neurologic Examination : Final Diagnoses : Blood pressure is 130/86; pulse is 78 and regular. 1. Lumbar spinal stenosis with right L5 General physical examination is normal includ- radiculopathy. ing good peripheral pulses. 2. Diabetic neuropathy. Ankle refl exes are absent. Vibration percep- tion is absent at the toes and diminished at the Case 8 A 46-year-old woman complains of a ankles. When testing vibration, the patient per- 1-week history of leg weakness and droopy eye- ceives it as pain. There is decreased perception of lids. Four days ago she could no longer climb pin up to the ankles bilaterally. He makes errors stairs and began to drag both feet. For 2 days she with light touch on the toes. Straight leg raising is has been short of breath and has had diffi culty negative. swallowing liquids. Since yesterday she has required a walker and voiding is impaired. Questions : Past medical history includes a 20-year history 1. Is peripheral vascular disease the cause of his of insulin-dependent diabetes mellitus. She has pain? had paresthesias of the toes for several years and 2. Where is the lesion causing his pain? this has been attributed to her diabetic neuropathy. Cases 185

Neurologic Examination : hypertension and tachycardia often occurs Blood pressure is 150/110; pulse is 120 and with AIDP. regular. 3. Associated illnesses include Epstein–Barr The patient has right ptosis and is mildly dys- virus, Hodgkin’s disease, cytomegalic virus, pneic, but her oxygen saturation is normal. She mycoplasma pneumonia and about 20% of has moderate bilateral facial weakness. Head patients have had campylobacter jejuni fl exion is 4/5. There is generalized proximal and infection. Diseases which may mimic AIDP distal weakness in the range of 4 − 4+/5. Refl exes include porphyria, HIV, hypokalemic periodic are 1+ other than absent brachioradialis and ankle paralysis, tic paralysis, and toxins such as refl exes. Plantars are fl exor. Vibration perception lead, arsenic, hexacarbons, nitrofurantoin, and is absent at the toes and diminished at the ankles. organophosphates. There is distal sensory loss to pin and tempera- 4. An EMG reveals a severe demyelinating poly- ture in a stocking-glove pattern. neuropathy. Cerebrospinal fl uid shows a pro- tein of 146 mg/dl and 4 wbc/cu mm (so-called Questions : “albuminocytologic dissociation”). This is 1. Where is the lesion or where are the lesions? actually a common nonspecifi c fi nding and 2. From the history alone what symptom is a seldom of diagnostic import. major clue to the diagnosis? 5 . Diagnosis : AIDP or Guillain–Barré syndrome. 3. What are the diseases associated with this Treatment is plasma exchange and/or intrave- disorder? nous immunoglobulin. Prognosis for improve- 4. What diagnostic tests are indicated? ment is good over several weeks to months. 5. What is the diagnosis and current treatment approach? Case 9 A 36-year-old woman complains of severe pain on the left lateral thigh. The pain is Case 8 Analysis : intermittent, sharp, and often worse with move- 1. This patient has a rapidly progressive illness ment. The severity is 8/10. It occurs several times which affects the right levator palpebrae supe- per hour and each pain lasts 5–10 min. She has rioris innervated by the right 3rd nerve, 7th had low back pain for many years but without any nerve innervated muscles, 11th nerve innervated defi nite radiation down the legs. muscles (head fl exion) and generalized weak- ness. This weakness is most compatible with an Neurologic Examination : acute neuropathy or a disorder of the neuromus- The patient is 5 ft. 4 in. and weighs 213 lb. Blood cular junction (myasthenia gravis). There are no pressure is 140/90, pulse is 86 and regular. signs of cerebral disease such as dementia, The only neurologic abnormality is hypesthe- aphasia, seizure, or homonymous hemianopsia sia to pin and light touch over a well-defi ned nor are there “crossed” fi ndings or eye signs region over the anterolateral portion of the left typical of brainstem pathology. Myelopathy is thigh. Straight leg raising elicits low back pain at ruled out because of involvement of eyelids and 80° bilaterally. face. The distal sensory loss and absent refl exes MRI (lumbar) discloses two herniated disks, indicate a neuropathy, acute or chronic. L3–L4 and L4–L5. At this point the primary diagnostic consid- Questions : erations include myasthenia gravis superim- 1. Where is the lesion? posed on a diabetic neuropathy and an acute 2. What causes the pain? neuropathy such as Guillain–Barré syndrome 3. Do the MRI fi ndings help with regard to the (acute infl ammatory demyelinating polyneu- diagnosis? ropathy (AIDP)) superimposed on a diabetic neuropathy. Case 9 Analysis : 2. The patient has bladder dysfunction which 1. The presence of pain suggests radiculopathy. does not occur with myasthenia gravis. The neurologic defi cit, however, is in the dis- Additionally, autonomic dysfunction such as tribution of the lateral femoral cutaneous 186 9 Neuromuscular Diseases: Neuroanatomic and Differential Diagnoses

nerve of the thigh and loss of perception of 7. What physiologic systems are commonly light touch is rarely found with radiculopathy, involved in this disorder? certainly not in such a well-outlined form. 8. What is the differential diagnosis? 2 . Diagnosis: Meralgia paresthetica due to com- 9. What laboratory tests are indicated? pression of the lateral femoral cutaneous nerve 10. What consultation should be requested of the thigh as it passes under the inguinal promptly? ligament. Obesity may be a predisposing Case 10 Analysis : factor. 1. Ptosis indicates weakness of levator palpe- 3. No. The test was unnecessary and resulted in brae superioris. Masseter and temporalis needless expense, unfortunately. muscle weakness and atrophy results in the Case 10 A 22-year-old college student requests partially open mouth and contributes to the an evaluation for weakness of both hands. The gaunt appearance. Orbicularis oculi and oris weakness has been slowly progressive since his weakness are evident because of weak eye teen years. Over the last few years his eyelids closure and inability to smile. Weak head have tended to droop. He has had diffi culty swal- fl exion indicates weak sternocleidomastoid lowing over the past year and has had occasional muscles. Dysphagia is due to poor esopha- choking spells. His grades have declined despite geal contraction. putting forth maximum effort. 2. No. Similar weakness occurs but impaired Past medical history is remarkable for three cognition does not, nor does muscular episodes of momentary loss of consciousness atrophy. causing an abrupt fall. Afterwards he is cold, 3. Muscle. sweaty, and nauseated. The patient is adopted. He 4. Cerebrum in view of the history of cognitive does not smoke or use alcohol. dysfunction. 5. The diagnostic bedside tests are grip and per- Neurologic Examination : cussion myotonia. The patient is requested to Blood pressure is 120/60; pulse is 88 with fre- grip two fi ngers of the examiner’s hand for quent irregular beats. several seconds and then quickly release them, The patient has a receding hairline and sunken which the patient is unable to do. Percussion cheeks giving him a gaunt appearance. He exhibits myotonia is detected by tapping the thenar bradyphrenia but he has a Mini-Mental State Exam eminence with a refl ex hammer. The thumb score of 30/30. He has mild bilateral ptosis, severe remains fl exed at the metacarpal-phalangeal weakness of eye closure and is unable to smile. joint for a few seconds. Tongue myotonia can There is early cataract formation. His mouth is occasionally be demonstrated by placing the slightly ajar and there is wasting of temporalis mus- edge of the tongue blade on the midline of the culature. Head fl exion is 4/5, intrinsic hand muscu- tongue and tapping the upper edge gently. lature 4/5, anterior tibialis 4+/5. His gait is normal 6 . Diagnosis: Myotonic dystrophy. It is an auto- other than diffi culty with getting up on his heels. somal dominant disorder associated with a Refl exes and sensory examinations are normal. defective gene on chromosome 19. There is Questions : an unstable trinucleotide repeat sequence, 1. What muscles of the eyes, face, and neck are cytosine, thymine, and guanidine (CTG). weak? The severity of the symptoms is proportional 2. Can a disorder of the neuromuscular junction to the number of excess repeats. explain the clinical fi ndings? 7. Endocrine disorders are common such as 3. Where is the primary pathology? testicular atrophy and thyroid disease. There 4. Is there another neuroanatomic site affected? are cardiac conduction defects and gastroin- 5. What bedside test is diagnostic? testinal motility problems due to smooth 6. What are the diagnosis and the etiology? muscle involvement. Cases 187

8. The differential diagnosis includes myotonia 2. Abduction of the arm and pressure placed at congenita (Thomsen’s disease). This is an the biceps and deltoid insertions cause pain autosomal dominant painless disease. Patients and thus the pathology is local. have generalized myotonia associated with 3. It isn’t. It is an expensive unnecessary test. well-developed and occasionally hypertrophic 4 . Diagnosis : Biceps tendonitis. muscles in the legs with myotonia provoked 5. The vital signs. These must be taken on every by exertion after rest. Proximal myotonic myo- visit. The patient has atrial fi brillation and is pathy is manifested by proximal weakness, promptly referred back to her internist. myotonia, and . Facioscapulohumeral Case 12 A 52-year-old man complains of a dystrophy may have similar patterns of weak- 3-year history of fatigue and a 2-year history of ness but there is no myotonia. leg weakness. For the past one year he has been 9. CK, Holter monitor, and EMG. The CK is troubled by dry mouth, constipation, impotence, normal and the Holter monitor reveals both and paresthesias. atrial and ventricular arrhythmias. The EMG Past medical history includes chronic obstruc- discloses characteristic features of myotonia. tive pulmonary disease associated with a 40 10. Cardiology. The patient has already had three pack-year history of smoking. syncopal events most likely secondary to a serious ventricular arrhythmia. Neurologic Examination : Blood pressure is 130/92; pulse is 96 and regular. Case 11 A 56-year-old woman complains of Pupils are 3 mm with a 1+/4 reaction to light. neck, right shoulder, and upper arm pain of Strength of deltoids, infraspinatus, iliopsoas, and 2 months duration. She believes the pain radiates quadriceps is 4/5. When deltoids are repeatedly from the neck down to the posterior part of the examined over a period of 15 s, the strength right upper arm. The pain is aggravated when increases to 4+/5. Refl exes are 1+ except for lying on her right side and she is awakened by it absent ankle jerks. Sensory examination is normal. frequently. Her right arm feels weak. Past medical history: The patient has type II Questions : diabetes. Medication is metformin. 1. What symptoms and signs are major clues to the diagnosis? Neurologic Examination : 2. What bedside tests may be diagnostic? Blood pressure is 115/80; pulse is 106 and irregu- 3. Where is the lesion and what is the diagnosis? larly irregular. 4. Why does weakness occur? The neurologic examination is normal except 5. What additional investigations are indicated? that pain interferes with testing the strength of the right arm. Case 12 Analysis : MRI (cervical). There are narrowed interver- 1. There is evidence of autonomic nervous sys- tebral foramina bilaterally at C5–C6 and on the tem dysfunction because of impotence, dry right side at C6–C7. mouth, and constipation. The poor pupillary light reaction is another sign of autonomic Questions : nervous system dysfunction, parasympathetic 1. What symptom suggests the source of the pain? in this case. 2. What bedside tests are diagnostic? 2. Repetitive use of weak muscles paradoxically 3. How is the MRI scan helpful? increases their strength. 4. What is the diagnosis? 3. The lesion is at the neuromuscular junction. 5. What part of this entire examination is most Diagnosis : Lambert–Eaton syndrome. important? 4. This is a presynaptic disorder with decreased Case 11 Analysis : quantal release of acetylcholine due to autoan- 1. Pain when lying on the right side suggests that tibodies to neuronal voltage-gated calcium the pathology is in the right shoulder. channels, P–Q type. Impairment of calcium 188 9 Neuromuscular Diseases: Neuroanatomic and Differential Diagnoses

channel function leads to reduced release of 2. The prototypical sign is the . acetylcholine. Localized fasciculations are often found with 5. An EMG with rapid repetitive stimulation radiculopathies but, if widespread, AHC dis- results in a markedly facilitated response, just ease is nearly certain. Plexus and nerve pathol- the opposite of myasthenia gravis. There is a ogy rarely cause fasciculations. Neuromuscular blood test to measure the level of autoantibod- junction and muscle diseases do not cause fas- ies. About 50% of patients have an occult ciculations. Weakness and atrophy are com- malignancy, primarily small cell lung cancer. mon with disease of the AHC. Thus, an intensive investigation for underly- 3. If a Babinski sign is found, amyotrophic lat- ing malignancy is required. eral sclerosis (ALS) would be the presumptive diagnosis because of concomitant upper and Case 13 A 28-year-old man requests an evalua- lower motor neuron signs. Limb girdle mus- tion because of progressive, painless weakness of cular dystrophy would be considered in the all extremities of over 4 years. He has particular absence of fasciculations. diffi culty raising his arms to reach for objects 4. EMG-NCV. over his head. For the last 4 months he has been 5 . Diagnosis: Spinal muscular atrophy. This is unable to climb stairs. He now needs assistance an inherited disorder with numerous varia- to get out of a chair. tions occurring in adults over age 20. The Past medical history is remarkable for insulin- inheritance can be autosomal recessive, auto- dependent diabetes mellitus since his teens. He somal dominant or X-linked recessive. Spinal has always had good control of his diabetes. muscular atrophy occurring in patients ages There is no family history of muscular disease. 2–20 is called Kugelberg–Welander disease. Neurologic Examination : In infancy the name given is Werdnig– Blood pressure is 120/70; pulse is 80 and Hoffman’s disease. regular. 6. There is no treatment. The prognosis for There is wasting and atrophy of shoulder and Werdnig–Hoffman’s disease in infancy is less thigh muscles. There is symmetrical 4/5 weakness than 2 years; Kugelberg–Welander in child- of deltoids, triceps, infraspinatus and supraspina- hood is 2–40 years; spinal muscular atrophy in tus. There is symmetrical 4+/5 weakness of gluteus adults is a normal life span. maximus, iliopsoas, and quadriceps musculature. Case 14 (Complex) A 54-year-old woman com- Fasciculations are frequent in triceps, deltoid, and plains of unsteadiness for 3 months. This has quadriceps muscles. Refl exes are barely elicitable been steadily progressing in severity. She adds a and sensory testing is normal. history of painful burning and prickly sensations Questions : in both feet. She has fallen a few times when 1. Where is the lesion? entering a dark room. Her hands feel clumsy but 2. What specifi c fi nding localizes the lesion? strength is unimpaired. She has no other illnesses 3. If Babinski signs were present, what would be and prides herself on taking no medications. She the presumptive diagnosis? If fasciculations has never had surgery. were not present what diagnosis would be Recent laboratory data includes a normal considered? CBC, FBS, liver functions, TSH, BUN, creati- 4. What test confi rms the diagnosis? nine, electrolytes, calcium, phosphorus, ANA, 5. What is the name of the disease and its ESR, and serum protein electrophoresis. variations? Neurologic Examination : 6. What is the prognosis? Blood pressure is 95/60; pulse is 88 and regular. Case 13 Analysis : There is 4+/5 strength of interossei and iliop- 1. The lesion is in the AHC, the lower motor soas musculature. She has moderate heel-to-shin neuron. ataxia and mild fi nger-to-nose ataxia. Gait is Cases 189 slightly wide-based and she has a positive a strong association with an occult malignancy, Romberg. Refl exes are 1+, symmetrical except probably 90%, but often this neuropathy pre- for absent biceps and ankle refl exes. Plantars are cedes the discovery of cancer by 1–2 years. fl exor. Sensory examination discloses absent A paraneoplastic panel for neuropathy should vibratory perception at toes, ankles, fi ngers, and be obtained, followed by a search for the occult wrists. Vibration is perceived as painful at elbows malignancy. Surgical resection of the malig- and knees. Position sense is impaired at toes, nancy can ameliorate the symptoms. ankles, and fi ngers. The patient expired from metastatic carcinoma A lumbar puncture reveals a CSF protein of of the breast. Small cell lung cancer is the most 107 mg/dl and 14 lymphocytes/cu mm. common etiology. Others include lymphoma, The patient expired 1 year later and an autopsy ovarian and prostate carcinomas. Pathologic was obtained. studies disclosed infl ammation in the dorsal root ganglia. Questions : 1. Where is the lesion? Case 15 A 42-year-old male truck driver requests 2. How can limb ataxia be explained? an evaluation for left hand weakness which has 3. What is the name for a vibration stimulus per- been progressive for over 6 months. He also has ceived as painful? occasional tingling sensations affecting the left 4. Why does the patient fall in the dark and why are 5th fi nger. He does mainly long distance her hands clumsy? Are there other reasons for hauling. falling in darkness if this is the only symptom? Neurologic Examination : 5. Does the rapid progression suggest the Blood pressure is 140/86; pulse is 88 and diagnosis? regular. 6. What could be the diagnosis and what addi- There is atrophy between the thumb and the tional tests should be performed? Could treat- index fi nger as well as the hypothenar eminence ment have been helpful? on the left side. He has mild weakness of exten- Case 14 Analysis : sion of the terminal phalanx of the left 4th and 1. Neuropathy. 5th fi ngers, moderate weakness of abduction and 2. Limb ataxia is commonly observed in patients adduction of the 2nd–5th fi ngers. There is mild with severe position sense loss. weakness of wrist fl exion medially and of thumb 3. Allodynia is the term used when a usually adduction. There is only mild numbness to pin painless stimulus is painful. over the 4th and 5th fi ngers. 4. Vision is necessary to maintain balance when Questions : patients have position sense loss. Loss of posi- 1. What muscles are involved? tion sense in the fi ngers interferes with dexter- 2. What questions should be asked of the patient ity. Vestibular dysfunction will also cause to establish the precise etiology of the instability in darkness since vision suppresses weakness? nystagmus of peripheral vestibular origin. 3. What is the anatomic differential diagnosis? Pure cerebellar disease rarely produces a posi- 4. What movement can be an aggravating tive Romberg and this is a controversial topic. factor? When it occurs, it is most likely due to inter- 5. What is the diagnosis? Can fractures cause ruption of vestibulocerebellar pathways. these symptoms and signs? What is a tardy 5. Yes. The rapid development with primarily ulnar palsy? sensory features suggests a paraneoplastic sensory neuropathy. Case 15 Analysis : 6 . Diagnosis : Paraneoplastic sensory neuropa- 1. Adductor pollicis, palmar and dorsal interos- thy. In this case it is due to the anti-Hu anti- sei (adduction and abduction, respectively), body, a neuronal antinuclear antibody. There is 3rd and 4th lumbricales, fl exor carpi ulnaris. 190 9 Neuromuscular Diseases: Neuroanatomic and Differential Diagnoses

2. Over the last 6 months has there been any Questions : change in your work habits? “Yes, I lost the air 1. Is there more than one lesion site? conditioning in my truck and have to leave the 2. Why was a neostigmine test performed? windows open.” Where is your left arm? “I 3. Is it common to have normal arterial blood lean it on the window ledge.” gases yet a decreased vital capacity? 3. Ulnar nerve vs. medial cord of the brachial 4. What are the major diagnostic clues for plexus vs. C8T1 radiculopathy. another disease process? 4. Frequent fl exion at the elbow aggravates the 5. What workup is indicated? neuropathy because it tightens the fl exor carpi 6. What is the diagnosis or what are the ulnaris aponeurosis, thus reducing the size of diagnoses? the cubital tunnel. 5 . Diagnosis: Ulnar neuropathy due to compres- Case 16 Analysis : sion of the nerve at the cubital tunnel. Tardy 1. Yes. The patient clearly has myasthenia gravis, ulnar palsy refers to an old elbow fracture with a neuromuscular junction disorder. The lack bony overgrowth causing ulnar nerve of response to either edrophonium or neostig- compression. mine regarding muscle strength suggests an additional disease process. Proximal weak- Case 16 A 60-year-old man requests an evalua- ness without fasciculations, refl ex, or sensory tion for a droopy left eyelid. He denies any other abnormalities is compatible with a myopathy. problems. On review of neurologic symptoms he 2. A neostigmine test is more often useful to assess acknowledges that he has been “aging” by which muscular strength since the patient can be exam- he means that his strength has decreased. He is a ined more carefully every 30 min up to 2 h. The paralegal and often has to retrieve large volumes edrophonium test is especially helpful for of tax laws from shelves above his head. He now patients with ptosis or eye movement weakness. requires assistance to do this. He used to climb 3. Yes. Testing the vital capacity is a direct fi ve fl ights of stairs to his offi ce for exercise but assessment of chest musculature integrity. now manages just one fl ight and then takes the Arterial blood gases are not reliable markers elevator. He is easily fatigued and short of breath for evaluating this neuromuscular weakness. with minimal exertion. They are often normal, immediately prior to a Past medical history is remarkable for hyper- respiratory arrest due to muscle fatigue. tension for which he takes atenolol. 4. The patient has hypertension and tachycardia Neurologic Examination : despite treatment with a beta blocker, atenolol. Blood pressure is 146/96; pulse is 112 and Furthermore, he has proximal muscle weakness regular. unimproved with anticholinesterase agents. He has moderate left ptosis, 4/5 strength of 5. Workup includes a CK, TSH, free T-4, ESR, head fl exion, deltoids, triceps, and infraspinatus ANA, CT chest with and without contrast, and bilaterally. Iliopsoas is 4+/5 on the right and 4/5 an EMG with repetitive nerve stimulation. on the left. Refl exes are 1+ with fl exor plantar The CK is 60 U/L, TSH is 0.2 µU/ml, free T-4 responses. Sensory examination is normal. is 3.8 ng/dl, ESR is 10 mm/h and ANA is neg- Laboratory data: FBS is 115 mg/dl. Vital ative. CT of the chest shows no thymoma capacity is 1.5 L. Arterial blood gases are which is present at about 15% of patients with normal. myasthenia gravis. The EMG shows typical An edrophonium test restores eyelid function fatigue with repetitive nerve stimulation and is to normal and vital capacity increases to 3 L but diagnostic of myasthenia gravis. muscle strength is unchanged. A neostigmine test 6 . Final Diagnosis : is performed later and muscle strength remains (a) Myasthenia gravis. unchanged. (b) Hyperthyroidism with myopathy. Cases 191

Case 17 A 72-year-old man complains of burn- 2. The patient’s blood pressure was dropping ing feet and “passing out.” Over the past 5 years consistently over 5 min. The readings should the patient has had intermittent, prickly sensa- have been continued until a minimum of tions on the soles of both feet. He ignored them 10–12 min had elapsed since he reports being up until 3 months ago when they became painful. symptomatic after 10 min of walking. When taking his usual 1-h walk in the park he has 3 . Diagnosis: Neuropathy plus autonomic found it necessary to sit down, invariably after neuropathy. 10 min elapsed, due to pain and feeling faint. He 4. Diabetes mellitus. Standard accepted tests are reports that intense pain usually precedes the hemoglobin A1C and a 2-h glucose tolerance faintness. On two occasions over the last 2 weeks, test. The hemoglobin A1C is 7.5%. The FBS he felt faint with only minor pain. He had the is 102 mg/dl and the 2-h glucose level is sudden onset of feeling cold, clammy, nauseated 220 mg/dl. and then fell. On these occasions there was com- 5. Amyloidosis, paraneoplastic syndromes, plete loss of consciousness for several seconds Guillain–Barré disease, porphyria, HIV, and without subsequent confusion. pandysautonomia. Additional considerations Past medical history: The patient has asthma include toxic drugs, especially vincristine, but currently takes no medications. amiodarone, and cisplatin as well as toxic sub- stances such as thallium, N-hexane, arsenic Neurologic Examination : and mercury. Hereditary sensory and auto- Blood pressure: supine 120/72, sitting 115/72, nomic neuropathies are rare etiologies. standing at 1 min 110/66, standing at 3 min 106/60, and standing at 5 min 100/56. The heart Case 18 A 27-year-old man complains of severe, rates are 72, 76, 78, 80, and 80, respectively. “searing,” right arm pain and weakness. The pain Strength of anterior tibialis is 4+/5 bilaterally began 1 week ago and, after 4 days had elapsed, he and ankle refl exes are absent. Vibration percep- noted weakness around the right shoulder. The pain tion is absent at toes and ankles and a few posi- began in the upper part of the shoulder blade, tion sense errors are made at the toes bilaterally. extended up to the neck and radiated down the outer There is distal sensory loss to pin and tempera- aspect of the upper arm. The severity was 8/10. ture in all extremities. Since the weakness developed, the pain has dropped to a level of 5/10. Movement aggravates the pain. Questions : 1. What is the most likely cause of this patient’s Neurologic Examination : syncope? Blood pressure is 120/70; pulse is 88 and 2. What part of the examination is incomplete? regular. 3. What is the single anatomic diagnosis? The strength of right deltoid, infraspinatus, 4. What is the most likely etiology and what tests and supraspinatus is 3/5. Strength of right biceps confi rm it? and brachioradialis is 4/5. There is winging of the 5. What is the differential diagnosis of neuropa- right scapula. Right biceps and brachioradialis thy with autonomic involvement? refl exes are absent. There is decreased perception of pin in a small round patch on the outer aspect Case 17 Analysis : of the right upper arm in the distribution of the 1. Delayed orthostatic hypotension. The etiology axillary nerve. of syncope is either a cardiac arrhythmia or severe hypotension. The former diagnosis is Questions : sporadic. The latter is more likely to be uni- 1. Where is the lesion? Why? What nerves inner- form with regard to the time it occurs after vate these muscles? assuming an upright posture. This is consis- 2. What is the signifi cance, if any, of the motor tent with the patient’s history. signs far exceeding any sensory loss? 192 9 Neuromuscular Diseases: Neuroanatomic and Differential Diagnoses

3. What is the most common etiology? Is it ever Questions : bilateral? 1. Where is the lesion? 4. What is the differential diagnosis? 2. Is the history of insulin-dependent diabetes mellitus pertinent? Case 18 Analysis : 3. What part of the history determines the seri- 1. Brachial plexus, upper trunk. It affects several ousness of the diagnosis? nerves, the axillary, long thoracic, suprascapu- 4. What is the differential diagnosis? lar, radial, and musculocutaneous. These 5. What is the diagnosis? innervate, respectively, the deltoid, serratus anterior, infra- and supraspinatus, brachiora- Case 19 Analysis : dialis, and biceps muscles. 1. This lesion affects both median and ulnar nerve 2. This is characteristic of brachial plexopathies. innervated muscles. Hence the anatomic diag- 3. A post-infectious disorder is the most com- nosis is brachial plexopathy, medial trunk. mon etiology in an outpatient practice. It has 2. No. been called Parsonage–Turner syndrome and 3. The duration of 4 years indicates a benign it can rarely be bilateral. process. 4. Associated diseases are diabetes, polyarteritis 4. The differential diagnosis includes neoplastic nodosa, and SLE. infi ltration, complication of radiotherapy, repet- (a) Post-vaccinal. Tetanus, infl uenza, typhoid- itive trauma, and the neurogenic thoracic outlet paratyphoid, and DPT. syndrome. Diabetes, vasculitis, post-infectious (b) Post-infectious. Epstein–Barr, cytomega- and post-vaccinal etiologies are excluded lic inclusion virus, infl uenza, diphtheria, because of the slow progressive course. typhoid, and typhus. 5 . Diagnosis: Neurogenic thoracic outlet syn- (c) Trauma. Causes include stretching, com- drome. This is most often due to a rudimen- pression, traction, direct injury, surgical tary cervical rib with an elongated transverse positioning, and intravenous drug abuse. process of the C7 vertebra. A fi brous band (d) Neoplasm. The etiology may be infi ltration arises from the tip of this rib, attaches to the of a malignant neoplasm originating from 1st thoracic rib, and compresses the medial the apex of the lung (Pancoast’s tumor). trunk of the brachial plexus. (e) Complication of radiotherapy. This syndrome is an extremely rare disorder but commonly diagnosed by physicians unfamil- Case 19 A 32-year-old woman complains of iar with the usual clinical signs. Wasting occurs pain in the left forearm for 4 years. Over the last and is most often prominent in median nerve 10 months she has noticed intermittent tingling innervated muscles and sensory symptoms are of the left 4th and 5th fi ngers. Over recent weeks, often present in the ulnar nerve distribution. The the strength of her left grasp has decreased. diagnosis of this disorder due to sensory symp- Past medical history is remarkable for insulin- toms alone is frequently made and rarely accu- dependent diabetes mellitus. rate. Numerous fi rst thoracic ribs have been Neurologic Examination : removed without amelioration of symptoms. Blood pressure is 100/60; pulse is 64 and regular. There is a vascular thoracic outlet syndrome, There is wasting of muscles of the left thenar equally rare, due to compression of the subcla- eminence. Strength of APB, fl exor digitorum vian artery by a fully developed cervical rib, profundus (2nd–5th digits) and fl exor pollicis which may result in post-stenotic dilatation of the longus are 4/5. Refl exes are normal. There is subclavian artery with thrombus formation and decreased pinprick sensibility over the dorsal and distal embolization. volar surfaces of the 4th and 5th fi ngers and volar Finally, there are instances of combined vas- surface of the adjacent medial hand. cular and neurologic symptomatology without Cases 193 defi nite signs, a nebulous concept. It may occur longus, and dorsifl exion of the 2nd–5th toes is in female patients with droopy shoulders and extensor digitorum longus. These are all inner- long necks resulting in stretching of the brachial vated by the deep peroneal nerve. Eversion of plexus. the foot is peroneus longus and brevis and decreased perception of pin, as described, are Case 20 (Complex) A 33-year-old man com- both mediated by the superfi cial peroneal plains of left foot weakness after spending an nerve. The refl ex at the ankle is intact since the evening gambling in Las Vegas 5 days previously. efferent response is mediated by the posterior He noticed the weakness after getting up from the tibial nerve which supplies the gastrocnemius. poker table where he had been sitting for 3 straight The following muscles and sensory loss are hours. He now has a dropped left foot. all radial nerve supplied. Right elbow exten- Last night, after several cocktails, he fell sion is triceps muscle, right elbow fl exion mid- asleep next to his girlfriend. Today he complains way between pronation and supination is of inability to raise his right hand. brachioradialis, extension of wrist to radial Past medical history: Several years ago, the side is extensor carpi radialis longus, extension patient was diagnosed with a “dropped foot” and of wrist to ulnar side is extensor carpi ulnaris, recovered without residual weakness after 3 extension of distal phalanx of thumb is exten- months. He is not certain which foot was sor pollicis longus, loss of refl exes are those of involved. the triceps and brachioradialis both innervated Neurologic Examination : by the radial nerve. The decreased perception Blood pressure is 138/88; pulse is 76 and regular. of pin over the dorsum of the thumb, index, Dorsifl exion of the left foot is 2/5. Dorsifl exion and middle fi ngers as well as proximal adja- of the left big toe is 2/5. Dorsifl exion of the 2nd– cent hand is via the superfi cial radial branch. 5th toes is 3/5. Eversion of the left foot is 2/5. 2. How does he sit at the card table? “I cross my Refl exes are normal and there is decreased per- legs.” Was his girlfriend on his right or left side ception of pin over the left lateral calf and dor- and where was her head? “Her head was on my sum of the foot. right shoulder.” Does anyone in the family Right elbow extension is 2/5. Flexion at the have similar problems? “Yes, my father.” elbow with the forearm between the pronated and 3 . Diagnosis : Hereditary neuropathy with liabil- supinated position is 2/5. Extension of the wrist ity to pressure palsies. This is an autosomal to the radial side is 2/5. Extensors of the fi ngers at dominant trait, presumed secondary to a con- the metacarpal phalangeal joints is 1/5. Extension genital abnormality of myelin which increases of the wrist to the ulnar side is 2/5 and exten- susceptibility to pressure. This disorder sion of the distal phalanx of the thumb is 1/5. resulted in common peroneal and radial neu- Refl exes are 2+ except for absent brachioradialis ropathies. Genetic testing for this disorder is and triceps on the right side. There is decreased available. perception of pin on the dorsum of the thumb, Case 21 Case 21 : An 88-year-old male retired index, and 3rd fi ngers as well as the proximal philosophy professor complains of diffi culty portion of the adjacent hand. swallowing liquids for 4 months. The dysphagia Questions : began abruptly and has persisted. It is unchanged 1. What muscles and nerves are involved? throughout the day. Over the last month he has 2. What additional diagnostic information may had diffi culty climbing stairs and replacing heavy be obtainable? books on shelves overhead. 3. What is the likely diagnosis? His past medical history includes rheumatoid arthritis, symptomatic for 15 years with early Case 20 Analysis : morning stiffness and migrating polyarthralgia. 1. Dorsifl exion of the foot is anterior tibialis, dor- This was diagnosed 1 year ago. After therapeutic sifl exion of the left big toe is extensor hallucis trials with a few medicines, he was placed on 194 9 Neuromuscular Diseases: Neuroanatomic and Differential Diagnoses prednisone 6 months ago and has improved with right and left nucleus ambiguus. Motor neuron a dose of 12 mg per day. He has hypertension disease is not of sudden onset and usually well-controlled with losartan. declares itself with fasciculations. Myasthenia gravis seldom begins abruptly but dysphagia Neurologic Examination : is often an early symptom. Although it often Blood pressure is 135/85; pulse is 60 and worsens later in the day, this is not required regular. for the diagnosis. Myopathies such as ocu- Palate movements are sluggish but gag is lopharyngeal dystrophy will not develop intact. Repeated checks of deltoid strength yield acutely. Myasthenia gravis must be the sus- 4/5 strength on the 12th try. Iliopsoas and quadri- pected diagnosis. ceps are 4+/5. Refl ex and sensory examinations 2. The CK is normal and antiacetylcholine recep- are normal. tor antibodies are elevated. Thyroid functions Questions : are normal. Pyridostigmine restores the patient 1. What are the primary diagnosis and the dif- to normal except for equivocal benefi t for leg ferential diagnosis? strength. 2. What tests are performed? 3. The patient is treated with pyridostigmine 3. What treatment is initiated? 60 mg. One hour ac t.i.d. 4. The CK is still normal. The EMG shows a few First follow-up visit 1 month later: scattered myopathic units in proximal muscles. The patient is pleased with his new treatment 5. The patient’s skin is fragile resulting in scat- regimen because of normal swallowing and the tered ecchymoses and he now has osteoporo- return of arm strength to normal. He has, how- sis. Both of these developments are ever, increasing diffi culty climbing and descend- complications of long-term treatment with ing steps although he can still manage one fl ight prednisone, usually with doses greater than of stairs. Examination is normal except for 4/5 10 mg/day. strength of iliopsoas and quadriceps. The treat- ment regimen is readjusted. Final Diagnoses : Second follow-up visit 2 months later: 1. Myasthenia gravis. One month ago the patient tripped and frac- 2. Steroid myopathy. tured his left hip which was pinned. Osteoporosis was diagnosed at that time. He can no longer Case 22 A 54-year-old male retired baseball climb a fl ight of stairs. player is admitted to the hospital with fever, pro- Examination reveals mottled skin with several ductive cough, and cachexia. He was diagnosed ecchymoses. There is 4−/5 strength of iliopsoas with carcinoma of the tongue 6 months ago and and quadriceps muscles. treated by hemiglossectomy and node resection. He has been anorectic since then and dropped Questions : 40 lb from 170 to 130. 4. What tests were ordered? Which were Past medical history is remarkable for a abnormal? 30-year history of using chewing tobacco daily. 5. What conditions support the diagnosis? Neurologic Examination : Case 21 Analysis : Temperature 101.5° F blood pressure is 140/100; 1. The anatomic differential diagnosis related to pulse is 110. dysphagia is a brainstem lesion (medulla- The patient is dehydrated, has a supple neck nucleus ambiguus), motor neuron disease, and there are rales, dullness to percussion and neuromuscular junction disorder, and myopa- decreased fremitus at the right base. The heart is thy. The sudden onset raises a suspicion of a enlarged. The liver is 4 cm below the right costal vascular event affecting the brainstem. But margin on deep inspiration and there is 3+ pedal there is no single artery which supplies both edema. Neurologic examination is normal. Cases 195

Laboratory data shows a white count of differential diagnosis of macrocytosis includes

22,000/cu mm with 92% neutrophils. Gram defi ciencies of vitamins B1 , B12, and folate as stain of sputum reveals a Gram-positive well as hypothyroidism, myelodysplasias and diplococci. drug toxicity e.g. phenytoin and azathioprine. The patient is admitted to the Intensive Care 5 . Final Diagnosis : Neuropathy, nutritional, Unit. After rehydration and treatment for pneu- associated with beriberi syndrome. The etiol- mococcal pneumonia, he recovers over 1 week. ogy is thiamin (vitamin B1 ) defi ciency. An additional diagnosis of congestive heart fail- ure is made and he is transferred to a rehabilita- Case 23 A 37-year-old woman complains of tion unit because of cachexia, malnutrition, and numbness, tingling, and pain in both feet and the dyspnea on exertion. On the rehabilitation unit he fi ngertips for 1 year. The pain has a needle-like complains of poor balance and falls in the shower quality. Three months ago she noted dimming of while washing his hair. vision with the right eye alone. Color perception Examination reveals moderately severe heel- was impaired. She has diffi culty closing the right to-shin ataxia, inability to perform tandem gait eye. For 2 months she has had tenderness and and a positive Romberg. He has absent position weakness of her shoulder muscles and joint pain sense at toes and ankles, absent vibration sense involving knees, elbows, and wrists. from the knees down and at the fi ngers. There is Past medical history includes a diagnosis of distal sensory loss to pin and temperature in all arthritis for the past 8 years, the etiology of which extremities and impaired light touch over the is unknown. toes. Refl exes in the legs are absent. General physical examination reveals a blood pressure of 130/80 and a regular pulse of 72. Questions : There is cervical adenopathy and an enlarged 1. Where is the lesion? liver which is 3 cm below the right costal margin. 2. Are all the neurologic abnormalities consis- There is also splenomegaly. tent with this lesion location? 3. What is the cause of the acute disorder? Neurologic Examination : 4. What routine blood tests and medical disorder The patient has a central scotoma OD to small provide support for this diagnosis? red targets and a visual acuity of 20/25. Optic 5. What is the diagnosis and what treatment disk margins are blurred OD with two splinter restores the patient to normal in 1 month? hemorrhages at the disk margin. There is an affer- ent pupillary defect and weakness of orbicularis Case 22 Analysis : oculi OD. Deltoids are weak with strength of 1. The patient has a neuropathy because of distal 4−/5 on the right and 4+/5 on the left along with sensory loss and absent refl exes in the legs. localized tenderness on the right side. Anterior 2. Yes. Heel-to-shin ataxia and poor tandem gait tibialis strength is 4/5 bilaterally. Ankle refl exes, are commonly present in patients with large brachioradialis and biceps are all absent. There is fi ber neuropathies involving vibration, posi- absent vibratory perception at the toes and dimin- tion, and touch. Cerebellar system involve- ished vibration perception at the ankles and fi n- ment remains a consideration, however. gers. Pinprick and temperature perception are 3. This patient was malnourished, cachectic, and diminished distally in all extremities. given intravenous fl uids without vitamins. This further depleted his already defi cient Questions : stores of vitamins. He could be assumed to 1. Where are the lesions? 2. What physical fi ndings are especially helpful have vitamin B 1 defi ciency as he has signs of beriberi syndrome, congestive heart failure, in making the diagnosis? and polyneuropathy. 3. What is the differential diagnosis? 4. A subsequent review of his laboratory data 4. What tests should be ordered? is signifi cant for macrocytic blood indices with 5. What is the diagnosis and what are the other an MCV of 105 Fl and MCH of 35 pg. The manifestations of this disease? 196 9 Neuromuscular Diseases: Neuroanatomic and Differential Diagnoses

Case 23 Analysis : Shortly after admission to the Intensive Care 1. The patient has an optic neuritis because of Unit he developed urosepsis and required treat- blurred disk margins, splinter hemorrhages, ment for Gram negative septicemia. One week and a central scotoma associated with an affer- later he developed ventricular tachycardia and ent pupillary defect. It is not papilledema became hypotensive on one occasion. Renal which is bilateral and does not affect central function subsequently declined as the BUN and vision. Weakness of orbicularis oculi indicates creatinine rose to 80 and 3.1 mg/dl, respectively. 7th nerve involvement. Tenderness and weak- After transfer back to a fl oor unit, he com- ness of deltoids indicates a myopathy. plained of weakness and was unable to walk Additionally, she has a sensorimotor neuropa- without support. thy because of distal weakness and impaired perception of vibration, pin, and temperature. Neurologic Examination : 2. Cervical adenopathy and splenomegaly raise The patient is disoriented to year and place. He consideration of sarcoidosis and lymphoma makes errors with short-term recall. There is 4/5 since both disorders could give rise to optic strength of deltoids, triceps, sternomastoids, and neuropathies, cranial neuropathies, neuropa- iliopsoas musculature. Anterior tibialis is 4−/5 thy, and myopathy. bilaterally. Brachioradialis, knee, and ankle jerks are absent. Vibration perception is absent at the 3. This is a multisystem disorder suggesting sev- toes, ankles, and knees. Position sense errors are eral diagnoses. These include HIV, lymphoma, made at the toes. There is distal sensory loss to Lyme disease, sarcoidosis, and vasculitis such pin and temperature in all extremities. as SLE, polyarteritis nodosa, and Sjögren’s syndrome. Questions : 4. A screening workup would be extensive and 1. Can a lesion in one anatomic site explain the include CBC, ESR, RA factor, serum protein neurologic abnormalities? electrophoresis, CK, renal and liver functions, 2. What is the differential diagnosis? What is the electrolytes, angiotensin converting enzyme diagnosis? level (for sarcoidosis), calcium, HIV, and 3. What underlying illnesses predispose the Lyme disease screening. A chest X-ray is patient to this disorder? essential. There was an increased alkaline 4. What is the prognosis? phosphatase of 250 U/L, increased gamma globulins, elevated CK of 300 U/L, increased Case 24 Analysis : ALT, AST to 60 and 70 U/L, and an abnormal 1. No. The patient has signifi cant proximal weak- chest X-ray which disclosed prominent hilar ness indicating the likelihood of a myopathy. adenopathy. An EMG-NCV disclosed fi ndings The distal weakness and sensory loss point to of neuropathy and myopathy. An MRI (brain neuropathy. Moreover, his impaired cognition and orbits) was normal. A lymph node biopsy indicates bilateral cerebral dysfunction, evi- revealed noncaseating granulomas. dently due to hypotension and an ischemic 5 . Final Diagnosis : Optic neuropathy, facial encephalopathy. neuropathy, polyneuropathy, and myopathy 2. The main differential diagnoses to consider in a due to sarcoidosis. patient who is critically ill, particularly with urosepsis, includes critical illness polyneuropa- Case 24 A 62-year-old man is admitted to the thy, critical illness myopathy, and a neuromus- Intensive Care Unit after a heart transplant. He cular junction disorder due to the use of had been suffering from a viral cardiomyopathy nondepolarizing neuromuscular blocking agents and congestive heart failure. (e.g., gentamycin). An acute cervical myelopa- His past medical history is signifi cant for thy can cause a quadriparesis and temporarily hypertension and chronic renal insuffi ciency with reduced refl exes. Metabolic abnormalities such a BUN of 50 mg/dl and a creatinine of 1.8 mg/dl. as hypokalemia, hypophosphatemia, and hyper- Cases 197

magnesemia should be ruled out. The sensory Case 25 Analysis : fi ndings in this case, however, are most compat- 1. Not likely. The relentlessly progressive neu- ible with neuropathy. Proximal muscle weak- ropathy despite good diabetic control would ness indicates a myopathy. Brainstem pathology be an exceptional manifestation of diabetes. can be excluded because of the absence of High arches and hypertrophic nerves do not crossed fi ndings, cranial nerve involvement, develop. and abnormal eye movements. 2. Wasting of distal musculature, high arch (pes Final Diagnosis : Critical illness neuropa- cavus), enlarged hard nerves are all character- thy and myopathy as well as an ischemic istic abnormalities of Charcot–Marie–Tooth encephalopathy. disease (CMT). 3. Sepsis, commonly urosepsis, and multiorgan 3. No. Although 50–70% of CMT cases are auto- failure are the underlying associated somal dominant, there are autosomal reces- illnesses. sive and rarely X-linked forms. 4. The prognosis is good when the medical ill- 4. Hereditary sensory and motor neuropathy. nesses are rapidly reversible. (a) HSMN 1 CMT 1 demyelinating. Case 25 A 28-year-old man complains of numb, (b) HSMN 2 CMT 2 axonal. weak feet. This has been a steadily progressive (c) HSMN 3 CMT 3 severe hypertrophic problem of over 10 years. He has been unable to run demyelinating neuropathy beginning in for 7 years. He reports that his foot and hand mus- infancy. This is known as Déjérine–Sottas cles are “shrinking.” After completing his station- disease. ary bicycle exercises he notes twitching in his calf Case 26 (Complex) A 56-year-old man com- muscles and he has severe cramps in his calves. plains of numbness and tingling of the feet asso- Past medical history is remarkable for insulin- ciated with unsteadiness. This began 6 months dependent diabetes mellitus beginning at age 12. ago and is progressively worsening. For the past It is under good control. His parents and three month he has had lancinating pains in the soles of siblings have no neuromuscular disorders. both feet which are nearly disabling. Neurologic Examination : The past medical history is negative and he Blood pressure is 120/75; pulse is 84 and regular. takes no medicines. Strength of interossei, APB, opponens polli- Neurologic Examination : cis, extensor pollicis longus, anterior tibialis, Blood pressure is 140/90; pulse is 72 and gastrocnemius, peroneus longus, and brevis mus- regular. cles are all 4/5. The patient has high arches. There He has normal strength and refl exes are 1+ is wasting of the small muscles of the feet and and symmetrical. Vibration perception is impaired hands, wrists, and ankles. An enlarged, hardened at toes and ankles and position sense is impaired ulnar nerve is palpated just above the left elbow at the toes. Light touch errors are made at the tips adjacent to the medial epicondyle. Ankle, knee, of the toes. brachioradialis, and biceps refl exes are absent. Vibration sense is impaired at the toes. Questions : 1. What nerve fi bers are involved in this Questions : neuropathy? 1. Could these fi ndings be consistent with a dia- 2. Is this neuropathy likely to be demyelinating betic neuropathy? or axonal? 2. What fi ndings suggest a hereditary A workup is initiated and the serum protein neuropathy? electrophoresis shows a large M-spike. An 3. Does the negative family history determine EMG–NCV discloses a demyelinating the diagnosis? neuropathy. 4. What are the most common types of heredi- The patient returns 2 months later having tary sensory and motor neuropathies? missed an earlier appointment. He complains 198 9 Neuromuscular Diseases: Neuroanatomic and Differential Diagnoses

of fatigue, mild weakness in both feet, and Past medical history is remarkable for carci- thickening of his skin. noma of the breast treated by mastectomy and The neurologic examination now discloses radiotherapy 8 years ago and mild hypertension. 4/5 strength of anterior tibialis, bilaterally. Current medicines are ramipril and Refl exes remain obtainable but there is a hydrochlorothiazide. markedly slow relaxing phase of brachioradia- General physical examination revealed a blood lis and ankle refl exes. pressure of 160/100; pulse 104; temperature 3. What part of the general medical examination 100.4°F. There is a grade 2/6 aortic systolic mur- should be repeated? mur, rales at the bases of both lungs and 2+ pit- 4. What tests should be ordered? ting edema of both legs. 5. What is the name of this syndrome? Neurologic Examination : 6. How does this compare with monoclonal gam- She has decreased rapid alternating movements mopathy of undetermined signifi cance and fi nger tapping of the left upper extremity and (MGUS)? decreased foot tap on the left side. There is 4/5 Case 26 Analysis : strength of left interossei, adductor pollicis, and 1. This is a large fi ber neuropathy since vibration fl exor pollicis. Wasting of the fi rst dorsal interos- sense, proprioception, and light touch percep- seus muscle is noted. There is 4+/5 strength of left tions are impaired. arm supination and fl exion at the elbow. The left 2. Demyelinating. anterior tibialis, peroneus longus, and brevis 3. The abdominal exam is repeated and discloses strength is 4/5. Strength of right anterior tibialis hepatosplenomegaly. is 4/5. Refl exes are 1+ and symmetrical in the legs, 4. The history of fatigue and, especially, the slow 2+ in the right arm and 1+ in the left arm. The sen- relaxing phase of the brachioradialis and ankle sory examination discloses decreased perception of refl exes suggest hypothyroidism. A TSH is pin and temperature distally in all extremities. There 70 μU/L. A metastatic bone survey reveals a is absent vibratory perception at toes and ankles sclerotic lesion in a thoracic rib. with barely perceived vibration at the knees. 5. This patient has POEMS syndrome which is Laboratory data. CBC shows a white count of polyneuropathy, organomegaly, endocrinopa- 12,000/cu mm with 90% polys. The hematocrit is thy, M-spike, and skin changes. This is an 32%. Urinalysis is signifi cant for 100 red cells osteosclerotic myeloma, a rare variant of mul- per high-powered fi eld. Renal and liver function tiple myeloma, ordinarily without hypercalce- studies are normal. mia, renal failure, anemia, and bone pain. Questions : 6. MGUS is a monoclonal gammopathy of unde- 1. Are the fi ndings compatible with a single neu- termined signifi cance. This refers to a para- roanatomic diagnosis? proteinemia unassociated with malignancy 2. What symptoms suggest a specifi c anatomic and with or without an associated neuropathy. locus? How could you describe the pattern of IgM is more frequent with neuropathy. Twenty weakness? percent of patients with MGUS will eventu- 3. Name every test which should be performed. ally acquire a malignant plasma cell disorder. 4. What is the differential diagnosis? Case 27 An 85-year-old woman is admitted to the medical service of the hospital because of Case 27 Analysis : hypertension, edema, and hematuria. On admis- 1. Yes. Neuropathy. Impaired rapid alternating sion, the patient adds a history of “sticking pains” movements, fi nger and foot tap will occur as a in both feet for the last 2 months. Over the past result of weakness and thus only indicate month she has noticed weakness of the left arm motor system dysfunction. The left-sided and leg. weakness is accompanied by decreased Cases 199

refl exes in the left arm and wasting of the fi rst protein electrophoresis, blood and urine dorsal interosseus muscle. These fi ndings cultures, 2D echocardiogram, chest X-ray. indicate peripheral (root or nerve) involve- The studies were all normal except for ment, but not neuromuscular junction or mus- ANCA-p. Subsequent renal angiograms cle. AHC involvement is possible but cannot disclosed multiple aneurysms typical of explain the “sticking pains” in both feet, typi- polyarteritis nodosa. An alternative diag- cal symptoms of neuropathy. nostic test would be a nerve and muscle 2. The “sticking pains” suggest neuropathy. biopsy which would likely demonstrate Muscular involvement of the left leg includes necrotizing vasculitis. the peroneus muscles (everters) innervated by 4 . Final Diagnosis : Mononeuritis multiplex sec- the superfi cial peroneal nerve and the anterior ondary to polyarteritis nodosa. tibialis innervated by the deep peroneal nerve. Case 28 A 47-year-old man complains of a Hence the left common peroneal nerve is 1-year history of loss of balance and a burning, affected. The right deep peroneal nerve is itchy sensation affecting both feet. The sensory involved because of a weak right anterior tibi- symptoms awaken him at night, but during the alis muscle. The left arm reveals ulnar nerve day he gets some relief with gabapentin. involvement (interossei, ADP and FP) and the One month ago he had the abrupt onset of severe musculocutaneous nerve (biceps). Thus this loss of vision OD. Funduscopic examination patient has mononeuritis multiplex. revealed a blurred optic disk margin, no venous 3. Vasculitis is the most likely etiology since the pulsations and several splinter and fl ame hemor- patient has mononeuritis multiplex, hyperten- rhages at the disk margins. The pupils were equal sion, and renal disease. and reactive, but there was an afferent pupillary Vasculitic diseases that cause mononeuritis defect OD. Visual acuity was fi nger counting only. multiplex include: Past medical history: The patient had lym- (a) Churg–Strauss disease. This is associated phoma 5 years ago and responded well to chemo- with pulmonary disease, especially asthma, therapy but had an adverse effect to vincristine allergic angiitis, and eosinophilia. which caused painful paresthesias and weak feet. (b) Polyarteritis nodosa. This is associated There was partial recovery after the medication with renal, gastrointestinal, central ner- was discontinued. Two years ago he had atrial vous system, joint, and muscular disease. fi brillation and tachycardia which was refractory (c) Rheumatoid arthritis. This is associated until treatment with amiodarone was initiated. He with joint, skin, pulmonary, and cardiac has hypercholesterolemia, coronary artery dis- disease. ease and required stents 4 years ago. He has a (d) Sjögren’s syndrome. This is associated 3-year history of diabetes and hypertension. with a sicca syndrome, pulmonary, mus- Current medications are amiodarone, simvas- culoskeletal, and central nervous system tatin, metformin, and ramipril. involvement. Neurologic Examination : (e) SLE. This disease involves joint, muscle, Blood pressure is 150/94; pulse is 80 and regular. skin, pulmonary, cardiac, renal, central ner- There is a large central scotoma OD with a vous system, and hematologic disorders. visual acuity of 20/100. An afferent pupillary (f) Wegener’s granulomatosis. This disease defect is present. Funduscopic examination reveals affects upper airway, lower airway, muscu- a blurred right optic disk margin and two splinter loskeletal system and causes renal disease. hemorrhages at the disk margin. There is bilateral Laboratory Workup : moderate-to-severe fi nger-to-nose and heel-to- CBC, metabolic profi le, ANA, anti-double- shin ataxia. Failure of check is noted in both arms. stranded DNA, SSA, SSB, ANCA-c Strength of anterior tibialis is 4/5 bilaterally. (Wegener’s), ANCA-p (polyarteritis and Vibration perception is absent at the toes, ankles, Churg–Strauss), RA factor, ESR, serum knees, and fi ngers. A few position sense errors are 200 9 Neuromuscular Diseases: Neuroanatomic and Differential Diagnoses made at the toes and there is distal sensory loss to Final Diagnosis: Optic neuritis, cerebellar pin and temperature in all extremities. Refl exes dysfunction, and neuropathy secondary to amio- are 1+ except for absent ankle jerks. darone toxicity.

Questions : Case 29 A 24-year-old man complains of tin- 1. Where are the lesions? Does this patient have gling over the sole of the right foot with the papilledema? exception of just under the heel. For 2 months he 2. Can the ataxia be due to sensory loss? has had pain in the ankle, which occasionally 3. Is there one etiology? radiates up the leg posteriorly to the buttocks. For 4. What is the most likely etiology? 2 weeks he has noted weakness when attempting to fl ex his toes. Past medical history is negative. Case 28 Analysis : 1. This patient has optic nerve disease, cerebellar Neurologic Examination : system dysfunction, and neuropathy. Unilateral Blood pressure is 150/112; pulse is 80 and regular. funduscopic abnormalities, central scotoma, Strength of toe fl exion is 3/5. There is numb- decreased visual acuity, and an afferent pupil- ness of the sole of the foot to pinprick stimulation lary defect all indicate an optic neuritis. except under the heel. Paresthesias are provoked Papilledema is bilateral; the pupils and vision in the valgus position (deviated away from the are ordinarily normal. Limb ataxia, inability midline) and they are alleviated in the varus posi- to perform tandem gait, and failure of check tion (toward the midline). There is a positive signal cerebellar system dysfunction. Distal Tinel’s sign when tapping over the lancinate liga- sensory loss, absent ankle jerks, and distal ment located just below the medial malleolus. weakness are signs of neuropathy. Eversion at the ankle causes paresthesias on the 2. Not in this case. The amount of proprioceptive sole of the foot. loss is minimal and affects only the toes. Questions : 3. The differential diagnosis includes: 1. What element of the examination is most (a) Toxic factors. important? (b) Diabetes. 2. Where is the lesion? Explain the anatomy. (c) Vasculitis. 3. What is the signifi cance of pain radiation (d) Paraneoplastic syndrome. upwards? 4. Diabetes causes neuropathy and predisposes to 4. What is the signifi cance of the distribution of an anterior ischemic optic neuropathy (AION), sensory symptoms and signs? but does not cause cerebellar ataxia unless there 5. What are the two diagnoses? is an associated stroke involving the cerebellar 6. What are your recommendations? system. The signs would then be focal. Vasculitis may cause an optic neuritis and Case 29 Analysis : neuropathy but not cerebellar ataxia without 1. Hypertension. Vital signs are the fi rst part of focality, which is expected with an ischemic any neurologic examination. The well-known complication. Paraneoplastic syndromes may complications of untreated hypertension man- cause neuropathy and cerebellar dysfunction date immediate recognition and treatment. but rarely a simultaneous optic neuropathy. 2. The patient has an entrapment neuropathy at the Simvastatin may rarely produce neuropa- tarsal tunnel affecting the posterior tibial nerve. thy but neither optic neuropathy nor cerebellar The sciatic nerve divides in the distal thigh to system dysfunction. Vincristine commonly the posterior tibial and peroneal nerves. The causes neuropathy but not the other fi ndings, posterior tibial nerve runs down the posterior and was discontinued years ago. Amiodarone calf then passes under the lancinate ligament is well-known to cause an optic neuritis, cer- after which it divides into medial plantar, lateral ebellar dysfunction, and neuropathy. plantar, and calcaneal branches. The calcaneal Cases 201

branches may separate before the nerve passes refl exes were absent and vibratory sense was lost under the ligament (about 40% of the time), and at the toes and ankles. Light touch was not per- therefore is often spared. ceived on the toes. 3. Retrograde pain radiation suggests that the Questions : origin of the pain is distal. 1. Is there a relationship between the lost ankle 4. Sensory symptoms and signs indicate involve- refl exes and the sensory loss with his pain ment of both medial and lateral plantar nerves. syndrome? The calcaneus nerve must branch off above 2. Where is the lesion that causes the pain? the tarsal tunnel. Valgus positioning (eversion 3. What is the name of his sensory symptom? of the ankle) will tighten the ligament over the 4. What is the most likely diagnosis? posterior tibial nerve and may provoke par- 5. What is the diagnosis? esthesias. The varus position (inversion) should relieve the pressure. Case 30 Analysis : 5 . Final Diagnoses : 1. Yes. (a) Hypertension. 2. The thoracic root or nerve. (b) Tarsal tunnel syndrome. 3. Dysesthesia. 6. Prompt referral to an internist or a family 4. Type II diabetes. The hemoglobin A1C is practitioner and, afterwards, an EMG-NCV. 7.3%. The fasting blood sugar is 132 mg/dl. Case 30 A 58-year-old man complains of excru- 5 . Final Diagnosis : Thoracoabdominal neuropa- ciating left upper abdominal burning pain awak- thy secondary to diabetes. ening him at 2:00 a.m. On arrival in the Emergency Case 31 A 91-year-old woman complains of Room, the pain intensity subsided slightly from a stiffness of the legs, loss of balance, and prickly 9/10 to an 8/10. The Emergency Room evalua- sensations in the feet. The prickly feelings tion included an ECG, cardiac enzymes, a cardi- become painful at night and keep her awake. ology, and surgical consultations. All evaluations Infrequently these symptoms affect the fi ngers of were normal but he was admitted for observation. both hands. There has been progressive develop- The day after admission he noted an electrical ment of this symptom complex over 1 year. sensation in the same region intermittently. Past medical history is remarkable for recur- Past medical history is negative other than rent gastrointestinal bleeding due to angiodyspla- asthma. He takes no medicine. Over the past year sia of the small intestine with resulting chronic he has lost about 30 lb, at least partly due to a iron defi ciency anemia. new exercise program but he has a good appetite Laboratory data includes normal liver func- and eats well. tions and metabolic panel. The CBC shows a A neurology consultation is requested on the hematocrit of 29% and normal blood indices. second hospital day after thoracic spine fi lms and an MRI (thoracic) were performed and were Neurologic Examination : normal. Blood pressure is 110/70; pulse is 68 and regular. There is a mild paraparesis with distal strength Neurologic Examination : 4/5 and proximal 4+/5. The patient exhibits mild The neurologist obtained additional history on spasticity. She has a positive Romberg. Vibration the morning of the second hospital day. The sense is absent at toes, ankles, and knees and abnormal sensation had extended upwards and position sense is absent at toes. Ankle and knee was in a dermatomal pattern encompassing refl exes are absent. There is a left Babinski sign. T6–T8 levels. Examination was remarkable for bulging of the left upper abdominal muscles. Questions : Touching the involved area produced a disagree- 1. What signs are compatible with neuropathy? able feeling which the patient could not explain 2. What signs suggest additional anatomic other than stating it was not painful. Ankle involvement? 202 9 Neuromuscular Diseases: Neuroanatomic and Differential Diagnoses

3. Could there be a single etiology? Questions : 4. Could degenerative disease be a factor? 1. What single fi nding has the greatest localizing 5. If there is a single diagnosis, what is the most value? likely one? 2. What part of the examination should be evalu- 6. What normal laboratory data obscures the ated in greatest depth? diagnosis? 3. What abnormal fi ndings in combination strongly suggest the underlying diagnosis? Case 31 Analysis : 4. What investigation will confi rm the diagnosis? 1. Distal weakness, vibration and proprioception loss, absent refl exes in the legs, and a positive Case 32 Analysis : Romberg. 1. Fasciculations. This is the prototypical sign of 2. Spasticity and left Babinski sign. Myelopathy AHC disease. does not explain the absent refl exes in the legs 2. The observation of fasciculations anywhere although all the other fi ndings are compatible mandates a careful search for additional fas- with it. ciculations on a disrobed patient. Fasciculations 3. Yes. noted elsewhere, such as in the pectoralis 4. Yes. The patient could have cervical myelopa- major and back musculature, will clinch a thy due to spinal stenosis and spondylosis diagnosis of ALS or motor neuron disease. superimposed on a neuropathy. This could Particular attention should be paid to the explain spasticity and the Babinski sign, but tongue where both fi brillations and fascicula- involvement of the dorsal column would be tions can be seen. Localized fasciculations exceedingly rare. Spondylotic myelopathy occur with radiculopathy, but this is ordinarily usually affects only the anterior and lateral accompanied by pain and decreased refl exes columns of the spinal cord. Absent refl exes in in the same location. the legs do not result from a myelopathy. 3. The increased refl exes in the weak leg support 5 . Diagnosis: Subacute combined degeneration a diagnosis of ALS.

plus neuropathy due to vitamin B 12 defi ciency. 4. Electromyography. 6. Normal blood indices are not expected with Case 33 A 38-year-old man complains of chest, vitamin B 12 defi ciency, but the chronic super- imposed iron defi ciency anemia prevents back, and upper arm pain associated with muscu- expression of an increased MCV. lar swelling of 4 days duration. This began with a prolonged workout which included heavy weight- Case 32 A 55-year-old man complains of rapidly lifting for 2 h. Although he felt muscular pain progressive weakness of the left leg for 6 months during the workout, it increased in severity within duration. There is no associated pain, numbness, 1 h and it has persisted without abatement. His or paresthesias. Past medical history is remarkable prior exercise programs never lasted more than for type II diabetes controlled with metformin. 15 min. Beginning about the age of 15 he noted muscle cramps and contractures after exercise. Neurologic Examination : Consequently, he independently limited his physi- Blood pressure is 110/70; pulse is 48 and regular. cal activity. After this particular exercise, he noted Strength of all left leg musculature is 4/5 that his urine turned brown. Now he has pain with except iliopsoas, 3/5. Tone is normal. There is any movement as well as nausea and malaise. decreased bulk of left thigh musculature. Fasciculations are numerous in the left calf and Neurologic Examination : thigh muscles. Sensory examination is normal. Blood pressure is 110/70; pulse is 70 and regular. Refl exes are 2+ except for 3+ in the left leg and Strength of pectoralis major is 4/5. Deltoid, plantars are fl exor bilaterally. supraspinatus and infraspinatus are 4+/5. Cases 203

The calf muscles appear hypertrophied. Refl exes Neurologic Examination : are 1+, symmetrical and the sensory examination Blood pressure is 135/80. is normal. He has a mild right ptosis. The right eye is abducted and slightly below the horizontal merid- Questions : ian. On attempted downgaze there is intorsion 1. What causes the brown urine? How does it OD. Eye movements are entirely normal OS. The occur? pupils are 4 mm equal and with a 3+/4 reaction to 2. What is the function of this substance? light bilaterally. The entire neurologic examina- 3. What medical complication may occur? tion is normal otherwise. 4. What blood tests are commonly abnormal? 5. What is a contracture? Why do muscles Questions : swell? 1. Why does the right eye intort on attempted 6. What is the suspected diagnosis? downgaze? 7. What test confi rms the diagnosis? 2. What nerve is involved? 3. Why is the pupil spared? Does sparing of the Case 33 Analysis : pupil lead to the diagnosis? 1. Myoglobin. Lysis of striated muscle (rhab- 4. If the right eye did not intort, would the diag- domyolysis) causes myoglobinuria. nosis change? Where would the lesion be? 2. Myoglobin transports oxygen from hemoglo- 5. Does the presence of excruciating pain sug- bin to mitochondria. gest the underlying etiology? 3. When severe, myoglobinuria may lead to 6. What is the diagnosis and prognosis? acute renal failure. 4. CK, BUN, creatinine. In this case the CK is Case 34 Analysis : 50,000 U/L and the BUN and creatinine are 1. The right eye intorts because the 4th nerve is markedly elevated. Hemodialysis is required. intact and its function when the eye is abducted 5. A contracture is a spontaneous shortening of is intorsion whereas, when adducted the eye muscle not caused by an act of contraction. moves directly downward. Muscles swell due to shift of extracellular 2. Third nerve. water into necrotic muscle. 3. The lesion is in the center of the nerve as the pupillary fi bers are peripherally located. This 6. Metabolic myopathy. McArdle’s disease. This implies ischemia secondary to small vessel is due to a myophosphorylase defi ciency with disease. A compressive lesion from a mass or resultant inability to metabolize glycogen. aneurysm would involve the pupil primarily in Although autosomal recessive, it is most often most cases. found in males and typically becomes symp- 4. Yes. Involvement of the 3rd and 4th nerves tomatic in the teen years. implies a lesion in the cavernous sinus where 7. Ischemic forearm exercise test. This measures the two nerves are close to each other. A pos- the rise in lactic acid with ischemic exercise. sible etiology would then include a carotid Failure to increase indicates defective metab- aneurysm or neoplasm. olism of glycogen. Defi ciency of myophos- 5. Pain is especially severe with an ischemic 3rd phorylase is only one of several metabolic nerve lesion, usually much more prominent than disorders that prevent glycogenolysis. with aneurysmal compression of the 3rd nerve. Case 34 A 72-year-old man complains of excru- 6. Diabetic 3rd nerve palsy. There are only a few ciating right eye pain of 2 days duration. On the reports of aneurysms causing a pupillary-sparing morning of this evaluation, his right lid drooped third palsy. These are located in the cavernous and he noted double vision in all directions of sinus. Consequently, an MRA and an MRI gaze but least when looking to the right. His past should be performed. The prognosis of a dia- medical history is remarkable for well-controlled betic 3rd nerve palsy is excellent as there is usu- hypertension. His only medicine is atenolol. ally a complete recovery within 3 to 6 months. 204 9 Neuromuscular Diseases: Neuroanatomic and Differential Diagnoses

Appendix

Screening evaluation for an idiopathic neuropathy Refsums disease CBC Most common toxic neuropathies BUN, creatinine, electrolytes Amiodarone Liver functions Phenytoin Calcium, phosphorus Cisplatin Free T-4, TSH Vincristine ANA Isoniazid B12 level Chloroquine Thalidomide ESR Nitrofurantoin Rheumatoid factor Vitamin B6 Serum protein electrophoresis Metronidazole Review of occupational exposure to toxic agents Taxols and social history Colchicine Review of family history Multiple mononeuropathies Neuropathy with prominent dysautonomia Sarcoidosis Diabetes Vasculitis Amyloidosis Leprosy Hereditary sensory and autonomic neuropathy Lyme disease Paraneoplastic sensory and autonomic HIV polyganglionopathy Cryoglobulinemia Acute pandysautonomia Multifocal motor conduction block neuropathy Guillain–Barré syndrome Diabetes Porphyria Neurofi bromatosis HIV Hereditary susceptibility to pressure palsies Toxic drugs. Amiodarone, vincristine, and cisplatin Case discussions related to anatomic site of pathology Toxic substances. N-hexane, arsenic, mercury, Anterior horn cell. Cases 13, 22 and thallium Radiculopathy. Cases 1, 7, 30 Demyelinating neuropathies Plexopathy. Cases 2, 18, 19 Guillain–Barré syndrome (AIDP) Neuropathy. Cases 3, 4, 5, 7, 8, 9, 14, 15, 17, 20, 22, CIDP 23, 24–29, 31 Osteosclerotic myeloma Neuromuscular junction. Cases 12, 16, 21 MGUS (monoclonal gammopathy of undetermined Myopathy. Cases 6, 10, 16, 21, 23, 24, 33 signifi cance) Myelopathy. Case 31 Hereditary susceptibility to pressure palsies Tendonitis. Case 11 Hereditary sensorimotor neuropathy, types 1 and 3 Type 1 CMT 1 Charcot–Marie–Tooth Type 3 CMT 3 Déjérine–Sottas disease Multifocal motor neuropathy Bibliography POEMS syndrome Diabetes Bird SJ, Rich MM. Neuromuscular complications of criti- HIV cal illness. Neurologist. 2000;6:2–11. Chusid JG. Correlative neuroanatomy and functional neu- Anti-MAG syndrome rology. 15th ed. Los Altos, CA: Lang Medical; 1973. Anti-Sulfatide syndrome Engel WK, Askanas V. Inclusion body myositis. Leprosy Neurology. 2006;66 Suppl 1:520–9. GALOP syndrome Karpati G, Hilton-Jones D, Bushby K, Griggs RC. Amiodarone Disorders of voluntary muscle. 8th ed. New York, NY: Chloroquine Cambridge University Press; 2010. Perihexaline Nicolle MW. Myasthenia gravis. Neurologist. 2002;8: 2–21. Bibliography 205

Pourmand R, editor. Neurologic clinics. In: Acquired neu- Rosenbaum R. Neuromuscular complications of connec- romuscular diseases. Philadelphia, PA: Saunders; tive tissue disease. Muscle Nerve. 2001;24:154–69. 1997. Ruts L, Drenthen J, Jacobs BC, Van Doorn PA. Distinguishing Rider LG, Miller FW. Deciphering the clinical acute onset CIDP from fl uctuating Guillain-Barré syn- presentations, pathogenesis, and treatment of the idio- drome. Neurology. 2010;74: 1680–6. pathic infl ammatory myopathies. JAMA. 2011;305: Selim MH. Neurologic aspects of thyroid disease. 183–90. Neurologist. 2001;7:135–46. Ropper AH, Gorson KC. Neuropathies associated with Stewart JD. Focal peripheral neuropathies. New York, paraproteinemia. N Engl J Med. 1998;338:1601–7. NY: Elsevier; 1987. Common Symptoms in the Neurology Clinic 10

Keywords Headache • Vertigo • Sleep apnea • Narcolepsy • Epilepsy • Syncope

The majority of patients referred to neurologists and facial pain, dizziness and vertigo, syncope and a large proportion of patients seen by primary and seizure, sleep disorders, and transient neuro- care providers complain of headaches, dizziness logic symptomatology. or vertigo, episodes of loss of consciousness, sleep disorders and transient symptoms such as confusion, amnesia, weakness, blurred vision and Headache numbness or tingling sensations. Since most resi- dency programs focus on in-patient experience, Although tension-type headache (TTH) is the the graduating resident is often unprepared for most common form of headache, the incidence is the deluge of patients with such complaints. He probably markedly lower in those people who or she must be able to sift through the history to seek medical attention. Certainly, in a neurology extract the critical information and, most impor- practice most patients have other types of head- tant, to ask the pertinent questions which elicit ache or facial pain, especially migraine. Passively diagnostic information. Lurking within this obtaining a history without probing questions is patient load are undoubtedly a significant number usually inadequate. Here is a typical example. with serious illnesses which must be detected and treated promptly. Many of them either do not Case 1 A 28-year-old woman requests an evalu- have abnormal laboratory values that easily facil- ation for recurrent “sinus headaches,” pressure- itate making a diagnosis or they provide mislead- like sensations in her forehead. They began about ing neuroimaging data that, if totally relied upon, 6 months ago; occur two or three times per week result in misdiagnosis. and last several hours. She has occasional nasal This chapter aims to provide an incentive drainage and “acetaminophen-sinus” relieves the to obtain a meticulous history which is the essen- pressure after 4 or 5 h. She specifically denies tial ingredient for making an accurate diagno- nausea, light and sound intolerance. When home sis. Additionally, there will be a focus on the she keeps busy around the house even when the ­interpretation of the patient’s responses. Short headache is severe. She is worried about keeping ­discussions of pertinent neurophysiology and her job since she loses 2–3 days per month due to neuroanatomy will be included. The symptoms these headaches. and disorders to be examined include headaches Additional history is obtained.

J.N. Alpert, The Neurologic Diagnosis: A Practical Bedside Approach, 207 DOI 10.1007/978-1-4419-6724-4_10, © Springer Science+Business Media, LLC 2012 208 10 Common Symptoms in the Neurology Clinic

Questions: D. During headache at least one of the 1. If you are home alone when you have a sinus following. headache, what would you do? “I would get all 1. Nausea and/or vomiting. of my housework done.” 2. Photophobia and phonophobia. 2. If you are home alone and all of your house- E. Not attributed to another disorder. work was already completed, what would you Migraine with aura* do? “My housework is never finished.” A. At least two attacks fulfilling criteria B–D. 3. Let us assume the nearly impossible situation B. Aura consisting of at least one of the follow- that your husband arranged for an excellent ing, but no motor weakness. service to clean your house and then took your 1. Fully reversible visual symptoms includ- two children camping. What would you then ing positive features (e.g., flickering lights, prefer to do if you had a severe “sinus head- spots or lines) and/or negative features ache”? “I would take a nap.” (i.e., loss of vision). 4. Would you prefer the lights on or off? “Off.” 2. Fully reversible sensory symptoms includ- 5. Would you have a snack before your nap? “No, ing positive features (i.e., pins and nee- I’m usually queasy and have no appetite.” dles) and/or negative features (i.e., (This patient uses the word “queasy” in place numbness). of nausea; nausea to her means vomiting). 3. Fully reversible dysphasic speech This patient has migraine without aura since disturbance. she loses her appetite and prefers to lie down in a C. At least two of the following. dark, quiet environment. Thus, the five key ques- 1. Homonymous visual symptoms and/or tions: Is there nausea, sensitivity to movement, unilateral sensory symptoms. light, sound, and smell? Nasal congestion is not 2. At least one aura symptom develops grad- unusual in migraine patients and response to ually over ³5 min and/or different aura “acetaminophen-sinus” is doubtful since she symptoms occur in succession over improves only after 4 h. The presence of severe ³5 min. intensity, a throbbing quality, and unilaterality 3. Each symptom lasts ³5 and £60 min. are not required for the diagnosis of migraine. D. Headache fulfilling criteria B–D for “migraine Headaches due to sinus diseases are relatively without aura” begins during the aura or fol- uncommon. Symptoms that support a diagnosis lows aura within 60 min. of a sinus headache include a profuse nasal dis- E. Not attributable to another disorder. charge which is often purulent, severe conges- tion, fever, and altered sense of smell. Severe Typical aura without headache* recurrent pain in maxillary or supraorbital loca- A. At least two attacks fulfilling criteria B–D. tions unaccompanied by major nasal symptoma- B. Aura consisting of at least one of the follow- tology is usually due to migraine. ing, with or without speech disturbance but no motor weakness. Migraine without aura* 1. Fully reversible visual symptoms includ- A. At least five attacks fulfilling criteria B–D. ing positive features (e.g., flickering lights, B. Headache attacks lasting 4–72 h (untreated or spots or lines) and/or negative features unsuccessfully treated). (i.e., loss of vision). C. Headache has at least two of the following 2. Fully reversible sensory symptoms includ- characteristics: ing positive features (i.e., pins and nee- 1. Unilateral location. dles) and/or negative features (i.e., 2. Pulsating quality. numbness). 3. Moderate or severe pain intensity. 4. Aggravation by or causing avoidance of *International Headache Society. The International routine physical activity (e.g., walking or Classification of Headache Disorders: 2nd edition. climbing stairs). Cephalalgia. 2004;24(Suppl 1):1–160 (www.i-h-s.org). Headache 209

C. At least two of the following. central pain modulation by activating trigeminal 1. Homonymous visual symptoms and/or axons and central projections in the trigeminal unilateral sensory symptoms. nucleus caudalis. What triggers CSD? Research 2. At least one aura symptom develops grad- studies suggest it is the excitatory amino acid, ually over ³5 min and/or different aura glutamate. symptoms occur in succession over Serotonin, which is 5-hydroxytryptamine ³5 min. (5-HT) is intimately associated with the develop- 3. Each symptom lasts ³5 and £60 min. ment of migraine. The main metabolite of sero- D. Headache does not occur during aura nor fol- tonin, 5-hydroxyindoleacetic acid, is increased in lows aura within 60 min. the urine in patients with migraine. Platelet sero- E. Not attributed to another disorder. tonin decreases at the onset of migraine and intra- venous serotonin aborts migraine. Hence, Case 2 A 48-year-old man complains of seeing serotonin agonists such as dihydroergotamine multicolored zigzag lines (fortification figures) in and sumatriptan treat migraine whereas reser- his left visual field. These gradually enlarge and pine, a 5-HT depleter, provokes migraine. There encompass most of the left visual field. He then are several 5-HT receptors and one of them, notes numbness and tingling of the left hand 5-HT1B/1D, is the major site of action of both which gradually ascends up to the face (march of DHE and triptan drugs. paresthesias) over 5 min. The total duration of Neurogenic inflammation (Waeber and symptoms is 30 min. This is a classical history Moskowitz 2005) is an integral part of the of a patient with a migrainous aura without migraine diathesis and is mediated by vasoactive headache. peptides, calcitonin gene related peptide (CGRP), Other visual phenomena include phosphenes vasoactive intestinal polypeptide, and substance (light flashes), teichopsia (“walled vision” as in a P. These are stored within axons projecting from homonymous hemianopsia), an arc of lights typi- the trigeminal ganglia. CGRP is predominant and cally in a C pattern, and geometric designs. Visual alone is elevated in external jugular vein blood illusions are much less frequent but most impor- during migraine. Inflammation of the meninges tant to be cognizant of since they are also mani- probably results from CGRP release and afferent festations of epileptiform events. Migraineurs pathways for pain pass from the meninges through may experience, for example, metamorphopsia, the trigeminal ganglion and synapse on second micropsia, macropsia, déjà vu, and jamais vu order neurons in the trigem­inocervical complex. symptomatology. This complex extends from the trigeminal nucleus A brief review of migraine physiology is now caudalis to the dorsal horn in the upper cervical appropriate. Approximately, 30% of patients cord. These neurons have axons which cross the with migraine have an aura. It may be associated midline and ascend to ­synapse with thalamic neu- with a phenomenon called cortical spreading rons. There are neural connections with the supe- depression (CSD). This terminology refers to a rior salivatory nucleus which results in decrease in electrical activity which begins in the parasympathetic outflow and vasodilatation. occipital lobe and migrates anteriorly at about The brainstem may be the site of initiation of 2–3 mm/min. The aura of fortification figures migraine as has been demonstrated by PET scans migrates at about the same velocity as CSD. which measure regional blood flow (Afridi et al. Coincident with CSD is a reduction in cerebral 2005). Activated areas include the periaqueductal blood flow which has been termed spreading gray near the dorsal raphe nucleus (neurons con- oligemia. The oligemia does not follow vascular taining serotonin) in the midbrain and dorsolat- territories but rather the location of the migrating eral pons adjacent to the locus ceruleus (neurons decrease of electrical activity. Hence, the under- containing norepinephrine). lying basis for migraine and migraine aura is likely to be primarily neuronal, not vascular as Case 3 A 42-year-old woman requests an evalu- originally believed. CSD may also play a role in ation because of severe headaches and a mood 210 10 Common Symptoms in the Neurology Clinic disorder. Although she has just one or two head- of the unilateral, throbbing, sharp eye pain aches a month, the prodromal phase and residual associated with photophobia lasting over 4 h. symptoms last 1–1/2 days and 1 day, respectively. 2. A generally accepted principle is that a “white” One of the headaches invariably occurs during eye is not the source of headache. Conjunctival the day before her menses. Prominent symptoms injection which occurs with iritis or conjuncti- are depression and fatigue. After the headache vitis and a cloudy cornea associated with acute has subsided her scalp is extremely sensitive and angle closure are the visible signs of it is too painful to comb her hair. This is known as ocular diseases which may cause headache. cutaneous allodynia. The one possible exception is the intermittent headache of subacute angle closure glaucoma, Comment: a rare disorder. These headaches usually last This patient exhibits a typical premonitory syn- less than 1 h, however. drome of migraine which can last 1–2 days. Some common symptoms are lassitude, hunger, depres- Case 5 A 56-year-old woman complains of sion, thirst, and irritability. Headache associated severe headaches subsequent to a motor-vehicle with menses most often occurs as the estrogen accident which occurred 10 months ago. Her car level falls. There is no proven association of was struck on the passenger side as she was cross- migraine with ovulation, however, despite the ing an intersection and she bumped her head on falling estrogen levels at that time. Cutaneous the car window. She did not lose consciousness allodynia manifested by this patient’s extreme or suffer a visible injury. Two days afterwards she scalp sensitivity is quite common after migraine had the first of numerous headaches occurring as well as either recurrence or persistence of the without warning. They are bilateral, frontal and premonitory symptoms. Cutaneous allodynia is pulsating, lasting 6–7 h, and they occur about due to sensitization of nocioceptive neurons in three times per week. Associated symptoms are the trigeminal nucleus caudalis. mild nausea, phonophobia, and aggravation with quick head movement. After the headache she Case 4 A 14-year-old girl complains of periodic feels dizzy, fatigued, and cannot concentrate. pain in the left eye over the previous 4 months. Because litigation was initially involved, she was The pain varies from sharp to throbbing and is diagnosed as having a compensation neurosis. usually associated with light sensitivity. It has The case was settled after 4 months but the head- been occurring a few times per week, each time aches increased in intensity afterwards. Her neu- persisting for 4–6 h. The pain is perceived as rologic examination is normal. within the eye or just behind it. Examination discloses normal fundi, visual Comment: fields and visual acuities of 20/60 O.D. and 20/50 This patient was initially diagnosed as having a O.S. Acuity is normal with pinhole. There is no compensation neurosis. The characteristics of the pain with eye movement. Perception of light and headache are, however, quite compatible with color are normal. There is no scleral injection. migraine. There is a pulsating quality with nau- The pupils are normal. sea and photophobia. The subsequent inability to concentrate associated with fatigue are more Questions: common as part of a premonitory phase but a not 1. Can this young girl’s headache be due to unusual occurrence afterwards. impaired visual acuity due to nearsight­ The diagnosis is posttraumatic migraine edness? (Weiss et al. 1991), a remarkably underdiagnosed 2. Is there any eye disease that might cause this and treatable disorder. This patient needs both patient’s headache? prophylactic treatment, since there is more than Answers: one headache per week, and abortive therapy. 1. No. does not cause this type of The physiologic basis for posttraumatic migraine headache. This patient has migraine because is unknown. Possible contributing factors include Headache 211 genetic predisposition, stress, or even the minor 3. The feared complication of temporal arteritis rotational forces from the injury. Of practical is loss of vision which is nearly always due to importance is its recognition and application of vasculitic involvement of the posterior ciliary treatment as with anyone who has migraine. arteries causing an anterior ischemic optic neuropathy. In this patient’s case the central Case 6 A 58-year-old woman complains of retinal artery is involved in view of amaurosis an unrelenting, bilateral, occipital headache for fugax, transient loss of vision in one eye. As 5 weeks. The headache has a boring-to-piercing this history demonstrates, the curtain-like quality, stronger on the left side. Two weeks ago effect is not the exclusive manifestation of she had a brief loss of vision O.S. lasting just amaurosis fugax and should not be relied upon 1 min. She describes it as blackening of vision for a diagnosis. Temporal arteritis is a rare eti- crossing her visual field from left to right. Carotid ology of amaurosis fugax but it must always angiography was promptly performed and was be suspected especially in the absence of either normal. A cardiology consultant found no cardiac ipsilateral internal carotid artery disease or a source of embolism. The evaluation included a cardiac source of embolism. transesophageal echocardiogram. Subsequent to this evaluation the patient had 4. Polymyalgia rheumatica frequently precedes at least four more episodes of visual loss O.S. or follows temporal arteritis. Fever of unknown lasting from 30 s to 2 min. origin may be the initial presentation of tem- What other questions might add diagnostic poral arteritis. information? The patient was asked the following Case 7 A 40-year-old man arrives in the questions: Emergency Room because of the abrupt onset of 1. Have you had fever, joint or muscle pain? a severe, throbbing, occipital headache occurring 2. Have you any pain on chewing? during orgasm. It is now 3 h since the onset of The patient has had repeated episodes of low- headache and it is beginning to subside. He com- grade fever without explanation in the past year. plains of nausea and photophobia. The neurologic She has mild, migrating muscle pain involving examination is normal including the absence of all extremities and aching in her jaws when nuchal rigidity although head movement increases chewing. the pain. Diagnosis: Comment: Temporal arteritis associated with amaurosis fugax, Headache with sexual activity can begin prior to jaw claudication, and polymyalgia rheumatica. or during orgasm (Frese et al. 2003). A rare third Comment: type is postural, occurs after intercourse and 1. Any patient over 50 who has new onset head- resembles a post-lumbar puncture headache. The ache must be evaluated for temporal arteritis preorgasmic form is commonly milder, builds up with a sedimentation rate and arguably a in intensity gradually and has features of a TTH. c-reactive protein. It is often aborted by ceasing sexual activity. The 2. The headache of temporal arteritis can be any- more alarming throbbing, explosive orgasmic where on the head, unilateral or bilateral. headache often prompts a search for immediate There is a predilection for involvement of medical care. It is estimated that in 4–11% of all arteries in the external carotid artery system. patients with subarachnoid hemorrhage the pro- When the internal maxillary artery is involved, voking factor is sexual activity. Thus, a thorough jaw claudication occurs. Most often the workup is necessary. superficial temporal artery is affected causing This patient had a CT scan followed by a lum- the typical temporal pain with associated bar puncture both of which were normal. The soreness. Involvement of the internal occipi- lumbar puncture was performed because CT tal artery may result in occipital pain and scans diagnose just 90–95% of patients with sub- soreness. arachnoid hemorrhage. 212 10 Common Symptoms in the Neurology Clinic

Case 8 A 56-year-old woman complains of 6. Visual symptoms simultaneously in both severe, frequent, left-sided headaches beginning temporal and nasal fields of both eyes. 2 years ago. They average two per week and 7. Ataxia. occur without an aura but during the first hour of 8. Decreased level of consciousness. the headache she feels unsteady and has brief 9. Simultaneously bilateral paresthesias periodic episodes of incapacitating vertigo, some- (abnormal or unpleasant sensation often times accompanied by tinnitus. Nausea and vom- described as numbness or as a prickly, iting can be severe. Occasionally, she notes stinging, or burning feeling). numbness and tingling around the mouth. The C. At least one of the following. headache itself lasts from 6 to 8 h. 1. At least one aura symptom develops gradu- Her neurologic examination is normal. MRI ally over ³5 min and/or different aura symp- and MRA studies are normal. toms occur in succession over ³5 min. Comment: 2. Each aura symptom lasts ³5 and £60 min. This patient has basilar migraine (Silberstein D. Headache fulfilling criteria B–D for “Migraine 2002). Ordinarily, patients with this disorder have without aura” begins during the aura or fol- brainstem symptomatology such as vertigo, lows aura within 60 min. ataxia, and facial paresthesias as an aura lasting E. Not attributed to another disorder. less than an hour prior to headache onset. A few Case 9 A 28-year-old man requests an evalua- patients, however, do not follow the prescribed tion for severe headaches provoked by playing pattern, and, if the headache is minor, will avoid tennis. They usually occur after playing for more mentioning it altogether. It is then incumbent on than 1 h. They are bilateral, pounding, associated the physician to elicit the diagnostic history. with nausea and photophobia, and last 1–2 h. Clearly, it remains a viable diagnosis only after This information prompts additional inquiries thorough evaluation which includes from the examining physician. neuroimaging. Initial descriptions of basilar migraine, also Questions: called Bickerstaff headache after the physician 1. Do you have headaches on other occasions? who initially delineated it, focus on its onset in “Yes, rarely I have similar headaches preceded youth. This is not the case as it occurs at any age by seeing bright lights directly in front of me in either sex but women more than men. which persist for 15 min.” 2. Do you play indoors or outdoors? “Outdoors.” Basilar-type migraine* A. At least two attacks fulfilling criteria B–D. Comment: B. Aura consisting of at least two of the follow- This patient has migraine with aura predisposing ing fully reversible symptoms, but no motor him to benign exertional headache. Heat and high weakness. altitude are provoking factors. 1. Dysarthria (impairments or clumsiness in Case 10 A 63-year-old woman is admitted to the the speaking of words due to diseases that hospital with a 2-day history of a severe, left affect the oral, lingual, or pharyngeal frontal headache and horizontal double vision. muscles). The double vision is evident mainly at distance 2. Vertigo. and looking to the left. She had a similar episode 3. Tinnitus. 8 years ago and does not recall the details but 4. Hypacusia (impaired hearing). medical treatment cured her in 2–3 weeks. 5. Diplopia (double vision). Neurologic examination: There is subjective impairment of color and light perception O.S. but visual acuity is 20/20 O.U., *International Headache Society. The International Classification of Headache Disorders: 2nd edition. visual fields are normal and there is no central sco- Cephalalgia. 2004;24(Suppl 1):1–160 (www.i-h-s.org) toma. Pupils are 4 mm and reactions to direct light Headache 213 stimulation are 4+/4 O.D. and 2+/4 O.S. consen- lasting just 3 min. This has been occurring over sual reactions are 3+/4 O.D. and 2+/4 O.S. The left the last 5 years typically while weightlifting. He corneal response is sluggish and the right is nor- works out on a regular basis and it occurs only mal. Corneal stimulation on the right elicits a strong when he uses heavy weights. One month ago, blink on the left. On red glass testing the left lateral during an upper respiratory infection, a cough rectus muscle is weak. The red glass covers the left would invariably produce an even more severe eye and the distal image on left lateral gaze is red. occipital headache.

Reminder: Questions: When looking left the image farthest to the left 1. What other history should be obtained? comes from the weak muscle, when looking right 2. What part of the neurologic examination the image farthest to the right comes from the requires special attention? weak muscle, etc. 3. What is the most likely diagnosis? Questions: Answers: 1. What cranial nerves are involved? 1. Pertinent questions include whether similar 2. Where is the lesion? headaches occur with any other activity. Does 3. What does the MRI show? he note any change in walking, leg strength, 4. What is the etiology? and balance? 2. Examination of eye movements and the motor Answers: system. 1. The left optic nerve is involved because of 3. The patient has a longstanding history of subjective impairment of color and light per- milder occipital headache with laughing and ception. The pupillary response to direct light when straining at stool. He has noted slight O.D. is better than consensual response O.D.; tightness in his legs when running. The neuro- thus afferent input O.S. is less indicating a left logic examination reveals mild spasticity in optic nerve lesion. the legs and a left Babinski sign. There are a The left 3rd nerve is affected since even the few downward beats of nystagmus on lateral consensual pupillary response O.S. remains gaze (asymptomatic in his case). decreased at 2+/4. In the absence of abnormal neurologic signs The left ophthalmic division of the trigemi- and with an acute presentation, the differential nal nerve is involved because of the sluggish diagnosis includes subarachnoid hemorrhage and corneal response. The strong left eyelid blink benign cough headache. Otherwise, lesions at the with right corneal stimulation excludes left foramen magnum which cause spinal cord and/or orbicularis oculi or 7th nerve pathology. cerebellar signs should be suspected. Compression The left 6th nerve is damaged as attested by of the cerebellar flocculus and nodulus produces left lateral rectus weakness. Diplopia due to downbeat nystagmus. This patient has Chiari 1 6th nerve palsies is most prominent at distance malformation due to herniation of the cerebellar when the eyes have to diverge. tonsils below the foramen magnum. Abnormal 2. The lesion is primarily in the cavernous sinus descent is usually considered 5 mm or more and since there is left 3rd, 5th, and 6th nerve an associated syrinx is not unusual. The presumed involvement. headache mechanism is the increased intratho- 3. The MRI shows an enlarged left cavernous racic pressure causing impaired venous return sinus due to abnormal tissue which enhances increasing cerebral blood volume which results with contrast and extends into the orbital apex. in an increased intracranial pressure. This is 4. This is Tolosa–Hunt syndrome, an idiopathic, transmitted through a pressure wave displacing often recurrent, granulomatous inflammation the cerebellar tonsils further into the foramen which responds to corticosteroid treatment. magnum. The no longer used Queckenstedt test, Case 11 A 45-year-old man requests an evalua- devised to diagnose spinal subarachnoid block, tion for a severe, pounding, occipital headache was performed by unilateral or bilateral jugular 214 10 Common Symptoms in the Neurology Clinic vein compression while measuring cerebrospinal The patient was a heavy smoker (two packs fluid pressure during a lumbar puncture. This eas- per day) until 4 years ago when he perma- ily increases CSF pressure by 50–150 mm. This nently ceased smoking. His neurologic exami- test confirms the connection between poor venous nation reveals ptosis and miosis, O.D. return and increased intracranial pressure. B. A 45-year-old woman complains of 20–30 headaches daily for 12 years. They are unilat- Case 12 A 54-year-old woman complains of eral, left-sided, periorbital and temporal, with daily headaches for 2 years. They last 8–10 h. a stabbing character and lasting 5–20 min. She She describes them as a constricting band around notes a , nasal congestion, and tearing. the head sometimes associated with tightness in Rarely there is a lid droop. Her neurologic the lower occipital or upper cervical region. examination is normal. About twice per week she has a severe, throb- C. A 48-year-old man complains of excruciating bing, bi-occipital headache accompanied by nau- orbital and temporal, right-sided, paroxysmal sea, photophobia, osmophobia, and it is headaches for 2 years. They last from 10 s to aggravated by movement. Consequently, she has 2 min and are associated with tearing, swollen common migraine (migraine without aura). Her eyelids, and reddening of the eye. His forehead neurologic examination is normal. on the right side becomes moist. His heart rate This patient has chronic daily headache due to declines from 60 to 42 beats per minute. There transformed migraine (Dodick 2006). Charac­ are no trigger points on examination. teristics of this relatively common disorder D. A 72-year-old man complains of frequent noc- include: turnal headaches invariably awakening him at 1. Greater than 15 days per month of headache. 2:00 a.m. for the last 5 years. He describes the 2. Average duration of untreated headache is pain as generalized, dull and with an intensity greater than 4 h. of 5–6/10, lasting 45 min and occurring 15 3. Episodic migraine which meets the ICHD-II cri- times per month. There are no associated teria and/or a history of increasing headache migrainous or autonomic features. After sit- frequency with decreasing migrainous features ting up and having a cup of coffee he obtains over more than 3 months and no sign of hemicra- modest relief in about 20 min. nia continua or new daily persistent headache. E. An 80-year-old woman complains of a 3-week A second perhaps more common cause of history of severe, sharp, stabbing pains in her chronic daily headache is medication overuse left cheek and forehead. They last for 30 s to headache the features of which are: 2 min, occur numerous times per day and are 1. Regular use of medication for greater than 3 provoked by chewing, less often brushing her months. teeth. In between the paroxysms of pain there 2. Use of headache medications for more than 15 is a constant dull ache. days per month. F. A 28-year-old woman arrives in the Emergency Cases 13A–13F Room because of “throat spasms” and left ear A. A 55-year-old man complains of a 3-year pain of 12 days duration. The pain is excruci- ­history of severe right-sided eye and facial pain. ating and most frequent during meals prevent- The pain is described as stabbing, mainly behind ing her from eating more than a few bites. The the eye and excruciating as it reaches an inten- ear pain has been particularly acute. These sity of 9–10/10. This prompts an occasional symptoms have resulted in a weight loss of Emergency Room visit. Infrequently it extends 5 lb over 1 week. During meals she would feel down to the upper jaw and teeth. He has had faint, turn pale, and become diaphoretic. tearing, nasal congestion and lid droop on only a few occasions. The pain lasts 45–60 min and What are the diagnoses? forces him to pace the room. The frequency is A. Cluster headache. two to three times per week throughout the year. B. Chronic paroxysmal hemicrania. Headache 215

C. Short-lasting, unilateral neuralgiform head- following autonomic symptoms always occurs: aches with conjunctival injection and tearing nasal congestion, rhinorrhea, and eyelid edema. (SUNCT). Bradycardia is sometimes present due to activa- D. Hypnic headache. tion of the parasympathetic system. E. Trigeminal neuralgia. Cluster headaches, paroxysmal hemicrania, F. Glossopharyngeal neuralgia. and SUNCT are described as the trigeminal auto- nomic cephalgias. Positron emission tomography Cluster headache: has shown activation of the posterior hypothala- Four characteristics of cluster headache include mus in all of them (Sprenger et al. 2004). On rare unilateral, mainly periorbital-temporal pain rarely occasions they may be symptomatic of underly- switching sides. Less often the pain manifests in ing structural pathology. There are instances of other locations such as upper or lower teeth, jaw, associated pituitary and paracavernous sinus cheek, nose, ear, and occipital area. The fre- lesions lending even more credence to a hypotha- quency is commonly two to six times per day. lamic origin. Internal carotid artery dissection has There are often autonomic features such as lacri- been discovered on a few occasions. mation, nasal congestion, conjunctival injection, eyelid swelling, ptosis, and Horner’s syndrome. Hypnic headaches: The etiology of the Horner’s syndrome may be Hypnic headaches usually occur in the older pop- edema of the internal carotid artery wall with ulation, nearly always over age 50 years. They compression of the cervical sympathetics. This erupt during nocturnal sleep and rarely during mechanism is supported by the absence of daytime naps. They tend to strike at the same anhidrosis in these patients. The headaches may time at night, last 15 min to 3 h, are of moderate- be episodic and seasonal or chronic. to-severe intensity, and generally described as The most useful diagnostic features are the throbbing or dull, bilateral, diffuse, or fronto- duration, 15 min to 2 h, and the patient’s behavior temporal. The majority of recorded cases have of restlessness and pacing the room. Conversely, occurred during REM sleep. Because of the the migraine patient prefers to lie down quietly ­unexpected prevalence in the older individual since movement aggravates pain. A smoking his- who may not have a history of headaches, struc- tory is extremely common. tural pathology must be excluded before the ­diagnosis is established. Paroxysmal hemicrania: These unilateral headaches have a strong simi- Trigeminal neuralgia: larity if not being nearly identical to cluster Trigeminal neuralgia is manifested by brief par- headache. The pain is usually fronto-temporal, oxysms (less than 2 min) of intense, sharp, lanci- excruciating, but last just 2–45 min and occur nating, or electricity-like pain in one, two, or all 20–40 times per day. Ordinarily they are associ- three divisions of the nerve. The second and third ated with autonomic symptoms of conjunctival divisions are most often affected. The examining injection, rhinorrhea, lacrimation, ptosis, and physician must be alert to the disorder since eyelid edema. Attacks may be provoked by many patients say that the pain is constant and head movement. They may also be episodic or may use other descriptive adjectives for the pain chronic but the episodic form can be diagnosed such as hot, twisting, or boring. A simple query only when there are remissions of at least regarding the presence of exacerbations will clar- 1 month. Genetic factors are important. ify the history. Triggering factors are nearly always present and include talking, chewing, SUNCT: shaving, and brushing the teeth. The examination These are severe, unilateral, orbital-temporal, is normal except for the frequent presence of stabbing headaches, or “pains” as many patients trigger points which when touched or pressed describe them. They last 5–250 s with a fre- provoke pain. An abnormal neurologic sign com- quency of 3–200 per day. At least one of the patible with a lesion in the cerebellopontine angle 216 10 Common Symptoms in the Neurology Clinic raises a high suspicion for neoplasm in that loca- Migraine tion. In patients under age 50, multiple sclerosis should be considered. Otherwise, compression A. Abortive of the nerve by an ectatic vascular loop, is the 1. Triptans (Loder 2010) – Pill, melt, nasal presumed etiology. The compression leads to spray, subcutaneous. demyelination, nerve irritability and possibly 2. Dihydroergotamine – Nasal spray, ephaptic transmission, which is nonsynaptic 0.5–1 mg IM, and 0.5–1 mg. I.V. q. 8 h for cross-talk between motor and sensory fibers. six doses preceded 20–30 min before with metoclopramide 10 mg IV each time. Glossopharyngeal neuralgia: 3. Isometheptene/dichloralphenazone/acet- Paroxysmal stabbing pains in the throat provoked aminophen (for mild migraine). by swallowing, talking, and coughing are the 4. Naproxen, ibuprofen, aspirin with or cardinal manifestations. Jacobson’s nerve, a without caffeine (for mild migraine). branch of the glossopharyngeal nerve, innervates 5. Diclofenac powder. the tympanic membrane, eustachian tube, and 6. Tramadol if options 1–5 are mastoid region. Thus, sharp ear pain is quite contraindicated. common and the disorder may present itself 7. Ketorolac – 60 mg IM or 30 mg IV, a solely with unilateral deep ear pain. Although the superb acute treatment. neuralgia is unilateral, the patient’s complaint 8. Valproate – 500 mg IV. will often refer to nonfocal throat pain which 9. Compazine (prochlorperazine) – 10 mg may obscure the diagnosis. Near-syncope, syn- p.o. or IV for headache or nausea; pro- cope, and convulsive syncope may occur since a methazine – IM only, for nausea. large volley of afferent neuronal discharges may 10. Metoclopramide 10 mg. I.V. q. 1/2 h × 4 incorporate the sensory fibers innervating the doses and diphenhydramine 25 mg. I.V. carotid sinus. This discharge, when reaching the with first and third dose. medulla, can provoke a reflex vagal response 11. Magnesium sulfate 1 g IV. causing bradycardia, rarely asystole, and result 12. Butorphanol nasal spray (last resort to in hypotension. The etiology is presumed to be prevent an Emergency Room visit). compression of the nerve by a vascular loop, 13. Avoidance of narcotics such as meperi- especially the posterior inferior cerebellar artery. dine, morphine, and hydromorphone Ephaptic transmission, cross-talk (non-synaptic) which usually require repetitive dosing between motor and sensory fibers may be the and are less effective than ketorolac as physiologic mechanism. well as possibly leading to addiction. Treatment options for headache disorders are B. Prophylaxis numerous. Drug selection often depends on 1. Beta blockers comorbid conditions which can be treated simul- a. Propranolol taneously, such as obesity. Avoidance of some b. Nadolol medications is critical such as beta blockers in a c. Timolol patient with bradycardia or asthma. The aim of 2. Anticonvulsants this brief and cursory introduction to treatment is a. Topiramate merely to acquaint the student or resident with b. Valproate the wide range of available therapies, both medi- c. Pregabalin cal and nonmedical. Dosing methods, adverse d. Gabapentin reactions, contraindications, and drug interac- 3. Tricyclics tions will not be included because of space limi- a. Nortriptyline tations. Obviously this list contains my personal b. Amitriptyline preferences (2011). c. Doxepin Headache 217

4. Miscellaneous Cluster Headache a. Duloxetine b. Verapamil A. Medications c. Lisinopril 1. Abortive d. Magnesium a. Oxygen 7–10 L/min by mask for e. Riboflavin 10–15 min. 5. Botulinum toxin b. Sumatriptan subcutaneous and possi- 6. Biofeedback bly nasal spray. C. Treatment of comorbidity 2. Prophylaxis Since depression is a frequent comorbid a. Corticosteroids for 3–4 weeks with factor, the addition of an SSRI or SNRI is tapering dose. often essential but is generally not efficacious b. Verapamil. as sole therapy. Dietary and hormone manip- c. Combined verapamil and corticoster- ulations are infrequently useful. When oids at treatment initiation. patients have hypertension or a tachycardia, d. Indomethacin. beta blockers are a good selection. When e. Lithium (for chronic cluster). obesity is a major issue topiramate is the ideal B. Surgical options (rare) treatment. 1. Radiofrequency lesion of trigeminal D. Hospital admission nerve. In-hospital treatment for 48–72 h should 2. Deep brain stimulation, still experimental. remain an option in cases of treatment failure. A protocol using DHE/Reglan combined with scheduled ketorolac IV is particularly useful. Chronic Paroxysmal Hemicrania

Trigeminal Neuralgia A. Prophylaxis only. 1. Indomethacin. A. Medications. 2. Verapamil. 1. Oxcarbazepine, carbamazepine, gabapentin, 3. Other anti-inflammatory medicines: aspi­rin, baclofen, and lamotrigine. diclofenac, naproxen and prednisone. B. Cyberknife, gammaknife radiotherapy, and radiofrequency lesions. C. Craniotomy with microvascular decompres- Hypnic Headache sion of the trigeminal nerve. D. Selection of one of the above options depends A. Abortive – caffeine. on numerous factors including age, drug B. Prophylaxis – indomethacin, lithium. complications, contraindications, and patient preference. Short-Lasting, Unilateral Neuralgiform Temporal Arteritis Headaches with Conjunctival Injection and Tearing (SUNCT) Corticosteroids are required daily for a minimum of 6 months and often lifelong at low dosage. A. Prophylaxis There is no satisfactory alternative. The initial 1. Lamotrigine. dose ranges from 60–100 mg with a gradual taper 2. Prednisone, gabapentin depending on headache and sedimentation rate 3. Microvascular decompression of the plus occasionally c-reactive protein. trigeminal nerve. 218 10 Common Symptoms in the Neurology Clinic

Glossopharyngeal Neuralgia but the remaining history and examination exclude this diagnosis. A. Prophylaxis only 3. Rupture of a thoracic meningeal diverticula 1. Oxcarbazepine can be demonstrated by CT/myelogram as 2. Carbamazepine contrast extravasates in the region of the CSF 3. Gabapentin leak. Radionuclide cisternography is less B. Surgical option is intracranial section of the precise. IX cranial nerve. 4. The abnormal CSF findings are due to decreased CSF volume. Case 14 A 33-year-old obese woman complains 5. The diagnosis is spontaneous intracranial of an intractable headache for 3 months. A few hypotension (Schievink 2006). Surgical repair weeks before the headache occurred she noted may cure this disorder. intermittent interscapular pain. Both the inter- scapular pain and the headache have persisted Case 15 A 35-year-old obese woman complains and occur only in the upright position. The sever- of a 4-month history of moderately severe gener- ity has gradually increased and has forced her to alized headache. She has periodic exacerbations remain in bed. associated with nausea and vomiting, episodic Neurologic examination is normal other than tinnitus, and occasional diplopia primarily when mild end nuchal rigidity. driving. She notes overlapping images of an A lumbar puncture yields clear fluid with approaching car at distance but the image 52 white cells /cu mm and a protein of 135 mg/dl. becomes single when the car is about one-half Questions: block away. 1. What additional information should be Past medical history: obtained from the lumbar puncture? The patient had an unexplained fever 6 months 2. What neuroimaging test may be diagnostic? ago for which she was hospitalized. Blood, urine, 3. What is the most common cause and how is it spinal fluid, and throat cultures were negative. A diagnosed? vasculitis evaluation was normal. She had a wbc 4. Why is the CSF abnormal? count of 16,000/cu mm with 87% neutrophils. 5. What is the diagnosis? Treatment with ampicillin was curative in 5 days. Answers: She does not recall why it was given. 1. What was the opening pressure? CSF pres- Neurologic examination: sure is low when it is less than 70 mm of H O. 2 Blood pressure is 130/95. Funduscopic examina- This patient’s opening pressure was 50 mm tion discloses bilateral blurred disk margins, H O. 2 absent venous pulsations, and a few small splinter 2. MRI (head) with contrast discloses diffuse hemorrhages at the disk margins. Red glass test- dural enhancement, present in about 90% of ing reveals diplopia on left lateral gaze. When the patients who have low intracranial pressure. red glass covers the right eye the image to the left The cerebellar tonsils may drop below the is white. CAT scan (head) is normal. foramen magnum which may cause nuchal rigidity which this patient demonstrated. Questions: Subdural fluid collections may develop. 1. Why does the patient have diplopia at Hence, a CT (head) would only provide distance? incomplete information and lead to inappro- 2. What does the red glass test demonstrate? priate surgical intervention. Only a thorough 3. What diagnosis is suspected? history uncovering the presence of orthostatic 4. Is the past medical history relevant? headache will prevent a therapeutic misadven- 5. What test is next? What is its most important ture. Obesity in a young woman always result? prompts consideration of pseudotumor cerebri 6. What examination should follow? Vertigo 219

Answers: and flocculus. There is vestibular representation 1. Diplopia at distance is common with 6th nerve in the cerebral cortex which is probably multifo- palsy since the eyes must diverge at distance. cal. The incidence of vertigo as a presenting 2. Left lateral rectus paresis. The distal image symptom of cerebral hemisphere disease is negli- comes from the weak muscle. gible, nearly always as a simple partial seizure 3. Pseudotumor cerebri (Friedman and Jacobsen with or without a secondary grand mal convul- 2002). sion. Consequently, the focus of the assessment 4. Yes. This patient had an otitis media. of vertigo must be on the differentiation between 5. Lumbar puncture. The opening pressure is the the major central and peripheral vestibular con- critical finding. nections and neurons as outlined above. 6. MRI (head) and MRV. The latter test reveals a The vestibular triad is composed of vertigo, lateral sinus thrombosis. nausea and/or vomiting, and diaphoresis (an autonomic component). These symptoms are the Final diagnosis: hallmark complex of vestibular system diseases. Increased intracranial pressure secondary to a Fortunately, patients seldom camouflage its iden- cerebral venous thrombosis (lateral sinus). tification with verbal obfuscation. If the sponta- Cases 14 and 15 are unusual examples of new neous complaints are vague a simple query such daily persistent headaches which are often major as “Do your surroundings seem to move or do diagnostic challenges. By definition the headache you sense your body moving?” may suffice to should be present for at least 3 months to be sub- evoke a diagnostic reply. Especially helpful addi- sumed under this category. Perhaps the most tional questions include, “Do the symptoms occur common etiology would be the development of as you lie down or turn over in bed?” or “Do they transformed migraine. In the elderly patient, tem- occur as you rise up from a supine to a sitting poral arteritis is always a consideration. position or when moving from a sitting to a standing position?” “Does the environment seem to shift when you move your head?” Vertigo The neurologic signs which may accompany these symptoms are nystagmus and postural Dizziness is one of the most frequent complaints instability. Usually, one of these is present but in the general medical and neurologic clinic. This with peripheral vestibular disorders, both may be symptom usually revolves about five disorders absent. A brief review of vestibular physiology which include vertigo (vestibular system patho­ may be helpful. logy), visual impairments, imbalance or gait A visible and measurable expression of the disorders, near-syncope (lightheadedness), and vestibular system is the generation of slow eye psychogenic factors (lightheadedness). This sec- movements. The visual system also produces tion will focus on vertigo. The other etiologies of slow eye movements through visual fixation and dizziness will be covered under other categories. ocular pursuit. These are the two primary afferent Vertigo, an objective or subjective illusory pathways which evoke eye movements involving sensation of movement, is the prototypical symp- slow system pathways. tom of the patient with a disorder of the vestibu- The origin of the vestibular afferents resides in lar system (Baloh and Kerber 2010). The anatomic the otolith organs, utricle and saccule, and the basis of vertigo is the peripheral vestibular appa- three semicircular canals which are the anterior, ratus, the semicircular canals and otolith organs, posterior, and horizontal. The otolith organs the 8th cranial nerve, the vestibular nuclei, and mediate linear motion and orientation according pathways within the brainstem to the cerebellum to gravity. The utricle senses movement in the and midline structures within the cerebellum. horizontal plane and the saccule in the sagittal Those pathways to the cerebellum travel primar- plane. The semicircular canals contain an ampulla ily to the inferior portion, especially the nodulus within which is the cupula sealing the canal. 220 10 Common Symptoms in the Neurology Clinic

When the head moves, the cupula bends and acti- an hour. One hour later, in the Emergency Room, vates the underlying hair cells innervated by a his examinations followed by a non-contrast CAT branch of the 8th nerve, a semicircular canal scan (brain) were normal. nerve. Thus, there is a transduction of rotational The following day the patient complained of movement into neural activity. vertigo. Specific questioning elicited a comment The left 8th nerve generates slow eye move- that he especially noted vertigo when he lay down ments to the right and vice versa. Therefore, sym- or got up from bed. A complete neurologic exam- metrical function results in no ocular deviation ination was normal. Past medical history was with eyes open or closed. If the left 8th nerve is negative. damaged, for example, the right 8th nerve is unop- Questions: posed. This generates eye movements to the left. 1. What is the next step? In the brainstem, the paramedian pontine reticular 2. What is the diagnosis? formation (PPRF) produces a compensatory quick 3. What treatment cures this disorder? response back to a central point of fixation, i.e., contralateral nystagmus. This occurs when the Case 17 A 57-year-old woman complains of eyes are closed but, if they are open, visual fixa- episodic vertigo lasting 15–20 min associated on a tion may override eye deviation induced by ves- few occasions with a brief loss of consciousness. tibular dysfunction thus preventing the observation The vertigo has occurred once or twice per month of the compensatory nystagmus. over the previous 6 months. The duration varies To summarize, an acute left 8th nerve lesion from 15 min to 2 h but is usually 30–60 min. generates right-beating nystagmus with a slight Witnesses have observed slurred speech and an rotatory counterclockwise element; this torsional unsteady, sometimes lurching gait. Past medical component is the summation of vectors produced history is remarkable for insulin-dependent dia- by the three semicircular canals. Central compen- betes mellitus. sation develops slowly, gradually, and inevitably. Neurologic examination is normal. Thus, the standard neurologic examination com- Questions: monly uncovers no abnormal signs even though 1. What questions may elicit a diagnostic answer the patient complains of typical vertigo. both from the present illness and past medical Special bedside examination techniques are history? required to assess these patients. The techniques 2. What is the diagnosis? are described in Chap. 4 (neurologic examination). These include observation of saccadic refixations, Case 18 A 56-year-old nurse who works in an the head thrust test, induction of oscillopsia by adjoining clinic arrives 10 min after the abrupt head shake, dynamic visual acuity test, stepping onset of severe vomiting, vertigo, and diaphore- (Fukuda) test, and Romberg and Dix–Hallpike sis (the vestibular triad). Past medical history is tests (see Chap. 4 for details). The examinations remarkable for hypertension under good control are quick and easy to accomplish. Other options with metoprolol. He has had prior episodes last- are pastpointing, head shake nystagmus, caloric ing as long as several hours. On one occasion he testing, and use of Frenzel glasses. Finally, an had an abrupt fall without explanation. indeterminate examination mandates a referral Neurologic examination: for electronystagmography (Fife et al. 2000). Blood pressure 140/95. Pulse is 56 and regular. Five of the most common disorders which cause Right-beating nystagmus with a slight counter- vertigo are described in the following case reports. clockwise rotation and an ataxic gait are noted. Answers will be given after the fifth report. The Romberg test is positive. A stat CT (brain) is Case 16 A 24-year-old longshoreman fell off a normal. Several hours later, MRI (brain) and platform striking the back of his head. He was MRA (head and neck) are normal. unconscious for approximately 10–15 s and Two hours after he is first evaluated, nystag- thought to be dazed plus mildly confused for half mus beats left on left lateral gaze with a slight Vertigo 221 rotatory clockwise component. With up and down Table 10.1 Duration of vertigo gaze the same nystagmus occurs. On right lateral BPPV Seconds gaze there is no nystagmus. The Romberg test is TIA – vertebrobasilar Few to several minutes positive. Migraine 5–60 min Ménière’s disease Hours Questions: Vestibular neuritis Days to weeks 1. What additional history should be obtained? 2. What diagnosis explains the features of this patient’s nystagmus? Neurologic examination: Case 19 A 67-year-old woman has the acute Blood pressure 130/80. Pulse 72, regular. There onset of vertigo, vomiting, and diaphoresis on a are no neurologic abnormalities. Saturday. Following treatment with over-the- Questions: counter meclizine, she feels sufficiently improved 1. What is the explanation of his visual over the next 48 h to return to work. She has a symptoms? residual feeling of unsteadiness and, when driv- 2. What anatomic areas are affected? ing and looking over her shoulder to change lanes 3. What is the diagnosis? cars seem to bounce. She is referred for a neuro- Answers: logic consultation. Past medical history is remark- The primary clue to the diagnosis of patients with able for insulin-dependent diabetes mellitus and vertigo (Furman and Cass 1999) is the duration an asymptomatic, critical right internal carotid of symptoms (Table 10.1). artery stenosis treated by endarterectomy. Neurologic examination discloses nystagmus Case 16: to the right on right lateral, upward, downward, 1. The Dix–Hallpike maneuver is performed and and direct forward gaze. The entire neurologic elicits torsional nystagmus for 10 s in the examination is otherwise normal. Non-contrast right-sided position after a latency of 3 s. CAT scan of the brain is negative. Subsequent 2. This indicates the presence of otoconia (cal- MRI (brain) and MRA (head and neck) discloses cium carbonate particles) in the right posterior T2 and FLAIR hyperintensities in subcortical semicircular canal. The otoconia were dis- white matter, brainstem, and cerebellum. The lodged from the tips of hair cells in the utricle MRA reveals mild basilar artery and bilateral by the head injury. supraclinoid carotid artery stenoses. The diagnosis is benign paroxysmal posi- tion ­vertigo (BPPV). Questions: 3. A canalith repositioning procedure (modified 1. What bedside neuro-otologic tests are likely to Epley maneuver) cures the patient (Fig. 10.1). be abnormal? 2. What could the cerebrospinal fluid studies show? Diagnostic key: 3. What electrophysiologic test may be diagnostic? Duration of seconds. 4. What is the diagnosis and the anatomic site of Case 17: pathology? 1. The patient is asked: “Do you have any dis- Case 20 A 62-year-old man is referred because comfort on your head?” “Have you ever had of a single episode of severe vertigo followed by severe headaches?” She responds: “My head loss of vision O.D. He describes the loss of vision feels sore after the vertigo subsides.” “I used as blank or “nothing there” but not black. Both to have sick headaches in my 20 s.” symptoms lasted 3 min. There was no headache. 2. Basilar migraine. Additional symptoms which Past medical history includes hypertension may occur include drop attacks, perioral and which is being treated successfully with lisinopril. limb paresthesias often manifested as a “march There is no history of migraine or severe of paresthesias” over several seconds to headaches. 15–20 min beginning in the hand and gradually 222 10 Common Symptoms in the Neurology Clinic

Fig. 10.1 Treatment of benign paroxysmal positional Position 2: While the patient’s head remains tilted back vertigo by the canalith repositioning procedure (modified his head is rotated 45° to the right. This position is main- Epley). In this example the left side is symptomatic. tained for 30 s or until the dizziness stops, whichever is S starting position. Position 1: The patient’s head is turned longer. Position 3: The patient is rolled over onto his right 45° to the left and then the patient is assisted to the supine shoulder so that his head is turned 45° down. This position position with his head tilted back over the edge of the is maintained for 30 s or until the dizziness stops, which- examining table. The patient is kept in this position for ever is longer. Position 4: The patient sits up slowly with 30 s or until the dizziness stops, whichever is longer. his head rotated to the right Vertigo 223

spreading up to the face. Visual scotomata, Solu-Medrol (40–60 mg) usually for 2–4 days, photopsias, and tinnitus are common. promethazine, meclizine. Diagnostic key: Diagnostic key: Duration of 5–60 min and recurrent attacks. Duration of symptoms varies from several days to weeks with a gradual resolution of symp­ Case 18: tomatology. Residual instability may last up to a. The patient is asked: “Do you have ringing 3 months. noises or a pressure sensation in the ears?” “Do you have any loss of hearing?” Reply: Case 20: “Yes, I have ringing in my right ear and my a. Loss of vision associated with occipital isch- hearing is slightly impaired on the right side.” emia is commonly described as blank or b. Eye movement abnormalities in an acute absent. Many people with homonymous hemi- Ménière’s attack, which is rarely observed, anopsias perceive this as affecting the eye are due to irritative phenomena resulting in which perceives the involved temporal field ipsilateral nystagmus. Later on there is con- which is larger than the nasal field. tralateral nystagmus, direction-fixed which b. Brainstem because of severe vertigo and obeys Alexander’s law. This states that nys- occipital lobe due to absence rather than dark- tagmus is most prominent when the eyes are ness or blackness of vision. deviated toward the direction of the quick c. The brief episode of 3 min is compatible with phase. The sudden drop attack is known as an a transient ischemic attack and the basilar otolithic crisis indicating involvement of this artery supplies the brainstem plus the occipital structure. Diuretics are usually given and an lobe via the posterior cerebral arteries, the left ENT referral is essential for management of being involved in this case. The etiology is Meniere’s disease. basilar artery stenosis producing brainstem ischemia resulting in vertigo and left occipital Diagnostic key: lobe ischemia. The latter results in a right Duration is usually in hours. homonymous hemianopsia, misinterpreted by Case 19: the patient as amaurosis O.D. a. Head shake at two cps provokes oscillopsia. Diagnostic key: The dynamic visual acuity test results in a Duration of symptoms is usually several minutes decline of visual acuity from 20/20 to 20/100. and in a vascular distribution. In this instance, the The head thrust test (Halmagyi and Curthoys simultaneous involvement of two separate struc- 1988) is abnormal from right to left as it pro- tures confirms the diagnosis. duces a saccadic refixation from left to right. Six other cases which are less often seen but b. Cerebrospinal fluid studies disclose 15 lym- easily recognized should also be kept in mind. phocytes/cu mm. A seventh patient with a rare eye movement dis- c. Electronystagmography demonstrates unilat- order will be added. Two or three case reports eral left-sided caloric weakness. will be presented consecutively followed by the d. Vestibular neuritis, left side (Hotson and Baloh explanations to the posed questions. This will 1998). Although a branch occlusion of the prevent the reader from immediately glancing at anterior inferior cerebellar artery could theo- the answers. retically cause this clinical picture, it would be quite rare and the spinal fluid findings support Case 21 A 50-year-old geologist complains of a viral etiology. The site of pathology is the dizziness and an unsteady gait which is worse in ampullary branches of the 8th cranial nerve. the dark. The environment seems to shift when he Treatment options include a 1–2-week course walks. These symptoms began 6 months ago after of a tapering dose of prednisone, low dose I.V. returning from a working trip to Venezuela where 224 10 Common Symptoms in the Neurology Clinic he was mapping regions for oil exploration. He Case 22: developed pneumonia, was treated with unknown a. Brief duration and stereotyped, frequent antibiotics and recovered quickly. Past medical symptoms. No. Concomitant medical illnesses history is negative. are not relevant. The history of the present ill- Neurologic examination reveals easily induced ness and the neurologic examination will oscillopsia, an abnormal dynamic visual acuity determine whether they are germane. test with his acuity declining from 20/20 to b. Electroencephalogram (EEG). 20/200 with head shake at two cps. He has a posi- c. Simple partial seizures. tive Romberg and a wide-based gait. d. Neoplasm. This patient had an extra-axial (out- side brain) mass overlying the posterior part Questions: of the insula. Extra-axial, well-circumscribed a. Where is the lesion? mass lesions are usually benign such as the b. What bedside test confirms the anatomic meningioma which this patient had resected location? successfully. c. What is the diagnosis? Case 23 A 50-year-old man complains of dis- Case 22 A 56-year-old woman has a 2-year equilibrium for 6 months. At the onset he had history of periodic intense vertigo associated vertigo, vomiting, and excessive perspiration with blurred vision lasting 2 min each time. The which resolved over 1 week. Past medical history episodes are identical and occur irregularly, once includes insulin-dependent diabetes mellitus, per month to several times per week. They have hypertension, and hypothyroidism. In addition to occurred supine, sitting, or standing. The frequency insulin the patient is being treated with amlo- has been increasing. She briefly lost consciousness dipine and thyroid replacement. on one occasion when it was particularly severe. Neurologic examination: Past medical history: Blood pressure is 140/100 and pulse 62 and Diabetes type II and hypertension. ­regular. Funduscopic examination O.U. reveals Medications: numerous scattered hemorrhages and exudates. metformin and hydrochlorothiazide. Venous pulsations are seen. The visual acuity is 20/100 O.U. due to macular disease and is uncor- Neurologic examination: rectable. Pupillary reactions are normal. He has a Blood pressure 160/110 and pulse 50 and regular. positive head thrust test, left to right. There are no The neurologic examination is normal. ankle reflexes, vibration sense loss is noted at the Questions: toes and ankles and a few position sense errors a. What symptoms are the key to the diagnosis? are made at the toes. The Romberg test is Is the past medical history relevant to the positive. diagnosis? Questions: b. What physiologic test is diagnostic? a. What illness caused this patient’s vertigo? c. What is the diagnosis? b. What conditions are contributing to his d. What etiology must always be suspected? disequilibrium? Answers: c. What is the significance of the funduscopic Case 21: findings? a. Semicircular canals or 8th nerve. d. Are the normal pupillary reactions expected? b. Caloric testing shows nearly absent responses e. What is the diagnosis? bilaterally. An electronystagmogram may be Case 24 A 32-year-old man complains of ver- used to quantify the deficit. tigo and right-sided neck pain after playing soc- c. Vestibular ototoxicity (Furman and Cass 1996) cer. He successfully scored the winning goal by of aminoglycosides is due to damage of ves- heading the ball into the goal from a corner kick. tibular branches of the 8th cranial nerve. The following day he saw his chiropractor Vertigo 225 because of a stiff neck. On his way home, shortly d. Right vertebral artery dissection causing isch- after treatment by neck adjustments, he had the emia in the right posterior inferior cerebellar onset of severe vertigo, nausea, and right facial artery distribution. This resulted in a right- numbness. His right-sided neck pain became sided Wallenberg’s syndrome or dorsolateral more intense. When eating he found it more com- medullary infarction. fortable to tilt his head to the left side. Case 25 A 77-year-old woman complains of epi- Questions: sodic vertigo for 1 month. Each spell lasts 5 min a. Which side of his body was likely to be numb? and, over the last week, she has averaged eight to b. What two neuro-ophthalmologic signs might ten per day. She has nausea but no vomiting. have been found? Additionally, she has noticed ringing and mild c. Why did he tilt his head to the left at mealtime? loss of hearing A.S. d. What is the diagnosis? Neurologic examination: Answers: There is slow large amplitude nystagmus beating Case 23: left on left lateral gaze and rapid low amplitude a. Vestibular neuritis. This history of vertigo, nystagmus beating right on right lateral gaze. vomiting, and diaphoresis gradually resolving Rinne’s test is negative on the left and Weber’s over several days is a typical history for this lateralizes right. disorder. Questions: b. Residual right 8th nerve disease, macular a. What cranial nerve and motor function should degeneration, and diabetic neuropathy. be evaluated carefully? c. They are stigmata of hypertension and diabe- b. What is the primary diagnosis to consider? tes. Hemorrhages are not at the disc margins and venous pulsations are present. Hence this Case 26 A 45-year-old man complains of vertigo is not papilledema. and double vision, “side-by-side.” These symptoms d. Yes. Pupillary reactions are normal in macular began 2 weeks ago and are beginning to subside. disease. Neurologic examination: e. Multisensory disequilibrium (Brandt and Blood pressure is 160/90. Pulse 90, regular. When Daroff 1980). the examiner uses a penlight, the patient notes Case 24: horizontal diplopia on left lateral gaze. With a red a. The left side. Vertigo plus right facial numbness glass covering O.D. the image on the far left is point to a right-sided brainstem lesion. The the red light. Slow, large amplitude nystagmus is latter typically produces crossed findings; right noted O.S. which beats to the left. There are facial numbness indicates involvement of the clumsy rapid alternating movements of the left ipsilateral spinal tract of the 5th cranial nerve hand, mild weakness of left interossei and wrist and is frequently accompanied by left hemihy- extension, and a left Babinski sign. pesthesia due to involvement of the crossed Questions: fibers of the lateral spinothalamic tract. a. What symptom localizes the site of the ves- b. Horner’s syndrome and horizontal gaze- tibular dysfunction? evoked nystagmus. The Horner’s syndrome is b. Are the findings consistent with a unilateral right-sided since sympathetic fibers remain anatomic site? ipsilateral throughout their anatomic pathway. c. What diagnosis is suspected but not proven? Nystagmus is caused by damage to the ves- tibular nuclei. Case 27 A 17-year-old female complains of per- c. Probably to improve swallowing since the ceiving a constant movement of her surroundings right soft palate is weak due to an infarction which seem to bounce around. These oscillations affecting the right nucleus ambiguus in the began 3 days ago and were accompanied by an medulla. occipital headache which lasted 1 day. Past medical 226 10 Common Symptoms in the Neurology Clinic

Fig. 10.2 Lateral view of the brainstem and cerebellopontine angle history is entirely negative. She takes no medica- Answers: tion and does not use recreational drugs. Case 25: Neurologic examination: a. Fifth, 7th, and 8th cranial nerves and cerebellar The blood pressure is 90/60 and pulse 90, regular. function. These are structures in the region of The patient exhibits constant, chaotic saccadic the cerebellopontine angle (Fig. 10.2). Cerebellar eye movements on direct forward gaze with function should be assessed with finger-to-nose attempts at visual fixation or in any gaze direc- and heel-to-shin testing, gait and tandem gait. tion. During sleep the eyes can be seen to be The vestibular component of the 8th nerve can moving in a similar fashion under closed lids or quickly be examined via the head thrust test. gentle elevation of the upper lids. An occasional b. Acoustic neuroma. brief jerk of either arm is seen. Case 26: Questions: a. Diplopia. This is a common symptom of brain- a. What is the name of this eye movement disor- stem lesions. der? What other phenomenon may occur b. Yes. The patient has a right-sided internuclear simultaneously? ophthalmoplegia (INO) (lesion of the right b. Where is the lesion? medial longitudinal fasciculus) because of c. What diagnoses should be considered. right medial rectus weakness discovered by Near-Syncope, Syncope, and Seizure 227

the red glass test. A right pons lesion may cause be challenged to identify the meaning of this left-sided motor signs, weakness, and clumsi- complaint. A standard method can be useful and ness of the left arm and left hyperreflexia due this will be outlined. to a right-sided corticospinal tract lesion. Since “blackout” has numerous interpretations, c. Multiple sclerosis. An internuclear ophthal- problem one is to distinguish between visual loss, moplegia (INO) is a characteristic sign of mul- dizziness, loss of consciousness, or other cata- tiple sclerosis. Multiple sclerosis cannot be strophic neurologic symptoms for which the patient proven, however, without either more than one uses this term. Once loss or near-loss of conscious- lesion or two lesions separated in space and ness is established, the next task is to meticulously time. Confirmation of the diagnosis will require review specific elements of the history. additional supportive abnormalities through General considerations: neuroimaging by MRI and/or cerebrospinal 1. Was the patient supine, sitting, standing, or fluid studies and/or evoked potentials. walking? Case 27: 2. What was the patient doing? For example, a. Opsoclonus, myoclonus. weightlifting? b. Cerebellum or brainstem, midbrain, and infre- 3. What time did it occur? For example, was it at quently pons. night after getting up to go the bathroom? c. Viral brainstem/cerebellar encephalitis is 4. What was the time relation to meals? probably the most common etiology. 5. What medicine, substance, liquid, or food was Paraneoplastic syndromes, especially asso- consumed and how long before this event ciated with neuroblastoma in children, small occurred? cell lung cancer, and breast cancer. 6. Was there any warning or symptom immedi- Multiple sclerosis. ately before loss of consciousness? Vascular disease including midbrain and Specific questions: pontine hemorrhage or infarction. 1. Was there dizziness, lightheadedness, a change Cerebellar , hereditary. in vision (especially dimness), sweating, or Hyperosmolar nonketotic coma. nausea prior to loss of consciousness? (Typical Toxic effect of drugs. of syncope). 2. Did this occur after urination or defecation? (Common before syncope). Near-Syncope, Syncope, and Seizure 3. Was there an unusual smell, taste, distortion of vision, or hearing? (Typical of seizure). 4. Was there déjà vu, jamais vu (strangeness of In the absence of head trauma, an episode of loss surroundings), forced thinking, or other psychic of consciousness is either syncope or seizure. experience? (Typical for seizure). Syncope (Grubb 1997) is by far more common and usually easily recognized. If the event is unwit- Duration: nessed and the patient is neither intelligible nor Duration of each symptom is critical towards perceptive, the diagnosis may be far from appar- making a diagnosis. For example, an unusual ent. Near-syncope is included in this discussion taste that lasts 2 h is unlikely to be due to a sei- since there are illnesses and syndromes which zure whereas that same taste which lasts 30 s to eventuate in loss of consciousness if not recog- 1 min might indicate a seizure. Consequently, the nized early and treated. patient’s reply to this question is of the utmost When the patient arrives at the clinic with a diagnostic significance. chief complaint of “I’m having blackouts,” the Responses to any of these questions may trig- physician may shudder because this symptom ger the need to pursue details. For example, the often requires a lengthy history which interferes patient may say that things look “different.” with his usual patient flow. Conversely, he may Follow-up questions might be: “Did they seem to 228 10 Common Symptoms in the Neurology Clinic be at a distance?” (teleopsia), “Did objects loom Case 29 A 32-year-old man is referred by a phy- up close?” (pelopsia), “Were they small or large?” sician’s assistant because of three seizures over (micropsia or macropsia). These examples are the past 10 days. The patient is unable to supply illusions. “Did you see things which were not any detailed information. He had no warning and actually there?” (hallucinations). If present, were his last recollection is standing next to the kitchen they patterns or geometrical figures or were they sink. He had been in a motor-vehicle accident animals, people or well-formed objects? Episodes 6 weeks previously and suffered a brief loss of of these visual disturbances, illusions, or halluci- consciousness of about 1 min with subsequent con- nations are strong indicators of simple partial fusion for a few minutes. He had no local injury and seizures if they are brief in duration. a CAT scan (head) without contrast was normal. Auditory hallucinations of epileptiform origin The patient’s wife witnessed all three of these can be verbal or nonverbal but not accusatory or spells. She reports that two episodes occurred commanding as occurs with psychoses. Examples before meals and one afterward. Two occurred of nonverbal noises are rumbling, machinery- sitting and one while standing. The most recent like, and whistling. Indistinct voices can be heard one lasted for 10 min during which time his arms and are often described as muffled. and legs jerked uncontrollably. Afterwards he Finally, and sometimes of greatest importance, seemed confused for 1 min. was there a witness to these occurrences? (Thijs a. Assuming the patient’s wife is a good observer, et al. 2008). A yes response requires immediate what specific questions might elucidate an contact with that individual preferably while the accurate diagnosis? patient is in the examining room or office. A b. If a physician is present, what examination delay in contacting the witness increases the techniques could establish a diagnosis? chance that this person will forget a pertinent detail of the event. Case 30 A 15-year-old girl was reported to have The case reports to follow will review some of passed out at school. She said she was feeling these points and explore other means of making a dizzy and noted blurry vision after rushing to her diagnosis. These will be presented consecutively math class for the final exam at 2:00 p.m. Her three at a time followed by the answers. teacher observed her to fall as she was about to sit down at her desk. She stiffened and then Case 28 A 17-year-old boy is referred to a began jerking all extremities. Simultaneously ­neurologist because of unexplained nausea and she lost control of her urine. In less than 1 min abdominal discomfort. The episodes began 6 she was alert and oriented. Shortly afterwards a months ago and are increasing in frequency to a general physical and neurologic examination few times per week. He has already seen his were normal. internist and two gastroenterologists. Endoscopy, a. What additional history would be useful? colonoscopy, and thorough stool evaluations b. What is the diagnosis? were entirely normal. A psychiatric assessment Answers: revealed a perfectly normal young man who is becoming increasingly anxious about his serious Case 28: symptoms. a. How long does the nausea and abdominal dis- a. What is the first question asked which virtu- comfort last? The nausea lasts 5–10 s, rarely ally makes the diagnosis? up to 1 min. This is a typical time course for b. What is the second question which secures the simple partial seizures. diagnosis? b. Does the nausea and discomfort travel? The c. What is the diagnosis and what examinations patient points to the epigastric region and ges- are indicated? tures upward indicating the location and sense d. If the examinations are normal should treat- of movement. Rising epigastric distress, occa- ment be initiated? sionally interpreted as nausea by some Near-Syncope, Syncope, and Seizure 229

patients, is a relatively common ictal sensa- 5. Are there undulating body movements? This tion. This simple partial seizure, previously is more common with pseudoseizures, rare called an aura, frequently precedes a grand with seizures. mal seizure. This sensation arises from an epi- leptiform discharge in the hippocampus. Case 30: c. The diagnosis is simple partial seizures and Since the student was rushing to take the final both an EEG and MRI (head) with and with- exam, perhaps she stayed up all night to study or out contrast should be performed. omitted meals. Presumably she was anxious. This d. Both studies are commonly normal in patients would be a perfect scenario for neurocardiogenic with epilepsy. Estimates of the incidence of a syncope (neurally mediated syncope or vasova- normal EEG in these circumstances is approxi- gal syncope). The presence of convulsive move- mately 25–50%. Treatment should be initiated. ments and urinary incontinence suggest seizure but the very brief postictal confusion is incom- Case 29: patible with a diagnosis of a grand mal seizure. a.1. Were his eyes open or closed? Psychogenic seizures (pseudoseizures) are usually asso- Diagnosis: ciated with eye closure whereas patients Convulsive syncope, neurocardiogenic (vasovagal) who have epileptic seizures ordinarily have (Grubb 2005). their eyes open (Chung et al. 2006). Neurocardiogenic syncope is characterized by 2. Was he blue or pale? Cyanosis is common a slew of premonitory symptoms and signs which with grand mal seizures and pallor with include: hyperventilation, lightheadedness, dim- syncope. ming or blurring of vision, nausea, pallor, sweat- 3. Did he bite his tongue? The lateral margins ing, muffled hearing, and generalized weakness. of the tongue are bitten with seizures. Tachycardia and an elevated blood pressure may 4. Was he incontinent? This is uncommon with precede these symptoms briefly. Bradycardia and pseudoseizures and common with grand mal hypotension quickly follow. The basis for these seizures. physiologic phenomena remains controversial. 5. Were the three spells you witnessed identi- Some individuals probably have a predisposition cal or were they somewhat different each to peripheral venous pooling with consequent time? Stereotyped episodes are more likely poor venous return. This produces vigorous ven- to be seizure. tricular contractions which activate mechanore- 6. Was he jerking, stiff, or limp? Pseudoseizures ceptors in the aortic arch and carotid sinus which are commonly clonic but seldom tonic usually respond only to stretch due to hyperten- (stiff). sion. This results in paradoxical reduction in 7. Was there confusion afterwards and how arterial resistance and reflex bradycardia caused long did it last. Confusion after syncope is by increased vagal discharge. usually less than 1 minute but can be several Convulsions are the brain’s response to minutes if a severe hypotensive episode hypoxia. The degree of cerebral hypoxia deter- occurred. Confusion after a grand mal sei- mines whether convulsive activity will occur. zure is several minutes to hours. The estimated incidence of convulsive activity b.1. Check for resistance to eyelid opening. The in neurocardiogenic syncope is nearly 10%. presence of resistance suggests a psycho- Syncopal myoclonus is arrhythmic, symmetrical, genic etiology. or asymmetrical. It affects all limbs and muscle 2. Check for tachycardia which is usually pres- groups equally and commonly involves the face. ent with a seizure. The duration is usually less than 30 s. Tonic mus- 3. Is there cyanosis? This is usually present cle activity is manifested by extension of head with grand mal seizures. and body and either flexion or extension of the 4. Are there inappropriate emotions? If these are arms. Vocalization such as moaning and complex evident, one should suspect pseudoseizures. movements such as licking the lips, chewing 230 10 Common Symptoms in the Neurology Clinic movements, and reaching upward have been sciousness. On the last occasion she had been sit- reported and thus may confound the diagnosis. ting on the couch, then got up slowly and carefully Ocular manifestations include eye opening, down­ walked to the kitchen. She felt cold, clammy, dizzy, beat nystagmus, lateral, or upward eye deviation. and passed out. She was unconscious for less than a Visual hallucinations, both formed and unformed, minute, according to her observations, and denies and auditory hallucinations such as rushing or confusion. Her neurologic examination is normal. machinery-like sounds and vague, unintelligible a. What could be the pathophysiologic voices have been described. mechanism? Case 31 A 44-year-old stockbroker complains of Answers: periodic brief dizziness when using his computer. Case 31: During a recent conversation with a client he lost a. No. Despite the importance of the past medi- track of the conversation for about 30 s. This lack cal history the present illness must be scruti- of attention prompted a request for a neurologic nized in its entirety prior to employing the evaluation. clues provided by his medical background. Past medical history: The abnormal neurologic signs, however, are Insulin-dependent diabetes mellitus, heavy drinker, a consequence of his medical illnesses. and smokes 1-1/2 packs of cigarettes per day. b. The diagnostic test is an EEG with photic stim- ulation. The EEG revealed epileptiform dis- Neurologic examination: charges during photic stimulation. Although Blood pressure 160/100. There are no ankle epilepsy induced by reading is an additional reflexes and vibration sense is absent at the toes. remote consideration, the probable provocation a. Is the past medical history relevant to his by a rapidly changing screen is most likely. symptoms? b. What is the diagnostic test? Diagnosis: Simple and complex partial seizures (Wyllie et al. Case 32 A 38-year-old woman complains of the 2010). gradual onset of feeling faint when walking more A review of seizure classification is now than 5 min. She describes nausea, excessive perspi- appropriate. ration, shortness of breath, neck pain, and her vision darkens. She recovers promptly after sitting. She has insulin-dependent diabetes mellitus. ILAE* Classification of Epileptic Seizures Examination: (Modified) Blood pressure supine is 140/80, sitting 135/85, standing at 1 min 130/90, and after 3 min 125/85. I. Partial (focal) seizures. The pulse varies from 80 to 84 in all positions. A. Simple partial seizures (consciousness Neurologic examination reveals absent reflexes preserved). in the legs, absent vibration perception at toes 1. Motor: Focal, Jacksonian, versive and ankles. (turning), postural, and speech arrest or vocalization. Diagnosis: 2. Somatosensory and special senses: Near-syncope. Visual, auditory, gustatory, olfactory, a. Why does she have neck pain, shortness of and vertiginous. breath, and darkening of vision? b. What test is diagnostic? c. What physiologic parameters should be targeted *ILAE. International League Against Epilepsy. Modified on treatment? from Commission on Classification and Terminology of the International League Against Epilepsy. Proposal for Case 33 A 35-year-old woman is seen 1 week revised clinical and electroencephalographic classifica- postpartum because of five episodes of loss of con- tion of epileptic seizures. Epilepsia 1981; 22: 489–501. Near-Syncope, Syncope, and Seizure 231

3. Autonomic: Epigastric sensation, pal- 2. Childhood epilepsy with occipital spikes. lor, sweating, flushing, piloerection, 3. Primary reading epilepsy. tachycardia, and pupillary dilatation. B. Symptomatic. 4. Psychic symptomatology: Aphasia, 1. Temporal lobe. affective change, déjà vu, jamais vu a. Simple partial: Autonomic, psychic, spe- with depersonalization, derealization, cial sensory, and epigastric sensations. illusions, and structured hallucinations. b. Complex partial. B. Complex partial seizures (consciousness c. Mesial temporal sclerosis. impaired). 2. Frontal lobe. 1. With or without automatisms. a. Simple partial: tonic, postural, and Automatism examples are lip smack- psychic. ing, fiddling with clothes or objects. b. Complex partial: automatism. 2. May begin with simple partial seizures. c. Supplementary motor (medial hemi- C. Partial seizures with secondary grand mal sphere), fencing posture, and speech seizures. arrest. 1. Simple partial to grand mal. d. Secondary grand mal seizures, postic- 2. Complex partial to grand mal. tal paresis (Todd’s phenomenon). 3. Simple partial to complex partial to 3. Parietal. grand mal. a. Simple partial sensory with or without II. Generalized seizures (initial involvement of secondary grand mal. both cerebral hemispheres). b. Complex partial if there is spread to the A. Absence seizure (loss of consciousness at temporal or frontal lobe. onset). c. Positive or negative phenomena: 1. Impaired consciousness only. Paresthesias, occasionally Jacksonian 2. One or more of the following elements: or asomatognosia (loss of awareness of Clonic, atonic, tonic, automatism, and 1/2 of the body). autonomic. 4. Occipital. 3. Atypical absence. a. Negative vs. positive phenomena, sco- a. Marked changes in tone. toma, hemianopsia, amaurosis vs. b. Onset and/or cessation is not flashes of light, geometric forms. abrupt. b. Visual illusions. B. Myoclonic seizures. c. Structured hallucinations. C. Tonic–clonic seizures. d. Illusions and structured hallucinations D. Tonic seizures. probably arise from epileptiform dis- E. Atonic seizures. charges in the temporoparieto-occipital III. Unclassified epileptic seizures. junction. C. Cryptogenic (presumed symptomatic but with unknown etiology). Classification According to Etiology* D. Generalized epilepsies and syndromes. 1. Idiopathic (age-related). A. Idiopathic. a. Childhood absence. 1. Benign childhood epilepsy with cen- b. Juvenile absence. trotemporal spikes. c. Juvenile myoclonic epilepsy. d. Grand mal seizures on awakening. *ILAE, International League Against Epilepsy. Modified from Commission on Classification and Terminology of 2. Cryptogenic or symptomatic. the International League Against Epilepsy. Proposal for a. West syndrome (infantile spasms). revised clinical and electroencephalographic classifica- b. Lennox–Gastaut syndrome. tion of epileptic seizures. Epilepsia 1981; 22: 489–501. 232 10 Common Symptoms in the Neurology Clinic

E. Situation-related seizures. clonic, partial, and myoclonus. Status epilep- 1. Febrile. Early childhood beginning at age ticus is common and seizures are often 3 months to 5 years. refractory to treatment. Mental retardation is 2. Toxic factors or events. Alcohol, drugs, evident. The EEG pattern is less than 3 Hz eclampsia, and nonketotic hyperglycemia. spike-wave discharges. The prognosis is poor.

Syndromes: Case 32: 1. Focal epilepsy of childhood, rolandic seizures. 1. Her vision becomes dark because of impaired These are associated with centrotemporal perfusion of the retinas due to orthostatic spikes, occur between the ages of 2 and 13, hypotension. There is impaired perfusion of and manifested by simple partial as well as the lung apices and neck musculature causing tonic–clonic seizures. They comprise 15–20% dyspnea and neck pain. of childhood epilepsies. Todd’s phenomena 2. Have her walk for at least 5 min or until symp- are common and a positive family history is toms occur. When blood pressure is taken a present in 15–30% of patients. decision about whether orthostatic hypoten- 2. Childhood absence. The onset is age 5–7 with sion is present cannot be made until the pres- frequently a genetic predisposition. The typi- sure is stabilized. This was not demonstrated cal EEG pattern is 3 Hz spike-wave discharges as it was declining when recordings were pre- which are bilateral, symmetrical, and syn- maturely discontinued. chronous. Grand mal seizures may occur at • Increase venous return. Elastic stockings. adolescence. Absence usually disappears • Increase blood volume. Treat with salt and/ afterwards. or mineralocorticoids (fludrocortisone). 3. Juvenile absence. Grand mal seizures are fre- • Increase peripheral resistance. Stimulating quent both before and after development. alpha-1 adrenergic receptors with midodrine. They begin at puberty. There are spike-wave • Exercise to avoid deconditioning. This discharges greater than 3 Hz. improves muscular tone which aids venous 4. Juvenile myoclonic epilepsy. Onset is at return. puberty. It manifests especially in the morning • Raise the head of the bed to avoid supine with myoclonus without loss of conscious- hypertension. ness. Grand mal seizures are common (90%), absence 30%. There is seldom remission and Case 33: lifelong treatment is usually required. There is a. A large blood volume and decreased vascular a strong genetic predisposition (about 50%). tone is present in pregnancy. After delivery, The EEG pattern is 3.5-6 Hz, multiple spikes/ blood volume returns to normal but vascular slow wave discharges. tone may remain decreased for several days. Hence, immediately after delivery women are 5. Grand mal seizures on awakening. Onset is in susceptible to significant drops in blood pres- puberty and there is a genetic predisposition. sure when standing up. 6. West syndrome (infantile spasms). This is cryptogenic or symptomatic and the age of Case 34 A 15-year-old boy is brought to the office onset is 4–7 months. Manifestations are flexor because of three recent episodes of vertigo, ringing or extensor spasms, psychomotor retardation, in the ears for several minutes and then abrupt loss hypsarrhythmia pattern on EEG. The progno- of consciousness for 1–2 min. On one occasion he sis is poor. saw flickering blue lights for 15 min, simultaneous 7. Lennox–Gastaut syndrome. This is a crypto- with vertigo, and then had abrupt loss of conscious- genic or symptomatic seizure disorder with ness. There was confusion for 30 s afterwards. age of onset between 1 and 8 years. The most The neurologic examination is normal. common seizures are tonic, atonic, and a. What questions should be asked? absence. Other seizure types include tonic- b. What is the diagnosis? Near-Syncope, Syncope, and Seizure 233

Case 35 A 32-year-old woman complains of minute or more than 120 beats/min upon chronic fatigue, faintness, periodic nausea, and ­standing or with minimal exertion. With tilt palpitations for 2 years. She can barely walk a table testing the increase occurs within 10 min half a block before requiring rest. of being upright. POTS syndrome is a well- known cause of the chronic fatigue syndrome. Examination: Blood pressure is 90/60 sitting, 86/60 standing, Case 36: and 88/58 after walking for 2 min. The pulse is a. Myoclonus, absence and tonic–clonic seizures. 90, 110, and 124, respectively. This history is typical of myoclonus, a sudden a. Do these findings explain her fatigue? involuntary muscle contraction generated by a b. What is the diagnosis? paroxysmal electrical discharge within the central nervous system (CNS). It can be epi- Case 36 An 18-year-old male high school student leptic or nonepileptic. Examples of the latter is brought to an internist because of clumsiness. include myoclonus associated with spinal cord At least a few times per week in the morning disease and hypnic jerks, sudden muscular he drops or throws either his glass of orange contractions in drowsiness. This patient also juice, bowl of cereal, or an eating utensil. He is has absence (staring) and a grand mal seizure. a good student, respectful, and despite intense b. The diagnosis is juvenile myoclonic epilepsy. determination on his part he has been unable to c. It is the most common cause of primary gen­ overcome this tendency. The neurologic exami- eralized epilepsy comprising about 10% of nation is normal. these epilepsies. Approximately 90% of these The patient’s internist refers him for neuro- patients have grand mal seizures and 30–35% logic evaluation. The day before the appointment have absence seizures. he has a generalized tonic–clonic seizure. The d. Lifelong treatment is usually required but with neurologist’s examination is normal other than a an excellent response to anticonvulsants. single episode of staring for 3 s. a. What phenomena are occurring? Case 37 A 72-year-old man requests an evalua- b. What is the diagnosis? tion because of an episode of loss of conscious- c. How common is this condition? ness while dining at his favorite restaurant. He d. What is the prognosis? had his usual dinner of a large bowl of pea soup, ribeye steak, mashed potatoes, and green beans. Answers: His beverage was one bottle of beer. His dessert Case 34: was bread pudding and ice cream. As he was fin- a. Is there any headache associated with these ishing dessert he felt nauseated for 1–2 min. One symptoms? Does he have a prior history of hour later while chatting with his family after headaches? The patient recollects a minimal dinner he began to perspire, appeared pale, and headache after regaining consciousness. He then passed out for 1 min. There were a few brief has had a few severe pulsating headaches asynchronous jerks of his extremities. On regaining associated with nausea and photophobia over consciousness he was confused for 30 s. the last year. a. Why did he lose consciousness? b. Basilar migraine. This disorder is associated b. What is the pathophysiology? with symptoms referable to the brainstem and Case 38 A 27-year-old psychologist is referred occipital lobe. for unusual and inappropriate laughter at a psy- Case 35: chology conference. She had been depressed and 1. Yes. under considerable stress since her boyfriend 2. This is POTS syndrome, paroxysmal orthos- canceled their marriage plans. As the small ten- tatic tachycardia syndrome, which is defined member meeting was discussing grief reactions, as an increase in heart rate of ³ 30 beats per the patient broke out into a peal of laughter which 234 10 Common Symptoms in the Neurology Clinic lasted 1 min. She seemed “dazed” afterwards and b. This is convulsive syncope due to hypotension was slow to respond for 2–3 min. The patient caused by compression of the inferior vena says that her friends reported three other very cava by the fetus decreasing venous return and brief similar events over the last 2 years when out thus resulting in poor cardiac output. for dinner. She did not believe her friends’ reports. Case 40 A 45-year-old man is referred because The neurologic examination is normal. of sudden loss of consciousness while watching a. Is her psychological state related to these TV after dinner. His wife was unable to rouse him episodes? for 2 min. An earlier spell, 1 month ago, occurred b. What are the two most important facts that while driving and she managed to get the car to suggest seizures? the side of the road. Over the past year while at Case 39 A 38-year-old woman is referred for dinner his wife has noticed that his jaw suddenly neurologic consultation because of a seizure in drops open on occasion. At those times he does bed. She is in the 8 month of pregnancy. The not respond to her for several seconds. Neurologic night before this evaluation, while supine in bed examination, MRI (brain), and EEG are normal. reading a book, she felt nauseated and then lost a. What additional questions might yield diag- consciousness. Her husband reports that his wife nostic information? stiffened, jerked a few times, and lost control of Case 41 An 80-year-old man is referred because her urine. She was moist with perspiration. of episodes of loss of consciousness associated a. What additional questions should be asked of with falling during the night. This has occurred the patient and husband to determine whether on four occasions always when standing. He feels this was a seizure or convulsive syncope? nauseated, dizzy (lightheaded), and short of b. What is a likely etiology of this event? breath before losing consciousness. There is no Answers: confusion according to the patient. a. What is the first question that should be asked? Case 37: b. What is the suspected mechanism of loss of a. Postprandial hypotension. consciousness? b. There is failure to maintain vascular resistance during blood pooling in the splanchnic bed. Case 42 A 66-year-old woman requests an eval- Postprandial syncope may occur up to 75 min uation because of sudden loss of consciousness following a meal. while reaching up to dust bookshelves in her living room. She was vaguely dizzy for 1–2 s before Case 38: losing consciousness. She awakened on the floor a. There is no definite connection. in a pool of urine having made note of the time b. The duration of laughing was 1 min, typical just prior to dusting and immediately after regain- for seizure phenomena. There was postictal ing consciousness. Just 2 min had elapsed. She confusion for 2–3 min. This patient has com- denies confusion before or after the event. plex partial seizures of gelastic character. Past medical history includes hypertension. Many patients with complex partial seizures Her only medicine is enalapril 10 mg q.d. do not believe or reject witnesses’ descriptions since there may be no preceding simple partial Neurologic examination: seizure (aura). Her MRI (head) disclosed a There is mild weakness of the left anterior tibialis hamartoma involving the hypothalamus, a muscle associated with a slow foot tap and clonus well-described cause of gelastic seizures. at the left ankle. A CAT scan (head) and an EEG are normal. A CBC shows a wbc count of 12,000/ Case 39: cu mm with 84% neutrophils. a. Was the patient confused afterwards and for how long? Was there tongue biting? The Neurologic examination 2 days later: patient was confused for 20 s and there was no The neurologic examination is normal and a CBC tongue biting. is normal. Sleep Disorders 235

a. Did this patient have a seizure or syncope? 48 h but signifies a structural lesion even if not b. What is the significance, if any, of the tran- visible by MRI. Rarely, minimal twitching can sient abnormal neurologic signs? occur in the affected muscles indicating focal c. Is there any other clue to support either syn- status epilepticus. An EEG should be obtained cope or seizure? promptly to see if there is ongoing seizure phenomena, simple partial status epilepticus, Answers: requiring immediate treatment. Case 40: c. An elevated wbc count with left shift occurs a. Was there any stiffening or jerking of the with seizures. extremities, urinary incontinence, tongue bit- Diagnosis: ing, or confusion? The answer is no. Seizure, complex partial, secondary to a right b. Has he been drowsy? Yes, he is always drowsy cerebral lesion. and frequently naps. An MRI (head) with and without contrast c. How does he respond when surprised? His reveals an intra-axial (within brain) mass in the legs give way occasionally. right frontal parasagittal region only visible on Diagnosis: the contrast-enhanced study. At surgery a small Narcolepsy with sleep attacks and cataplexy. grade 3 astrocytoma is removed. Astrocytomas are graded from benign to malignant (1–4). Grade Case 41: 4 is a glioblastoma multiformae. a. Did you urinate or move your bowels before losing consciousness? Response: Yes, I just finished urinating. Sleep Disorders b. Micturition or postmicturition syncope. Bladder contraction is mediated by the para- One century ago a textbook of neurology intro- sympathetic system. A strong vagal response duced an eight-page chapter on disorders of sleep may produce bradycardia, reflex vasodilation, with the following first sentence: “The disorders hypotension, and then syncope. Since sleep is and disturbances of sleep, while mainly symp- associated with a drop in blood pressure, get- tomatic, in some instances reach an important ting up at night to urinate is likely to add a development and almost obtain the dignity of a factor of orthostatic hypotension. disease.”* Conversely, Hippocrates is reported to Straining to urinate, especially in men with have said: “Sleep and watchfulness, both of them prostatic obstruction, will increase intrathoracic when immoderate, constitute disease.” Just a few pressure which may reduce venous return fol- decades ago practicing physicians thought of lowed by a subsequent drop in cardiac output, patients with sleep disorders as occurring in two hypotension, and syncope. This mechanism categories, the insomniac and the somnolent, the occurs with straining of any type such as with former to be bludgeoned with barbiturates and defecation, weightlifting, and coughing. the latter to be activated with amphetamines. Case 42: Today, sleep disorders (Kryger et al. 2010) are a. The history is indeterminate with regard to the known to aggravate or cause severe medical diagnosis. The denial of confusion cannot be ­disorders as well as provide diagnostic clues to either accepted or refuted since there was no explain the source of many patient complaints. witness. Urinary incontinence is common with Sleep apnea predisposes to or precipitates cardio- seizure but may also occur occasionally with vascular and cerebrovascular events and has other syncope. widespread adverse ramifications. Narcolepsy, if b. The transient abnormal neurologic signs are called a Todd’s phenomenon, a sign of a sei- *Church A, Peterson F. W.B. Saunders. 3rd ed, 1902; zure with focal elements. It can last minutes to p. 616–624 236 10 Common Symptoms in the Neurology Clinic untreated, thwarts a normal lifestyle, promotes augur the subsequent development of a neurode- disability and is responsible for more motor-vehi- generative process. PLMS and restless legs syn- cle accidents than epilepsy. Rapid eye movement drome (RLS) will be discussed together because (REM) behavior disorder of sleep may predict of clinical similarities; but their differences and the eventual emergence of Parkinson’s disease. etiologies will be contrasted. Sleep-related move- Parasomnias may simulate seizure disorders. Jet ment disorders, nocturnal paroxysmal dyskine- lag and shift work negatively influence sleep and sias and bruxism warrant attention, the former sap the energy of those affected individuals. because it resembles or may be a seizure disorder Chronic insomnia and idiopathic hypersomnia and the latter because of its prevalence. Lastly, may lead to chronic disability. Lastly, sleep hab- insomnia, one of the most common ailments in its are the window into the psyche such that, the general public will be reviewed. when diagnostic considerations are refuted by appropriate examinations, investigation of sleep patterns may guide the physician to an unex- Sleep Apnea pected psychiatric diagnosis. Just two to three decades ago the standard Obstructive Sleep Apnea medical history either omitted or did not empha- a. The incidence of obstructive sleep apnea size a sleep history. It is now, or it should be, a (OSA) in the elderly population (greater than routine element of every history and physical 65 years) is believed to reach 25% in those liv- examination by a primary care provider. The pur- ing independently and 42% in nursing home pose of this abbreviated review is to acquaint the populations. student or resident with common sleep disorders b. Symptoms and clinical features. that have major health consequences. There are manifold symptoms of sleep apnea There are eight major categories of sleep and many of them masquerade this disorder. disorders. Hence, itemization of the presenting manifes- 1. Insomnia tations is worthwhile. These include: 2. Sleep-related breathing disorders • Daytime somnolence* 3. Hypersomnia of central origin • Chronic fatigue* 4. Circadian rhythm sleep disorders • Insomnia* 5. Parasomnias • Frequent nocturnal urination. 6. Sleep-related movement disorders • Dry mouth. 7. Isolated symptoms and normal variants • Decreased libido and impotence. 8. Other sleep disorders • Nocturnal sweating. Only a small number of these conditions will • Morning headaches. be selected for discussion. Sleep-related breath- • Snoring, choking, and snorting* ing disorders, obstructive and central sleep apnea, • None of the above symptoms should be have had a prominent impact on caring for attributed to OSA without adequate inves- patients with heart disease and stroke since omis- tigation. Nevertheless, OSA should be part sion of treatment may lead to catastrophic conse- of the differential diagnosis. The asterisks quences (Yaggi et al. 2005). Narcolepsy and indicate hallmark characteristics. idiopathic hypersomnia should be recognized by c. Diseases aggravated or produced by OSA: every practicing physician as adequate treatment • Augmentation of GERD. will restore a nonfunctioning individual to a nor- • Cardiac arrhythmias. mal or near-normal life. The parasomnias such as • Stroke. REM behavior disorder (RBD) of sleep and peri- • Pulmonary heart disease and right heart odic leg movements of sleep (PLMS) are being strain. increasingly recognized as relatively common. • Impaired cognition. The former causes mayhem in the bedroom and • Hypertension. often leads not only to marital discord but may • Sudden death. Sleep Disorders 237

Central sleep apnea Obstructive sleep apnea

Airflow:

Respiratory effort:

Fig. 10.3 Distinguishing central from obstructive sleep apnea

d. Predisposing factors: often-snorers and non-snorers is 1.9. Smoking • Obesity. and alcohol are additive factors. • Nasopharyngeal abnormalities. Snoring and brain infarction are associated. • Retrognathia. The risk ratio of cerebral infarction between • Micrognathia. habitual snorers and occasional or nonsnorers e. Pathogenesis. is 10.3. In one study of 70 patients with cere- OSA comprises about 84% of all cases of bral infarctions during sleep, 68.6% were sleep apnea. It is an upper airway obstruc- habitual snorers. Arterial hypertension is com- tion disorder particularly common in obese mon in patients who have OSA but in epide- patients since pharyngeal muscles relax dur- miological studies habitual snoring persisted ing sleep and they can be infiltrated by adi- as a risk factor for stroke even after an adjust- pose tissue which results in further narrowing ment for arterial hypertension. More recent of the pharyngeal airway. Furthermore, in studies have used polysomnograms and demo- obese patients there may be enlargement of graphic data and have found a clear associa- the tongue, uvula, and palate causing further tion of severe sleep apnea (AHI equal or constriction of the airway passage. Other fac- greater than 30) with increased risk of stroke tors which play a role are cigarette smoking, or deaths from any cause. (AHI stands for age, male gender, and use of alcohol or hyp- apnea hypopnea index). notics. The age factor may correlate with the The basis of increased cardiovascular and observed increase in weight of the elderly. stroke risk may be due to increased sympa- Alcohol and hypnotics probably decrease the thetic nervous system activity because of the respiratory drive through their depressant work required to breathe which results in an effect on the CNS. increased heart rate and blood pressure. This f. Manifestations in sleep. increased activity can lead to an enlarged heart Patients with OSA may snort, gasp, choke, and arrhythmias with their attendant conse- and snore heavily. There are body jerks and quences of congestive heart failure, hyperten- they may make flailing arm movements. They sion, and stroke. are nearly always heavy snorers which is h. Polysomnography features. equivalent to partial upper airway obstruction. Apnea durations often range between 20 Thus, assessments of medical illness in these and 40 s. They occur in light sleep, stages 1 patients who snore heavily, even in the absence and 2, not deep sleep. Apnea is associated with

of polysomnographic documentation, bear bradycardia, tachycardia, and O2 desaturation scrutiny. of less than 90%. There must be five or more g. Disease association. obstructive apneas per hour to confirm the Hypertension and ischemic heart disease, the diagnosis. latter especially in men, are more prominent Apnea/Hypopnea Index (AHI): 5–15 is in heavy snorers. The prevalence of hyperten- mild. sion in habitual snorers is 10.5% as compared 15–29 is moderate. to 6.5% in non-snorers. The age-adjusted risk Equal to or greater than 30 is severe (see ratio of ischemic heart disease between Fig. 10.3). 238 10 Common Symptoms in the Neurology Clinic

Central Sleep Apnea g. Polysomnographic characteristics. a. Clinical features and symptoms. Apnea lasts for 10–30 s followed by 10–60 s Central sleep apnea (CSA) is relatively uncom- of hyperventilation. It usually occurs between mon as it comprises about 10% of sleep apnea wakefulness and sleep and when the patient cases. It occurs in patients with a normal body is supine. One or more of these events are habitus and snoring is not prominent; thus the present: diagnosis is often concealed. Symptoms differ • Frequent arousals with apnea. somewhat from patients who have OSA. They • Bradycardia/tachycardia.

include: • O2 desaturations with apnea. • Chronic fatigue. Criteria for diagnosis is an AHI equal or • Insomnia, primarily difficulty maintaining greater than 5 per hour; 5–15 is mild; 15–29 is sleep. moderate; equal or greater than 30 is severe • Daytime somnolence. (see Fig. 10.3). • Morning headache. • Impaired cognition. • Depression. Parasomnias • Sexual dysfunction is infrequent. b. Diseases provoked or aggravated by CSA. REM Behavior Disorder of Sleep • Hypertension. REM sleep comprises about 20–25% of sleep. • Cardiac arrhythmias. Features include episodic bursts of REM, EEG • Congestive heart failure. activation with beta activity (greater than 13 Hz), • Pulmonary hypertension. and it is most prominent in the latter one-third of c. Predisposing factors, causes. sleep. During REM sleep there is loss of muscle • Congestive heart failure. tone so that dreaming, which occurs during this • High altitude. sleep period, is unaccompanied by physical activ- • Neurologic disorders that affect central ity. In REM behavior, disorder of sleep, muscle control. atonia is absent and thus violent dreams are likely d. Pathogenesis. to be accompanied by violent physical activity.

The PCO2 is the primary determinant of ven- This disorder occurs primarily in the elderly popu- tilation during sleep. Conversely, large fluc- lation, especially beginning in the sixth or seventh

tuations in the PO2 around the normal range decades, and about 80% of the patients are male. have little effect on ventilation efforts. CSA is The abnormal behavior can be simple or com- probably due to an altered sensitivity to plex, motor, and/or verbal. Manifestations of simple hypercapnia. behavior include laughing, swearing, crying as well e. Manifestations in sleep. as limb and body jerking. Complex behavioral Gasping, choking, frequent body movements, features are often aggressive such as reaching out and cyanosis. to grab the bed partner, kicking, punching, jumping f. Disease association. out of bed, and running. Any disease which affects the medulla can Clinical criteria for the diagnosis, as described cause CSA. Thus, patients with autonomic above, are insufficiently specific. Other diagno- dysfunction such as in multiple system atro- ses such as night terrors (pavor nocturnus in chil- phy, neoplasms infiltrating or compressing the dren, incubus in adults), nightmares, sleepwalking medulla, polio and syringobulbia can cause (somnambulism), obstructive apnea, posttrau- CSA. Post-polio syndrome can affect medul- matic stress disorder, confusional arousal, and lary neurons earlier than spinal cord neurons nocturnal seizures can produce similar behavior. and thus CSA can be the initial presenting Polysomnography provides the required evidence problem. to document the diagnosis since the abnormal sleep behavior must occur during REM sleep. Sleep Disorders 239

RBD occurs in diseases associated with alpha- the calves, often asymmetric, lasting minutes to synuclein deposition. The most common is hours. They are relieved by movement; thus the Parkinson’s disease but it may also occur in mul- patient gets out of bed and walks around. The dif- tiple system atrophy, Lewy body disease and pro- ferential diagnosis includes akathisia but this dis- gressive supranuclear palsy. The incidence of order is not relieved with movement. development of such a disorder may reach 40% Features that support the diagnosis of RLS over a 10-year period. include the presence of PLMS which are evident Treatment of choice is clonazepam 0.5–2 mg h/s. in 80% of patients, a positive family history It may work by suppressing motor manifestations (autosomal dominant), and a good response to rather than REM sleep itself. Melatonin is occasion- the standard treatment with dopamine agonists, ally helpful at dosages of 3–12 mg before bed. Of pramipexole, or ropinirole. Dyskinesias do not great importance is the aggravation of this disorder emerge since nigrostriatal neurons are intact by specific medicines which include tricyclics, obviating denervation supersensitivity. MAO inhibitors, SSRIs or SNRIs, tramadol, and Disorders which precipitate or are associated bisoprolol. Caffeine, nicotine, and alcohol may with RLS include iron deficiency anemia, ure- aggravate the disorder and should be avoided. mia, dialysis, rheumatoid arthritis, peripheral arterial disease, congestive heart failure, neurop- Periodic Leg Movements of Sleep, athy, and Parkinson’s disease. Incidence in preg- Restless Legs Syndrome, Hypnic Jerks, nancy is quite high, 15% after the 20th week. and Nocturnal Leg Cramps Iron deficiency anemia, particularly with low There is much confusion among clinicians about ferritin levels, is important to recognize as iron these four disorders. Each has unique characteris- supplements can be an effective remedy. tics and management differs. PLMS and hypnic There are numerous alternative treatments jerks (HJ) are motor system disorders whereas, should dopamine agonists be ineffective or iron RLS and NLC are both sensory and motor with a supplements not indicated. These include clon- prominent sensory component. azepam, gabapentin, baclofen, tramadol, pregab- PLMS have wide ramifications. Prevalence in alin and, as a last resort, opioids. the elderly is near 50%. The movements are ste- NLC (“charleyhorses”) are painful involuntary reotyped, slow, and manifested by extension of muscle contractions that awaken the patient. The the big toe with flexor movements at ankle, knee, cramps involve gastrocnemius and soleus muscles and infrequently hip. They can begin with a myo- thus causing flexion of toes and ankle. They are clonic jerk but last up to 5 s and may occur two to unilateral, last seconds to minutes, and occur in all three times per minute. They emerge in stage 1 or sleep states. They are provoked by stretching the 2 sleep, not REM sleep. Partial arousal often muscle and relieved by dorsiflexion of the foot. occurs; thus sleep fragmentation with insomnia Some result in insomnia and daytime fatigue. and daytime somnolence can result. Underlying NLC are most common in the elderly and disorders include iron and folate deficiency, ane- pregnant women after the 25th week of preg- mia, renal failure, neuropathy, and radiculopathy. nancy. Predisposing diseases are numerous such They may occur with other sleep disorders such as hypertension, renal disease, hypocalcemia, as CSA, OSA, RBD, and narcolepsy. hypokalemia, diabetes, hypothyroidism, vascular RLS occurs prior to sleep onset and is charac- disease, liver disease, and venous insufficiency. terized by an irresistible urge to move both legs. Medicines which may provoke cramps are sta- They are most common just after lying down at tins, diuretics, donepezil, vincristine, bronchodi- night. Patients may awaken during the night lators, and oxcarbazepine. severely symptomatic. It may be prevalent in Efficacious treatment includes quinine sulfate daytime hours, especially when the patient rests. and gabapentin. Quinine should be avoided, how- The arms may be affected. Patients describe ever, in the elderly and in patients with renal dis- “creeping,” crawling, and tingling sensations in ease. It decreases the excitability of the motor 240 10 Common Symptoms in the Neurology Clinic endplate with a resultant risk for cardiac arrhyth- sleep. The patient recalls the dream and mias. There are additional numerous toxic side when awakened has normal cognition. effects which include thrombotic thrombocy- 2) Sleep paralysis. This often occurs with topenic purpura, interstitial nephritis, and the narcolepsy but can be a sole symptom. hemolytic uremic syndrome. Magnesium can be During this period, usually a few minutes, helpful in pregnant women. there is a with areflexia HJ are sudden single muscle contractions in but retention of eye and chest movements. the legs occurring at onset of sleep and hence The patient is severely anxious and may have been called sleep starts. Infrequently, the have a sense of suffocating. A touch or arms and head are involved. HJ are nearly uni- sound may terminate the episode which versal experiences in the general population. occurs commonly at the end of a REM Caffeine and other stimulants provoke them. The sleep period. only medical care required is reassurance and 3) RBD (discussed in an earlier section). avoidance of pharmacotherapy. c. Sleep-wake transition. 1) Rhythmic movement disorders. These Miscellaneous Parasomnias are stereotyped repetitive movements a. Arousal disorders. mostly occurring in infants and children 1) Sleep terrors (pavor nocturnus in children, usually at sleep onset. The movements incubus in adults). primarily involve the head, neck, and These are manifestations of extreme fright body. Head banging against the wall (jac- with concomitant autonomic discharges tatio capitis nocturna) can easily be mis- manifested by diaphoresis, pupillary dila- interpreted as a seizure particularly when tation, tachycardia, and tachypnea. These heard but not witnessed. Rarely there are phenomena are initiated by a piercing skull injuries. Prolonged, repetitive, for- scream and erupt from slow wave sleep in ward and backward body rocking is the first third of the night. A common pro- another manifestation. voking scenario is attempting to arouse a 2) Sleeptalking (somniloquy). The individual child who is in stage 3 sleep. The patient is speaks briefly or utters sounds without amnestic for the event. emotional content. It occurs in all sleep 2) Sleepwalking (somnambulism). stages but most often with REM sleep. This occurs during slow wave sleep and d. Nocturnal paroxysmal dystonia. when awakened the patient is often confused. During non-REM sleep, stages 2 or 3, the indi- It can be associated with somniloquy vidual exhibits complex motor behavior, espe- (sleeptalking). Inappropriate behavior is cially dystonic but occasionally ballistic or not unusual such as urinating in a corner choreoathetoid. These are either brief, less than of a room. 1 min, or prolonged attacks lasting up to 1 h. The 3) Confusional arousal. brief spells simulate frontal lobe seizures and Physical violence can be precipitated by have been described as extrapyramidal seizures. sleep drunkenness. This typically occurs Although they are most common in infants they with forced awakening from slow wave have been reported to begin in middle age. They sleep in the first third of the night. Upon respond to anticonvulsant treatment. arousal the patient is disoriented and think- e. Sleep bruxism. ing is slow. The duration varies from sev- This is a common stereotyped movement dis- eral minutes to several hours. Restraining order manifested by grinding and crunching of the patient can provoke aggression. This is the jaws. It is often associated with anxiety most common in children. and is common in children with mental retarda- b. REM sleep disorders. tion and cerebral palsy. Complications include 1) Nightmares or threatening dreams occur excessive wear on the teeth, temporal man- in the last third of the night during REM dibular joint pain, and periodontal disease. Sleep Disorders 241

The diagnosis can be particularly important 5. Include regular exercise in daytime activities. since the noise of bruxism may raise the ques- 6. Get out of bed at the same time daily, irrespec- tion of seizures. tive of the amount of sleep. 7. Avoid caffeine, alcohol, nicotine, and stressful activities in the evening. Chronic Primary Insomnia There are different opinions on pharmacologic therapy. My view is to use medications that have Chronic primary insomnia (CPI) (Silber 2005) is a short half-life such as zaleplon and zolpidem an accurate diagnosis provided two conditions which have half-lives of 1 and 2.5–3 h, respec- are met; there is nonrestorative sleep which inter- tively. These are agonists at the benzodiazepine feres with work or social life and the insomnia is receptor component of the GABA receptor com- not secondary to drug use, medical disease, psy- plex and are less likely to interfere with sleep chiatric illness, or another sleep disorder. Thus architecture. Short-term treatment is the goal. sleep apnea, parasomnias, and circadian dys- An extremely effective benzodiazepine is alpra- rhythmias cannot be present. Acute CPI is less zolam. Although this medicine is notoriously than 1 month with subacute 1–6 months and addictive, a dose of 0.125–0.5 mg given only at chronic greater than 6 months duration. bedtime is a lifesaver for many patients. Common medical conditions that interfere Intermittent use is always the goal. Trazodone, a with sleep include asthma, COPD, CHF, GERD, weak SSRI, is often quite helpful. Standard SSRIs rheumatic disorders, ESRD, and hyperthyroid- are additional options but their successful use ism. Common substances that interfere with sleep implies the presence of depression as the under- are caffeine, nicotine, alcohol, and illegal drugs. lying disorder. Psychological intervention should Perhaps the most important illness to consider is be considered and the topic at least broached with depression since sleep disorders are an integral patients although many are resistant. Mirtazapine part of the depressive diathesis as well as its com- is a very potent sedating antidepressant and par- mon predecessor. ticularly efficacious. The adverse effects of Circadian dysrhythmias are a misalignment increased appetite, weight gain, and daytime between the natural endogenous circadian clock somnolence associated with its long half-life of which is 24.2 h and the social/physical environ- 20–40 h makes its successful long-term use ment. The circadian clock is located in the supra- unlikely. Lastly, quetiapine 25 mg at bedtime, chiasmatic nucleus of the hypothalamus. Light is with a half-life of only 6 h, may be very useful. the stimulus which affects this rhythm. Melatonin, Problematic drugs, especially in the elderly, produced by the pineal gland, is the modulator of should be recognized. The tricyclic compounds this system. Its secretion is controlled by neurons such as amitriptyline, nortriptyline, and doxepin in the suprachiasmatic nucleus. The two most must be used carefully, if at all, because of their commonly recognized of these disorders are jet anticholinergic side effects. These include cardiac lag and shift work. The physiology of this symp- arrhythmias, impaired cognition, somnolence, tom and management techniques are beyond the orthostatic hypotension, aggravation of glaucoma scope of this text. and prostate disease, decreased sexual function, Treatment approaches for CPI focus on behav- and exacerbation of psychosis. A general principle ior, sleep hygiene, and lastly, pharmacotherapy. is to avoid their use in elderly patients. Antihis­ Behavioral techniques and education should take tamines, an ingredient of over-the-counter drugs, precedence. Some of the principles are self- are used with success in younger individuals but evident: in the elderly often cause grogginess and impaired 1. Go to bed when sleepy. cognition the following day. Sedatives which have 2. Use the bedroom only for sleep and sex. a longer half-life such as flurazepam and, to a 3. Avoid prolonged daytime naps. lesser extent temazepam, are useful only if there 4. Avoid TV use in bed since exposure to bright is concomitant chronic anxiety which can be light has an arousing effect. ameliorated the following day. 242 10 Common Symptoms in the Neurology Clinic

The serious adverse consequences of hypnot- auditory, visual, or somesthetic. The latter is a ics in general, but particularly those with long sense of feeling touched or an out-of-body expe- half-lives, include aggravation of depression, rience. To some patients the hallucinations appear daytime somnolence, impaired cognition, and so real that the specter of schizophrenia is raised. increased risk of falls.

Diagnostic Criteria for Narcolepsy Narcolepsy and Cataplexy 1. EDS occurring almost daily for at least The excessive daytime somnolence (EDS) of 3 months. narcolepsy has several unique characteristics. 2.* Cataplexy. There is an irresistible urge to sleep but the naps 3. Confirmation by nocturnal polysomnography are short, refreshing, and typically last 15–20 min. followed by a daytime multiple sleep latency The sleep attacks commonly occur at inappropri- test (MSLT) or low hypocretin-1 concentra- ate times such as driving, eating or in the midst of tion in CSF (1/3 mean control values). conversations. Automatic behavior, perhaps due Polysomnography and the MSLT are the to “microsleeps,” may occur after which patients accepted and currently required examinations for have no recollection of what has transpired. They a firm diagnosis of narcolepsy. Nocturnal poly- may describe them as blackouts or memory somnography rules out other causes of EDS which lapses. Reported behaviors have included putting may mimic narcolepsy. The use of the MSLT for vegetables in the dishwasher or taking incompre- diagnostic purposes requires a minimum of 6 h hensible notes at a lecture. Differentiating these sleep the night before. There are five nap times at events from complex partial seizures with autom- 2-h intervals. Sleep onset REM periods atisms is critical. Nocturnal sleep is usually dis- (SOREMPs) of less than 15 min are considered turbed in narcoleptic patients. There are frequent definitely pathologic. Narcolepsy is confirmed if awakenings some of which may be due to RBD there are two or more of them. The mean onset and PLMS. sleep latency is less than 8 min. Low hypocretin-1 The main criterion for the diagnosis of narco- CSF levels are 99% specific, confirm the diagno- lepsy is the presence of cataplexy (Dauvilliers sis, although this examination is seldom neces- et al. 2007), a sudden loss of muscular tone typi- sary and not readily available today. Lastly, the cally provoked by a surprise or sudden, forceful, presence of human leukocyte antigen (HLA) emotional event. Ordinarily it lasts less than DQB1*0602 is common, supportive of the diag- 2 min. It is often manifested by a fall, head drop, nosis but is neither required nor does it establish or jaw opening. Falls can be associated with the diagnosis. injury or put the patient at risk. One such alarm- A brief discussion of the physiology of narco- ing incident described by a patient was an abrupt lepsy is warranted. Hypocretins are produced and collapse when observing a car approaching at an secreted by neurons located in the posterior hypo- unexpectedly rapid speed. The famous tetrad of thalamus. Ninety percent of narcoleptic patients narcolepsy adds two other common but not have low CSF levels (Ripley et al. 2001). Hypocretin required phenomena. These are sleep paralysis, neurons excite brainstem neurotransmitter sys- an occurrence upon awakening or falling asleep tems, locus ceruleus (norepinephrine), dorsal raphe of complete loss of motor function with retention of pons (serotonin), substantia nigra (dopamine), of eye and respiratory movements lasting seconds and lateral dorsal tegmental (LDT)/pedunculopon- to several minutes. This may be an isolated tine tegmental (PPT) area (acetylcholine). During ­phenomenon and hence is not diagnostic of nar- REM sleep there is activation of the LDT/PPT colepsy. Hypnagogic and hypnopompic halluci- nations may occur, the former at the onset of *There is now a recognized category of narcolepsy with- sleep and the latter on awakening. These can be out cataplexy. Other Transient Neurologic Symptoms 243 neurons and inactivation of other brainstem tant autonomic dysfunction. Headache disorders ­neurotransmitter systems. Additionally, the basal are common. It is associated with an HLA-Cw2 ­forebrain (acetylcholine) and tuberomammillary allele. Treatment is the same as for narcolepsy nucleus (histamine) neurons are activated. other than avoiding sodium oxybate. Adverse Conversely, neurons in the anterior hypothalamus reactions to medical treatment are more common (GABA-producing) promote non-REM sleep via compared to patients with narcolepsy. projections to the thalamus and brainstem nuclei. Hypocretin-producing neurons are reduced in number in the posterior hypothalamus (Lodi et al. Other Transient 2004). This has been documented by proton MR Neurologic Symptoms spectroscopy. N-acetylaspartate (NAA) is the neuronal marker which can be used to measure The majority of transient neurologic symptoms the number of neurons. The number is reduced in have been covered. But there are additional neuro- the hypothalamus of narcoleptic patients with or logic occurrences which may be more challenging without cataplexy. The etiology is unknown but to diagnose. This section will contain a cursory there is an autoimmune hypothesis supported by review of important symptoms and a few rare con- successful treatment with intravenous immuno- ditions. The purpose of including rare phenomena globulin of a patient at the onset of the illness. is to stimulate an interest in carefully listening to Treatment of cataplexy includes sodium oxy- and observing patients as well as to avoid discard- bate, SSRIs, and SNRIs. Sodium oxybate is used ing descriptions as either inconceivable or psy- immediately before bed and stimulates GABA chogenic. Three case reports will be presented receptors. Narcolepsy can be treated with modafinil, consecutively followed by the answers. methylphenidate, or amphetamines. Modafinil is the first choice since it is long-acting, has little “Drop Attack” (The Sudden abuse potential and has been proven effective in a Inexplicable Fall) double blind, placebo-controlled study. Meth­ ylphenidate and amphetamines increase dopamine Case 43 A 42-year-old man complains of several and norepinephrine release at the synapse and falling episodes without warning. His physician block their reuptake. questions him about what he was doing at the instant of falling. Did he briefly lose consciousness and does he remember hitting the ground? Did he Idiopathic Hypersomnia have chest pain, palpitations, sweating, nausea, dizziness, visual disturbance, sudden weakness, Idiopathic hypersomnia comprises 5–10% of or numbness of one or more extremities? The patients evaluated at sleep disorder clinics. The patient replies that while shopping at the super- clinical features are considerably different than market he unexpectedly ran across an old college those of narcolepsy. Patients are usually less than friend he had not seen for 15 years. Neurologic age 25 at onset. The condition is marked by long examination is normal. periods of sleep, normal nocturnal sleep and yet, What question was omitted that could have patients remain drowsy throughout the day. They easily lead to the diagnosis? have prolonged naps of 1–2 h which are com- posed of non-REM, non-refreshing sleep instead Case 44 A 58-year-old woman has fallen on sev- of the brief 15–20-min naps of patients with nar- eral occasions over the past year. She brings her colepsy who feel rejuvenated. MSLT discloses daughter who is invited to share her observations. short latencies of less than 10 min but a normal She has observed her mother to collapse abruptly latency of REM sleep. Associated symptoms may sometimes striking her head and never able to include Raynaud’s syndrome, syncope and ortho- break the fall. The patient feels as though she is static hypotension all of which suggest concomi- thrown to the ground. She denies headache, visual 244 10 Common Symptoms in the Neurology Clinic disturbances, vertigo, altered speech and mem- such as when driving and stopped at a red light. ory, nausea, sweating, numbness or weakness of The diagnosis is narcolepsy with the full tetrad of the extremities, or impaired balance. The neuro- findings, sleep attacks, cataplexy, sleep paralysis, logic examination is normal. and hypnagogic hallucinations. The patient’s What questions were omitted that might lead sudden fall was due to cataplexy provoked by the to the diagnosis? surprise of meeting an old college friend. Case 45 A 37-year-old male, a high-achieving Case 44: mechanical engineer, is brought to his internist The patient is asked: Do you have a loss of hear- by his wife because of several abrupt falls which ing or ringing in the ears? She responds: I have occurred without evident provocation over the poor hearing and ringing in my right ear. previous 3 months. The patient reports tripping This patient has Ménière’s disease and the but his wife says the sidewalks are smooth and condition is called an otolithic crisis or Tumarkin’s without obstacles. Moreover, he denies all neuro- otolithic crisis. The etiology is presumed to be a logic complaints and exhibits no overt signs of sudden disturbance of otolithic function. This fear or anxiety. His wife reports that she believes affects the vestibulospinal system and precipi- her husband was unsteady on those occasions. tates loss of postural tone. Neurologic examination is normal including a Case 45: thorough mental status examination. The social history must be thoroughly investigated. Problem: Alcohol abuse or use of drugs must be suspected. How does one distinguish pathological denial Case 46 A 45-year-old retarded woman is brought (anosognosia) from willful prevarication, igno- to her family physician because of numerous, sud- rance, or a behavioral quirk such as fear of den falls. She is the product of a complicated physicians? breech delivery, delayed developmental mile- Anosognosia is usually generalized to all stones, and numerous seizures in childhood. These questions and accompanied by dementia such as were myoclonic, atypical absence, rarely complex Alzheimer’s disease or a nondominant hemi- partial, and generalized tonic–clonic. They were sphere lesion. These disorders are easily exposed eventually controlled by valproate with the excep- after additional history followed by an assiduous tion of a rare myoclonic jerk. Her neurologic neurologic examination. examination is unremarkable other than mental The normal patient who is anxious and brought retardation manifested by the attainment of only a in under duress, after a few moments of adjust- fourth grade level of education. ment, ordinarily replies truthfully to other ques- What is the etiology of these falls? tions. The patient described above fits the Do you know the name of this syndrome? category of prevarication. What part of the history should be meticu- Case 47 A 63-year-old man suddenly collapses lously and dexterously handled? on the floor without warning in an airport. He denies loss of consciousness. He looks pale, feels Answers: cold, and his skin is moist when he is examined Case 43: by a physician who rushes over to attend to him Do you feel drowsy during the day? Other related arriving only 1 min after the fall. His pulse is questions might include: Has this occurred to you strong and regular at 72 beats per minute. He on other occasions when you are surprised? Do reports that an identical event occurred on one you wake up at night and find that you are unable previous occasion at home. General physical and to move? Do you have visual hallucinations when medical examinations are normal performed a you are falling asleep or waking up? few hours later on both occasions. The patient responds yes to all questions and What is the most likely diagnosis and what adds that he frequently naps at inopportune times specific test is indicated? Other Transient Neurologic Symptoms 245

Case 48 A 77-year-old woman has fallen with- This patient has a high grade basilar artery steno- out warning on three occasions over the past sis with ischemia of the basis pontis, the location 2 weeks. She reports that her legs suddenly gave of the corticospinal tracts. way and she could not break her falls. She has a longstanding history of hypertension and smoked Confusion/Amnesia one pack of cigarettes per day until 10 years ago. Several years ago she had three episodes of dou- Case 49 A 52-year-old man asks his wife what ble vision lasting about 10 min each. She recalls concert they were planning to attend. They had that they were associated with a striking tilt of the just finished dinner and discussed the event 2 min environment of about 45°. before. His startled wife reminded her husband. What is the most likely diagnosis? They got into their car with the patient driving. Answers: He asked his wife “Where are we going?” His question came after 15 min of highway driving. Case 46: He appeared puzzled and she insisted that they These falls are likely to be caused by astatic sei- return home. He drove home uneventfully. She zures. The etiology is sudden loss of muscular excused his behavior as due to overwork, lack of tone or sudden tonic spasms with abrupt falling. sleep and believed that a nap would restore him They are often accompanied by myoclonus. to normal. After a nap of 45 min she awakened This patient has Lennox–Gastaut syndrome him. He appeared annoyed and asked the identi- manifested by the triad of multiple seizure types, cal question “Where are we going?” She replied, retardation, and EEG findings of slow spike-wave “To the Emergency Room.” When the Emergency discharges (less than 3 Hz). Room physician walked into the examining room Case 47: the patient became alarmed and said “How did 1. Pallor, moist, and cold skin occurs with syn- I get here? I don’t remember anything since cope. Cardiac arrhythmia is suspected. ­dinner except briefly driving into the garage” on 2. The lack of premonitory signs makes hypoten- their return home earlier that evening. He com- sion much less likely. plains of a dull, generalized headache. The neu- 3. This patient is likely to have had complete heart rologic examination including a detailed mental block or another critical sudden cardiac arrhy­ status evaluation is normal. thmia. A 24-h Holter monitor is indicated. What is this syndrome called and what is the currently proposed etiology? Case 48: What other neurologic disease is this syn- When there are brief symptoms, such as a few drome associated with? minutes, that suggest a TIA or other paroxysmal disturbance, diplopia and dysphagia are strong Answers: indicators of brainstem ischemia or dysfunction. This patient has transient global amnesia (Fisher Vertigo and dysarthria, although commonly due 1982), a well-known but uncommon syndrome to brainstem pathology are, as sole symptoms, often prompting an Emergency Room visit as the insufficient to localize an ischemic event. Vertigo patient suddenly becomes aware that he has lost occurs with any vestibular system disorder, cen- his memory for the preceding few or several tral or peripheral. Dysarthria is a nonlocalizable hours and may find himself in an unexpected symptom even if commonly due to brainstem location. During the period of amnesia, most dysfunction. The visual illusion of a tilted envi- often 2 to several hours, rarely longer, the patient ronment occurs with disorders of the vestibular can appear to function normally except for asking system. The combination of sudden loss of leg the same questions repeatedly due to absence of strength plus the visual illusions (tilt of environ- short-term recall. There may be islands of mem- ment) is thus most likely indicative of brainstem ory such as this patient had as he recalls driving ischemia due to vertebral or basilar artery disease. into his garage. 246 10 Common Symptoms in the Neurology Clinic

Studies have shown dysfunction of neurons in Case 52 A 52-year-old woman complains of the hippocampus (Gonzalez-Martinez et al. 2010). repeated episodes of brief loss of vision O.S. when Diffusion-weighted imaging MRI studies have dis- walking into a bright environment. This has closed hyperintensities in this region for a few days occurred daily for 3 months and is increasing in afterwards. Etiologic speculations include gluta- frequency such that she expects loss of vision every mate release producing metabolic stress on the sus- time she leaves the house on a sunny day. Her past ceptible neurons and venous flow abnormalities. medical history is remarkable for hypertension and There is a strong association with migraine smoking two packs of cigarettes per day. and many patients do complain of headaches, Neurologic examination: although often mild and nonspecific, afterwards. Blood pressure is 140/90. There are no carotid Some precipitating factors include: sexual inter- bruits. The entire neurologic examination is nor- course, exposure to cold and traumatic emotional mal other than the funduscopic exam. This shows experiences. dot and blot hemorrhages in the midperiphery associated with dilated retinal veins and arterio- lar narrowing. Transient Visual Impairment a. What does this funduscopic picture suggest? b. Why does the patient lose vision in bright Case 50 A 38-year-old woman complains of fre- light? quent “blackouts” affecting either eye which last c. What is the most likely cause? seconds. These have occurred numerous times usually affecting the left eye, rarely the right. Case 53 A 28-year-old woman complains of loss When they began 4 months ago they occurred of vision O.S. when she looks to the left. This about twice weekly but now several times per symptom has been present for 1 month. She has day. The past medical history is entirely negative no illnesses. The neurologic examination is nor- and the patient takes no medications. She has mal except for diminished color perception O.S. been on the Adkin’s diet for obesity and now Visual acuity is 20/20 O.U. a. What causes decreased color perception? weighs 190 lb at 5¢3″ tall. She weighed 240 lb 1 year ago. An MRI scan of the brain is normal. b. What could be the etiology of this rare a. What additional questions would lead to the condition? diagnosis? Answers: b. What part of the neurologic examination is Case 50: most important? a. Do you have headaches? The history of brief Case 51 A 65-year-old man complains of a gray visual loss affecting either or rarely both eyes curtain being drawn across his right eye from simultaneously is typical of papilledema. right to left lasting 30 s and completely obliterat- b. Funduscopic examination. The three cardinal ing his vision. The visual loss persists for 3–4 min. signs are blurred optic disk margins, absence These episodes have occurred 10–15 times over of venous pulsations (usually lost when the the last 6 weeks. The most recent occurrence was pressure exceeds 200 mm H2O), and hemor- 10 min ago while in the waiting room. He has rhages at the disk margin. Only the presence type II diabetes controlled with metformin. of hemorrhages secures the diagnosis as the a. What may be seen on funduscopic other two elements can occur as a normal vari- examination? ation. It must be emphasized that, apart from b. What is the etiology? these events, most patients with papilledema c. Is the manner of the loss of vision horizontal have normal vision. This formerly morbidly or vertical (like a curtain descending) of diag- obese patient has papilledema and normal nostic significance? vision. The diagnosis is pseudotumor cerebri. Other Transient Neurologic Symptoms 247

A lumbar puncture confirms the diagnosis as 20/200 O.U. The pupils are normal. Specifically

the opening pressure is 280 mm H2O. there is no afferent pupillary defect. a. What is a positive central scotoma and what is Case 51: its anatomic source? a. Gray particles, which are platelet-fibrin emboli, b. What is the etiology of the hallucinations and may be seen passing through the retinal arteri- what name is attached to them? oles. Hollenhorst plaques which are glistening, yellow, and lodging at vessel bifurcations arise Case 55 A 48-year-old man complains of visual from the aorta or carotid arteries. Chalky white hallucinations occurring simultaneously with plaques which are composed of calcium origi- smelling “rotten eggs.” The hallucinations are nate from calcified aortic or mitral valves. composed of seeing little men wearing overalls b. The etiology is usually extracranial carotid and working on machinery in his left visual field. atheromatous disease. They are only a few inches tall and they approach c. No. him without any visible sign of hostile intent. The neurologic examination reveals a left Case 52: superior homonymous quadrantanopsia. a. Chronic low perfusion. The slowed circula- a. What is the name of these hallucinations? tion time causes focal dilation and congestion b. What single question will determine the mech- of retinal veins. There is a breakdown of capil- anism of these events? lary walls producing infarctions with hemor- c. Where is the lesion? rhage (Klijn et al. 2002). This is venous stasis retinopathy. Case 56 An unusual phenomenon b. Exposure to bright light requires increased A 47-year-old man was referred for neurologic retinal metabolism which cannot be sustained evaluation because of persistent visualization of because of chronic retinal ischemia. a television scene after turning away from the TV c. Severe, often near-total carotid occlusion from set to answer a question from his wife who stood any cause. on his left side. He observed the same picture he just saw on the TV superimposed on his wife. Case 53: It lasted nearly 4 min. a. Optic nerve pathology. Neurologic examination discloses a left b. An orbital mass lesion produces torsion and homonymous inferior quadrantanopsia, position stretching of the optic nerve with eye movement sense loss affecting the fingers of the left hand, in a specific direction depending on its location. astereognosis with the left hand and impaired This patient has a metastasis from carcinoma rapid alternating movements of that hand. of the breast. a. Do you know what this visual phenomenon is called? b. Where is the lesion and what is the suspected Transient Visual Illusions diagnosis? and Hallucinations Case 57 A rare phenomenon A 91-year-old woman arrives in the Emergency Case 54 A 95-year-old woman requests an eval- Room because of severe vertigo and double vision uation for visual hallucinations. She sees idyllic of 5 h duration. She has a longstanding history of country scenes which include farmers and cattle hypertension and hypercholesterolemia. moving about in brownish-green pastures. They occur frequently, are present for a few minutes, Neurologic examination: and appear most often in a dimly lit environment. Blood pressure is 150/100. She has horizontal She is curious about them but not frightened. gaze-evoked nystagmus. There is paresis of left Neurologic examination discloses bilateral, superior rectus, inferior oblique, inferior rectus positive, central scotomas plus visual acuities of and medial rectus, right superior rectus, and she 248 10 Common Symptoms in the Neurology Clinic has bilateral ptosis due to weak levator palpebrae Diagnosis: superioris muscles. Simple partial seizures secondary to a right tem- poral lobe cysticercosis cyst. Hospital course: The patient recovers after 2 more hours but then Case 56: reports visual hallucinations. She describes beau- a. This patient has palinopsia (or palinopia), the tiful scenes of gardens with pink and green flowers. perseveration of a visual image (Critchley These persist for several days. 1951). a. Do you know the name of these hallucinations? b. Patients usually have a right parieto-occipital b. How can the eye signs be explained and what lesion with impaired vision in the left periph- could be the etiology of the hallucinations? eral field. This can be an ictal event. The patient had a visual field defect, sensory loss Answers: manifested by astereognosis, and abnormal Case 54: position sense and impaired rapid alternating a. Positive central scotomas are dark or brown- movements. These findings are all consistent ish due to pathology in the macula as opposed with a right parietal lesion. The patient has an to absence or blurriness which is common astrocytoma grade 4 (glioblastoma multifor- with optic nerve disease. mae) in the right parietal lobe. b. Sensory deprivation of the occipital cortex Case 57: causes increased spontaneous activity result- a. The patient has peduncular hallucinosis ing in these hallucinations which are called (Geller and Bellur 1987). The conjunction of release hallucinations. vertigo and diplopia are characteristic of Diagnosis: brainstem pathology. Charles Bonnet syndrome (Siatkowski 1990). b. The exam reveals a left 3rd nerve lesion plus right superior rectus paresis which is expected Case 55: when the lesion is in the oculomotor nucleus. a. Lilliputian. This name was used in a novel by There is contralateral innervation of the supe- Jonathan Swift, Gulliver’s Travels, for an rior rectus muscle, a peculiar anatomic fact. imaginary race of men of minute proportions. The only other crossed innervation among the b. How long do the visual and olfactory halluci- cranial nerves is the 4th nerve. The pathology nations last? They last about 2 min and thus is in the midbrain. This is the usual location of are consistent with the definition of simple the lesion which produces peduncular halluci- partial seizures. nosis although thalamic lesions have also c. Temporal lobe. The olfactory cortex is in the been documented. The precise cause of orbitofrontal and adjacent temporal cortex. peduncular hallucinosis is unknown but it Formed visual hallucinations are usually may be due to reticular formation damage. derived from the temporal lobe whereas This can prevent ascending inhibitory infor- unformed hallucinations originate from the mation from reaching the lateral geniculate occipital lobe. The conjunction of brief olfac- nucleus. Thus, excitation of neurons in this tory and formed visual hallucinations is char- nucleus may be the source of peduncular acteristic of seizures due to temporal lobe hallucinosis. lesions. The fact that the visual hallucinations were in his left visual field and that the exami- nation disclosed a left superior homonymous Transient Motor and Sensory quadrantanopsia indicates that the site of the Disturbances lesion is in the right temporal lobe. The CT (head) and MRI (head) with and without contrast are abnormal showing a calcified Case 58 A 66-year-old woman reports an epi- lesion typical of cysticercosis. sode of clumsiness of the left hand lasting 10 min. Other Transient Neurologic Symptoms 249

She was getting dressed at that time and could Diagnosis: not button her blouse. She gives no history of Right cerebral ischemia and probable infarction pain, numbness, or tingling. Neither her face nor secondary to right internal carotid artery high her legs were affected. Her past history is remark- grade stenosis with embolism. able for a one pack per day smoking habit for This is confirmed by angiography. Carotid 45 years. She has always been normotensive and Dopplers were not done since angiography was there is no history of diabetes. indicated whether or not the carotid Dopplers The neurologic examination reveals a blood were abnormal. pressure of 110/70, pulse 70, regular. She has Case 59: slightly impaired rapid alternating movements of a. The anatomic basis for brief single diffuse limb the left hand. sensory symptoms could be cerebral, unlikely a. Where is the lesion? What is the most likely to be brainstem or spinal cord, very likely root etiology? and less likely nerve. The patient has no symp- Case 59 A 25-year-old man reports an episode toms or signs of brainstem involvement. A spi- of intermittent numbness and painful tingling of nal cord lesion almost invariably gives bilateral the entire left leg lasting 6 h 1 week ago. This persistent symptoms. Since the entire limb is occurred once previously about 2 months ago but involved, single nerve compression or was less severe although more prolonged, about entrapment cannot explain the symptoms. 10 h. The patient had optic neuritis 2 years ago at Consequently, a cerebral or lumbar root lesion which time an MRI (brain) with and without con- is most likely. Multiple sclerosis relapses are trast was normal. A thorough neurologic exami- defined as greater than 24 h. An ischemic vascu- nation was entirely normal. Cerebrospinal fluid lar event ordinarily lasts less than 10 min, studies included a normal IgG index and no oli- exceedingly rare in this age group, anyway. goclonal bands were found. Since the tingling is painful, lumbar root pathol- a. What is the significance of this history? ogy is suspected. b. What additional questions and bedside tech- b. When queried, the patient says his symptoms niques might reveal the anatomic source of were most severe when sitting. He has lumbar these symptoms which would permit an accu- radiculopathy supported by a positive straight rate diagnosis? leg raising test, the specific diagnostic exami- nation technique. Lumbar root symptoms are Answers: commonly diffuse. An MRI (lumbar spine) discloses a herniated disk at L4–L5 compress- Case 58: ing the left L5 root. a. Brief clumsiness of the left hand with residual impairment of rapid alternating movements Case 60 A 77-year-old man complains of nonra- could be due to either right corticospinal tract diating cold, tingling and gripping sensations of or left cerebellar system pathology. The brief the left calf and foot. These are stereotyped, duration of 10 min is characteristic of a vascu- began 4 months ago and occur whether supine, lar event. Focal ischemic events involving sitting or standing. He denies back pain, weak- cerebellar pathways in brainstem or cerebellar ness, restlessness or aggravation of symptoms hemisphere causing clumsiness of one hand with either prolonged sitting or walking. are exceedingly rare whereas they are com- The neurologic examination is normal other mon with internal carotid artery distribution than mild left anterior tibialis weakness. TIAs. Hence, an embolus from a right internal a. Where can the lesion be? carotid artery plaque is the most likely etiol- b. What additional information could be obtained ogy especially since the patient is a lifelong which would lead to the diagnosis? smoker. Cardioembolism is much less likely Case 61 A 27-year-old woman complains of tin- in the absence of known cardiac disease. gling sensations involving the left arm and face. 250 10 Common Symptoms in the Neurology Clinic

They begin in the left hand and gradually travel Case 62 A 25-year-old graduate student is up the left arm to eventually deposit on the left referred because of periodic involuntary move- cheek. The movement of the paresthesias up to ments for 6 months. He notes the movement on the face develops over 40 min and the cheek is getting out of his chair quickly after working on involved for 5 more minutes. Her neurologic his computer or reading for a few hours. The examination is normal. involuntary movements are either rapid, affecting a. What question should the patient be asked and the hands, or of a sustained posture such as inver- what is the diagnosis? sion of a foot. They last between 30 and 60 s. If he is surprised they can be provoked. Answers: a. What types of movements are they? Case 60: b. Do you know the name of this rare condition? a. As in the previous case cerebral and root Case 63 A 19-year-old college student com- pathology are most likely. RLS could be con- plains of several spells, over a few months, of sidered but the patient is not restless. paralysis during the night. He may awaken Claudication may produce gripping sensations at 2:00 a.m. and be unable to move any extremity but it is not expected to occur at rest. Nerve until 4:00 or 5:00 a.m. Speech and swallowing root compression is commonly exacerbated are unaffected. The episodes occur most often with prolonged sitting, not with this patient. early Sunday morning or the night prior to exam- Weakness of anterior tibialis is compatible inations. The neurologic examination is normal. with nerve root or CNS pathology. a. Why might this occur early Sunday morning b. The diagnostic question is “What is the dura- or before an examination? tion of the symptoms?” The answer? It varies b. Any thoughts about the diagnosis? from 30 s to 2 min, never more. This duration with stereotypical symptoms indicates simple Answers: partial seizures. An MRI (head) reveals a Case 62: parasagittal meningioma, right parietal lobe. a. The rapid distal involuntary movements are Case 61: choreiform. A sustained abnormal posture, a. The patient should be questioned about head- inversion of the foot, is dystonia. ache. The history is typical of the “march of b. The disorder is paroxysmal kinesigenic chore- paresthesias” which is a common aura of oathetosis and dystonia (Demirkiran and migraine. The duration of symptoms is char- Jankovic 1995). Provoking factors are quick acteristic. This patient reports a mild headache movements after a prolonged period of rest and beginning when the paresthesias reach their a sudden surprise. This is a genetic disorder, peak. From her perspective, the headache is of usually autosomal dominant, begins in youth no concern compared to the severe paresthe- and tends to increase in frequency with age. sias. This has been called the cheiro-oral man- Other forms of paroxysmal dyskinesia are non- ifestations of migraine. kinesigenic exertion-induced and hypnogenic. The nonkinesigenic form is provoked by fatigue, alcohol, excitement and lasts minutes Paroxysmal Motor Phenomena to hours. The exertion-induced form occurs after prolonged exercise and lasts 5–30 min. The following two case reports are of rare disor- The hypnogenic form occurs during non-REM ders but they are representative of a large group sleep and a seizure disorder must be ruled out. of patients. The purpose of discussing these con- The presumed etiology of paroxysmal dyski- ditions is merely to expose the reader to the exis- nesias is dysfunction within the basal ganglia. An tence of these paroxysmal disorders of involuntary ictal origin must be considered since patients movements and paralysis. respond to anticonvulsants. References 251

Case 63: Vertigo a. The occurrence of paralysis early Sunday morning suggests a provoking activity the Baloh R, Kerber K. Clinical neurophysiology of the ves- preceding night. A common activity for a col- tibular system. 4th ed. New York, NY: Oxford University Press; 2010. lege student on a Saturday night is alcohol Brandt T, Daroff RB. The multisensory physiological and consumption and before exams there is a fac- pathological vertigo syndromes. Ann Neurol. 1980;7: tor of emotional stress. Both are known to pre- 195–203. cipitate an attack of periodic paralysis. Fife TD, Tusa RJ, Furman JM, et al. Assessment: vestibu- lar testing techniques in adults and children. Neurology. b. There are hyperkaliemic and hypokaliemic 2000;55:1431–41. periodic paralyses (Venance 2006). This patient Furman JM, Cass SP. Balance disorders. A case study has the hypokaliemic type which can manifest approach. Philadelphia, PA: F.A. Davis; 1996. itself as rarely as a few occasions in a lifetime Furman JM, Cass SP. Benign paroxysmal positional ver- tigo. N Engl J Med. 1999;341:1590–6. or as often as daily. The attacks are usually Halmagyi GM, Curthoys IS. A clinical sign of canal pare- nocturnal, last a few hours and can be mani- sis. Arch Neurol. 1988;45:737–9. fested by quadriplegia with areflexia. This is Hotson JR, Baloh RW. Acute vestibular syndrome. N Engl most often due to a defect in the calcium chan- J Med. 1998;339:680–5. nel related to a mutation on chromosome 1q. Near-Syncope, Syncope and Seizure References Chung SS, Gerber P, Kirlin KA. Ictal eye closure is a reli- able indicator for psychogenic nonepileptic seizures. Headache Neurology. 2006;66:1730–1. Grubb BP. Neurocardiogenic syncope. N Engl J Med. Afridi SK, Giffin NJ, Kaube H, et al. A positron emission 2005;352:1004–10. tomographic study in spontaneous migraine. Arch Grubb BP, Olshansky B, editors. Syncope: mechanisms Neurol. 2005;62:1270–5. and management. Armonk, NY: Futura; 1997. Dodick DW, Chronic daily headache. N Engl J Med 2006; Thijs RD, Wagenar WA, Middlekoop HAM, et al. 354:158–65. Transient loss of consciousness through the eyes of a Frese A, Eikermann A, Frese K, et al. Headache associ- witness. Neurology. 2008;71:1713–8. ated with sexual activity. Neurology. 2003;61: Wyllie E, Gupta A, Lachhwani DK, editors. The treatment 796–800. of epilepsy. principles and practice. 5th ed. Baltimore, Friedman DI, Jacobsen DM. Diagnostic criteria for idio- MD: Williams and Wilkins; 2010. pathic intracranial hypertension. Neurology. 2002;59: 1492–5. Headache Classification Subcommittee of the International Headache Society. International Classification of Sleep Disorders Headache Disorders. Cephalalgia. 2004;24(Suppl 1): 1–160. Dauvilliers Y, Arnulf I, Mignot E. Narcolepsy with cata- Loder E. Triptan therapy in migraine. N Engl J Med. plexy. Lancet. 2007;369:499–511. 2010;363:63–70. Kryger MH, Roth T, Dement WC. Principles and practice Schievink W. Spontaneous spinal cerebrospinal fluid leaks of sleep medicine. 5th ed. Philadelphia, PA: Elsevier; and intracranial hypotension. JAMA. 2006;295: 2010. 2286–96. Lodi R, Tonon C, Vignatelli L, et al. In vivo evidence of Silberstein SD, Lipton RB, Goadsby PJ. Headache in clinical neuronal loss in the hypothalamus of narcoleptic practice. 2nd ed. London: Martin Dunitz; 2002. patients. Neurology. 2004;63:1513–5. Sprenger T, Boeker H, Tolle TR. Specific hypothalamic Ripley B, Overeem A, Fujiki N, et al. CSF hypocretin/ activation during a cluster headache attack. Neurology. orexin levels in narcolepsy and other neurologic con- 2004;62:516–7. ditions. Neurology. 2001;57:2253–8. Waeber C, Moskowitz MA. Migraine as an inflammatory Silber MH. Chronic insomnia. N Engl J Med. 2005; disorder. Neurology. 2005;64(Suppl 2):S9–15. 353:803–10. Weiss HD, Stern BJ, Goldberg J. Post-traumatic migraine: Yaggi HK, Concato J, Kernan WN, et al. Obstructive sleep Chronic migraine precipitated by minor head or neck apnea as a risk factor for stroke and death. N Engl J trauma. Headache 1991;31:451–59. Med. 2005;353:2034–41. 252 10 Common Symptoms in the Neurology Clinic

Transient Neurologic Symptoms Ishiyama G, Ishiyama A, Jacobson K, Baloh RW. Drop attacks in older patients secondary to an atologic Critchley M. Types of visual perseveration. “Paliopsia” cause. Neurology. 2001;57:1103–6. and illusory visual spread. Brain. 1951;74: 267–99. Klijn CJM, Kapelle J, Van Schooneveld MJ, et al. Venous Demirkiran M, Jankovic J. Paroxysmal dyskinesias: clinical stasis retinopathy in symptomatic carotid artery occlu- features and classification. Ann Neurol. 1995;38:571–9. sion. Stroke. 2002;33:695–701. Fisher CM. Transient global amnesia. Arch Neurol. Siatkowski RM, Zimmer B, Rosenberg PR. The Charles 1982;39: 605–8. Bonnet syndrome. Visual perceptive dysfunction in Geller TJ, Bellur SN. Peduncular hallucinosis. Magnetic sensory deprivation. J Clin Neuro-Ophthalmology. resonance imaging confirmation of mesencephalic 1990;10:215–18. infarction during life. Ann Neurol. 1987;21:602–4. Venance SL, Cannon SC, Fialho D, et al. The primary Gonzalez-Martinez V, Comte F, de Verbizier D, Carlander B. periodic paralyses: diagnosis, pathogenesis and treat- Transient global amnesia. Arch Neurol. 2010;67: 510–1. ment. Brain. 2006;129:8–17. Diagnostic Dilemmas 11

Keywords Case reports • Dementia • Demyelinating • Infection • Vascular

The aim of this chapter is to review categories of Neurologic examination : neurologic diseases not covered in preceding Blood pressure is 140/100 in the right arm and sections. The method will be case studies with a 125/90 in the left arm. There are bilateral carotid focus on clinical-neuroanatomic correlation fol- bruits. Mental status examination is normal other lowed by a differential diagnosis. Analysis of than bradyphrenia and poor short-term recall. laboratory data, especially cerebrospinal fl uid, Downgaze is absent and upgaze is paretic. The will be included. There will be a potpourri of dis- vertical oculocephalic maneuver, performed with eases, many uncommon, yet all likely to be diffi culty because of nuchal rigidity, elicits up encountered at one time in a general neurologic and down gaze. The patient exhibits mild bra- and some medical practices. The requirement to dykinesia, truncal instability with retropulsion, be conversant in internal medicine will be appar- and cogwheel rigidity. ent in several case reports. These case analyses Case 1 Questions : and differential diagnoses are likely to go beyond 1. Can a focal lesion explain the symptoms and the level of most readers of this text but chal- signs? lenges often pique curiosity and stimulate the 2. Is the past medical history relevant in making student or physician to think beyond the mun- the diagnosis? Are the blood pressure fi ndings dane, rote requirements of daily practice. and carotid bruits helpful? Case 1 A 76-year-old man complains of impaired 3. Where is the pathology? memory, loss of balance, falling, and a stiff neck. 4. Does the history help to exclude some types There was an insidious onset of these symp- of pathology? toms beginning 7 months ago. They are steadily 5. What is the diagnosis? increasing in severity. 6. Are there other diseases that present with The past medical history includes type II dia- similar, if not identical, manifestations? betes, hypertension, and coronary artery disease. Medications are metformin, ramipril, clopidogrel, Case 1 Analysis : and atenolol. 1. No. The abnormal mental status manifested by bradyphrenia and short-term memory loss

J.N. Alpert, The Neurologic Diagnosis: A Practical Bedside Approach, 253 DOI 10.1007/978-1-4419-6724-4_11, © Springer Science+Business Media, LLC 2012 254 11 Diagnostic Dilemmas

indicates cerebral pathology and vertical gaze 4.5 mm OS with a 2+/4 light reaction. Downgaze paresis is a sign of midbrain involvement. elicits intorsion OS. There is mild weakness of 2. No to both questions. Many elderly people right frontalis, orbicularis oculi, and orbicularis have multiple illnesses, especially type II dia- oris muscles. Taste is impaired on the right side betes and hypertension. The asymmetric blood of the tongue. Leg strength is 4+/5 bilaterally pressures in the arms which are mildly ele- except for 4/5 anterior tibialis on the right and 3/5 vated and the bilateral carotid bruits suggest on the left. Tone is spastic in both legs and she atherosclerotic disease, but there is no history has bilateral Babinski signs. of stroke or TIA to explain this patient’s neu- A lumbar puncture discloses normal opening roanatomic pathology. pressure with ten lymphocytes/cu mm and pro- 3. A lesion in the riMLF results in paresis of tein 70 mg/dl. There is an elevated IgG index and downgaze with preservation of the vertical oligoclonal bands are present. An MRI scan of oculocephalic maneuver. Dementia indicates the brain is normal. bilateral cerebral involvement. 4. The history of progressive multifocal neuro- Case 2 Questions : logic impairments and the absence of discrete 1. Where are the lesions most likely located? focal events of neurologic dysfunction is 2. What diagnosis should be considered but dis- indicative of a degenerative process and virtu- carded and why? ally rules out a vascular etiology. 3. What simple examination may be diagnostic? 5. The diagnosis is progressive supranuclear 4. What do the cerebrospinal fl uid fi ndings palsy, a tau (+) disorder (Williams and Lees indicate? 2009 ) . The clinical presentation includes 5. What treatment reverses the defi cits and dementia, parkinsonian symptoms and signs, returns the patient to normal within 2 weeks? vertical gaze paresis especially downgaze, and Case 2 Analysis : nuchal rigidity. Tau protein is derived from 1. Left 3rd nerve, right 7th nerve, and spinal one gene on chromosome 17 and the hyper- cord. Intorsion OS indicates preservation of phosphorylated form is the primary compo- the 4th nerve. nent of neurofi brillary tangles which interfere 2. Multiple sclerosis. Dyspnea and impaired with axonal transport and lead to cell death. taste are not presenting symptoms. Loss of 6. This disease is one of the Parkinson-plus taste occurs with peripheral facial nerve syndromes which include multiple system lesions. The spinal fl uid protein is almost atrophy and corticobasal ganglia degenera- invariably normal. Lyme disease could be tion. Multiple system atrophy encompasses considered but it rarely causes a myelopathy. previously described conditions which include 3. The chest X-ray shows hilar adenopathy. olivopontocerebellar atrophy, striatonigral Subsequently drawn, an angiotensin-converting degeneration, and Shy-Drager syndrome. enzyme level is elevated. This patient has Case 2 A 32-year-old woman requests an evalu- sarcoidosis (Terushkin et al. 2010 ) which ation because of double vision, right facial weak- commonly affects cranial nerves, especially ness, impaired taste, stiff legs, and shortness of the facial nerve. breath of 2 weeks duration. She has a past medi- 4 . I n fl ammation such as occurs with immune cal history of insulin-dependent diabetes mellitus disorders especially. Other diagnoses which and migraine. produce either oligoclonal bands or an elevated IgG index include systemic lupus erythemato- Neurologic examination : sus and Lyme disease. Blood pressure 120/78, pulse 88, and regular. 5. Prednisone. Pertinent fi ndings include disconjugate gaze with the left eye deviated down and to the left. Pupils Case 3 A 58-year-old woman is referred by a are 4 mm OD with a 4+/4 light reaction and psychiatrist because of suspected mild cognitive 11 Diagnostic Dilemmas 255 impairment. She had refused neuropsychological alitis may produce psychiatric disorders, sei- testing. Two years ago, she sought treatment for zures, memory loss, agitation, and altered depression, but over recent months has exhibited mental status. Vascular disease is unlikely to inappropriate behavior which has included steal- present as a slowly progressive disorder. The ing antiques from a friend’s home, no longer degenerative disease that is the primary con- bathing regularly and exhibiting loss of initiative sideration is frontotemporal dementia or interest in any activity. (Arvanitakis 2010 ; Hu et al. 2007 ) . Past medical history is remarkable for hypo- 4. The family history is critical since frontotem- thyroidism, hypercholesterolemia, and small cell poral dementia is autosomal dominant and, lung cancer resected successfully 3 years ago. therefore, a positive family history for demen- Medications are venlafaxine, thyroid replace- tia is in the range of 40–50%. This patient’s ment, and atorvastatin. mother had abnormal behavior followed by dementia beginning in her early 60s, a typical Neurologic examination : sequence of events in frontotemporal demen- Blood pressure 140/95, pulse 92 with an occasional tia which is the second most common demen- irregular beat. Her spontaneous speech is limited tia in patients less than age 65. and answers to questions are brief. Echolalia is This patient has frontotemporal dementia, noted occasionally and naming is impaired. most often tau-positive (there are tau-negative The Mini-Mental State Exam score is 27/30. forms which are ubiquitin-positive, another Errors made in this examination are in naming and pathogenic protein). It is due to a progranulin repetition. She has bilateral grasp and palmomen- mutation on chromosome 17. tal refl exes. 5. The forms of this disease include. Case 3 Questions : (a) Behavioral variant frontotemporal dementia 1. Does this patient have symptoms which sug- which is described in this case. Another gest focal pathology? uncommon manifestation is the Klüver– 2. Does this patient have clinical signs which Bucy syndrome, a behavior disorder charac- suggest focal pathology? terized by hyperreactivity to visual stimuli, 3. What diagnoses should be considered? hyperoral, and hypersexual behavior. 4. What additional history should be reviewed? (b) Semantic dementia. This disorder is 5. What are the three forms of this disease and manifested by defi cient expressive and how do they differ? receptive vocabulary which develops insidiously. Anomia and lack of emotional Case 3 Analysis : responses are common. Inability to use 1. Yes. The behavioral changes suggest frontal utensils or common tools, apraxia, is lobe disease. common. The patients have poor recall of 2. Yes. Paucity of speech, dysnomia and echola- names and identifi cation of faces. MRI lia are indicative of left cerebral pathology, scans reveal anterior temporal lobe especially frontal lobe. Grasp refl exes are involvement. often found in patients with frontal lobe (c) Nonfl uent progressive aphasia. The typi- pathology. Palmomental refl exes are com- cal characteristics are phonemic parapha- monly present in patients with dementia, but sias, poor repetition, anomia, halting can be seen in the normal population. speech, alexia, and agraphia but, initially, 3. Degenerative disease, neoplasm, both meta- preservation of word meaning. static, carcinoma of the lung and benign neo- plasms such as meningioma. A paraneoplastic Case 4 A 46-year-old male petroleum engineer syndrome such as limbic encephalitis is an has just returned from a 6-month job in Saudi unlikely but possible etiology. Limbic enceph- Arabia. For the last 2 weeks, he has had severe mid- 256 11 Diagnostic Dilemmas back pain and progressive right leg weakness. He in view of position and vibration sense loss now requires a cane. His past history is negative. primarily on the right side. The sensory level at L1 on the left side indicates lateral spinotha- Neurologic examination : lamic tract involvement on the right as these Pulse 66, regular, blood pressure 150/90. fi bers immediately cross the midline in the Abnormal fi ndings include 4/5 proximal and dis- anterior commissure. Consequently, this tal weakness of right leg musculature. He walks patient most likely has a thoracic myelopathy. with a cane and circumducts the right leg. He has The level should be at T7 or higher in view of no abdominal or cremasteric refl exes on the right loss of the abdominal refl exes on that side. side. There is unsustained right ankle clonus. A sensory level varies from one to two seg- Vibration perception is absent at the right toes, ments below the lesion, but to a much lower ankle, knee, and left toes. Position sense errors level on frequent occasions. are made at the right toes. He has an L1 sensory 4. An MRI of cervical and thoracic spine is level on the left side. He is tender to percussion ordered with and without contrast as the diag- over the midthoracic spine. nosis is myelopathy, most likely thoracic, but a cervical lesion cannot be entirely excluded. Case 4 Questions : Generally, myelopathies require both cervical 1. How does one explain the refl ex abnormalities. and thoracic visualization if the diagnosis has 2. Does this patient have abnormal muscular not yet been made. tone? 5. The differential diagnosis includes transverse 3. What syndrome does the patient have and myelitis, transverse myelitis as a manifesta- what is the localizing value of the abnormal tion of multiple sclerosis, herniated thoracic fi ndings? disk, neoplasm, infection, and vascular dis- 4. An MRI scan of the cervical and thoracic ease. Transverse myelitis usually develops spine is performed. Why would both cervical more rapidly than 2 weeks. Multiple sclerosis and thoracic regions be imaged if the lesion is is most defi nitely a consideration since the likely to be at T -T . The absent abdominal 7 9 patient is age 46 (usual age range is 15–50). refl exes on the right side support this level of Herniated thoracic disks are rare. Neoplasm, involvement. especially a malignancy such as lymphoma, is 5. What is the differential diagnosis of a rapidly a defi nite consideration. A benign lesion such progressive lesion affecting the thoracic spinal as a meningioma or schwannoma would be cord? most unlikely to develop in this rapid fashion. 6. If the MRI scan shows an ill-defi ned lesion at T 7 Infections such as osteomyelitis with abscess on the right side which is extradural, what is the formation are a major possibility. next step? What studies should be performed? The neuroanatomic diagnosis, as expected, Case 4 Analysis : will still determine the differential diagnosis. 1. The loss of superfi cial refl exes, the abdominal There are three anatomic sites of lesions and cremasteric, and the unsustained right which affect the spinal cord, intramedullary, ankle clonus are caused by a lesion of the cor- extramedullary, and extradural. Transverse ticospinal tract on the right side of the spinal myelitis with or without associated multiple

cord at T7 or above. sclerosis and gliomas are intramedullary (within 2. Yes. Circumduction of the right leg occurs the spinal cord). Meningiomas and schwanno- with spasticity. mas are usually extramedullary (outside the 3. The patient has a partial Brown-Séquard syn- spinal cord but within the dura). Herniated drome as there is involvement of the corti- disks and spinal stenosis are extradural (outside cospinal tract on the right side causing right the dura). Mass lesions such as metastatic neo- leg weakness with spasticity, right ankle clo- plasm or abscess are most often extradural. nus, and loss of the superfi cial refl exes on the Consequently, the extradural lesion in this case right side. The posterior columns are affected is most likely metastatic neoplasm or abscess. 11 Diagnostic Dilemmas 257

6. A lumbar puncture is performed and the fl uid Case 5 Analysis : is sent for cell count, protein, glucose, gram 1. Balint’s syndrome (Hecaen and De stain, acid-fast smear, fungal smears, crypto- Ajuriaguerra 1954 ) . coccal antigen, VDRL, and cytology. 2. Simultanagnosia. This is demonstrated by the The spinal fl uid glucose is 20 mg/dl, pro- patient’s ability to identify each element of a tein 120 mg/dl, and there are 58 white cells/cu scene, but inability to appreciate its meaning. mm all of which are mononuclears. The acid- Oculomotor apraxia. This is the inability to fast smear shows gram-negative rods. The generate saccades on command. diagnosis is thoracic myelopathy secondary to Optic ataxia. This is the inability to reach an epidural tuberculous abscess associated or touch a target which is clearly seen. with osteomyelitis (Scheld et al. 2004 ) . 3. DWI on the MRI scan shows bilateral isch- emic lesions in the parieto-occipital and pretec- Case 5 A 67-year-old male lawyer was brought tal regions. Convergence-retractory nystagmus, to the Emergency Room because of confusion. eyelid retraction, upgaze paresis, and large, While speaking to a client, he noted the sudden round, fi xed pupils occur with bilateral lesions onset of total loss of vision and could no longer of the pretectum. remember the elements of a contract he had just 4. Acute confusion and agitation occurs with composed. His speech was incoherent and he acute bilateral ischemic changes, especially became agitated. involving parieto-occipital regions. Acute, Past medical history includes coronary artery unilateral, nondominant parietal lesions have disease, three myocardial infarctions, and con- rarely been associated with this behavioral gestive heart failure. Current medications are change. digoxin and clopidogrel. 5. The abrupt onset indicates vascular disease. Neurologic examination : Encephalitis could be considered but pro- Blood pressure 140/80 and pulse 82 with an dromal symptoms ordinarily occur. infrequent irregular beat. After the agitation sub- 6. A 2D echocardiogram discloses an ejection sided, about 1 h later, the patient could be exam- fraction of 20%. Thus, this patient had ined and he was disoriented to month and year. cardioembolism to the distal portion of the When shown a picture of a volleyball game he basilar artery with resultant ischemia in both was able to identify and describe each player, but posterior cerebral artery distributions. This could not comprehend the action that was being is one manifestation of “top-of-the-basilar portrayed. He was unable to generate saccadic syndrome” (Mohr et al. 2004 ) . eye movements to command. There was paresis Case 6 A 55-year-old man is brought to the of upgaze, eyelid retraction, and convergence- Emergency Room because of the sudden onset of retractory nystagmus. Each quick phase of nys- left-sided weakness and accelerating headache. tagmus produced convergence and retraction of A pulsating headache began 2 days before admis- the globes. This was easily produced by moving sion. Yesterday, the patient had two episodes of an optokinetic tape downward. Pupils were 6 mm, brief loss of vision affecting each eye indepen- equal, round, and fi xed to light. He was unable to dently lasting about 15–20 s. reach a target that he could clearly see. The patient has had periodic low back pain and Case 5 Questions : anorexia for 3 months with a weight loss of 18 lb. 1. What is the name of this syndrome? He has a 2-month history of chronic fatigue. 2. What are its elements? 3. Where is or where are the lesions on the MRI Past medical history : scan? The patient has a fi ve-year history of type II dia- 4. Does this explain the confusion and agitation? betes and has been treated with metformin. 5. What is the differential diagnosis and the Approximately, 10 months ago he was started on probable diagnosis? atorvastatin because of an LDL of 200 mg/dl and 6. What test should confi rm it? triglycerides of 386 mg/dl. 258 11 Diagnostic Dilemmas

Neurologic examination : 4. This is a classic although uncommon present- Pulse is 52 and the blood pressure 150/105. ing manifestation of superior sagittal sinus Funduscopic examination shows blurred disk thrombosis which frequently causes papille- margins on the superior and nasal portions of dema (Mohr et al. 2004 ; Biousse and Bousser both optic disks. Venous pulsations are not seen. 1999 ) . A single splinter hemorrhage is noted on the disk 5. Patients with superior sagittal sinus thrombo- margin OS. Pupils, visual fi elds, and eye move- sis have hemorrhagic infarctions and hemor- ments are normal. He has a left arm pronator drift rhage is a cortical irritant. with diminished rapid alternating movements and 6. This patient has cancer of the pancreas which fi nger tap on the left side. Strength of the left arm may cause a coagulopathy and is associated is 4+/5 except for interossei at 4/5. with venous sinus disease as well as its better- CAT scan of the head is normal. Hematocrit is known connection to deep venous thrombosis. 32%. Routine metabolic screen is normal. Case 7 A 60-year-old man is brought to the Hospital course : clinic by his wife because of frequent falling over The next morning the patient has a right focal sei- the previous 3 months. The patient believes that zure involving arm, face, and leg associated with he has been clumsy on these occasions and has nystagmoid jerks to the right. This lasts 1 min. simply tripped. From his viewpoint, he feels Afterwards, he has a right hemiparesis with entirely well and he denies all neurologic strength 3/5 except for 2/5 interossei and wrist symptoms. extensors. At this time, the left arm has returned Past medical history is remarkable for alco- to normal. hol abuse (which he denies), peptic ulcer disease, and refl ux. He smokes one pack of cigarettes Case 6 Questions : per day. Medications are omeprazole and 1. What caused the two brief episodes of loss of multivitamins. vision affecting each eye independently? 2. Does this patient have cerebral or brainstem Neurologic examination : pathology? Why? Pulse 60 and regular and blood pressure 110/70. 3. What do the funduscopic changes reveal? There are bilateral carotid bruits. He has several 4. What disease does a shifting hemiparesis sug- large ecchymoses on both arms and one leg. The gest which causes these funduscopic changes? patient’s mental status examination is normal and 5. Why did a seizure occur? this includes short-term recall. Palpebral fi ssures 6. What is the relevance of the back pain, weight are asymmetric, left much greater than right. loss, and anorexia? Pupils are 3 mm, equal, and reactive to light. He has impaired fi nger tap on the left side. When Case 6 Analysis : seated on the edge of the table, he falls back- 1. Increased intracranial pressure. Brief episodes wards. He cannot perform tandem gait. of visual loss, unilateral or bilateral, may occur in association with papilledema, often Case 7 Questions : many times per day. 1. What part of the present illness localizes the 2. Cerebral. Focal seizures do not occur with disorder? brainstem lesions. 2. What is the meaning of the asymmetric palpe- 3. Papilledema. Blurred disk margins often begin bral fi ssures? Does this specifi c fi nding fi t with over superior and inferior margins of the optic the history? disk. Venous pulsations may be absent in 3. Does inability to perform tandem gait indicate 5–10% of the normal population, but the sin- cerebellar dysfunction in this patient? gle splinter hemorrhage at the disk margin OS 4. When there is retropulsion (falling backwards) confi rms the diagnosis of papilledema. what anatomic structures are often affected? 11 Diagnostic Dilemmas 259

5. Can the past medical history help to explain Case 8 A 27-year-old woman complains of a left the clinical presentation? What is the differen- temporal headache and impaired speech of 5 h tial diagnosis? duration. Other than endometriosis she is entirely well. She has a past medical history of ulcerative Case 7 Analysis : colitis and was in a motor-vehicle accident 1. Denial. Despite the obvious presence of sev- 2 months ago, her car being struck from the rear. eral large bruises, he dismisses the signifi - There was no loss of consciousness. cance of his falls. This syndrome is called anosognosia although anosodiaphoria might Neurologic examination : be more apt. This latter term refers to dismiss- Blood pressure 130/100 and a regular pulse of 80. ing the signifi cance rather than the complete Her speech is halting, nonfl uent, and naming is denial of the falls. impaired. Repetition is normal. Abnormal neuro- 2. Since the pupils are equal and reactive, the logic fi ndings include asymmetric pupils which asymmetric palpebral fi ssures must be due to are 4 mm OD and 3 mm OS with a 3+/4 reaction weakness of the left orbicularis oculi which is to light bilaterally. There is a good reaction to innervated by the right cerebral hemisphere via near. The right palpebral fi ssure is greater than corticobulbar fi bers which cross in the pons to the left. The patient has a right Hoffmann and synapse at the left facial nucleus and provide Babinski sign. There are 5 beats of clonus at the innervation via the left facial nerve. Patients with right ankle and 2 at the left. right cerebral hemisphere lesions often have left Case 8 Questions : facial weakness which includes eye closure and 1. What kind of aphasia does this patient have? lower facial movement, but only rarely the fron- 2. What is the meaning of the pupillary asym- talis muscles which are bilaterally innervated. metry? Why are the lids asymmetric? 3. No. Tandem gait is impaired in many disor- 3. Is the pupillary asymmetry helpful in the dif- ders, especially those with severe sensory loss, ferential diagnosis? What artery could be especially involving position sense, vestibular affected? Which part of that artery? disease, and any of the three motor systems, 4. What is the treatment and prognosis? cerebellar, extrapyramidal, and corticospinal. 4. Retropulsion is common with midline pathol- Case 8 Analysis : ogies which include lesions involving the cer- 1. The patient has a transcortical motor aphasia ebellar vermis, hydrocephalus, large ventricles, which is nonfl uent speech along with usually Parkinson’s disease, and a shift of midline normal comprehension and good repetition. structures. In Broca’s aphasia repetition is impaired. 5. Yes. One must suspect that the patient suffered 2. A pupillary asymmetry of greater than 0.5 mm a head injury as a result of some of his falls. must be considered abnormal. The lid asym- The differential diagnosis includes primarily metry is either due to a right orbicularis oculi trauma and neoplasm. The neoplasm could be weakness or a left Horner’s syndrome. The benign but more likely malignant, if present at former may occur with a left cerebral lesion, all, because of the rapid development of neu- but most likely, in this case, in which motor rologic signs. This patient has a subdural function is normal and the left pupil is smaller, hematoma, chronic, right-sided and with a a Horner’s syndrome is the most likely etiol- right-to-left midline shift which causes his ogy. The pupillary fi ndings can be rechecked truncal instability. Chronic subdural hemato- in dark and, with a sympathetic lesion, the mas may cause progressive dementia or remain asymmetry increases when the pupils are mea- stable for years resulting in mild cognitive sured after 5–15 s have elapsed. In this case, impairment. Any patient with cognitive the normal right pupil increases to 5 mm and impairment requires at least a CT (brain) to the left to 3.5 mm, an increase in the asym- rule out this treatable lesion. metry by 0.5 mm. 260 11 Diagnostic Dilemmas

3. The sympathetic pathway runs through the 4. What is the diagnosis? What is the prognosis? carotid artery sheath and is commonly injured 5. What other diseases have a similar etiology? with carotid dissections which is the diagnosis Case 9 Analysis : in this case (Jensen et al. 2008 ) . Dissections of 1. There is bilateral cerebral hemisphere involve- extracranial arteries are far more common ment. The grand mal seizure, memory loss, than intracranial dissections. They may be and abnormal behavior indicate cerebral hemi- spontaneous or traumatic with the trauma sphere pathology. The myoclonic jerks are often quite trivial. Usually blood dissects likely to be of cerebral origin, although myo- between the intima and the media causing clonus may occur with brainstem or spinal stenosis, but, if between media and adventitia, cord disease. Brainstem/posterior commis- pseudoaneurysms may result. Dissections sure/cerebellar system involvement is present are typically located at the C2–C3 level. because of upbeat nystagmus, paresis of Angiograms demonstrate a tapered high-grade upgaze, limb ataxia, failure of check, and a stenosis resulting in an extended thin column wide-based ataxic gait. Anterior horn cell dis- of contrast called the “string sign.” ease is indicated by the presence of fascicula- 4. The prognosis is good for both clinical recov- tions. The Babinski sign indicates corticospinal ery and spontaneous resolution of the dissec- tract pathology. tion over 3–6 months. Standard treatment is 2. Corticospinal, extrapyramidal, and cerebellar. warfarin, although some neurologists advo- The Babinski sign is the quintessential mark cate clopidogrel. of corticospinal tract disease. Bradykinesia is Case 9 A 55-year-old man is brought to the a manifestation of extrapyramidal disease. Emergency Room after a tonic–clonic seizure. Failure of check, limb, and gait ataxia indicate His postictal state lasts about 1 h. When awake cerebellar system involvement. and alert, the following information is obtained. 3. Electroencephalogram. There are periodic, He has noticed a decline in his memory over the bilateral, sharp triphasic complexes at 1 Hz. prior 2 months and has had occasional involun- Cerebrospinal fl uid: 14-3-3 protein is pres- tary jerky movements of his extremities causing ent. This is a protein kinase inhibitor, a non- him to drop his dinner plate and eating utensils. specifi c marker for cell death. PCR testing His walking is unsteady. His wife adds that his may soon be available and replace testing for behavior is odd as he has been withdrawn and CSF 14-3-3 protein taciturn, a sharp departure from his usual ebul- MRI. This reveals T2 hyperintensities in lient nature. the basal ganglia. 4. Creutzfeldt–Jakob disease (Johnson and Gibbs Neurologic examination : 1998 ) . Blood pressure 120/70 and pulse 96 and regular. 5. This is a prion disease, a transmissible The patient is afebrile. A Mini-Mental State spongiform encephalopathy. The prion is a Exam score is 24/30. The patient has upbeat nys- proteinaceous particle. Other prion diseases tagmus on attempted upgaze which is incomplete, include: mad cow disease, fatal familial bilateral heel-to-shin and fi nger-to-nose ataxia, insomnia, Kuru, and Gerstmann–Sträussler– and a wide-based ataxic gait. He has moderate Scheinker disease. bradykinesia and failure of check in both arms. Case 10 A 45-year-old man complains of the Fasciculations are prominent over left thigh mus- sudden onset 1 week ago of horizontal double culature. He has a left Babinski sign. vision when looking to the left. The double vision Case 9 Questions : is slowly resolving. He has no other neurologic 1. Where is the pathology located? symptoms. He did have one episode of blurred 2. What motor systems are involved? vision affecting the right eye 3 years ago. His 3. What tests should be ordered and what do they ophthalmologist found a normal eye 1 week after show? the episode resolved. 11 Diagnostic Dilemmas 261

Past medical history is remarkable for hyper- (b) Protein. This is expected to be normal. If tension for which he takes atenolol and enalapril. it is increased, other etiologies should be considered. Neurologic examination : (c) IgG index. This is commonly but not Blood pressure 130/90 and pulse 100, regular. always increased. The patient has normal pursuit eye movements in (d) Oligoclonal bands. These are commonly all directions. His saccadic eye movements show but not always present. a decreased velocity of right eye saccades to the left. There is decreased perception of light and Case 11 A 14-year-old girl complains of head- color OS, but a normal visual acuity, 20/20 OD, ache, fever, and neck pain of 3 days duration. For and OS. The left pupil shows a better consensual 3 months, she has had fatigue and migratory joint reaction than direct response to light. and muscle pain. Two months ago, she developed a mild left facial weakness while traveling with Case 10 Questions : her family in Mexico during summer vacation. 1. What eye movement syndrome does this A diagnosis of Bell’s palsy was made, treatment patient have? with prednisone given but no changes in the facial 2. Is there more than one lesion? weakness have been noted since then. Over the 3. What diagnosis is virtually certain? past 2 weeks, she has had shooting pains and 4. What studies may help to confi rm the diagnosis? tingling down the left leg. Case 10 Analysis : 1. Internuclear ophthalmoplegia or MLF syn- Neurologic examination : drome. The MLF is mainly a quick system The patient has a temperature of 100°F. Blood pathway; thus evaluation by assessing sacca- pressure is 90/60 and pulse 88 and regular. There des is much more sensitive than evaluating is mild nuchal rigidity and left facial weakness pursuit. Nystagmus may or may not be present which involves frontalis, orbicularis oculi, orbic- in the contralateral abducting eye. Convergence ularis oris, and platysma. is not invariably preserved. Cerebrospinal fl uid studies show 88 wbc/cu mm 2. Yes. There is an optic nerve lesion OS since and all are mononuclear. Protein is 110 mg/dl and the patient perceives less light and color with glucose 70 mg/dl. The IgG index is elevated and this eye. Visual acuity is a less sensitive test. oligoclonal bands are present. The better consensual response to light OS Case 11 Questions : objectively confi rms the presence of optic 1. What part of the general physical examination nerve pathology. could be diagnostic? 3. Multiple sclerosis (Weiner 2009 ) . There are 2. Does geography matter? two symptomatic lesions separated in space 3. What are the two main diagnostic and time. A normal ophthalmologic evaluation considerations? at the time of his symptoms does not negate the 4. What tests may distinguish between these history and current fi ndings on examination. diagnoses? 4. Since there are two attacks by history and 5. What is the most likely diagnosis? fi ndings, additional supporting data are not required. Nevertheless, a baseline MRI (head) Case 11 Analysis : with and without contrast is indicated. A nor- 1. Careful inspection of the skin may reveal an mal result would still not refute the diagnosis. erythematous ring-like lesion with a clear cen- A contrast-enhancing lesion would indicate tral area, erythema migrans. active ongoing disease. 2. Yes. This family lives in New Jersey, a state Cerebrospinal fl uid studies of interest where the prevalence of Lyme disease is high. would include: 3. Sarcoidosis and Lyme disease. Both diseases (a) Cell count. This often shows a modest can cause polyarthralgia, polymyalgia, facial lymphocyte pleocytosis in the range of paresis, meningeal infl ammation, and abnor- 6–20 wbc/cu mm. mal spinal fl uid. The cerebrospinal fl uid fi nd- 262 11 Diagnostic Dilemmas

ings which may occur in both diseases include Case 12 Questions : a mononuclear pleocytosis, elevated protein, 1. What test was performed which established increased IgG index, and oligoclonal bands. In the original diagnosis? contrast, multiple sclerosis patients frequently 2. What fi nding on the second neurologic exami- have an increased IgG index, oligoclonal nation helps to determine the next test? bands, but a normal cerebrospinal fl uid pro- 3. What is the next test? tein and a mild, if any, pleocytosis. 4. What could this examination show? 4. For Lyme disease, serologic tests of blood Case 12 Analysis : and spinal fl uid are useful. For sarcoidosis, 1. The test performed was a sedimentation rate angiotensin-converting enzyme levels may which was 96 mm/h. The patient refused a be increased and gallium scans abnormal. temporal artery biopsy. The headache with Biopsy of lymph nodes or muscle and nerve temporal arteritis may be unilateral or bilat- are occasionally required for the diagnosis of eral and located anywhere on the head. Any sarcoidosis. person over the age of 50 who has a persistent 5. Lyme disease (Pachner and Steere 1985 ) . headache must have an ESR drawn. Jaw clau- Meningeal symptoms and signs, peripheral dication due to ischemia related to infl amma- facial weakness, and radiculopathy are typical tory involvement of the internal maxillary neurologic fi ndings. arteries is common. The Lyme disease triad is meningitis, cranial 2. Eyeball tenderness may occur with meningeal neuritis, and an infl ammatory radiculopathy. irritation. This fi nding in addition to fever and Case 12 An 88-year-old woman complains of a photophobia raises the distinct possibility of severe bilateral occipital headache and jaw pain meningeal irritation due either to subarach- with chewing for 1 week. She has type II diabetes noid hemorrhage or infection. Infection must and chronic obstructive pulmonary disease, the lat- be suspected in any patient with fever who is ter a result of a one pack per day smoking history taking prednisone. for 55 years. She stopped smoking 5 years ago. She 3. Lumbar puncture. takes metformin and uses an Albuterol inhaler. 4. The patient has turbid cerebrospinal fl uid with Neurologic examination is normal. Laboratory 700 wbc/cu mm of which 90% were mononu- tests are performed and she is started on predni- clears. The protein is 102 mg/dl. There is a sone 60 mg q.d. positive India ink preparation and cryptococ- Over the next 6 months, the patient’s dose of cal antigen. prednisone is gradually reduced to 10 mg q.d. Diabetes mellitus and immunosuppression Her blood tests are now completely normal, but predispose patients to infections. The diagno- her headache returns and increases in intensity to sis is cryptococcal meningitis (Scheld et al. a level of 9/10. She is readmitted to the hospital. 2004 ) . Neurologic examination reveals a temperature Case 13 An 82-year-old man is admitted to the of 101°F, blood pressure 140/90, and heart rate 90 hospital because of acute confusion, fever, and regular. She has severe photophobia and pho- cough, and hypotension. Past medical history is nophobia. Her eyeballs are tender to palpation. unknown and there are no family members Examination otherwise is normal. Pertinent labo- available. ratory tests revealed a hematocrit of 39%, white count of 16,000/cu mm with 80% neutrophils, and Neurologic examination : there is a normal metabolic panel. The sedimenta- Blood pressure 80/50, pulse 60, temperature tion rate is 20 mm/h. An MRI scan of the brain 101°F. shows scattered T2/FLAIR hyperintensities, The neck is supple and there is no adenopa- which are not unusual in number and size for age. thy. There is a grade 3/6 systolic murmur at the 11 Diagnostic Dilemmas 263 apex; rales, decreased fremitus, and breath Case 14 A 52-year-old woman is referred to the sounds are present at the left base. The spleen is neurology clinic because of a 1-week history of palpable. severe, diffuse headache, impaired vision on the left side, and a clumsy left hand. She is currently Case 13 Questions : being treated with cyclosporine for aplastic ane- 1. What diagnosis is suspected and proven? mia. The initial neurologic examination reveals a 2. What examinations are abnormal? blood pressure of 180/110. Abnormal fi ndings Treatment is initiated. The patient improves include disorientation to month and year, diffi - over the course of 1 week. The patient’s men- culty performing simple addition such as 14 + 7, tal status returns to normal. and 0 recall of 3 words after 3 min have elapsed. On the eighth hospital day, the patient com- She has a left homonymous hemiachromatopsia, plains of a sudden severe headache, joint pain, poor rapid alternating movements of the left arm, and stiff neck. and position sense loss at the left fi ngers. Neurologic examination : She is admitted to the hospital, has a grand Blood pressure is 120/70, temperature 99.6°F, mal seizure on the night of admission and, when pulse 100. There is mild nuchal rigidity. Speech questioned afterwards, says that her vision is nor- is fl uent but he makes phonemic paraphasias such mal. Neurologic examination discloses an alert as requesting a “goose paper” instead of a news- patient, disoriented to place and date. She is paper. He makes position sense errors at the right unable to count fi ngers, but can perceive light fi ngers. A stat CAT scan (head), noncontrast, is fl ashes. Pupils are 4 mm equal and reactive to normal. light at 4+/4. She has a left arm pronator drift, absent position sense at the left fi ngers, and Case 13 Additional Questions : impaired rapid fi nger tap on the left. There is sen- 3. What test is performed? sory extinction of the left side with double simul- 4. What are the most likely results? taneous stimulation. 5. What diagnosis should be high on the list? An MRI shows large T2 hyperintensities, mainly parieto-occipital white matter, right greater Case 13 Analysis : than left. 1. This patient has subacute bacterial endocardi- The patient is treated with labetolol and pheny- tis. He has the murmur of mitral insuffi ciency, toin. Blood pressure returns to normal at 116/80 signs of a left lower lobe pneumonia, and an very quickly, within 2 h. Cyclosporine is with- enlarged spleen. held. Forty-eight hours later, the patient has a nor- 2. He has hematuria, positive blood cultures with mal mental status and vision returns to normal. streptococcus viridans, and a mass on the The left arm is no longer clumsy, but she still mitral valve seen on echocardiogram. makes a few position sense errors at the fi ngers. 3. Lumbar puncture. A repeat MRI (head) 3 days later is normal. 4. The CSF is bloody and the supernatant is xantho- Case 14 Questions : chromic. There are 250 wbc/cu mm with 90% 1. What clinical syndrome did this patient have? neutrophils and RBCs too numerous to count. 2. What is the diagnosis? Gram stain is positive with cocci in chains. 3. What conditions are associated with this 5. Mycotic aneurysm (Mohr et al. 2004 b). disorder? Angiography demonstrates an aneurysm in a 4. What could be the pathogenesis? distal branch of the left middle cerebral artery, 5. What are the treatment considerations? a typical location for mycotic aneurysm. The fi rst symptom of an embolic event were the Case 14 Analysis : phonemic paraphasias which would be the 1. Anton’s syndrome. This is denial of blindness optimal time for neurologic investigations. which may occur with bilateral occipital lobe 264 11 Diagnostic Dilemmas

lesions. Pupillary responses are intact since to rapid fi nger count. Optokinetic slow phases to the light refl ex pathway leaves the optic tract, the right are impaired and quick phases to the left travels through the brachium of the superior appear slow. He has a right arm pronator drift colliculus, then to the pretectum, Edinger– upwards and loss of position sense at the right Westphal nucleus, 3rd nerve, ciliary ganglion, fi ngers and wrist. sphincter pupillae. MRI (head) shows two T2 hyperintensities, 2. Posterior reversible encephalopathy syndrome right parieto-occipital and left parietal. There is (PRES) (Lee et al. 2008 ) due to cyclosporine no contrast enhancement on a T1 study. toxicity. This is also known as hyperperfusion Case 15 Questions : encephalopathy. 1. What is the name of the visual syndrome? 3. (a) Pre-eclampsia and eclampsia. 2. What are the four salient features of the clini- (b) Hypertensive encephalopathy. cal presentation? (c) Postcarotid endarterectomy with ipsilat- 3. What is the localizing value of each abnormal eral hyperperfusion especially with prior fi nding? severe, critical stenoses. Occasionally, 4. What is the differential diagnosis? this may be bilateral. 5. What tests were positive and diagnostic? (d) Postsurgical treatment of arteriovenous 6. What other disorders occur with this disease? malformations. (e) Lupus nephritis. Case 15 Analysis : (f) Drug overdose. 1. Illusory spread of color, more specifi cally (g) Chemotherapy for neoplasms. erythropsia (red) in this case (Critchley 1966 ) . (h) Re nal disease. This phenomenon is occasionally an ictal (epi- 4. Altered cerebral autoregulation. Cerebral ves- leptiform) event. It is usually associated with a sels supplying the occipital lobe have less nondominant parieto-occipital lesion. sympathetic innervation and are therefore less 2. The patient has a 1-year history of chronic protective for hypertensive events. fatigue, visual disturbances, cognitive impair- 5. Avoid steroids which may produce endothelial ment, and multifocal signs. dysfunction, discontinue provoking drugs and 3. The inability to perform serial sevens and the treat with beta blockers such as labetalol or impaired short-term recall are nonlocalizing calcium channel blockers. fi ndings which together indicate bilateral cerebral hemisphere disease. The left hom- Case 15 A 47-year-old male lawyer is referred onymous inferior quadrantanopsia to rapid by an ophthalmologist because of visual distur- fi nger count indicates a lesion affecting the bances. The patient describes four episodes over optic radiations in the right parietal lobe. the last 2 weeks of altered color vision. His entire Conversely, a temporal lobe lesion which visual fi eld turned a deep red on each occasion interrupts optic radiation pathways would for 2–4 min. Additional symptoms include produce a superior quadrantanopsia. chronic fatigue and a gradual decline in his per- Optokinetic slow phases to the right are formance at work over 1 year. He feels unmoti- impaired and quick phases to the left appear vated and seldom contributes to discussions in slow. This is also compatible with a right group meetings. He has to struggle to recall cerebral (parietal in this case) lesion. It is eas- details of contracts he has drawn up. iest to observe the impaired contralateral Neurologic examination : quick phases for localization purposes. Blood pressure 130/80, pulse 76 and regular, Conversely, the right arm pronator drift temperature 98.6 F. upwards and loss of position sense at the right Abnormal fi ndings include inability to per- fi ngers and wrist indicate a left parietal lesion. form serial sevens and poor short-term recall. He Of note is the intact vibration perception com- has a left homonymous inferior quadrantanopsia monly preserved with cerebral lesions. 11 Diagnostic Dilemmas 265

4. The differential diagnosis includes infec- hemiglossectomy and radical neck dissection for tion, demyelinating disease, neoplasm and carcinoma of the tongue. She has been anorectic, paraneoplastic syndrome as the primary lost 26 lb, weighs 82 lb, and is dehydrated. Her considerations. pneumonia is treated successfully with antibiot- A primary neoplasm such as a glioma is ics and she is rehydrated. On the third hospital nearly always unilateral. Metastatic neoplasm day, she complains of dizziness and the nurses could be a reasonable explanation, but neither fi nd that her level of consciousness varies from illness is likely to have been slowly progres- alert to lethargic with occasional episodes of sive over 1 year. Conversely, a paraneoplastic being diffi cult to rouse. syndrome is a consideration. Demyelinating Neurologic examination : disease is likely by MRI criteria. Multiple Blood pressure 94/50, pulse 100, temperature 98.6 F. sclerosis, the epitome of a demyelinating dis- The patient is lethargic but is rousable and order, presents initially with optic nerve, then completely oriented with normal speech, brainstem, or spinal cord involvement, at vari- naming, and repetition. She spells, adds, and ance with this clinical presentation. An infec- reverses words correctly. There is horizontal tious etiology whether fungal, bacterial, viral, gaze-evoked nystagmus, paresis of the left infe- or parasitic are diagnostic possibilities. All of rior oblique and superior rectus muscles, and them are likely to be acute and devastating, right superior oblique muscle. She has moderate incompatible with this alert, mildly dysfunc- bilateral heel-to-shin and fi nger-to-nose ataxia tional individual. and an ataxic, wide-based gait. This patient has progressive multifocal leu- koencephalopathy (Toothaker and Rubin, Case 16 Questions : 2009), a demyelinating disorder caused by the 1. Where is the lesion? Why? JC virus which is carried by more than 90% of 2. What does each eye movement sign signify? the population in the latent form in the kidney. Why is there ataxia? The virus infects the oligodendrocyte, the glial 3. What is the diagnosis? cell responsible for myelination. It has a pre- 4. What treatment restores the patient to a nor- dilection for parieto-occipital white matter. mal neurologic examination within 48 h? Although the natural course is usually sub- Case 16 Analysis : acute there are chronic cases. 1. Midbrain. The altered sensorium indicates 5. The positive diagnostic tests were HIV serol- pathology affecting the upper pontine, mid- ogy and PCR for JC virus in the spinal fl uid. brain, or thalamic reticular formation. The Additionally, he may have HIV dementia, typ- eye signs and ataxia support midbrain ically subcortical, as he exhibits memory localization. impairment and abulia which is manifested by 2. Horizontal gaze-evoked nystagmus means his loss of initiative, apathy, and paucity of nystagmus to the right on right lateral gaze speech. and nystagmus to the left on left lateral gaze. 6. HIV predisposes to any opportunistic central Nystagmus present in two directions is always nervous system infection whether bacterial, of central nervous system origin (brainstem or protozoan, viral, or fungal. Neoplasms, espe- cerebellum) or a toxic effect of drugs on the cially primary central nervous system lym- central nervous system. An exception would phoma and metastatic Kaposi sarcoma, are be the occasional patient with congenital nys- considerations. Additional associated diseases tagmus who usually informs the examiner that include vacuolar myelopathy, sensory poly- his eye fi ndings are of longstanding. Horizontal neuropathy, and myopathy. gaze-evoked nystagmus does not have a more Case 16 A 54-year-old woman is admitted to the precise localizing value. The left inferior hospital because of fever and cough productive of oblique and superior rectus pareses indicate a yellow sputum. Six months ago, she had a left partial left 3rd nerve palsy due to a lesion 266 11 Diagnostic Dilemmas

affecting adjacent 3rd nerve fascicles. The Case 17 Analysis : superior oblique weakness OD indicates a 1. Gait apraxia. right 4th nerve lesion or a left trochlear nucleus 2. Questions that might yield diagnostic lesion which is likely in this case since the left responses relate to memory loss, bladder func- 3rd nerve fascicles are also involved. The tion, tremor, handwriting, and initiating move- ataxia is a result of damage to the brachium ments such as turning in bed or getting out of conjunctivum (superior cerebellar peduncle). a chair. In other words, does this patient have 3. Wernicke’s encephalopathy (Victor et al. a dementia or an extrapyramidal disorder such 1989 ) . This patient has the triad of altered as Parkinson’s disease? sensorium, eye signs, and ataxia of this disor- 3. Falling backwards, retropulsion, may occur der. The patient was obviously poorly nour- with Parkinson’s disease, communicating ished, cachectic, and evidently defi cient in hydrocephalus and, rarely, neoplasms affect-

stores of the B vitamins, primarily B1 . ing midline structures. Hydration with D5W without vitamin supple- 4. Yes. The patient had meningitis 20 years ago.

mentation resulted in depleting whatever B 1 5. The differential diagnosis is hydrocephalus, stores remained with subsequent precipitation Parkinson’s disease, and midline neoplasm. of this syndrome. The symptoms and signs of hydrocephalus

4. Vitamin B1 100 mg. given IV and then by include a triad of gait apraxia (not ataxia), mouth daily restored the patient to normal dementia, and urinary incontinence. This is a neurologic function. communicating hydrocephalus, i.e., there is free fl ow of cerebrospinal fl uid from the lat- Case 17 An 82-year-old woman complains of eral ventricles to the lumbar subarachnoid dizziness only when walking. This has progres- space. The gait apraxia nearly always occurs sively increased over the last 3 months. She feels fi rst. When the syndrome begins with demen- well otherwise. tia, degenerative disease is much more likely. Past medical history includes repair of an A resting tremor, micrographia and bradyki- ascending aortic aneurysm, hypertension, and nesia, would support a diagnosis of resection of colon carcinoma 1 year ago. She had Parkinson’s disease. Hydrocephalus may be a bacterial meningitis at age 62 recovering quickly sequela of meningitis and subarachnoid hem- and without sequelae. orrhage as well as due to the idiopathic syn- Neurologic examination : drome of normal pressure hydrocephalus. Blood pressure is 130/76 and pulse 106 and regu- This patient has communicating hydrocepha- lar. The patient has an abnormal gait manifested lus due to bacterial meningitis which likely by diffi culty initiating a step after getting out of a caused adhesions and partial obstruction of chair. After she does take a fi rst step, her subse- spinal fl uid resorption into the venous sinuses. quent gait is shuffl ing and she takes extra steps The disorder may develop slowly over when turning. When seated on the examining decades, but more often as a subacute com- table, she falls backwards and must be supported plication. The treatment is a ventriculoperito- by a nurse standing behind her. neal shunt. Case 17 Questions : Case 18 A 40-year-old bachelor, living alone, is 1. What is the name of this gait disorder? brought to the Emergency Room by a neighbor 2. What other symptoms should the patient be because of confusion fi rst noted 1 week ago and questioned about? progressing in severity. His neighbor initially 3. What is the signifi cance of falling backwards? thought he was drunk since he had obviously 4. Is the past medical history relevant? been inebriated on several occasions during the 5. What is the differential diagnosis and the preceding 6 months. His past medical history is diagnosis? otherwise unknown. 11 Diagnostic Dilemmas 267

Neurologic examination : 4. HSV-1 resides in latent form in the trigeminal Blood pressure is 120/70, pulse 86, regular, and he ganglion (Gasserian) located under the tempo- is afebrile. The patient is alert and has fl uent, rapid, ral lobe and thus there is a predilection for and unintelligible speech. Naming and repetition temporal lobe involvement. The patient is are impaired. He states his name and follows two treated successfully with acyclovir. but not three-step commands. Perseveration is Case 19 A 22-year-old male college student is common. He has a right Hoffmann sign and the found on the street, unresponsive. The patient’s right plantar response is neutral, whereas the left is family was contacted and they report no known fl exor. illnesses nor does he take any medications. There Case 18 Questions : is neither a history of drug abuse nor signs of 1. Where is the lesion? Explain the abnormal trauma. neurologic signs. Neurologic examination : 2. What are the diagnostic considerations? Blood pressure is 150/70, pulse 100, temperature 3. What tests should be performed? 100.2 F, and the neck is supple. The patient 4. What is the source of this disease. responds to pain with movement of all extremi- Case 18 Analysis : ties. The oculocephalic maneuver elicits full hor- 1. Fluent, rapid, unintelligible speech with poor izontal and vertical eye movements. Caloric repetition is compatible with a fl uent aphasia, testing with cold water AS produces ipsilateral Wernicke’s type. Poor comprehension is deviation and contralateral nystagmus and vice characteristic. Perseveration, repeating the versa AD. The right pupil is 6 mm with a 2+/4 same response to a new request, is a common reaction to light and the left pupil is 3 mm with a accompaniment, although not a pathogno- 2+/4 reaction to light. monic sign of an aphasia. The refl ex asymme- CAT scan (head) is normal. CBC reveals a tries support the localization to the left cerebral hematocrit of 42%, 13,000 white cells /cu mm hemisphere. with 82% neutrophils. A lumbar puncture dis- 2. Neither an ischemic nor hemorrhagic stroke closes 27 wbc/cu mm of which 80% are neutrophils. is likely to present with a steadily progres- The protein is 30 mg/dl and glucose 70 mg/dl. sive decline in language function. An isch- Gram stain, fungal smear, and acid-fast smear are emic infarction with secondary edema or an normal. The cryptococcal antigen is negative. intracerebral hematoma with recurrent bleed- A subsequent MRI scan of the brain is normal. ing or edema could be the etiology, but it is Case 19 Questions : not typical. Neoplasm, especially metastatic 1. What test is now done immediately? with secondary hemorrhage, is an additional 2. What could it show? possibility. An infection, mainly herpes sim- 3. What clinical and laboratory fi ndings does this plex encephalitis (Steiner et al. 2007 ) (HSV- patient have which are consistent with the 1), fi ts the clinical presentation and is the eventual diagnosis? etiology. 4. What treatment restores the patient to normal? 3. The electroencephalogram shows periodic lat- eralized epileptiform discharges in the left Case 19 Analysis : temporal leads. The MRI reveals T2 and 1. Electroencephalogram. An electroencephalo- FLAIR (fl uid attenuated inversion recovery) gram is the only commonly used neurologic hyperintensity in the left temporal lobe. test that evaluates function, not structure. MRI Cerebrospinal fl uid contains 90 lymphocytes/ (head) is often normal in a comatose patient as cu mm and 300 rbc/cumm, indicating the pres- in this case. ence of a hemorrhagic encephalitis. PCR dis- 2. The EEG discloses frequent, generalized, sharp, closes HSV-1 DNA. and slow wave discharges indicating a seizure 268 11 Diagnostic Dilemmas

disorder. Since some of this epileptiform activ- Case 20 Analysis : ity lasts 8 s, the patient’s prolonged unrespon- 1. The sensory and gait examinations are most sive state is due to ongoing epileptiform activity, useful. The patient has vibration sense loss at nonconvulsive status epilepticus (Kaplan toes and ankles and no position sense at the 2005 ) . toes. The Romberg test is positive. 3. Low grade fever, unilateral mydriasis, and 2. Light-near dissociation. This is not a pathog- abnormal caloric testing because of ocular nomonic sign. It may occur with midbrain deviation may occur in postictal states. and, when there is associated visual impair- Unilateral mydriasis can be present in the ment, optic nerve lesions. absence of focal pathology. 3. The patient has Argyll–Robertson pupils The mild leukocytosis with left shift and which are usually small, irregular, and poorly the CSF pleocytosis are well-known occur- or nonreactive to light, but with a good reac- rences in patients with seizures. tion to the near stimulus. This patient has tabes 4. The patient is treated with intravenous val- dorsalis, a complication of tertiary proate and returns to normal after 2 hours have (Roos 1992) . The pathology is in the dorsal elapsed. roots with secondary degeneration in the pos- terior columns. The autonomic nervous sys- Case 20 A 67-year-old man arrives at the tem is commonly involved and, in this case, Emergency Room with intense paroxysmal includes orthostatic hypotension and impaired abdominal pain. This is his fourth visit over the parasympathetic innervation of the bladder. previous 3 months. In the last 3 years, he has had The latter results in urinary retention and over- an appendectomy, cholecystectomy, and two fl ow incontinence. exploratory laparotomies. He has periodic 4. The patient has a Charcot joint which is dizziness and an unsteady gait. He has diffi culty associated with sensory loss, cartilage rupture, initiating urination and additionally has urinary and bone overgrowth. The same fi ndings may incontinence. occur in patients with diabetes. Past medical history includes osteoarthritis with an enlarged deformed left ankle. He was a Case 21 A 68-year-old man requests an evalua- cocaine addict for many years. tion for memory loss of 6 months duration. This primarily involves recall of names. He has been Neurologic examination : depressed since his retirement 1 year ago. He has Blood pressure 130/80 sitting and 90/50 stand- a history of ulcerative colitis which is quiescent. ing. The pulse is 72 and 76, respectively. Pupils He was a heavy smoker of two packs per day are 2.5 mm, irregular and with a 2+/4 reaction to until 5 years ago when he was hospitalized for 1 light, but 4+/4 to near. Visual acuity is 20/30 OD week because of pneumonia at which time he and 20/40 OS, but with pinhole 20/20 OU. There was able to quit smoking. At about the same is no central scotoma. time, he noted loss of his sensation of taste. Case 20 Questions : Additionally, he has insomnia with early morn- 1. What other parts of the neurologic examina- ing awakening. tion could yield diagnostic information? 2. What is the name given for this type of pupil- Neurologic examination : lary reaction? Is this a pathognomonic sign? Blood pressure 150/90. There are bilateral carotid 3. What is the name of this patient’s pupillary bruits. The pulse is 96 and irregularly irregular. The abnormality and the disease this patient is suf- patient has a fl at depressed affect. His Mini-Mental fering from? What other manifestations are State Exam score is 26/30. This includes recalling present? one of three words after 3 minutes have elapsed, 4. What is the name of the deformed, enlarged knowing the month but not the date and he makes joint? one error when reversing fi ve-letter words. 11 Diagnostic Dilemmas 269

Case 21 Questions : 2010 ) , his wife requests an additional study 1. Could the carotid bruits tie in with his memory to support this conclusion. What routine pro- loss? cedure could be useful? 2. Could the cardiac arrhythmia be a factor? Case 21 Analysis : 3. What part of the cranial nerve examination 1. No. This patient has not had transient isch- might yield a diagnostic clue? emic attacks. 4. What diagnoses should be considered? 2. No. This patient has not had transient isch- A workup for dementia is indicated with a emic attacks or stroke. CBC, TSH, B , glucose, calcium, electrolytes, 12 3. Sense of smell. The patient complains of loss BUN, creatinine and liver functions, and RPR. of taste, but the sense of smell provides most An MRI is performed. The patient is seen by a of the nuance of taste. cardiologist and is treated with atenolol and 4. Mild cognitive impairment, pseudodementia warfarin. Fluoxetine is added. of depression, Alzheimer’s disease. Re-evaluation 2 months later : 5. Normal. The patient feels much better. He sleeps well and 6. Paratonic rigidity and grasp refl exes. no longer feels depressed. His wife is present and 7. Neuropsychological testing. This disclosed she is disturbed by his inertia. He prefers to stay at evidence of a degenerative dementia. home and watch television when he used to be urg- 8. Anosognosia. ing his wife to go out to dinner and to the movies. 9. Abulia. 10. Lumbar puncture. Patients with Alzheimer’s Neurologic examination : disease have markedly decreased levels of The blood pressure is 124/70 and the pulse 64 ß-amyloid and signifi cantly elevated and irregularly irregular. A Mini-Mental State 1–42 levels of tau protein. Exam score is 28/30 (improved). Case 22(A) A 55-year-old woman complains of Case 21 Additional Questions : double vision, neck, and low back pain of 10 days 5. What were the most likely results of the stud- duration. She notes that the double vision is hori- ies performed? zontal, present only when focusing on distant 6. What abnormal signs should be searched for objects and perhaps more evident when looking with the motor and refl ex examination? to the left. The neck pain bothers her when she 7. What examination is likely to be the most puts on her shoes. Her back pain is constant, valuable in this clinical setting? unchanged by any position and occasionally radi- Re-evaluation 4 months later : ates around to the navel on the left side. Two days The patient reports that his memory is now nor- ago, she suffered a burn on her right leg when mal. His wife says he is getting lost when driving testing the temperature of her bath water. in the neighborhood. He no longer goes for daily Past medical history : walks and he exhibits no emotion. The patient has hypertension, chronic renal insuf- Neurologic examination : fi ciency with a BUN of 48 mg/dl and creatinine Examination is remarkable for a Mini-Mental 2 mg/dl. She has chronic bronchitis associated State Exam score of 20/30. with a one pack per day smoking habit for 35 years. Case 21 Additional Questions : 8. What name can be given for the patient’s Neurologic examination : assessment of his memory? Blood pressure 140/95, pulse 72, temperature 9. What condition is his wife describing? 98.6°F. She has horizontal diplopia only on left 10. Although neuropsychological testing shows lateral gaze when looking at an object beyond evidence for a degenerative cause for demen- 15 ft. The left palpebral fi ssure is wider than tia, namely Alzheimer’s disease (Mayeux the right. 270 11 Diagnostic Dilemmas

Case 22(A) Questions : Case 22(B) Analysis : 1. What is the signifi cance of diplopia at distance? 1. Cytology with a 10 cc aliquot of fl uid, acid- 2. The history mandates an evaluation for and fast smear and culture, gram stain, cryptococ- comment about what neurologic signs? cal antigen, fungal smear, and culture. The 3. Can a single lesion explain the neurologic cytology is positive. symptoms and signs? What are the anatomic 2. The diagnosis is leptomeningeal carcinomato- considerations? sis due to oat cell carcinoma of the lung (Little 4. A CT (head), noncontrast because of renal et al. 1974 ; Clarke et al. 2010 ) . insuffi ciency, is normal. This small commu- 3. There are at least two possible explanations. nity hospital has no MRI unit. What is the next The tumor cells may utilize glucose or there study indicated? may be defective glucose entry into the cere- brospinal fl uid. IV glucose given to patients Case 22(A) Analysis : with leptomeningeal carcinomatosis does not 1. Diplopia at distance occurs with 6th nerve pal- increase CSF glucose levels, whereas, in nor- sies. This patient has a left 6th nerve lesion mal controls, CSF glucose is increased. since diplopia is more evident when looking to 4. A repeat lumbar puncture with another 10 cc the left. At distance, the eyes must diverge and of cerebrospinal fl uid should be sent for cytol- thus a lateral rectus paresis is easier to detect. ogy. This should further increase the yield of 2. Since head fl exion (putting on shoes) causes positive cytology to over 90%. neck pain, meningeal irritation may be present. A comment about nuchal rigidity or Brudzinski Case 23 A 48-year-old man complains of severe sign should be made. This patient does have perianal pain. This began 5 weeks ago and is nuchal rigidity. increasing in intensity. Over the past 2 weeks, he 3. No. The patient has a left 6th and 7th nerve has had diffi culty voiding and moving his bow- paresis. The latter is surmised because of the els. For the past week, he has been dragging his wider left palpebral fi ssure implying weak left left foot. Past medical history is remarkable for eye closure (orbicularis oculi). This is likely to Crohn’s disease and alcohol abuse. be a cranial neuropathy, although a lesion at Examination : the facial colliculus in the pons where the 7th Blood pressure is 90/60 and pulse 64 and regular. nerve fi bers loop around the 6th nucleus can- The general medical examination is remarkable not be excluded. Thoracic radiculopathy is for splenomegaly, a suprapubic mass, and large likely to explain the pain radiating around to inguinal nodes. the navel. Nuchal rigidity indicates meningeal Neurologic examination reveals absent knee involvement. and ankle jerks, 4+/5 strength of left quadriceps, 4. Lumbar puncture. hamstrings, gastrocnemius, and 4/5 strength of Case 22(B) The cerebrospinal fl uid protein is left anterior tibialis muscles. Sensory examina- 80 mg/dl. The CSF glucose is 18 mg/dl. A simul- tion of the feet is normal. taneous serum glucose is 62 mg/dl. The CSF Case 23 Questions : wbc count is 220 /cu mm with 50% neutrophils. 1. What part of the sensory examination has been The IgG index is abnormal and oligoclonal omitted? bands are present. 2. What part of the routine medical examination Case 22(B) Questions : should be performed? 1. What other spinal fl uid tests should be 3. What refl ex can be used to check the S3 and performed? S4 roots? 2. What diagnosis is most likely? 4. What could the suprapubic mass be composed 3. Why is the glucose so low? of? 4. If all studies are normal, what should be done 5. Where is the lesion? next? 6. What is the suspected etiology? 11 Diagnostic Dilemmas 271

Case 23 Analysis : 3. Does this patient have unilateral or bilateral 1. Sacral sensation. This shows perianal sensory disease? loss to pin and touch; there is no sensory 4. If an MRI (head) and an MRA (head and neck) dissociation. are normal, what diagnoses should be 2. Rectal examination. With any history suggest- considered? ing sacral root involvement such as bladder 5. An MRI (head) shows three acute infarctions; dysfunction, a rectal examination is mandatory. one is subcortical in the right centrum semio- The patient had poor rectal sphincter tone. vale, a second is in the right temporal lobe, 3. The bulbocavernosus refl ex. The examiner and the third is in the left parietal region. An pricks the glans penis with a pin while a fi nger MRA (head and neck) is normal. What diag- in the rectum feels contraction of the anal noses should be considered? sphincter. 6. With either MRI fi ndings what test should be 4. The mass is a large distended bladder. next? What results could be anticipated? 5. Cauda equina. 7. What additional tests should be performed 6. Lymphoma (Bierman and Giglio 2005 ) . now? Adenopathy and splenomegaly strongly sug- Case 24 Analysis : gest this diagnosis. An inguinal node biopsy 1. Phonemic paraphasias, such as “fl indow,” and confi rms the diagnosis of lymphoma. neologisms, “vintel” and “fl atsar.” A neolo- Case 24 A 52-year-old man is admitted to the gism is making up a new word for an object. hospital because of severe headaches and 2. Transcortical sensory aphasia. This is a fl uent confusion developing over 3 days. On admission, aphasia similar to Wernicke’s aphasia, but he is disoriented to place and date, but speech with intact repetition. The lesion is usually content and fl uency are normal. Three days after outside the perisylvian region in the left pari- admission, his speech becomes incomprehensible etal cortex. but fl uency remains normal. He uses occasional 3. Bilateral because of a confusional state at the nonsensical words. He has a 20-year history of onset. hypertension controlled with medication. A CBC, 4. (a) Infection. Viral encephalitis is a consider- ESR, metabolic panel, liver functions, and ANA ation, especially herpes simplex encephali- are all normal. A neurologic consultation is tis which often presents with an aphasia, requested. but has a predilection for the temporal lobe. The MRI is usually abnormal. Neurologic examination : Another virus causing encephalitis is pos- Blood pressure 140/100, pulse 100, regular. There sible. Meningitis is not likely as the neck is are no carotid bruits. The patient is afebrile and his supple. Fungal disease causing an associ- neck is supple. His speech is rambling, fl uent, rela- ated vasculitis and human immunodefi - tively rapid, and he makes numerous paraphasias ciency virus (HIV) should be considered. such as “fl indow” instead of “window,” “sty” (b) Neoplasm, especially primary central ner- instead of “fl y.” He calls a watch a “vintel” and a vous system lymphoma. shoe, “fl atsar.” Repetition is normal. He does not (c) Vascular respond to visual threat in the right visual fi eld Vasculitis such as primary angiitis of when testing each eye alone. Finger tapping on the the central nervous system. Other types right side is slow. such as polyarteritis nodosa or systemic Case 24 Questions : lupus erythematosus usually have sys- 1. What two types of paraphasias does he temic signs. exhibit? Cardioembolism, especially subacute 2. What type of aphasia is this and what is its bacterial endocarditis, is a consideration localizing value? except for the absence of systemic signs. 272 11 Diagnostic Dilemmas

Reversible vasoconstriction syndrome musculature and mild weakness of intrinsic hand (Ducros et al. 2007 ) . Etiologies include musculature. There is mild but defi nite heel-to- migraine, illicit drug use, and over-the- shin ataxia and mild spasticity of all extremities. counter medicines for colds that include Refl exes are 3+ with a right Hoffmann sign and pseudoephedrine or other similar com- bilateral Babinski signs. Vibration perception is pounds. It may occur during the puerpe- absent at toes, ankles, and knees. Position sense rium. CSF is normal in these disorders as is absent at the toes. There is prominent distal vasoconstriction is the etiology of these impairment of pain and temperature perception symptoms. in all extremities. 5. Vascular etiologies as described above. Case 25 Questions : 6. The next test is a lumbar puncture which 1. What is the anatomic basis of the neurologic shows: 110 wbc/cu mm of which 95% are symptoms and signs? Is there more than one lymphocytes, 200 rbc/cu mm, protein 94 mg/dl, neuroanatomic site of involvement? glucose 77 mg/dl with a simultaneous blood 2. Could this entire disorder be due to degenera- sugar of 110 mg/dl. Gram stain, India ink, tive or neoplastic disease? cryptococcal antigen, VDRL, AFB smear, and 3. What disease category is likely to be the PCR for herpes simplex are negative. etiology? 7. A brain and leptomeningeal biopsy should be 4. What tests should be ordered? performed obtaining a specimen from the tip 5. What two diagnoses are most likely? of the nondominant temporal lobe. Angiography can show beading of vessels indicating a prob- Case 25 Analysis : able infl ammatory process such as vasculitis, 1. Myelopathy is present because of spasticity, but beading may be present with fungal infec- unilateral Hoffmann sign, and bilateral tions, fi bromuscular dysplasia, infectious Babinski signs. Symmetrical hyperrefl exia vasculitis, and some neoplasms. The absence alone is normal. Vibration loss precedes posi- of systemic symptoms and signs supports a tion sense loss with both neuropathies and diagnosis of primary angiitis of the central myelopathies. As an isolated fi nding, neurop- nervous system which is the diagnosis as the athy is a far more frequent etiology. When biopsy shows Langerhans-type giant cells, myelopathic signs are present, as in this case, lymphocytes, histiocytes, and plasma cells. the posterior column is most likely to be the site of pathology. Diffuse weakness in the legs Case 25 A 63-year-old woman complains of a is more likely with myelopathy than a poly- progressively unsteady gait for 3 months. She has neuropathy with the possible exception of an tingling sensations in the toes and weakness in acute or chronic infl ammatory demyelinating the feet and hands. She had blurred vision with polyneuropathy, not present here because of the left eye over the past week. myelopathic signs. Distal sensory loss of pain The past medical history is remarkable for type and temperature supports a diagnosis of neu- II diabetes, a cholecystectomy 30 years ago and ropathy. Heel-to-shin ataxia is commonly gastric bypass surgery 15 years ago. She receives associated with proprioception loss. Optic B12 injections every 2 weeks and takes multivita- nerve disease is evident because of poor color mins, but has not always been compliant. perception and an afferent pupillary defect Neurologic examination : despite normal visual acuity. Blood pressure 110/70, pulse 72, and regular. Thus, the neuroanatomic pathology is myel- The patient is afebrile. She has poor color per- opathy, neuropathy, and optic neuropathy. ception OS associated with an afferent pupillary 2. Neoplasm is not considered because of the defect with a visual acuity of 20/20 and a normal multiple neuroanatomic regions affected. A fundus. She has mild, symmetrical weakness of degenerative disease involving these three both legs, equally involving proximal and distal sites is not apparent. 11 Diagnostic Dilemmas 273

3. Nutritional disease is always suspected after 5. What is the most likely diagnosis as well as gastric bypass surgery. other illnesses which may produce similar 4. When nutritional disease is suspected, espe- presentations?

cially postgastric bypass, vitamin B12 , RBC 6. What is the etiology? folate, copper, ferritin, serum iron and total iron, 7. What evaluations are useful to make this binding capacity levels should be ordered. diagnosis? 5. Either vitamin B or copper defi ciency may 12 Case 26 Analysis : cause myelopathy, neuropathy, and optic neu- 1. Psychosis, memory loss, and movement ropathy. The etiology is copper defi ciency in disorder. this case (Kumar 2006 ) . 2. Ocular motor apraxia. It is an early manifesta- Case 26 A 53-year-old woman is brought to her tion of this illness. internist under duress by her family because of a 3. Hyperkinetic and choreiform. 5-month history of behavioral changes, loss of 4. Repetitive squeezing of the examiner’s fi ngers memory, and poor balance. She has become with- as well as intermittent tongue protrusion are drawn, depressed, and paranoid. She accuses her both common with chorea. The intermittent husband of having a mistress and cannot be dis- squeezing of the examiner’s fi ngers has been suaded from this fi xed belief. Her only medical called “milkmaid’s grip.” illness is asthma which is well-controlled with 5. Huntington’s chorea (Jankovic and Tolosa albuterol. The family history is signifi cant as her 2007 ) . The major differential diagnoses are: father died of Parkinson’s disease, bedridden, at (a) Metabolic disorders which include: hyper- age 62. One of four siblings has tremor and a gait thyroidism, hypoparathyroidism, hyperna- disorder. tremia, hyponatremia, hypomagnesemia, hyperglycemia, hypoglycemia, nutritional Neurologic examination : defi ciencies, and acquired hepatocerebral Blood pressure 110/70 and pulse 96, regular. degeneration. The patient appears sullen and hostile. Her (b) Infectious diseases: Sydenham’s chorea, Mini-Mental State Exam score is 24/30. Her encephalitis, and Creutzfeldt–Jakob speech is dysarthric. She has diffi culty initiating disease. saccades which are clearly slow and are accom- (c) Genetic (very rare): neuroacanthocytosis panied by simultaneous head movements. and Wilson’s disease. Optokinetic nystagmus is diffi cult to elicit and (d) Drugs: phenytoin, amphetamines, neurolep- quick phases of nystagmus are slow. She is unable tics, dopaminergic drugs, tricyclics, oral to protrude her tongue for more than 1 s. When contraceptives. checking for arm drift, facial grimacing is promi- (e) Degenerative diseases such as multiple nent. She appears restless and fi dgety. Her system atrophy. strength is normal but grasp is characterized by (f) Sen ile chorea. repetitive squeezing of the examiner’s fi ngers. 6. Genetic basis of Huntington’s chorea: There is Her gait is ataxic and lurching. a mutation on chromosome 4 which contains Case 26 Questions : an unstable expansion of CAG (cytosine, ade- 1. What are the three major elements of the nine, guanine) repeats. This causes an increase patient’s illness? in the Huntingtin protein which has a predilec- 2. What is the name of this patient’s eye move- tion for damaging neurons in the caudate ment disorder and does it have diagnostic nucleus. signifi cance? 7. The family history must be scrutinized as the 3. What type of movement disorder does this disease has an autosomal dominant inheri- patient have? tance pattern. MRI scans show cerebral and 4. What is the signifi cance of repetitive squeezing caudate nucleus atrophy. DNA testing is of the examiner’s fi ngers when checking grasp? available. 274 11 Diagnostic Dilemmas

Case 27 A 45-year-old woman is brought to the Case 27 Additional Questions : Emergency Room because of the abrupt onset of 4. Since the lesion is vascular, what additional left-sided weakness and double vision. She has information should be obtained from the no previous history of neurologic disease other patient? than migraine and a remote history of an optic 5. What additional laboratory tests should be neuritis with mild residual visual impairment. drawn? There is no family history of neurologic disease 6. What diagnosis is suspected and proven? or stroke. She is a lawyer, married for 20 years, Case 27 Analysis : and has no children although she has wanted 1. The previous optic neuritis could be either on them. She has mild hypertension treated with an ischemic or demyelinating basis. If it was atenolol and under good control. She smokes a ischemic, the diagnosis was likely to be ante- half a pack of cigarettes per day. rior ischemic optic neuropathy. The lesion Neurologic examination : which explains the horizontal paresis of the Blood pressure 140/105. Pulse 82, regular. right eye would be a right MLF lesion causing OD: Visual acuity 20/30, central scotoma to a right adduction paresis and a right PPRF red matches, and an afferent pupillary defect. lesion which causes a right gaze paresis. OD: Remains midline and has full up and Hence, the right eye has no horizontal move- down gaze. ment. The left eye deviates to the left because OS: Deviated to the left and also has full vertical of the right PPRF lesion. eye movements. 2. One-and-a-half syndrome. She has a mild left arm pronator drift, impaired 3. Vascular and demyelinating disease. fi nger and foot tap on the left side and poor rapid 4. The patient should be questioned about alternating movements of the left arm. whether she had spontaneous abortions. She She has a left hyperrefl exia, Hoffmann and responds that she had three. Babinski sign. 5. Lupus anticoagulant, anticardiolipin antibodies, Laboratory data: CBC is normal except for a ANA, RPR, FTA-ABS. platelet count of 86,000 /cu mm. Metabolic pro- Results are elevated anticardiolipin anti- fi le, liver functions, PT and ESR are normal. The bodies, positive test for lupus anticoagulant, PTT is elevated by 1 s. ANA positive 1:320, RPR positive, FTA-ABS negative. The patient has already had a plate- Case 27 Questions : let count which was low at 86,000 /cu mm and 1. What structures are affected that produce these a PTT elevated by 1 s. The abnormal PTT is a neurologic abnormalities? major clue for this diagnosis even if minimally 2. What is the name of the eye movement elevated. syndrome? 6. Antiphospholipid antibody syndrome (Levine 3. What two disease categories are the primary et al. 2002 ) . Common manifestations of this considerations? syndrome include migraine, visual distur- Neurologic evaluation : bances, a history of spontaneous abortion, MRA (head and neck) are normal. thrombophlebitis, pulmonary embolism, and MRI (head) shows a few scattered T2 and recurrent ischemic stroke. Acute encephalop- FLAIR hyperintensities which are subcortical in athy, seizures, and disseminated intravascular the brainstem but not periventricular. Diffusion- coagulation have been reported. weighted imaging (DWI) shows a hyperintense The primary treatment is anticoagulation lesion in the pontine tegmentum and basis pontis. without which probably the majority of The fi nding supports a diagnosis of an acute patients would have recurrent complications infarction. including ischemic stroke. Prednisone is gen- Cerebrospinal fl uid is normal including IgG erally not effective nor is it used unless antico- index and oligoclonal bands. agulation with warfarin is unsatisfactory. 11 Diagnostic Dilemmas 275

Case 28 A 31-year-old woman is brought to the 5. What diagnosis should be considered? Emergency Room because of the sudden onset of 6. What tests should be ordered? mild confusion and rapid, jerky, uncontrollable movements of both hands. Four years ago, she Case 28 Analysis : had the abrupt onset of loss of consciousness, 1. There is both diffuse and focal cerebral hemi- facial grimacing, and lip smacking lasting 2 min. sphere involvement. The focal involvement is An MRI (head) with and without contrast was manifested by the history of seizures due to a normal. An EEG disclosed a right temporal sharp right temporal lobe epileptic focus. Diffuse wave focus. The patient had four similar complex involvement is manifested by the confusional partial seizures until a satisfactory lamotrigine state. level was achieved over 2 months. She has had no The abnormal involuntary movements indi- seizures since then. Three months ago, she noted cate an extrapyramidal disorder. burning, tingling sensations in the feet associated Neuropathy is evident because of absent with slowly progressive weakness of both legs, refl exes, distal greater than proximal weak- and poor balance. ness, and distal sensory loss. There are no signs of myelopathy such as Neurologic examination : spasticity, asymmetric, increased or patho- Blood pressure 160/90. Pulse 84, regular. The logic refl exes, or a sensory level. patient is disoriented to month and year and 2. Chorea. The rapid, jerky movements with pre- recalls 1/3 words after 3 minutes have elapsed. dilection for distal musculature are character- She has rapid, jerky movements of both hands. istic fi ndings. There is mild proximal and moderate distal leg 3. Electroencephalography and lumbar puncture. weakness. She has an unsteady gait with bilateral The EEG shows a diffusely slow pattern footdrop. Refl exes are absent in the legs. Position with a few right temporal sharp waves, an sense is impaired at the toes and there is loss of epileptiform fi nding. vibratory perception at toes and ankles. There is Cerebrospinal fl uid shows six wbc/cu mm distal sensory loss to pin in both legs and both (lymphocytes), protein of 77 mg/dl, increased hands. IgG index, and oligoclonal bands. Case 28 Questions : 4. No. Multiple sclerosis does not cause neurop- 1. Where is or are the lesions? Could this patient athy and rarely, if ever, an acute confusional have a myelopathy? state. Although abnormal involuntary move- 2. What is the name of the movement disorder? ments may occur, they are rare. The cerebro- spinal fl uid abnormalities, elevated IgG index, Laboratory data : and the presence of oligoclonal bands are CBC, electrolytes, and glucose are normal. characteristic, but not pathognomonic of mul- Creatinine is 2 mg/dl and BUN 39 mg/dl. Liver tiple sclerosis. Increased CSF protein is not functions are normal. ordinarily found in patients with multiple The patient is treated with haloperidol and sclerosis. improves. On the third hospital day, she is con- 5. Systemic lupus erythematosus (Joseph et al. fused. The neurologic examination reveals an 2007) . Seizures, an acute confusional state, alert patient oriented × 1 only, who follows just abnormal involuntary movements – especially simple commands. She is unable to add 14 + 7 chorea – and neuropathy are well-described and reverse the word “hand.” Her short-term manifestations of this disease. The major diag- recall is 0 after 3 minutes have elapsed. nostic clue is impaired renal function indicat- Case 28 Additional Questions : ing a systemic disorder with mainly neurologic 3. What neurologic studies are indicated? What manifestations. might they show? 6. ANA is positive in 98% of patients with neu- 4. Could this patient have multiple sclerosis? ropsychiatric manifestations of SLE (NPSLE). 276 11 Diagnostic Dilemmas

Anti-double-stranded DNA is positive in 3. An MRI (head) with and without contrast is 60% of patients with NPSLE. normal. A lumbar puncture discloses 60 wbc/ SSA, SSB, anti-Smith, anticardiolipin, and cu mm, 80 mg/dl of which are neutrophils. A lupus anticoagulant studies are frequently CSF protein is 80 mg/dl. Oligoclonal bands positive. are not resent. (a) Can an MRI (head) be normal in patients Additional comments : with multiple sclerosis? An acute confusional state may be due to anti- (b) Are these CSF fi ndings compatible with a neuronal antibodies, but this is a diagnosis of diagnosis of multiple sclerosis? exclusion in view of other possible etiologies 4. What other test is indicated? such as venous thrombosis, multiple strokes, sei- 5. What diagnosis is suspected? zures, metabolic encephalopathy, and opportu- nistic infections, particularly fungal diseases. Case 29 Analysis : Chorea may be due to small vessel ischemic dis- 1. Optic nerve and spinal cord. Impaired color ease or antineuronal antibodies. Seizures are perception is an early sign of optic nerve thought to be related to antiphospholipid antibod- disease and may be a residual fi nding after ies as well as antineuronal antibodies. recovery from an optic neuritis or retrobulbar Case 29 A 30-year-old woman complains of dif- neuritis. fi culty using her right hand for delicate move- 2. The patient has a partial Brown-Séquard syn- ments required for making jewelry. This began drome involving the lateral column of the cer- just 1 week ago simultaneous with numbness of vical spinal cord. There are signs of corticospinal the left leg. She has been healthy otherwise except tract disease on the right side because of ipsi- for a transient episode of visual impairment when lateral spasticity, right arm weakness, right on a 2-week backpacking trip on the Appalachian Hoffmann, and right Babinski signs. The lateral Trail 2 years before. She had blurred vision with spinothalamic tract is also affected because of the left eye lasting for about 1 week. When she impaired pain and temperature perception on saw her ophthalmologist 1 week later, she felt the left side up to T10. The sensory level is not well, had normal vision, and the examination was always up to the lesion level. normal. (a) Yes. (b) No. The CSF protein is usually normal in Neurologic examination : multiple sclerosis patients. The CSF white Blood pressure is 120/70 and pulse 80, regular. count seldom exceeds 40 wbc/cu mm and Funduscopic examination shows optic disk pallor are all mononuclear. OD, decreased color perception OD as red matches 3. MRI of cervical and thoracic spine. When a look maroon. She has 20/20 vision and an afferent diagnosis of myelopathy of unknown origin is pupillary defect OD. She has mild weakness of made, the entire spinal cord should be imaged. right interossei, opponens pollicis, abductor pol- This patient has a three-segment lesion in the licis brevis, extensor pollicis longus, wrist exten- cervical cord from C3–C6 on the right side. sors. There is mild spasticity of the right arm and An NMO-IgG test is ordered. the right leg as well as decreased fi nger and foot 4. Neuromyelitis optica (Devic’s disease) tap on the right side. She has a right Hoffmann (Wingerchuk et al. 2007 ) . This is now believed and right Babinski sign. There is a T10 sensory to be due to an antibody which targets the level on the left side to pin and temperature. blood-brain barrier water channel, aquaporin Case 29 Questions : 4. This disease has different CSF fi ndings than 1. Where is the pathology? multiple sclerosis. There is usually a high CSF 2. Can a lesion in one location explain all of the protein level, no oligoclonal bands, and fre- fi ndings except for the ophthalmologic mani- quently a marked pleocytosis with sometimes festations? What pathways are involved? a predominance of neutrophils. The MRI and 11 Diagnostic Dilemmas 277

CSF fi ndings support the diagnosis of neuro- Warmth indicates the side of the anhidrosis. It myelitis optica and the NMO-IgG test is posi- was not present in this patient which suggests tive confi rming the diagnosis. a lesion of the sympathetic system distal to the bifurcation of the common carotid artery as Case 30 A 62-year-old man is seen because of the sudomotor fi bers which regulate sweating an episode of impaired speech 1 week ago. He and cool the cheek follow the external carotid was struggling to fi nd the correct word and his artery. Otherwise, the sympathetic fi bers travel perceptive wife detected that either the syllables in the sheath of the internal carotid artery. were incorrect or the chosen word spoken cor- 3. The 6th nerve is located just lateral to the rectly did not fi t the context of the sentence. The internal carotid artery in the cavernous sinus. event lasted just 5 min. Thus, a lesion in the cavernous sinus is likely Past medical history is remarkable for hyper- to explain the fi ndings. tension, gout, and mild renal insuffi ciency. 4. There should be a vascular lesion in view of Neurologic examination : the transient ischemic attack. This patient has a Blood pressure is 115/60 and pulse 80, regular. left cavernous carotid aneurysm, which proba- The patient has normal speech and there is bly contains a thrombus which is the source of weakness of abduction OS. The pupils measure an embolic event to a distal branch of the left 2 mm OS and 3 mm OD. They both react well to middle cerebral artery (Stiebel-Kalish et al. light. The palpebral fi ssures are asymmetric, right 2005 ) . greater than left. Case 31 A 78-year-old widow complains of Case 30 Questions : burning feet and unsteadiness of 3 weeks dura- 1. The wife reported abnormal speech. What did tion. She has fallen twice in the past week with- she describe? What is the diagnosis of this out suffering an injury. She is having progressive event? diffi culty climbing stairs over the last 6 months. 2. Explain the pupillary fi ndings. What bedside Past medical history is remarkable for an test might help to localize the lesion which idiopathic cardiomyopathy and she has been causes the pupillary asymmetry? treated with digoxin. She is a nonsmoker, has a 3. Is there any connection between the pupillary glass of wine with dinner, and eats well. fi ndings and the lateral rectus weakness? Can a single lesion cause both fi ndings? Neurologic examination : 4. How can these disparate fi ndings by history Blood pressure 88/60. Heart rate 66, regular. and examination be related to each other? Anterior tibialis 4/5, iliopsoas 4+/5, deltoids 4+/5, and infraspinatus 4+/5. There is moderate Case 30 Analysis : heel-to-shin and mild fi nger-to-nose ataxia. She 1. The incorrect syllables were phonemic para- has a wide-based ataxic gait. phasias. The incorrect word was a semantic Refl exes are absent except for 1+ triceps paraphasia. The patient had a transient isch- bilaterally. emic attack in the left carotid-middle cerebral Vibration perception is absent at toes and artery distribution. ankles. There is distal sensory loss to pin and 2. The patient has a signifi cant pupillary asym- temperature in all extremities and the patient does metry because the difference is greater than not perceive light touch on the toes. 0.5 mm. The good reaction to light makes the larger right pupil not likely to be part of a 3rd Case 31 Questions : nerve palsy. The smaller pupil and palpebral 1. Where is the pathology? fi ssure on the left, ptosis and miosis, supports 2. In view of the location of the pathology what the presence of a partial Horner’s syndrome. should the patient be questioned about in more Anhidrosis is detected by placing the dorsum detail? of the examiner’s hands on both cheeks. 3. What are the diagnoses? 278 11 Diagnostic Dilemmas

Case 31 Analysis : Case 32 Analysis : 1. (a) The patient has a neuropathy because of 1. Sensory fi ndings are expected. Most patients absent refl exes, distal weakness in the display sensory loss off the midline and feet, and distal sensory loss as described. towards the side of involvement because of (b) The patient has a myopathy because of overlapping sensory fi bers. This is more appar- proximal weakness in upper and lower ent on the body than the face. extremities. 2. The most common location is in the thalamus, (c) The patient has cerebellar disease because but parietal lobe and brainstem lesions involv- of heel-to-shin and fi nger-to-nose ataxia ing the lateral spinothalamic tract can produce as well as a wide-based ataxic gait. similar syndromes. This patient has a thalamic 2. Alcohol consumption history must be carefully syndrome (Wilkins and Brody 1969 ; questioned. This is frequently underestimated Bogousslavsky et al. 1988 ) . especially in elderly patients. Sometimes the 3. The posterior cerebral artery irrigates the thal- only clue may be macrocytic blood indices. amus and an occlusion, due to embolism or 3. Alcoholic myopathy (Perkoff 1971 ) (chronic), intracranial vascular disease, of a branch of neuropathy, and cerebellar atrophy especially this artery is the most likely cause. involving the vermis. It should be noted that Treatment is diffi cult, but current options alcoholic myopathy can be acute or chronic. include: gabapentin, pregabalin, and tricyclic The neuropathy can develop even with a nor- compounds. mal diet. Examination for cerebellar fi ndings Case 33 A 16-year-old boy fell off his bike and may yield diagnostic information in patients struck his head on a concrete embankment. He with idiopathic neuropathies. When cerebellar was unconscious for 2 h. The day after the injury ataxia is present in patients with neuropathy, he complained of double vision with the images alcohol abuse must be strongly considered. “on top of one another.” Case 32 A 72-year-old man had the acute onset Neurologic examination : of paresthesias of the left face and body 1 week Blood pressure 110/80 and pulse 86, regular. On ago. He was on a European vacation and was right lateral gaze, the left eye is hypertropic. unable to return until just yesterday. The par- esthesias have gradually evolved into a burning Case 33 Questions : sensation which is becoming more intense and 1. What structure was affected to cause the loss now is nearly unbearable. of consciousness? Past medical history is remarkable for hyper- 2. What causes the hypertropia? tension and he is being treated with losartan. 3. What bedside test confi rms the localization? 4. Is this type of diplopia rare or common with Neurologic examination : head injury? Blood pressure 140/100 and pulse 90, regular. The patient has hypesthesia to pin on the left Case 33 Analysis : face and body with normal perception perceived 1. The dorsal midbrain is shifted against the ten- on the body 2 cm to the left of the midline and on torium cerebelli and the midbrain reticular the face 0.5 cm to the left of the midline. formation is injured resulting in loss of consciousness. Case 32 Questions : 2. Paresis of the left superior oblique muscle due 1. Are the sensory fi ndings unusual? to a left 4th nerve lesion (Burger et al. 1970 ) . 2. Where could the lesion be and what is the syn- 3. Bielschowsky test. The head is tilted to the left drome called? which increases the hypertropia and diplopia. 3. What is the most likely etiology? Head tilt to the right corrects the diplopia since 11 Diagnostic Dilemmas 279

it eliminates the need for intorsion. This may 2. He has REM behavior disorder of sleep which result in a persistent unconscious head tilt, a is a common early symptom of Parkinson’s major clue in the diagnosis. disease and may precede the development of the clinical signs. Case 34 A 73-year-old man is referred for loss 3. He was begun on carbidopa/levodopa. of balance. Over the past 3–4 months, he has 4. The patient has an extrapyramidal disorder. noted a tendency to fall backwards. His wife adds The diagnosis is Lewy body dementia that his movements have slowed considerably. (McKeith et al. 2005 ) . The clinical features He prefers to sit in a chair with armrests so he can are psychosis, hallucinations, impaired execu- push himself up to stand. Over the last week, he tive function, dementia, and parkinsonism has required assistance to stand. Despite the with an inadequate response to levodopa. This decline in motor function, he has been “acting disorder may begin with signs of Parkinson’s out his dreams” violently, sometimes attempting disease, dementia, or behavioral abnormali- to punch his wife who struggles to restrain him. ties. When the initial feature is the extrapyra- Case 34 Questions : midal disorder, dementia or behavioral 1. What four fi ndings should be searched for on abnormalities develop within 1 year of the the examination? onset of the disease. The dementia is more 2. Is the sleep disorder relevant? severe than Parkinson’s disease dementia and 3. What medicine is likely to have been started? is associated with a greater degree of cortical The patient returns 4 months later. For atrophy. The Lewy body, present in Parkinson’s three-and-a-half months, he had modest disease, Lewy body disease, and Alzheimer’s improvement in motor function as he had been disease, is an eosinophilic cytoplasmic inclu- able to get out of a chair independently. During sion which contains a pathogenic protein, the past 2 weeks, however, he has had vivid alpha-synuclein. frightening hallucinations, on one occasion Case 35 A 66-year-old retired banker complains seeing a group of foxes scampering across the of a 6-month history of poor memory and noctur- front lawn. He has complained of a foul odor nal headaches which force him out of bed. These in the house on several occasions, each lasting are occasionally associated with nausea but no 1–2 h. He has been accusing his wife of vomiting. He has had insulin-dependent diabetes infi delity. mellitus since age 18 and has been meticulous Neurologic examination : about his care. A Mini-Mental State Exam score has dropped Neurologic examination : from 28/30 to 22/30. His short-term recall is zero. Blood pressure is 160/100 and pulse 62, regular. His attention waxes and wanes. He exhibits hypo- Abnormal fi ndings include bradyphrenia, bra- mimia (masked facies), bradykinesia, cogwheel dykinesia, a stooped posture, and poverty of rigidity, and retropulsion as he needs support on associated movements. Normal laboratory stud- his back to sit on the examining table. ies include a CBC, thyroid functions, metabolic Case 34 Additional Questions : panel, liver functions, calcium, phosphorus, RPR,

4. What motor system is involved and what is the and vitamin B12 level. An MRI (head) noncon- diagnosis? trast is normal. Case 34 Analysis : Case 35 Questions : 1. The patient exhibits retropulsion, diffi culty 1. What blood test was omitted? initiating movements, and cogwheel rigidity. 2. What additional examination would be Thus, he has three of the four cardinal mani- helpful? festations of Parkinson’s disease which are 3. Treatment was initiated and the patient returns postural instability, bradykinesia, and cog- for follow-up in 6 weeks feeling well. What wheel rigidity, but not a resting tremor. treatment was given? 280 11 Diagnostic Dilemmas

Case 35 Analysis : Case 36 Analysis : 1. ESR. Any patient over age 50 with headaches 1. Hypertension, bradycardia, and a slow respi- requires this examination to rule out temporal ratory rate indicate the Cushing’s response arteritis. The ESR is 16 mm/h. which signals increased intracranial pressure. 2. Neuropsychological testing. This patient had 2. The right hemiataxia supports the presence of no evidence of impaired cognition, but there a right cerebellar hemisphere lesion. were indications of depression. 3. The impaired level of consciousness implies 3. The patient was treated with fl uoxetine and dysfunction of the reticular formation in the returned to normal in 6 weeks. midbrain or thalamus. One of the most common causes of noctur- 4. Eye deviation to the right is consistent with nal pain, including headaches, is depression either a left PPRF lesion or a lesion above the (Cummings 1989 ) . This treatable disorder oculomotor decussation (pontomesenceph- should never be overlooked. alic) on the right. In this case, since the mid- brain reticular formation is involved, Case 36 A 90-year-old man complains of the compression of the oculomotor pathway in the sudden onset of an occipital headache and stag- right midbrain is most plausible. Paresis of gering. He has a longstanding history of hyper- upward gaze is due to pathology affecting the tension treated with ramipril and indapamide. posterior commissure which is located at the The patient is promptly brought to the Emergency junction of the aqueduct of Sylvius and the Room where a brief screening neurologic exami- third ventricle. nation discloses an alert, oriented man in acute 5. Thus, the sequence of events is consistent with distress due to a severe occipital headache. He a rapidly enlarging mass in the right cerebellar has right hemiataxia and a wide-based ataxic gait. hemisphere, obstructing the aqueduct of A CAT scan (noncontrast) is ordered stat and Sylvius which then causes acute hydrocephalus neurology consultation requested. and increased intracranial pressure. Neurologic examination : Compression of the right midbrain then results Neurologic examination 15 min later discloses a in a decreased level of consciousness because blood pressure of 180/120, pulse 48, and respira- of involvement of the reticular formation, ipsi- tory rate of 6/min. The patient is poorly respon- lateral eye deviation, and eventually upgaze sive, but briefl y arousable with a painful stimulus paresis which is due to transtentorial upward and is then oriented. His eyes are deviated to the herniation (Posner et al. 2007 ) . right. Vigorous oculocephalic maneuvers elicit The CAT scan shows a large right cerebellar transient gaze to the left and vertical head move- hemisphere hematoma. This is evacuated suc- ments cause down but not upgaze. Pupils are cessfully and the patient recovers well returning 5 mm and 1+/4 reactive to light. Bilateral Babinski to near-normal. Of interest is that large portions signs are present. of the cerebellar hemisphere can be removed often with little residual neurologic defi cit. Case 36 Questions : 1. What explains the abnormal vital signs? 2. Where is the lesion that causes the right hemiataxia? References 3. Why is the patient poorly responsive? 4. Why are the eyes deviated to the right? Why is Arvanitakis Z. Update on Frontotemporal Dementia The there an upgaze paresis? Neurologist. 2010;16:16–22. Bierman P, Giglio P. Diagnosis and treatment of central ner- 5. Explain the sequence of events and name the vous system involvement in non-Hodgkin’s lymphoma. physiologic process that is occurring. North Am Hematol Oncol Clin. 2005;19(4):597–609. References 281

Biousse V, Bousser MG. Cerebral venous thrombosis. Little JR, Dale AJD, Okazaki H. Meningeal carcinomatosis. Neurologist. 1999;5:326–49. Arch Neurol. 1974;30:138–43. Bogousslavsky J, Regli F, Uske A. Thalamic infarcts: Mayeux R. Early alzheimer’s disease. N Engl J Med. clinical syndromes, etiology and prognosis. Neurology. 2010;362:2194–201. 1988;38:837–48. McKeith IG, Dickson DW, Lowe J. Diagnosis and man- Burger L, Kalvin N, Smith JL. Acquired lesions of the agement of dementia with Lewy bodies. Neurology. fourth cranial nerve. Brain. 1970;93:567–74. 2005;65:1863–72. Clarke JL, Perez HR, Jacks LM, et al. Leptomeningeal Mohr JP, Choi D, Grotta JC, Weir B, Wolf PA, editors. metastasis in the MRI era. Neurology. 2010;74: Stroke. Pathophysiology, diagnosis and management. 1449–554. 4th ed. New York, NY: Churchill Livingstone; 2004. Critchley M. The parietal lobes. New York, NY: Hafner; Pachner AR, Steere AC. The triad of neurologic manifes- 1966. tations of Lyme disease. Neurology. 1985;35:47–53. Cummings JL. Dementia and depression: an evolving Perkoff GT. Alcoholic myopathy. Annu Rev Med. enigma. J. Neuropsychiatry Clin Neurosci. 1989; 1971;22:125–32. Summer 1:236–42. Posner J, Saper C, Schiff N, Plum F. Diagnosis of stupor and Ducros A, Boukobza M, Porcher R, et al. The clinical and coma. New York, NY: Oxford University Press; 2007. radiological spectrum of reversible cerebral vasocon- Roos KL. . Semin Neurol. 1992;12: striction syndrome. A prospective series of 67 patients. 209–12. Brain. 2007;130:3091–101. Scheld WM, Whitley RJ, Marra CM. Infections of the Hecaen H, De Ajuriaguerra J. Balints syndrome: psychic central nervous system. Philadelphia, PA: Lippincott paralysis of visual fi xation and its minor forms. Brain. Williams and Wilkins; 2004. 1954;77:373–400. Steiner I, Kennedy PGE, Pachner AR. The neurotropic Hu WT, Mandrekar JN, Parisi JE, et al. Clinical features herpes viruses: herpes simplex and varicella-zoster. of pathologic subtypes of behavioral-variant fronto- Lancet Neurol. 2007;6:1015–28. temporal dementia. Arch Neurol. 2007;64:1611–6. Stiebel-Kalish H, Kalish Y, Bar-On RH, et al. Presentation, Jankovic J, Tolosa E, editors. Parkinson’s disease and natural history and management of carotid cavernous movement disorders. 5th ed. Baltimore, MD: Williams aneurysms. Neurosurgery. 2005;57:850–7. and Wilkins; 2007. Terushkin V, Stern BJ, Judson MA, et al. Neurosarcoidosis. Jensen MB, Chacon MR, Aleu A. Cervicocerebral arterial Neurologist. 2010;16:2–15. dissection. Neurologist. 2008;14:5–6. Toothaker TB, Rubin M. Paraneoplastic neurological syn- Johnson RT, Gibbs CJ. Creutzfeldt-Jakob disease and dromes. A review. The Neurologist. 2009;15:21–33. related transmissible spongiform encephalopathies. Victor M, Adams RD, Collins GH. The Wernicke- N Engl J Med. 1998;339:1994–2004. Korsakoff syndrome. 2nd ed. Philadelphia, PA: FA Joseph FG, Lammie GA, Scolding NJ. CNS lupus. Davis; 1989. Neurology. 2007;69:644–54. Weiner HL. The challenge of multiple sclerosis: how do Kaplan P. The clinical features, diagnosis and prognosis we cure a chronic heterogeneous disease? Ann Neurol. of nonconvulsive status epilepticus. Neurologist. 2009;65:239–48. 2005;11:348–61. Wilkins RH, Brody LA. The thalamic syndrome. Arch Kumar N. Copper defi ciency myelopathy (human sway- Neurol. 1969;20:559–62. back). Mayo Clinic Proc. 2006;81:1371–84. Williams DR, Lees AT. Progressive supranuclear palsy: Lee VH, Wijdicks EFM, Manno EM, Rabenstein AA. clinicopathological concepts and diagnostic chal- Clinical spectrum of reversible posterior leukoenceph- lenges. Lancet Neurol. 2009;8:270–9. alopathy syndrome. Arch Neurol. 2008;65:205–10. Wingerchuk DM, Lennon VA, Luccinetti CF, et al. The Levine JS, Branch W, Rauch J. The antiphospholipid spectrum of neuromyelitis optica. Lancet Neurol. syndrome. N Engl J Med. 2002;346:752–63. 2007;6:805–15. Index

Diseases in Case Reports

A Convulsive syncope, 228Ð229 Acute infl ammatory demyelinating CreutzfeldtÐJakob, 260 polyneuropathy (AIDP), 184Ð185 Cryptococcal meningitis, 262 Alcoholism, 244, 258Ð259, 277Ð278 Cysticercosis 248 Alzheimer’s disease, 268Ð269 Amiodarone toxicity, 199Ð200 Aneurysm D cavernous carotid, 277 Depression, 279Ð280 middle cerebral, 21 mycotic, 262Ð263 posterior communicating, 20Ð21, 159Ð160 E Angiitis of the CNS, 271Ð272 Encephalitis, 225, 227, 266Ð267 Anterior horn cell disease Exertional headache, 212 amyotrophic lateral sclerosis, 202 spinal muscular atrophy, 188 Anterior ischemic optic neuropathy, 154Ð155 F Antiphospholipid antibody syndrome, 274 Fibromuscular dysplasia, 21 Arterial dissection, 171, 259Ð260 Fracture, compression, 33 Arteriovenous malformation, 161Ð162 Fronto temporal dementia, 254Ð255

B G Back pain, 33 Glossopharyngeal neuralgia, 214, 216 Benign paroxysmal positional vertigo (BPPV), 220 Guillain-Barré syndrome, 184Ð185

C H Cardioembolism, 6, 7, 171Ð172, 257 Hematoma, cerebellar, 157Ð159 Carpal tunnel syndrome, 181, 183 Hemiballism, 7Ð8 Cataplexy, 243, 244 Hereditary neuropathy with liability to pressure palsies, 193 Cauda equina syndrome, 270Ð271 Herniation, 171Ð172 Cavernous sinus syndrome, 277 Herpes simplex encephalitis, 266Ð267 Cerebellar ataxia, 277Ð278 HIV complications, 264 Cerebellar hematoma, 152, 280 Huntington’s chorea, 273 Charles Bonnet syndrome, 247, 248 Hydrocephalus, communicating, 266 Chiari I, 213, 214 Hypnic headache, 214, 215 Chorea Huntington’s, 273 systemic lupus erythematosus, 275Ð276 I Chronic paroxysmal hemicrania, 214, 215 Infarction Cluster headache, 169Ð170, 214, 215 brainstem, 9Ð10, 10Ð11, 152, 171, 224Ð225

J.N. Alpert, The Neurologic Diagnosis: A Practical Bedside Approach, 295 DOI 10.1007/978-1-4419-6724-4, © Springer Science+Business Media, LLC 2012 296 Index

Infarction (cont) N cerebral, 6Ð7, 152Ð154, 171Ð172, 257, 259Ð260, Narcolepsy, 234, 235 262Ð263, 271Ð272 Neoplasm subthalamus, 7Ð8 acoustic neuroma, 225, 226 thalamus, 278 astrocytoma, 234, 235 Intracranial hypotension, 218 glioblastoma multiforme, 247, 248 hamartoma, 233, 234 meningioma, 224, 249Ð250 J orbital mass, metastatic, 246, 247 Juvenile myoclonic epilepsy, 233 schwannoma, 5Ð6 Neurocardiogenic syncope, 225, 226 Neuromyelitis optica, 276Ð277 L Neuropathy Lacunar infarction, 152 AIDP (Guillain-Barré syndrome), 184–185 LambertÐEaton, 18Ð19, 187Ð188 alcoholic neuropathy, 277Ð278 Leptomeningeal carcinomatosis, 269Ð270 carpal tunnel syndrome, 182Ð183 Lewy body disease, 279 CIDP, 16Ð17, 181Ð182 Lyme disease, 261Ð262 copper defi ciency, 272Ð273 Lymphoma, 270Ð271 critical illness, 196Ð197 diabetic, 30Ð31, 173Ð174, 181, 184, 191, 201 hereditary neuropathy with liability to pressure M palsies, 193 McArdle’s disease, 202Ð203 hereditary sensory and motor neuropathy, 197 Ménière’s disease, 220Ð221, 223, 243, 244 hypothyroid, 181 Meningitis, 22Ð23, 262 meralgia paresthetica, 185Ð186 Meralgia paresthetica, 185Ð186 mononeuropathy multiplex, 198Ð199 Metabolic encephalopathy, 4Ð5 nutritional, 194Ð195, 277Ð278 Migraine paraneoplastic, 188Ð189 with aura, 31Ð32, 209Ð210, 249Ð250 POEMS, 197Ð198 basilar, 212, 220, 221, 223 rheumatoid arthritis, 19Ð20 post-traumatic, 210Ð211 tarsal tunnel, 200Ð201 transformed, 214 ulnar neuropathy, 189Ð190 without aura, 207Ð208, 210 Multiple sclerosis, 11Ð13, 225, 226Ð227, 260Ð261 O Multiple system atrophy, 173 Obstructive sleep apnea, 33 Multisensory disequilibrium, 224, 225 , 203 Myasthenia gravis, 190, 193Ð194 Optic neuropathy, 272Ð273, 274 Myelopathy Orgasmic headache, 211 amyotrophic lateral sclerosis Orthostatic hypotension, 30Ð31, 173Ð174, 191, 230, 232 (ALS), 202 Otolithic crisis, 243Ð244 cervical spinal stenosis, 13Ð14 Chiari I, 213Ð214 copper defi ciency, 272Ð273 P epidural abscess, 255Ð257 Paraneoplastic neuropathy, 188Ð189 multiple sclerosis, 11Ð13 Parkinson’s disease, 5Ð8 neuromyelitis optica, 276Ð277 Paroxysmal kinesigenic choreoathetosis, 250 schwannoma, 5Ð6 Paroxysmal orthostatic tachycardia syndrome (POTS), subacute combined degeneration, 201Ð202 174, 233 Myopathy Periodic paralysis, hypokaliemic, 250, 251 alcoholic, 277Ð278 Plexopathy hyperthyroid, 190 brachial, 191Ð192 hypokaliemic periodic lumbosacral, 14Ð15, 180Ð181 paralysis, 250, 251 POEMS, 198 inclusion body myositis (IBM), 183Ð184 Polyarteritis nodosa, 198Ð199, 263Ð264 McArdle’s disease, 202Ð203 Posterior reversible encephalopathy myotonic dystrophy, 186Ð187 syndrome (PRES), 263Ð264 steroid, 19Ð20, 193Ð194 Progressive multifocal leukoencephalopathy, 264Ð265 Myositis, inclusion body, 183Ð184 Progressive supranuclear palsy, 253Ð254 Index 297

Pseudoseizures, 228, 229 T Pseudotumor cerebri, 246Ð247 Tabes dorsalis, 268 Tarsal tunnel, 200Ð201 Temporal arteritis, 211, 262 R Tendonitis, biceps, 187 Radiculopathy Thalamic syndrome, 278 cervical, 180 Thoracic outlet syndrome, 192Ð193 lumbar, 184, 249 TolosaÐHunt syndrome, 212Ð213 Toxicity alcohol, 277Ð278 S aminoglycoside, 223, 224 Sarcoidosis, 195Ð196, 254 amiodarone, 199Ð200 Seizure Transient global amnesia, 245Ð246 astatic, 244, 245 Transient ischemic attack complex partial, 230, 233, 234, 235, 267Ð268 carotid, 151Ð152, 246, 247, 248Ð249 simple partial, 224, 228, 229, 230, 247, 248 vertebrobasilar, 142, 154, 221, 223, status epilepticus (nonconvulsive), 267Ð268 245, 247, 248 Spinal muscular atrophy, 188 Transtentorial herniation, 280 Spinal stenosis Trigeminal neuralgia, 214Ð215 cervical, 13Ð14 Trochlear neuropathy, 278Ð279 lumbar, 184 Tuberculosis (epidural abscess), Subacute bacterial endocarditis, 262Ð263 255Ð257 Subarachnoid hemorrhage, 20Ð21, 21Ð22, 159Ð161, 262Ð263 Subclavian steal, 154 U Subdural hematoma, 258Ð259 Ulnar neuropathy, 189Ð190 SUNCT, 214, 215 Syncope convulsive, 228, 229Ð230 V heart block, 244, 245 Venous stasis retinopathy, 246, 247 near syncope, 230, 232 Venous sinus thrombosis, 257Ð258 neurocardiogenic, 228, 229Ð230 Vertebral artery dissection, 171 postmicturition, 234, 235 Vestibular neuritis, 221, 223 postpartum, 230, 232 postprandial, 233, 234 pregnancy, 234 W Syringomyelia, 170Ð171 Wallenberg’s syndrome, 9Ð10 Systemic lupus erythematosis, 275Ð276 Wernicke’s encephalopathy, 265Ð266